Download as pdf or txt
Download as pdf or txt
You are on page 1of 148

MATHEMATICS

(Standard)
DELHI
2023 CBSE Solved Paper

Time allowed : 3 hours Maximum Marks : 80


GENERAL INSTRUCTIONS:
Read the following instructions very carefully and strictly follow them:
(i) This question paper contains 38 questions. All questions are compulsory.
(ii) This question paper is divided into five Sections - A, B, C, D and E.
(iii) In Section A, Questions no. 1 to 18 are multiple choice questions (MCQs) and questions number 19 and 20 are Assertion-
Reason based questions of 1 mark each.
(iv) In Section B, Questions no. 21 to 25 are very short answer (VSA) type questions, carrying 2 marks each.
(v) In Section C, Questions no. 26 to 31 are short answer (SA) type questions, carrying 3 marks each.
(vi) In Section D, Questions no. 32 to 35 are long answer (LA) type questions carrying 5 marks each.
(vii) In Section E, Questions no. 36 to 38 are case study based questions carrying 4 marks each. Internal choice is provided in
2 marks questions in each case-study.
(viii) There is no overall choice. However, an internal choice has been provided in 2 questions in Section B, 2 questions in
Section C, 2 questions in Section D and 3 questions in Section E.
22
(ix) Draw neat diagrams wherever required. Take π = wherever required, if not stated.
7
(x) Use of calculators is not allowed.

(c) Mode = 2 Median – 3 Mean


Section-A (d) Mode = 2 Mean – 3 Median
1. The ratio of H.C.F to L.C.M of the least composite number 7. The pair of linear equations 2x = 5y + 6 and 15y = 6x – 18
and the least prime number is: (1 Mark)
represents two lines which are: (1 Mark)
(a) 1 : 2 (b) 2 : 1 (c) 1 : 1 (d) 1 : 3
(a) Intersecting (b) Parallel
2. The roots of the equation x2 + 3x – 10 = 0 are: (1 Mark)
(c) Coincident (d) Either intersecting or parallel
(a) 2, –5 (b) –2, 5 (c) 2, 5 (d) –2, –5
8. If a, b are zeroes of the polynomial x2 – 1, then value of
3. The next term of the A.P. 6, 24, 54 is: (1 Mark)
(a + b) is: (1 Mark)
(a) 60 (b) 96 (c) 72 (d) 216 (a) 2 (b) 1 (c) –1 (d) 0
4. The distance of the point (– 1, 7) from x-axis is: (1 Mark)
9. If a pole 6 m high casts a shadow 2 3 m long on the
(a) –1 (b) 7 (c) 6 (c)50 ground, then sun’s elevation is: (1 Mark)
5. What is the area of a semi-circle of diameter ‘d’?  (a) 60° (b) 45° (c) 30° (d) 90°
 [OS]* (1 Mark)
10. secq when expressed in terms of cot q, is equal to: (1 Mark)
1 1 2 1 1 2
(a) πd 2 (b) πd (c) πd 2 (d) πd 1 + cot 2 θ
16 4 8 2 (a) (b) 1 + cot 2 θ
cot θ
6. The empirical relation between the mode, median and
mean of a distribution is: (1 Mark) 1 + cot 2 θ 1 − cot 2 θ
(a) Mode = 3 Median – 2 Mean (c) (d)
(b) Mode = 3 Mean – 2 Median cot θ cot θ
*[OS] denotes Out of the Syllabus questions.
11. Two dice are thrown together. The probability of getting 17. If a, b are the zeroes of the polynomial p(x) = 4x2 – 3x – 7,
the difference of numbers on their upper faces equals  1 1
to 3 is: (1 Mark) then  +  is equal to: (1 Mark)
α β
1 2 1 1 7 −7 3 −3
(a) (b) (c) (d) (a) (b) (c) (d)
9 9 6 12 3 3 7 7
12. 6 cm 18. A card is drawn at random from a well-shuffled pack of
A C R
52 cards. The probability that the card drawn is not an
ace is: (1 Mark)
x
5 cm

3 cm
1 9 4 12
(a) (b) (c) (d)
13 13 13 13
Questions number 19 and 20 are Assertion and Reason based
B Q P
questions carrying 1 mark each. Two statements are given, one
In the given figure, DABC ~ DQPR, If AC = 6 cm , BC = 5 cm, labelled as Assertion (A) and the other is labelled as Reason (R).
QR = 3 cm and PR = x; then the value of x is: (1 Mark) Select the correct answer to these questions from the codes (a),
(a) 3.6 cm (b) 2.5 cm (c) 10 cm (d) 3.2 cm (b), (c) and (d) as given below.
13. The distance of the point (–6, 8) from origin is: (1 Mark) (a) Both Assertion (A) and Reason (R) are true and Reason
(a) 6 (b) –6 (c) 8 (d) 10 (R) is the correct explanation of the Assertion (A).
(b) Both Assertion (A) and Reason (R) are true, but Reason
14. In the given figure, PQ is a tangent to the circle with
(R) is not the correct explanation of the Assertion (A).
centre O. If ∠OPQ = x, ∠POQ = y, then x + y is: (1 Mark)
(c) Assertion (A) is true, but Reason (R) is false.
Q
(d) Assertion (A) is false, but Reason (R) is true.
19. Assertion (A): The probability that a leap year has 53
x y
P O 2
Sundays is .
7
Reason (R): The probability that a non-leap year has 53
5
(a) 45° (b) 90° (c) 60° (d) 180° Sundays is . (1 Mark)
7
15. In the given figure, TA is a tangent to the circle with centre O 20. Assertion (A): a, b, c are in A.P. if and only if 2b = a + c.
such that OT = 4 cm, ∠OTA = 30°, then length of TA is:
 (1 Mark) Reason (R): The sum of first n odd natural numbers is n2.
 (1 Mark)

Section-B
O
21. Two numbers are in the ratio 2 : 3 and their L.C.M is 180.
m
4c What is the H.C.F. of these numbers? (2 Marks)
30°
T A 22. If one zero of the polynomial p(x) = 6x2 + 37x – (k – 2) is
(a) 2 3 cm (b) 2 cm (c)
2 2 cm (d) 3 cm reciprocal of the other, then find the value of k.(2 Marks)
16. In DABC, PQ||BC. If PB = 6 cm, AP = 4 cm , AQ = 8 cm, 23. (a) Find the sum and product of the roots of the quadratic
find the length of AC. (1 Mark) equation 2x2 – 9x + 4 = 0. (2 Marks)
A OR
(b) Find the discriminant of the quadratic equation
4x2 – 5 = 0 and hence comment on the nature of roots
P Q of the equation.
24. If a fair coin is tossed twice, find the probability of getting
‘atmost one head’. (2 Marks)
B C 2 2 2
25. (a) Evaluate 5cos 60° + 4sec 30° − tan 45°  (2 Marks)
(a) 12 cm (b) 20 cm (c) 6 cm (d) 14 cm sin 2 30° + cos 2 30°

164 MATHS
OR (b) If AD and PM are medians of triangles ABC and
PQR, respectively where DABC ~ DPQR, prove that
(b) If A and B are acute angles such that sin(A – B) = 0 and
AB AD
2cos(A + B) – 1 = 0, then find angles A and B. = 
PQ PM

Section-C 34. A student was asked to make a model shaped like a


cylinder with two cones attached to its ends by using a thin
26. (a) How many terms are there in an A.P. whose first and aluminium sheet. The diameter of the model is 3 cm and its
fifth terms are –14 and 2, respectively and the last term total length is 12 cm. If each cone has a height of 2 cm, find
is 62. (3 Marks) the volume of air contained in the model. (5 Marks)
OR 35. The monthly expenditure on milk in 200 families of a
Housing Society is given below:
(b) Which term of the A.P. 65, 61, 57, 53, .... is the first
negative term?

1000–1500

1500–2000

2000–2500

2500–3000

3000–3500

3500–4000

4000–4500

4500–5000
Monthly
27. Prove that 5 is an irrational number. (3 Marks) Expenditure
(in `)
28. Prove that the angle between the two tangents drawn from
an external point to a circle is supplementary to the angle
Number of
subtended by the line-segment joining the points of contact 24 40 33 x 30 22 16 7
families
at the centre. (3 Marks)
Find the value of x and also, find the median and mean
sin A − 2sin 3 A expenditure on milk. (5 Marks)
29. (a) Prove that = tan A (3 Marks)
2 cos3 A − cos A
OR Section-E
(b) Prove that
36. Two schools ‘P’ and ‘Q’ decided to award prizes to their
sec A(1 – sin A)(sec A + tan  A) = 1.
students for two games of Hockey ` x per student and
30. Two concentric circles are of radii 5 cm and 3 cm. Find the Cricket ` y per student. School ‘P’ decided to award
length of the chord of the larger circle which touches the a total of ` 9,500 for the two games to 5 and 4 students
smaller circle. (3 Marks) respectively; while school ‘Q’ decided to award ` 7,370 for
the two games to 4 and 3 students respectively.
31. Find the value of ‘p’ for which the quadratic equation
px(x – 2) + 6 = 0 has two equal real roots. (3 Marks) Based on the above information, answer the following
questions:
Section-D
32. (a) A straight highway leads to the foot of a tower. A
man standing on the top of the 75 m high tower
observes two cars at angles of depression of 30° and
60°, which are approaching the foot of the tower. If
one car is exactly behind the other on the same side
of the tower, find the distance between the two cars.
(use 3 = 1.73 ) (5 Marks)
OR
(b) From the top of a 7 m high building, the angle of (i) Represent the following information algebraically
elevation of the top of a cable tower is 60° and the (in terms of x and y). (1 Mark)
angle of depression of its foot is 30°. Determine the (ii) (a) What is the prize amount for hockey? (2 Marks)
height of the tower. OR
33. (a) D is a point on the side BC of a triangle ABC such that (b) Prize amount on which game is more and by how much?
∠ADC = ∠BAC, prove that CA2 = CB.CD (5 Marks) (iii) What will be the total prize amount if there are
OR 2 students each from two games? (1 Mark)

CBSE 2023 (DELHI) 165


37. Jagdish has a field which is in the shape of a right angled 38. Governing council of a local public development authority
triangle AQC. He wants to leave a space in the form of a of Dehradun decided to build an adventurous playground on
square PQRS inside the field for growing wheat and the the top of a hill, which will have adequate space for parking.
remaining for growing vegetables (as shown in the figure). After survey, it was decided to build rectangular
In the field, there is a pole marked as O. playground, with a semi-circular area allotted for parking
Based on the above information, answer the following at one end of the playground. The length and breadth
questions: of the rectangular playground are 14 units and 7 units,
respectively. There are two quadrants of radius 2 units on
Y one side for special seats.

I
A
Parking
Playground

7 units
area
B
S II
R
14 units

Based on the above information, answer the following


questions:
X′ C P O Q X
(–200, 0) (200, 0) (i) What is the total perimeter of the parking area? 
 [OS]* (1 Mark)
(i) Taking O as origin, coordinates of P are (–200, 0) and
of Q are (200, 0). PQRS being a square, what are the (ii) (a) 
What is the total area of parking and the two
coordinates of R and S? (1 Mark) quadrants? [OS]* (2 Marks)
(ii) (a) What is the area of square PQRS? (2 Marks) OR
OR (b) What is the ratio of area of playground to the area
(b) What is the length of diagonal PR in square PQRS? of parking area? [OS]*
(iii) If S divides CA in the ratio K : 1, what is the value (iii) Find the cost of fencing the playground and parking
of K, where point A is (200, 800)? (1 Mark) area at the rate of ` 2 per unit. [OS]* (1 Mark)

EXPLANATIONS

1. (a) Least composite number = 4, least prime number = 2


We can write it as 6, 24, 54, 96
H.C.F (4, 2): L.C.M (4, 2) = 2 : 4 = 1 : 2
Therefore, the ratio of H.C.F to L.C.M of the least Hence, the next term is 96 .  (1 Mark)
composite number and the least prime number is 1 : 2. 4. (b) The distance of any point from x-axis is the
 (1 Mark) y-coordinate.
2. (a) Given: x2 + 3x – 10 = 0
Therefore, the distance of the point (–1, 7) from x-axis
By factorisation method, we get
is 7. (1 Mark)
⇒ x2 + 5x – 2x – 10 = 0 ⇒ x(x + 5) – 2(x + 5) = 0
5. (c) Out of the Syllabus
⇒ (x + 5)(x – 2) = 0 ⇒ x = –5, 2 (1 Mark)
6. (a) Empirical relationship between mean, median and
3. (b) Given series is 6, 24, 54
mode is: Mode = 3 Median – 2 Mean (1 Mark)
We can write this series as 6, 2 6, 3 6 which is an
7. (c) The equation can be written as
A.P. and common difference is 6 2x – 5y – 6 = 0 and 6x – 15y – 18 = 0
Next term can be obtained by adding common Here a1 = 2, b1 = –5, c1 = –6 and a2 = 6, b2 = –15,
difference 6 then 6, 2 6, 3 6, 4 6 c2 = –18

*[OS] denotes Out of the Syllabus questions.

166 MATHS
a1 2 1 b1 −5 1 c1 −6 1 ∠OQP = 90° (since the radius through the point of contact
= = ,= = and = = of a tangent to a circle is perpendicular to the tangent.)
a2 6 3 b2 −15 3 c2 −18 3
Thus, y + 90° + x = 180° ⇒ x + y = 90° (1 Mark)
a1 b1 c1
⇒ = = 15. (a) We know tangent makes 90° with the centre.
a2 b2 c2
Given ∠ATO = 30° & OT = 4 cm
Hence, the given pair of linear equation represents
AT AT 3 AT
coincident line. (1 Mark) cos30° = = ⇒ =
OT 4 2 4
8. (d) Given that, a and b are the zeros of x2 – 1
3
Here a = 1, b = 0, c = –1 ⇒ AT = 4 × = 2 3 cm  (1 Mark)
2
−b 0 16. (b) Given, in DABC, PQ || BC
\ α += β = = 0. (1 Mark)
a 1
Also, PB = 6 cm, AP = 4 cm and AQ = 8 cm
9. (a) Given that, A
Let QC = x cm
Height of the pole = 6 m
6m A
Length of shadow = 2 3 m
q 4 cm 8 cm
Let q be the elevation of the C B
2 3m P Q
sun, then
AB 6 6 cm x cm
⇒ tan q = ⇒ tan
= θ = 3 = tan 60°
BC 2 3
\ q = 60° (1 Mark) B C
10. (c) We know, sec q = 1 + tan q
2 2 Using basic proportionality theorem,
AP AQ 4 8
1 cot 2 θ + 1 ⇒ = ⇒ = ⇒ x = 12 cm
sec2 q = 1 + ⇒ sec2 q = PB QC 6 x
cot 2 θ cot 2 θ
So, AC = AQ + QC = 8 cm + 12 cm = 20 cm (1 Mark)
2
2
cot θ + 1 cot θ + 1
sec q = ⇒ sec q =  (1 Mark) 17. (d) p(x) = 4x2 – 3x – 7
cot θ2 cot θ −b 3
Here, α + β = ⇒ α + β = ...(i)
11. (c) Total number of outcomes = 36 a 4
Possible favourable outcomes = (1, 4), (2, 5), (3, 6), c −7
Also, αβ = ⇒ αβ = ...(ii)
(4, 1), (5, 2), (6, 3) a 4
Favourable outcomes = 6 3
1 1 α+β
\ Probability of getting difference of number 3 Now, + = = 4  [From (i) and (ii)]
α β αβ −7
6 1
= =  (1 Mark) 4
36 6 1 1 −3
⇒ + =  (1 Mark)
12. (b) Given, ∆ABC ~ ∆QPR α β 7
AC QR 6 3 18. (d) Total number of cards = 52
\ = ⇒ = ⇒ x = 2.5 cm (1 Mark)
BC PR 5 x So the number of possible outcomes = 52
13. (d) From the distance formula, we have Let E be the event of getting a non-ace card.
Distance of point (–6, 8) from O(0, 0) There will be 48 non-ace cards.
= Number of favourable outcomes = 48
(8 − 0) 2 + (−6 − 0) 2 = 64 + 36
Number of favourable outcomes 48 12
= 100 = 10 unit (1 Mark) P(E) = = = 
Number of possible outcomes 52 13
14. (b) In DOPQ, 12
Hence, the probability of getting a non-ace card is
.
We know that, ∠POQ + ∠PQO + ∠OPQ = 180° 13
[⸪ Sum of angles of a triangle = 180°] (1 Mark)

CBSE 2023 (DELHI) 167


19. (c) The leap year has 366 days i.e., 52 week and 2 days. OR
These two days can be; Sun-Mon, Mon-Tue, (b) Given equation: 4x2 − 5 = 0
Tue-Wed, Wed-Thu, Thu-Fri, Fri-Sat, Sat-Sun. Comparing with ax2 + bx + c = 0 we get,
2 a = 4, b = 0, c = –5 ½ Mark)

Thus, the probability of 53 Sundays = Therefore, D = b2 − 4ac
7
So, Assertion (A) is true. D = 02 − 4(4)(−5) = 80 > 0 ½ Mark)

Now, the non-leap year has 365 days i.e., 52 week Thus, D > 0
and 1 day. Hence, roots are real and distinct (1 Mark)
1 24. When a coin is tossed twice, the possible outcomes are
Thus, the probability of 53 Sundays =
7 {TT, HH, TH, HT} ½ Mark)

So, Reason(R) is false. (1 Mark) Number of possible outcomes = 4
20. (b) Assertion (A) is the definition of an arithmetic Favourable outcomes = {TT, HT, TH} ½ Mark)

progression. If a, b and c are in AP, then the common Number of favourable outcomes = 3
difference is d = b − a = c − b. Thus, we have 3
b = (a + c)/2, which is equivalent to 2b = a + c. \ Probability of getting atmost one head  (1 Mark)
4
Reason (R) is also true.
5cos 2 60  4sec 2 30  tan 2 45
Since, sum of first n odd natural number is n2. 25. (a) Given,
Hence, both Assertion (A) and Reason (R) are true, sin 2 30  cos 2 30
2 2
and Reason (R) is not the correct explanation of 1  2  2
Assertion (A).  (1 Mark) 5  + 4  − (1)
2  3
⇒  ½ Mark)

21. Given, ratio of the numbers = 2 : 3, L.C.M of numbers = 180 2 2
1  3
We know that, product of L.C.M and H.C.F of two   +  
 2   2 
numbers is equal to the product of the numbers.
⸫ L.C.M × H.C.F = a × b (a, b are the two numbers) 5 16
+ −1
15 + 64 − 12 67
 ½ Mark)
(½ ⇒ 4 3 ⇒ =  ½ Mark)

Let, numbers be 2x and 3x. 1 3 12 12
+
\ L.C.M = 2 × 3 × x = 6x 4 4
180 5cos 2 60  4sec 2 30  tan 2 45 67
\ 6x = 180 ⇒ x = = 30 ½ Mark)
(½ Hence,  (1 Mark)
6 2 2
12
sin 30  cos 30
 
\ Numbers are 30 × 2 = 60 and 30 × 3 = 90 (½ ½ Mark)
Now, L.C.M × H.C.F = ab(a, b are the numbers) OR
\ 180 × H.C.F = 60 × 90 ⇒ H.C.F = 30 (½ ½ Mark) (b) Given, sin(A – B) = 0 and 2cos(A + B) – 1 = 0
22. Given, one zero of p(x) = 6x + 37x − (k − 2) is
2 sin(A – B) = 0 = sin 0° ⇒ A – B = 0° ...(i) (½ ½ Mark)
reciprocal to the other. 1
cos(A + B) = = cos60°
Let α be the one root of the polynomial. 2
1 ⇒ A + B = 60° ...(ii) (½
½ Mark)
So, the other root will be . ½ Mark)
(½ By adding equation (i) and (ii), we get
α
−(k − 2) A – B + A + B = 0 + 60°
Thus, the product of the roots =  (½
½ Mark) 60°
6 ⇒ 2A = 60° ⇒ A = = 30° ½ Mark)

1 −(k − 2) 2
⇒ α× = ⇒ 6(1) = –k + 2 ½ Mark)

α 6 From equation (i), A = B = 30° ½ Mark)

⇒ k = 2 – 6 = –4 26. (a) Given, first term of the A.P. a  14
Hence, the value of k is –4. ½ Mark)
(½ Let the common difference be d. ½ Mark)

23. (a) 2x2 – 9x + 4 = 0 ⇒ 2x2 – 8x – x + 4 = 0 ½ Mark)
(½ Given, a5 = 2
⇒ 2x(x – 4) – 1(x – 4) = 0 ⇒ (2x – 1)(x – 4) = 0 a + 4d = 2   [⸪ an = a + (n – 1)d] ½ Mark)

1 –14 + 4d = 2 ⇒ 4d = 16 ⇒ d = 4 ½ Mark)

⇒ 2x – 1 = 0 or x – 4 = 0 ⇒ x = or x = 4 (½ ½ Mark)
2 Now the last term will be given by
1 9
Hence, sum of roots = 4 + = ½ Mark)
(½ l = a + (n – 1)d ⇒ 62 = –14 + (n – 1)4 (1 Mark)
2 2
1 ⇒ 76 = (n – 1)4 ⇒ (n – 1) = 19
Product of roots = 4 × = 2 ½ Mark)
(½ Hence, n = 20 ½ Mark)

2

168 MATHS
OR Proof: ∠OAP = 90° [⸪ Tangent is ⊥ to radius]
(b) Here, first term (a) = 65, second term = 61 Also, ∠OBP = 90° [⸪ Tangent is ⊥ to radius]
Common difference (d) = 61– 65 = – 4 ½ Mark)
(½  ½ Mark)

Let the nth term of the given A.P. be the first negative In quadrilateral OAPB, the sum of all interior angles
term.
= 360°
⸫ an < 0 ⇒ a + (n – 1)d < 0 ½ Mark)
(½ ⇒ ∠OAP + ∠OBP + ∠BOA + ∠APB = 360°
65
⇒ 65 + (n – 1)(–4) < 0⇒ n − 1 >  ½ Mark)
(½ ⇒ 90º + 90º + ∠BOA + ∠APB = 360°
4
⇒ ∠BOA + ∠APB = 180°
65 69 1
⇒n> +1 ⇒ n > ⇒ n > 17  ½ Mark)
(½ Hence proved. ½ Mark)

4 4 4
1 sinA − 2sin 3 A sinA  1 − 2sin 2 A 
Since, 18 is the natural number just greater than 17
4 =
29. (a) L.H.S =   (½
½ Mark)
So, n = 18 (½½ Mark) 2cos3 A − cosA cosA  2cos 2 A −1 
∴18th term is first negative term. ½ Mark)
(½ We know, sin2θ + cos2θ = 1
27. Let’s assume that 5 is a rational number. If 5 is  
sinA  sin 2 A + cos 2 A − 2sin 2 A 
a rational number, that means it can be written in the = ½ Mark)

form of a/b, where a and b are integers that have no  (
cosA  2cos 2 A − sin 2 A + cos 2 A ) 

common factor other than 1 and b is not equal to zero.
sinA  cos 2 A − sin 2 A 
 ½ Mark)
(½ =   = tanA = R.H.S. (1 Mark)
5 a cosA  cos 2 A − sin 2 A 
  5b  a
1 b Hence, L.H.S. = R.H.S.
On squaring both sides, we get Hence, proved. (1 Mark)
5b2 = a2 ⇒ b2 = a2/5  ...(i) (½
½ Mark)
OR
This means 5 divides a .2

That means it also divides a. (b) L.H.S = sec A 1  sin Asec A  tan A
a We write everything in terms of sinA and cosA
⇒  c ⇒ a = 5c ½ Mark)

5
1  1 sin A 
On squaring both sides, we get a 2  25c 2 = (1 − sin A )  +   (½
½ Mark)
cos A  cos A cos A 
On putting the value of a2 in equation (i), we get
5b2 = 25c2 ⇒ b2 = 5c2 ⇒ b2/5 = c2 ½ Mark)
(½ (1 − sin A)  1 + sin A  (1 − sin A)(1 + sin A) (½
½ Mark)
=   =
cos A  cos A  cos A × cos A
This means b is divisible by 5 and so b is also
2

divisible by 5. Therefore, a and b have 5 as a common (=


1 − sin A ) (1 − sin A )
2 2 2

factor. But this contradicts the fact that a and b are =


cos 2 A cos 2 A
coprime. This contradiction has arisen because of our
incorrect assumption that 5 is a rational number. So, cos 2 A
= = 1 = R.H.S.(1 Mark)
we conclude that 5 is irrational. (1 Mark) cos 2 A

28. Given: A circle with centre O having tangents PA and Hence, L.H.S. = R.H.S.
PB drawn from external point P. ½ Mark)
(½ Hence proved. (1 Mark)
To prove: ∠APB + ∠AOB = 180° ½ Mark)
(½ 30. Given: Two concentric circles are of radii 5 cm and 3 cm.
P  ½ Mark)

To find: The length of the chord of the larger circle
A which touches the smaller circle. ½ Mark)

Solution: PQ is chord of a larger circle and tangent of
a smaller circle.
B (1 Mark) As, tangent is perpendicular to the radius at the point
O
of contact S.
⸫ OSP = 90° ½ Mark)



CBSE 2023 (DELHI) 169


OR
(b) B
O
Q

h
S
P 60° E Line of sight (1 Mark)
A
In OSP (Right angled triangle) using Pythagoras 30°
Theorem, we get
OP2 = OS2 + SP2 ⇒ 52 = 32 + SP2 ½ Mark)

7m
7m
⇒ SP2 = 25 – 9 = 16 ⇒ SP = ±4
SP is length of tangent and cannot be negative
30°
⸫ SP = 4 cm. ½ Mark)

D C
QS = SP (Perpendicular from center bisects the chord
considering the larger circles) Let AD be the building and AC be the cable tower
Therefore, QS = SP = 4 cm Let AE = DC = x cm and height of tower be BC (½
½ Mark)
Length of the chord, PQ = QS + SP = 4 + 4 = 8 cm BE h h
In ∆ABE, tan60  ⇒ 3 ⇒x (1 Mark)
Therefore, the length of the chord of the larger circle AE x 3
is 8 cm. ½ Mark)
(½ AD 1 7
In DADC, tan30   

31. Given that the roots of the quadratic equation ⇒ (1 Mark)
DC 3 x
px( x  2)  6  0 are equal and real.
On solving, we get, px2 – 2px + 6 = 0 ½ Mark)
(½ 1 7


3 h ⇒ h = 21 m ½ Mark)

Comparing with general equation ax 2  bx  c  0,
we get, a = p, b = –2p and c = 6 ½ Mark)
(½ 3
Condition for two equal and real root is Hence, height of tower  21  7  28 m.  (1 Mark)
D = b2 – 4ac = 0 ½ Mark)
(½ 33. (a) Given: D is a point on the side BC of a triangle ABC
⇒ (–2p) – 4 · p · 6 = 0 ⇒ 4p – 24p = 0
2 2 such that ∠ADC = ∠BAC. ½ Mark)

⇒ 4p(p – 6) = 0 ⇒ 4p = 0 or (p – 6) = 0 To prove: CA = CB.CD
2
½ Mark)

⇒ p = 0 or p = 6 ½ Mark)
(½ A
Putting p = 0 in equation given we get 6 = 0 that is not
possible. ½ Mark)

Hence, value of p is 6 for which the equation has two
 (1 Mark)
equal real roots. ½ Mark)

32. (a) D E 
30° Line of sight B D C
60°
Proof: In ΔADC and ΔBAC, 
75 m

∠ADC = ∠BAC (Given)  ½ Mark)



(1 Mark) ∠ACD = ∠BCA (Common)  ½ Mark)

∴ ΔADC ~ ΔBAC(AA similarity criterion) (½
½ Mark)
60° 30°
We know that corresponding sides of similar triangles
C B x A
are in proportion.
Let x be the distance between two cars. ½ Mark)
(½ CA CD
75 75 ∴ = ⇒ CA2 = CB.CD ½ Mark)

In DCBD, tan60  
⇒ BC   25 3 m CB CA
BC 3
 (1 Mark) Hence proved. (1 Mark)
75 1 75 OR
In DACD, tan30  ⇒ 
 (1 Mark)
AC 3 25 3  x (b) Given: ΔABC ~ ΔPQR ½ Mark)

 
75 3 ⇒ x = 50 3 
⇒ x  25 3  ½ Mark)

To prove:
AB AD
=  ½ Mark)

⇒ x = 50 × 1.73 = 86.5 m (1 Mark) PQ PM

170 MATHS
P Radius (r) of cylindrical part = Radius of conical part
A 3
= cm = 1.5 cm  ½ Mark)

2
 (1 Mark) Volume of air present in the model = Volume of
B CQ cylinder + 2 × Volume of a cone ½ Mark)

D M R
Volume of cylinder = πr h2

Proof: ∆ABC ~ ∆PQR (Given)


= π × 1.52 × 8 = 18π cm3 (1 Mark)
We know that the corresponding sides of similar
triangles are in proportion. 1 2
Volume of cone = πr h
AB AC BC 3
∴ = =  …(i) (½
½ Mark) 1
PQ PR QR = π× 1.52 × 2= 1.5π cm3  (1 Mark)
3
Also, ∠A = ∠P, ∠B = ∠ , ∠C = ∠R…(ii) (½
½ Mark)
Since, AD and PM are medians, 2 cm
BC QR
∴ BD = and QM = …(iii) (½
½ Mark)
2 2 3 cm
From equations (i) and (iii), we get
12 cm ½ Mark)

AB BD
= …(iv) (½
½ Mark)
PQ QM
In ΔABD and ΔPQM, ∠B = ∠Q  [using eq. (ii)]
AB BD
From equation (iv), we have, =
PQ QM Volume of air present in the model = 18π + 2 × 1.5π
∴ ΔABD ~ ΔPQM  ½ Mark)
(½ 22
= 18π + 3π = 21π =21× =66 cm3  ½ Mark)

(SAS similarity criterion) 7
AB BD AD Hence, volume of air present in the model is 66 cm3.
⇒ = = (by defination of similarity)   ½ Mark)

PQ QM PM
Hence proved. ½ Mark)
(½ 35. To find the value of x, we need to use the information
that the total number of families is 200.
34. For the given statement first draw a diagram, In this
diagram, we can observe that ⸫ 200  24  40  33  x  30  22  16  7
Height (h1) of each conical part = 2 cm ⇒ 200 = 172 + x ⇒ x = 28
Height (h2) of cylindrical part = 12 – 2 – 2 = 8 cm So, there are 28 families who spend between ₹2500
 ½ Mark)
(½ and ₹3000 per month on milk. ½ Mark)

Frequency xi  A x  3250
Class Mid value (xi) di  A fi  di C.f
(fi) h 500
1000  1500 24 1250 –4 –96 24

1500  2000 40 1750 –3 –120 64


2000  2500 33 2250 –2 –66 97
2500  3000 28 2750 –1 –28 125
3000  3500 30 3250 = A 0 0 155
3500  4000 22 3750 1 22 177
4000  4500 16 4250 2 32 193

4500  5000 7 4750 3 21 200


∑ fi = 200 ∑ fi . di = –235
(2 Marks)

CBSE 2023 (DELHI) 171


Σf i d i −235 OR
Mean x =
A+ × h = 3250 + × 500
Σf i 200 (b) On substituting the value of x in (i), we get
= 3250 + (–1.175) × 500 = 3250 – 587.5 = 2662.5 5(980) + 4y = 9500 ½ Mark)

 (1 Mark) 9500 − 4900 4600
⇒y= = = 1150  (½ ½ Mark)
N 4 4
From the table, N = ∑ fi = 200 ⇒ ∑ fi = 200
2 Hence, prize amount of cricket is more by
⸫ Median class = 2500 – 3000 1150 – 980 = `170  (1 Mark)
Then l = 2500, C.f = 97, f = 28 and h = 500 (½
½ Mark) (iii) If there are 2 students each from two games, then
total prize money = 2x + 2y
N 
 − C. f  = 2(980) + 2(1150) = `4260 (1 Mark)
Median = l +  2 × h
 f  37. (i) Given, coordinates of P and Q are (–200, 0) and (200, 0).

( 200 − ( −200)) − ( 0 − 0)
2 2
 100 − 97   (1 Mark) Distance PQ =
2500 + 
=  × 500  2553.5
 28 
= 4002 400 
= ½ Mark)

36. (i) Given, Hockey `  x per student and cricket `  y per Therefore, the y-coordinates of R and S must be 400
student. For school ‘P’, the total prize amount units away from the x-axis. Since, R is vertically
for hockey and cricket is ` 9500. The number of above Q and S is vertically above P.
students awarded for hockey and cricket are 5 and 4 Hence, coordinates of R should be (200, 400) and of S
respectively. should be (–200, 400) ½ Mark)

Hence, 5x + 4y = 9500  …(i) (½
½ Mark) (ii) (a) Area of square PQRS = (side)2 (1 Mark)
For school ‘Q’, the total prize amount for hockey and = PQ = 400 = 160000 sq. units.
2 2
(1 Mark)
cricket is ` 7370. The number of students awarded are OR
4 and 3 respectively. (b) Length of diagonal PR in square PQRS
Hence, 4x + 3y = 7370 …(ii) (½
½ Mark) = 2 side = 400 2 units (2 Marks)
(ii) (a) Given equations are: (iii) Let’s say point S divides CA in the ratio K : 1 coordinates
of A are (200, 800) and y coordinates of C is 0.
5 x + 4 y= 9500 × 3
4 x + 3 y= 7370 × 4 Hence, using section formula for y coordinate of S, we
800 K + 1( 0 )
15x + 12y = 28500 ½ Mark)
(½ get 400 =  ½ Mark)

1+ K
16x + 12y = 29480 ½ Mark)

⇒ 400 + 400K = 800K ⇒ 400K = 400 ⇒ K = 1
–x = – 980 ½ Mark)
(½  ½ Mark)

Hence, prize amount for hockey is `980  ½ Mark)
(½ 38. Out of the Syllabus

172 MATHS
Outside DELHI
2023 CBSE Solved Paper

Time allowed : 3 hours Maximum Marks : 80


GENERAL INSTRUCTIONS:
Read the following instructions very carefully and strictly follow them:
(i) This question paper contains 38 questions. All questions are compulsory.
(ii) This question paper is divided into five Sections - A, B, C, D and E.
(iii) In Section A, Questions no. 1 to 18 are multiple choice questions (MCQs) and questions number 19 and 20 are
Assertion-Reason based questions of 1 mark each.
(iv) In Section B, Questions no. 21 to 25 are very short answer (VSA) type questions, carrying 2 marks each.
(v) In Section C, Questions no. 26 to 31 are short answer (SA) type questions, carrying 3 marks each.
(vi) In Section D, Questions no. 32 to 35 are long answer (LA) type questions carrying 5 marks each.
(vii) In Section E, Questions no. 36 to 38 are case study based questions carrying 4 marks each. Internal choice is provided
in 2 marks questions in each case-study.
(viii) There is no overall choice. However, an internal choice has been provided in 2 questions in Section B, 2 questions in
Section C, 2 questions in Section D and 3 questions in Section E.
22
(ix) Draw neat diagrams wherever required. Take π = wherever required, if not stated.
7
(x) Use of calculators is not allowed.

5. In the given figure, DE || BC. If AD = 3 cm, AB = 7 cm and


Section-A EC = 3 cm, then the length of AE is: (1 Mark)
1. If the zeroes of the quadratic polynomial x2 + (a + 1) x + b A
are 2 and –3, then (1 Mark)
m
3c

(a) a = –7, b = –1 (b) a = 5, b = –1


m

D E
7c

(c) a = 2, b = –6 (d) a = 0, b = –6
2. The number of quadratic polynomials having zeroes –5 3 cm
and –3 is: (1 Mark)
(a) 1 (b) 2 B C
(c) 3 (d) more than 3 (a) 2 cm (b) 2.25 cm (c) 3.5 cm (d) 4 cm
3. If the sum of the first n terms of an A.P. be 3n2 + n and its 6. The LCM of smallest 2-digit number and smallest
common difference is 6, then its first term is: (1 Mark) composite number is: (1 Mark)
(a) 2 (b) 3 (a) 12 (b) 4 (c) 20 (d) 40
(c) 1 (d) 4
7. The distance of the point (–4, 3) from y-axis is: (1 Mark)
cos 2 θ 1 (a) –4 (b) 4 (c) 3 (d) 5
4. 2
− 2 , in simplified form, is: (1 Mark)
sin θ sin θ
8. If one zero of the polynomial x2 + 3x + k is 2, then the
(a) tan2θ (b) sec2θ value of k. (1 Mark)
(c) 1 (d) –1 (a) –10 (b) 10 (c) 5 (d) –5
9. The point of intersection of the line represented by Assertion – Reason Based Questions: In question numbers 19
3x – y = 3 and y-axis is given by: (1 Mark) and 20, a statement of Assertion (A) is followed by a statement
(a) (0, –3) (b) (0, 3) (c) (2, 0) (d) (–2, 0) of Reason (R). Choose the correct option out of the following:
10. If the quadratic equation ax2 + bx + c = 0 has two real and (a) Both Assertion (A) and Reason (R) are true; and Reason (R)
equal roots, then ‘c’ is equal to: (1 Mark) is the correct explanation of Assertion (A).
−b 2 2 (b) Both Assertion (A) and Reason (R) are true; but Reason (R)
b −b b
(a) (b) (c) (d) is not the correct explanation of Assertion (A).
2a 2a 4a 4a (c) Assertion (A) is true but Reason (R) is false.
11. A card is drawn at random from a well shuffled deck of (d) Assertion (A) is false but Reason (R) is true.
52 playing cards. The probability of getting a face card is:
(1 Mark) 19. Statement A (Assertion): For 0 < θ ≤ 90o, cosec θ – cot θ
and cosec θ + cot θ are reciprocal of each other. (1 Mark)
1 3 4 1
(a) (b) (c) (d) Statement R (Reason): cosec2 θ – cot2 θ = 1
2 13 13 13
20. Statement A (Assertion): If 5 + 7 is a root of a quadratic
12. If ∆PQR ∼ ∆ABC ; PQ = 6 cm, AB = 8 cm and the
equation with rational co-efficients, then its other root
perimeter of ∆ABC is 36 m, then the perimeter of ∆PQR is:
is 5 − 7 . (1 Mark)
 [OS]* (1 Mark)
Statement R (Reason): Surd roots of a quadratic equation
(a) 20.25 cm (b) 27 cm
with rational co-efficients occur in conjugate pairs.
(c) 48 cm (d) 64 cm
13. The volume of a right circular cone whose area of the base Section-B
is 156 cm2 and the vertical height is 8 cm, is: (1 Mark)
(a) 2496 cm3 (b) 1248 cm3 21. Find the length of the shadow on the ground of a pole of
(c) 1664 cm 3
(d) 416 cm3 height 18 m when angle of elevation θ of the sun is such
14. The circumferences of two circles are in the ratio 4 : 5. 6
that tan θ = . (2 Marks)
What is the ratio of their radii? [OS]* (1 Mark) 7
A
(a) 16 : 25 (b) 25 : 16 (c) 2 : 5 (d) 4 : 5 22. (a) In the given figure, ABC
15. In the given figure, PA and PB are tangents from external is a triangle in which x x+2
point P to a circle with centre C and Q is any point on the DE || BC. If AD = x, DB
= x – 2, AE = x + 2 and D E
circle. Then the measure of ∠AQB is: (1 Mark)
EC = x – 1, then find the x – 2 x–1
A value of x.(2 Marks)
B C
OR
P 55° C Q
(b) Diagonals AC and BD of trapezium ABCD with
AB || DC intersect each other at point O. Show that
B OA OB .
=
1o OC OD
(a) 62 (b) 125° (c) 55° (d) 90°
2 D C
16. A box contains 90 discs, numbered from 1 to 90. If one
disc is drawn at random from the box, the probability that
it bears a prime number less than 23 is: (1 Mark) O
7 1 4 9
(a) (b) (c) (d)
90 9 45 89
17. The coordinates of the point where the line 2y = 4x + 5 A B
crosses x-axis is: (1 Mark)
23. (a) Show that 6n can not end with digit 0 for any natural
 −5   5  −5   −5  number ‘n’. (2 Marks)
(a)  0,  (b)  0,  (c)  , 0  (d)  , 0 
 4   2  4   2  OR
18. (cos4 A – sin4 A) on simplification, gives: (1 Mark) (b) Find the LCM and HCF of 72 and 120.
(a) 2sin2A – 1 (b) 2sin2A + 1 24. Find the points on the x-axis, each of which is at a distance
(c) 2cos A + 1
2
(d) 2cos2A – 1 of 10 units from the point A(11, –8). (2 Marks)

*[OS] denotes Out of the Syllabus questions.

174 MATHS
25. In the given figure, PA
is a tangent to the circle Section-D
A
drawn from the external 32. 250 apples of a box were weighed and the distribution of
point P and PBC is masses of the apples is given in the following table:
130o
the secant to the circle P C Mass (in grams) 80–100 100–120 120–140 140–160 160–180
with BC as diameter. If B O
Number of a
∠AOC = 130 , then find
o
20 60 70 x 60
apples
the measure of ∠APB,
where O is the centre of the circle. (2 Marks) (i) Find the value of x and the mean mass of the apples.
 (3 Marks)
(ii) Find the modal mass of the apples. (2 Marks)
Section-C 33. (a) Two tangents TP and TQ are drawn to a circle with
centre O from an external point T.
B
26. In the given figure, AB and Prove that ∠PTQ = 2∠OPQ. (5 Marks)
CD are diameters of a circle
with centre O perpendicular P
to each other. If OA = 7 cm, D O C
find the area of shaded region. T O
 [OS]* (3 Marks)
A Q
27. If sin θ + cos θ = p and sec θ + cosec θ = q, then prove that
q(p2 – 1) = 2p. (3 Marks) OR
(b) A circle touches the side A
28. (a) The sum of two numbers is 15. If the sum of their
BC of a ∆ABC at a point
3
reciprocals is , find the two numbers. (3 Marks) P and touches AB and AC B P C
10 when produced at Q and
Q R
OR R respectively. Show that
1
(b) If α and β are roots of the quadratic equation AQ = (Perimeter of ∆ABC).
x2 – 7x + 10 = 0, find the quadratic equation whose 2
roots are α2 and β2. 34. A solid is in the shape of a right-
circular cone surmounted on a hemisphere, the radius of
29. Find the ratio in which the line segment joining the points each of them being 3.5 cm and the total height of the solid
A(6, 3) and B(–2, –5) is divided by x-axis. (3 Marks) is 9.5 cm. Find the volume of the solid. (5 Marks)
30. In the given figure, E is a point on the side CB produced of 35. (a) Find the sum of integers between 100 and 200 which
an isosceles triangle ABC with AB = AC. If AD ⊥ BC and are (i) divisible by 9 (ii) not divisible by 9.(5 Marks)
EF ⊥ AC, then prove that ∆ABD ~ ∆ECF.(3 Marks)
OR
A (b) Solve the equation: –4 + (–1) + 2 + 5 + ... + x = 437.

Section-E
F 36. “Eight Ball” is a game played on a pool table with 15 balls
numbered 1 to 15 and a “cue ball” that is solid and white.
Of the 15 numbered balls, eight are solid (non-white)
coloured and numbered 1 to 8 and seven are striped balls
E B D C numbered 9 to 15.
31. (a) Find the HCF and LCM of 26, 65 and 117, using
prime factorisation. (3 Marks)
OR
(b) Prove that 2 is an irrational number.

*[OS] denotes Out of the Syllabus questions.

CBSE 2023 (Outside DELHI) 175


The 15 numbered pool balls (no cue ball) are placed in a 38. A coaching institute of A
large bowl and mixed, then one ball is drawn out at random. Mathematics conducts classes
Based on the above information, answer the following in two batches I and II and fees
questions: for rich and poor children are B
different. In batch I, there are
(i) What is the probability that the drawn ball bears
20 poor and 5 rich children,
number 8? (1 Mark)
whereas in batch II, there are 5
(ii) What is the probability that the drawn ball bears an poor and 25 rich children. The 45o
even number? (2 Marks) total monthly collection of fees 30o
from batch I is ₹ 9000 and from P 36 cm O
OR
batch II is ₹ 26,000. Assume that each poor child pays ₹ x
What is the probability that the drawn ball bears a number,
per month and each rich child pays ₹ y per month.
which is a multiple of 3?
(iii) What is the probability that the drawn ball is a solid
coloured and bears an even number? (1 Mark)
37. Radio towers are used for transmitting a range of
communication services including radio and television.
The tower will either act as an antenna itself or support one
or more antennas on its structure. On a similar concept,
a radio station tower was built in two Sections A and B.
Tower is supported by wires from a point O.
Distance between the base of the tower and point O is
36 cm. From point O, the angle of elevation of the top of
the Section B is 30° and the angle of elevation of the top Based on the above information, answer the following
of Section A is 45°. questions:
Based on the above information, answer the following (i) Represent the information given above in terms of
questions: x and y. (1 Mark)
(i) Find the length of the wire from the point O to the top
(ii) Find the monthly fee paid by a poor child. (2 Marks)
of Section B. (1 Mark)
OR
(ii) Find the distance AB. (2 Marks)
Find the difference in the monthly fee paid by a poor
OR child and a rich child.
Find the area of ∆OPB. (iii) If there are 10 poor and 20 rich children in batch II,
(iii) Find the height of the Section A from the base of the what is the total monthly collection of fees from
tower. (1 Mark) batch II? (1 Mark)

EXPLANATIONS

1. (d) Let a and b are the zeroes of the polynomial 2. (d) Given, the zeroes of the polynomial are –5 and –3.
x2 + (a + 1)x + b As we know that the general form of the polynomial
Here, a = 2 and b = –3 with zeroes a and b is given by:
−(a + 1) k[x2 – (a + b)x + ab], where k is any real number
Sum of zeroes, α + β =
1
= k[x2 – (–5 – 3)x + (–5)(–3)] = k[x2 + 8x + 15]
⇒ 2 + (–3) = – a – 1 ⇒ a = 0
k can take infinite value. Therefore, the number of
b
Product of zeroes, αβ = ⇒ 2(–3) = b ⇒ b = – 6 polynomials having zeroes –5 and –3 is more than 3.
1 (1 Mark)
  (1 Mark)

176 MATHS
3. (d) Given, Sn = 3n2 + n 11. (b) Total number of outcomes = 52
First term, S1 = 3(1)2 + 1 = 4 (1 Mark) No. of face card = 12
12 3
2
cos 2 θ − 1 P(getting a face card)
= =  (1 Mark)
4. (d) cos θ − 1 = 52 13
2 2
sin θ sin θ sin 2 θ 12. (b) Out of the Syllabus
− sin 2 θ
= = −1 [⸪ sin2q + cos2q = 1] (1 Mark) 13. (d) Given, area of the base
sin 2 θ
i.e., pr2 = 156 cm2
5. (b) Given, AD = 3 cm, AB = 7 cm and EC = 3 cm.
Vertical height of cone,
Let AE = x cm, then AC = AE + EC = x + 3 cm
h = 8 cm h
A
As we know, volume of cone
m x
3c 1 2 1
m = πr h = × 156 × 8
7c E 3 3 r
D
= 52 × 8 = 416 cm2 (1 Mark)
3 cm
14. (d) Out of the Syllabus
B C A
15. (a) Given, ∠APB = 55°
In DABC, DE || BC
Also, ∠PAC = ∠PBC = 90° C Q
AD AE 55°
∴ = [By Thale's theorem]  [Tangent is ⊥ to radius] P
AB AC
Now, in quadrilateral PACB B
3 x
⇒ = ⇒ 3(x + 3) = 7x ⇒ 3x + 9 = 7x ∠APB + ∠PAC + ∠PBC + ∠ACB = 360°
7 x+3
9 [⸪ Sum of interior angles in quadrilateral = 360°]
⇒ 7x – 3x = 9 ⇒ 4x = 9 ⇒ x = = 2.25 cm (1 Mark)
4 ⸫ 55° + 90° + 90° + ∠ACB = 360° ⇒ ∠ACB = 125°
6. (c) Smallest 2-digit number = 10 As we know, that the angle subtended by an arc at the
Smallest composite number = 4 centre is double the angle subtended by an arc at the
By prime factorisation, we get 10 = 2 × 5, 4 = 2 × 2 remaining part of the circle.
Now, L.C.M of 4, 10 is 2 × 5 × 2 = 20
125° 1
Therefore, the L.C.M of smallest 2-digit number and ∴ ∠ACB = 2 ∠AQB ⇒ ∠AQB
= = 62 °
smallest composite number is 20. (1 Mark) 2 2
(1 Mark)
7. (b) The distance of the point (x, y) from y-axis is its
16. (c) Prime numbers less then 23 are 2, 3, 5, 7, 11, 13, 17,
x-coordinate.
and 19.
Hence, the distance of the point (–4, 3) from y-axis is
4 units. (1 Mark) No. of favourable outcomes = 8
8. (a) Given, polynomial f(x) = x2 + 3x + k Total no. of possible outcomes = 90
It is also given that 2 is one of its zeroes. Hence, P(getting a disc having a prime number less
∴ f(2) = 0 ⇒ (2)2 + 3(2) + k = 0 No. of favourable outcomes 8 4
⇒ 4 + 6 + k = 0 ⇒ k = –10 than 23) = = = 
Total no. of possible outcomes 90 45
Hence, the value of k is –10. (1 Mark)
 (1 Mark)
9. (a) Given equation of line is 3x – y = 3
As we know that at the y-axis, x = 0 17. (c) Given equation of line is 2y = 4x + 5
On substituting the value of x in the given equation, As we know, any point where the line crosses x-axis is
we have, 3(0) – y = 3 ⇒ y = –3 of (x, 0) i.e., its y-coordinate is 0.
∴ The point of intersection is (0, –3). (1 Mark)
On putting y = 0 in given equation of line, we get
10. (d) Given, quadratic equation is ax2 + bx + c = 0 5
Condition for real and equal roots is D = 0 2(0) = 4x + 5 ⇒ x = −
4
∴b2 – 4ac = 0 [ D = b2 – 4ac]  −5 
Hence, the given line crosses x-axis at  , 0  .
b 2  4 
⇒ b2 = 4ac ⇒ c =  (1 Mark)  (1 Mark)
4a

CBSE 2023 (Outside DELHI) 177


18. (d) Given, cos4A – sin4A = (cos2A)2 – (sin2A)2 In DABC, DE || BC (1 Mark)
= (cos A – sin A) (cos A + sin A)
2 2 2 2 AD AE x x+2
∴ =⇒ ½ Mark)
= [By BPT] (½
[ a2 – b2 = (a – b) (a + b)] DB EC x − 2 x −1
= (cos2A – sin2A) (1) [ cos2A + sin2A = 1] ⇒ x (x – 1) = (x + 2) (x – 2)
= cos A – (1 – cos A)[ sin2A = 1 – cos2A]
2 2 ⇒ x2 – x = x2 – 4 [ (a – b) (a + b) = a2 – b2]
= 2 cos2A – 1 (1 Mark) ⇒ x = 4 ½ Mark)

19. (a) Since, cosec2q – cot2q = 1 OR
⇒ (cosec q – cot q) (cosec q + cot q) = 1 (b) Given: ABCD is a trapezium where AB || DC and
diagonals AC and BD intersect each other at O.
[ a – b = (a – b) (a + b)] 2 2
A B
1
⇒ cosec θ − cot θ =
cosec θ + cot θ E
O  ½ Mark)

∴ It is clear that (cosec q – cot q) and (cosec q + cot q)
are reciprocal of each other.
Hence, Reason is also true and the correct explanation D C
of Assertion. (1 Mark)
OA OB
20. (a) If one root of a quadratic equation with rational To prove: = . ½ Mark)

OC OD
coefficient is irrational number, then other root will be
its irrational conjugate pair. Construction: From the point O, draw a line EO
touching AD at E such that, EO || DC || AB.
If one root is 5 + 7 , then other root will be 5 − 7 .
Proof: In ΔADC, we have OE || DC
Hence, Assertion and Reason both are true and Reason
Therefore, by using Basic Proportionality Theorem
is the correct explanation of Assertion. (1 Mark)
AE AO
21. Let AB be a pole of height 18 m and length of the =  …(i) (½ ½ Mark)
ED CO
shadow be x m.
Now, In ΔABD, OE || AB
A
Therefore, by using Basic Proportionality Theorem
AE BO
= …(ii)
 ½ Mark)
(½ ED DO
18 m
From equation (i) and (ii), we get
AO BO OA OB
q = ⇒ =  ½ Mark)

CO DO OC OC
B C
Hence proved.
AB ½ Mark)

In DABC, tan θ =  23. (a) For a number 6n to end with the digit zero (0), it must
BC be divisible by 5, as we already know that any number
6 18 6 having unit place as 0 or 5 is divisible by 5.
⇒ = [Given,
= tan θ =and AB 18 m]
7 BC 7 Prime factorisation of 6n = (2 × 3)n (1 Mark)
As we can see that, the prime factorisation of 6n
18 × 7
⇒ BC = = 3 × 7 = 21 m ½ Mark)
(½ doesn’t contain prime number 5.
6
Therefore, it is clear from above that, 6n is not divisible
Hence, length of the shadow = 21 m ½ Mark)
(½ by 5 for any natural number n and hence, it proves
A that 6n can never end with the digit 0 for any natural
22. (a)
number n. (1 Mark)
x+

x OR
2

D E (b) The prime factorisation of 72 and 120 is given by


½ Mark)
72 = 2 × 2 × 2 × 3 × 3, 120 = 2 × 2 × 2 × 3 × 5 (½
2

x–
x–

H.C.F (72, 120) = Product of common factors with


B C lowest power = 2 × 2 × 2 × 3 = 24 (½½ Mark)

178 MATHS
L.C.M (72, 120) = Product of prime factors with ⇒ 10 × 15 = 3x(15 – x) ⇒ 150 = 45x – 3x2
½ Mark)
highest power = 2 × 2 × 2 × 3 × 3 × 5 = 360(½ ½ Mark)
⇒ 3x2 – 45x + 150 = 0 ⇒ 3(x2 – 15x + 50) = 0 (½
Hence, the H.C.F and L.C.M of 72 and 120 are 24 and
⇒ x2 – 10x – 5x + 50 = 0 ⇒ x(x – 10) –5(x – 10) = 0
360 respectively. ½ Mark)

⇒ (x – 5) (x – 10) = 0 ⇒ x = 10 or 5 ½ Mark)

24. Let the point on x-axis be B(x, 0) which is at a distance
of 10 units from A(11, –8). If first number = 10, then other number = 5
∴ Distance AB = 10 units and if first number = 5, then other number = 10
∴ Two required numbers are 5 and 10. (1 Mark)
⇒ ( x − 11) 2 + (0 − (−8)) 2 =10
OR
2
⇒ x + 121 − 22 x + 64 = 10  ½ Mark)
(½ (b) Given, a and b are the roots of x2 – 7x + 10 = 0
On squaring both sides, we get ⇒ x2 – 7x + 10 = 0 ⇒ x2 – 5x – 2x + 10 = 0(½ ½ Mark)
⇒ x2 – 22x + 185 = 100 ⇒ x2 – 22x + 85 = 0 (½ ½ Mark) ⇒ x(x – 5) –2 (x – 5) = 0 ⇒ (x – 2) (x – 5) = 0
⇒ x – 17x – 5x + 85 = 0 ⇒ x(x – 17) –5(x – 17) = 0
2 ⇒ x = 2 and x = 5 ½ Mark)

⇒ (x – 17) (x – 5) = 0 ⇒ x = 17 or 5 ∴ a = 2 and b = 5
Hence, the required points are (17, 0) and (5, 0). (1 Mark) Hence, a2 = 4 and b2 = 25
25. As we know that the A Now, we need to find the quadratic equation having
tangents at a point to a roots a2 and b2 i.e., 4 and 25. ½ Mark)

circle is perpendicular to P 130°
As we know, quadratic equation can be written as
the radius at the point of B O C
contact. ½ Mark)
x2 – (sum of roots)x + product of roots = 0 (½
∴ ∠PAO = 90° ...(i) (1 Mark) x – (4 + 25)x + (4 × 25) = 0
2

Now, in DPAO x2 – 29x + 100 = 0 (1 Mark)


∠PAO + ∠APB = ∠AOC m n
29.
 [Exterior angle property of a triangle] A(6, 3) P(x, 0) B(–2, –5)
⇒ 90° + ∠APB = 130° [From (i)] We know that co-ordinate of y on x-axis is 0.
⇒ ∠APB = 130° – 90° = 40° (1 Mark) Let P(0, y) divides AB in the ratio m : n
26. Out of the Syllabus So we have, co-ordinate of y.
27. Given, sin q + cos q = p and sec q + cosec q = q my2 + ny1 m(−5) + n × 3
y= ⇒ 0=  (1 Mark)
L.H.S. = q(p2 – 1) = (sec q + cosec q) [(sin q + cos q)2 – 1] m+n m+n
⇒ –5m + 3n = 0 × (m + n)  (1 Mark)
 1 1  2 2
m 3
=  +  [sin θ + cos θ + 2sin θ cos θ − 1] ⇒ 5m = 3n ⇒ =
 cos θ sin θ  (1 Mark)
 n 5
 sin θ + cos θ  So, m : n = 3 : 5 (1 Mark)
=   [1 + 2sin θ cos θ − 1]  (1 Mark)
 sin θ cos θ  30. Given, ABC is an isosceles triangle in which AB = AC.
[ sin2q + cos2q = 1] Also, AD ⊥ BC and EF ⊥  AC. ½ Mark)

sin θ + cos θ A
= × 2sin θ cos θ = 2(sin q + cos q)
sin θ cos θ
 [sinq + cosq = p]
= 2p = R.H.S
Hence, L.H.S = R.H.S proved. (1 Mark) F
28. (a) Given, sum of two numbers is 15.
Let one of the number be x, then the other number be
15 – x. ½ Mark)
(½ E B D C
According to the question, we have To prove: ∆ABD ~ ∆ECF ½ Mark)

1 1 3 15 − x + x 3 Proof: ABC is an isosceles triangle.
+ = ⇒ = (½ ½ Mark)
x 15 − x 10 (15 − x) x 10  AB = AC (Given)

CBSE 2023 (Outside DELHI) 179


⸫ ∠ABD = ∠ECF (Angles opposite to equal sides (i) Total number of apples = 250
are equal) ...(i) (½½ Mark) 210 + x = 250 ⇒ x = 40 ½ Mark)

In ∆ABD and ∆ECF,
∠ADB = ∠EFC (Each 90°) ½ Mark)
(½ Now, mean ( x ) =
∑fx i i
=
27700 + 150 x

∠ABD = ∠ECF [From equation (i)] (½ ½ Mark) ∑f i 210 + x

½ Mark)
∆ABD ~ ∆ECF (by AA similarity criterion) (½ 27700 + 150 × 40
= = 134.8 ½ Mark)

Hence proved. 210 + 40
31. (a) The prime factorisation of 26, 65 and 117 is given by (ii) Here, modal class is 120–140 as it has maximum
26 = 2 × 13, 65 = 5 × 13, 117 = 9 × 13 (1 Mark) frequency.
H.C.F. (26, 65, 117) = Product of common factor with \ l = 120, h = 20, f1 = 70, f0 = 60, f2 = x = 40
lowest power = 13  (1 Mark)  ½ Mark)

L.C.M. (26, 65, 117) = Product of prime factors with  f1 − f 0 
Mode = l + × h  ½ Mark)

highest power = 2 × 5 × 9 × 13 = 1170 (1 Mark) 2
 1 − f0 − f2 
f
Hence, the H.C.F and L.C.M of 26, 65 and 117 are
13 and 1170 respectively.  70 − 60 
120 + 
=  × 20
 2 × 70 − 60 − 40 
OR
10 10
(b) Let us assume, to the contrary, that 2 is a rational 120 +
= × 20 = 120 + × 20
140 − 100 40
number. If 2 is a rational number, then it can be
a = 120 + 5 = 125 (1 Mark)
written in form of , where a and b are co-prime
integers and b ≠ 0. b 33 (a) Given: TP and TQ are the tangents to a circle having
a centre O. ½ Mark)

Now, 2 = ⇒ 2b = a ½ Mark)

b P
On squaring both sides, we get ⇒ 2b2 = a2…(i)
⇒ a2 is divisible by 2 ⇒ a is divisible by 2 (½½ Mark)
So, we can write a = 2c (where c is any integer)
T x O 
On putting the value of a in equation (i), we get
b2
⇒2b2 = (2c)2 ⇒ = c2
2 Q
⇒ b2 is divisible by 2 ⇒ b is divisible by 2 (½½ Mark)
Since, a and b are both divisible by 2 then 2 is a To prove: ∠PTQ = 2 ∠OPQ ½ Mark)

common factor of a and b. Proof: We know that the tangents drawn from an
But this contradicts the fact that a and b are co-primes external point are equal.
 ½ Mark)

\ PT = TQ ⇒ ∠TPQ = ∠TQP ½ Mark)

This contradiction has arisen because of our incorrect
assumption that 2 is a rational number. ½ Mark)
[Equal sides have equal oppisite angles](½
Hence, 2 is a irrational number. (1 Mark) In ∆PTQ, ∠PTQ + ∠TPQ + ∠TQP = 180°
32.  [Sum of interior angles of a triangle]
Mass (in Number of xi fi xi ⇒ x° + ∠TPQ + ∠TPQ = 180° ⇒ 2∠TPQ = 180° – x°
grams) apples (fi) x
⇒ ∠TPQ= 90° − …(i) (1 Mark)
80-100 20 90 1800 2
100-120 60 110 6600 Since, tangent is ⊥ to the radius at the point of contact.
120-140 70 130 9100 \ ∠OPT = 90° ⇒ ∠OPQ + ∠TPQ = 90°
140-160 x 150 150x x x
⇒ ∠OPQ + 90° – = 90° ⇒ ∠OPQ =  (1 Mark)
160-180 60 170 10200 2 2
∑ fi = 210 + x ∑ fi xi = 27700+150x ⇒ 2∠OPQ = ∠PTQ [∠PTQ = x] (1 Mark)
 (2 Marks) OR

180 MATHS
(b) We know that the length of tangents drawn from an ½ Mark)
tn = a + (n –1)d ⇒ 198 = 108 + (n – 1)9 (½
external point to a circle are equal. ⇒ 90 = (n – 1)9 ⇒ 10 = n – 1 ⇒ n = 11 (1 Mark)
A n
Now, = Sn [ a + l ] [⸪ l = tn= last term]
2
11 11 × 306
= [108 + 198] = = 11 × 153 = 1683
 2 2 (1 Mark)
P
B C
(ii) Total numbers between 100 and 200 are 99.
Q R
101, 102, ........199 ½ Mark)

(1 Mark) 99
n 99
Sn
Now, =
2
[=
a + l]
2
[ ]
101 + 199=
2
× 300

= 99 × 150 = 14850 (1 Mark)



AQ = AR …(i) [Tangents from A] \ Sum of integers not divisible by 9 between
100 and 200
BP = BQ …(ii) [Tangents from B]
= Sum of all integers between 100 and 200 – Sum
CP = CR …(iii) [Tangents from C] (1 Mark)
of integers divisible by 9 between 100 and 200
Now, Perimeter of DABC = AB + AC + BC = 14850 – 1683 = 13167 (1 Mark)
= AB + AC + BP + CP OR
= AB + AC + BQ + CR [From (ii) & (iii)] (1 Mark) (b) Given, (–4) + (–1) + 2 + 5 + ... + x = 437
= (AB + BQ) + (AC + CR) = AQ + AR The given series is an A.P.
= AQ + AQ = 2 AQ
[From (i)] (1 Mark) ½ Mark)
Here, a = – 4 and d = – 1 – (–4) = – 1 + 4 = 3 (½
1 n
\ AQ = (Perimeter of DABC) (1 Mark) Now, Sn = [2a + (n – 1)d] ½ Mark)

2 2
34.
n
⇒ 437 = [2 (–4) + (n – 1) (3)] ½ Mark)

2
½ Mark)
⇒ 874 = n [– 8 + 3n – 3] ⇒ 874 = n [3n – 11] (½
⇒ 3n2 – 11n – 874 = 0

9.5 cm ⇒ 3n2 – 57n + 46n – 874 = 0 ½ Mark)



⇒ 3n (n – 19) + 46 (n – 19) = 0
3.5 cm (1 Mark)
−46
⇒ (3n + 46) (n – 19) = 0 ⇒ n = or n = 19 (1 Mark)
3
\ n = 19 ( n cannot be in fraction)
 Now, tn = a + (n – 1) d ½ Mark)

Radius of Cone = Radius of Hemisphere = 3.5 cm = (– 4) + (19 – 1) 3 = –4 + 18 × 3 = –4 + 54 = 50
Height of Cone = (9.5 – 3.5) cm = 6 cm (1 Mark) Hence, The value of x is 50. (1 Mark)
Volume of Solid = Volume of Hemisphere + Volume 36. (i) Total number of balls = 15
2 1 2 π 2
of Cone = πr 3 + = πr h r [ 2r + h ]  (1 Mark) Number of ball bears numbers 8 = 1
3 3 3
22 1
= × 3.5 × 3.5 [ 2 × 3.5 + 6]  (1 Mark) P(drawing a ball bears number 8) =  (1 Mark)
7×3 15
22 (ii) Even number balls are 2, 4, 6, 8, 10, 12 and 14
= × 0.5 × 3.5 [ 7 + 6] = 166.83 cm3 (1 Mark)
3 Total number of even balls = 7 (1 Mark)
35. (a) (i) Numbers between 100 and 200 divisible by 9 are 7
P(drawing a ball bears an even number) = (1 Mark)
108, 117, .... 198 15
Here, a = 108 and d = 9 OR

CBSE 2023 (Outside DELHI) 181


Number of ball bears a number which is a multiple of OR
3 are 3, 6, 9, 12 and 15 (1 Mark) 1
\ Total number of ball bears a number, which is a As we know, Area of DOPB = × Base × Height
2
multiple of 3 = 5.  (1 Mark)
P(drawing a ball bears a number having a multiple 1 1
5 1 = × OP × BP = × 36 × 12 3
= =
of 3)  (1 Mark) 2 2
15 3
(iii) Number of solid coloured balls = 8 = 36 × 6 3 = 216 3 cm3 (1 Mark)
Number of solid coloured balls having an even (iii) Height of section A from base of towers is PA.
number = 4 ⸫ PA = 36 cm. (1 Mark)
P(drawing a solid coloured and bears an even number 38. (i) Let the monthly fees of poor children be ` x and of
4 rich children be ` y.
ball) =  (1 Mark)
15 For Batch I:
37. (i) A 20x + 5y = 9000 …(i)
For Batch II:
5x + 25y = 26000 …(ii) (1 Mark)
B (ii) Multiply equation (ii) by 4 and substract from
equation (i), we get
20x + 5y = 9000
45° 20x + 100y = 104000
30° – – –
P O –95y = –95000
36 cm
Let AP be the tower. ⇒ y = 1000 ½ Mark)

Hence, monthly fees paid by rich child is ` 1000.
OP 3 36
In DBOP, cos30° = ⇒ =[Given, OP = 36 cm] On substituting value of y in equation (i), we get
OB 2 OB
20x + 5 (1000) = 9000
36 × 2 72 72 3
⇒ OB= = = × ⇒ 20x + 5000 = 9000 ½ Mark)

3 3 3 3
⇒ 20x = 9000 – 5000
72 3
= = 24 3 cm  (1 Mark) 4000
3 =
⇒ x = 200
20
BP 1 BP
(ii) In DBOP, tan 30° = ⇒ = Hence, the monthly fees paid by poor child is ` 200.
OP 3 36  (1 Mark)
36 3 36 3 OR
⇒ BP = ×= = 12 3 cm (1 Mark)
3 3 3 Difference in monthly fees paid by Rich child and
AP poor child = 1000 – 200 = ` 800 (2 Marks)
Now In DAOP, tan 45° =
OP (iii) No. of poor children = 10
AP No. of rich children = 20
⇒ 1 = ⇒ AP = 36 cm [Given, OP = 36 cm]
36 Total monthly collection of fees from batch II
Hence, AB = AP – BP = 36 – 12 3
= 10 × 200 + 20 × 1000 = 2000 + 20000 = ` 22000
= 12(3 – 3) cm  (1 Mark)  (1 Mark)

182 MATHS
DELHI Term-II
2022 CBSE Solved Paper

Time allowed : 2 hours Maximum Marks : 40


GENERAL INSTRUCTIONS:
Read the following instructions carefully and strictly follow them:
(i) This question paper contains 14 questions. All questions are compulsory.
(ii) This question paper is divided into three Sections viz. Section A, B and C.
(iii) Section A – comprises of 6 questions (Q. Nos. 1 to 6) of two marks each. Internal choice has been provided in
two questions.
(iv) Section B – comprises of 4 questions (Q. Nos. 7 to 10) of three marks each. Internal choice has been provided in
one question.
(v) Section C – comprises of 4 questions (Q. Nos. 11 to 14) of four marks each. An internal choice has been provided in
one question. It also contains two case study based questions.
(vi) Use of calculator is not permitted.

X
Section-A
1. A solid piece of metal in the form of a cuboid of dimensions
11 cm × 7 cm × 7 cm is melted to form ‘n’ number of solid O
A
7
spheres of radii cm each. Find the value of n.  40o
2
 [OS] (2 Marks)
Y B
2. (a) In Figure, AB is diameter of a circle centered at O. BC
is tangent to the circle at B. If OP bisects the chord 11 1 49
, −3, − ,.... is ?
3. (a) Which term of the A.P. −
AD and ∠AOP = 60°, then find m∠C. (2 Marks) 2 2 2
(2 Marks)
A OR
P (b) Find a and b so that the numbers a, 7, b, 23 are in A.P.
60o 4. Find the sum of first 20 terms of an A.P. whose nth term is
O given as an = 5 – 2n. (2 Marks)
D
5. Solve the quadratic equation: x2 – 2ax + (a2 – b2) = 0 for x.
(2 Marks)
B C 6. If mode of the following frequency distribution is 55, then
find the value of x.  (2 Marks)
OR
Class 0–15 15–30 30–45 45–60 60–75 75–90
(b) In Figure, XAY is a tangent to the circle centered at O.
If ∠ABO = 40°, then find m∠BAY and m∠AOB. Frequency 10 7 x 15 10 12
Section-B P

7. Heights of 50 students of class x of a school are recorded


and following data is obtained: (3 Marks) T O
R

130–135

135–140

140–145

145–150

150–155

155–160
Height
Q
(in cm)
13. Case Study-1:
Number of
4 11 12 7 10 6
Students Kite Festival
Find the median height of the students. Kite festival is celebrated in many countries at different
times of the year. In India, every year 14th January is
8. (a) The mean of the following frequency distribution is celebrated as International Kite Day. On this day many
25. Find the value of f. (3 Marks) people visit India and participate in the festival by flying
Class 0–10 10–20 20–30 30–40 40–50 various kinds of kites.
Frequency 5 18 15 f 6 The picture given below, shows three kites flying together.

d B
OR A
(b) Find the mean of the following data using assumed

60 m
50 m
mean method:
Class 0–5 5–10 10–15 15–20 20–25 30o 60o
D C E
Frequency 8 7 10 13 12
In Figure, the angles of elevation of two kites (Points
9. Two men on either side of a cliff 75 m high observe the A and B) from the hands of a man (Point C) are found
angles of elevation of the top of the cliff to be 30° and 60°. to be 30° and 60° respectively. Taking AD = 50 m and
Find the distance between the two men. (3 Marks) BE = 60 m, find:
B (1) The lengths of strings used (take them straight) for
kites A and B as shown in the figure. (2 Marks)
75 m

(2) The distance ‘d’ between these two kites. (2 Marks)


30o 60o 14. Case Study-2:
C D
A
A ‘circus’ is a company of performers who put on shows of
10. Construct a pair of tangents to a circle of radius 3 cm which acrobats, clowns etc. to entertain people started around 250
are inclined to each other at an angle of 60°.[OS] (3 Marks) years back, in open fields, now generally performed in tents.
One such ‘Circus Tent’ is shown below.
Section-C
11. (a) The sum of two numbers is 34. If 3 is subtracted from
one number and 2 is added to another, the product of
these two numbers becomes 260. Find the numbers.
(4 Marks)
OR
(b) The hypotenuse (in cm ) of a right angled triangle is The tent is in the shape of a cylinder surmounted by a
6 cm more than twice the length of the shortest side. conical top. If the height and diameter of cylindrical part are
If the length of third side is 6 cm less than thrice the 9 m and 30 m respectively and height of conical part is 8 m
length of shortest side, then find the dimensions of the with same diameter as that of the cylindrical part, then find:
triangle. (1) The area of the canvas used in making the tent. (3 Marks)
12. In Figure, PQ is a chord of length 8 cm of a circle of radius (2) The cost of the canvas bought for the tent at the rate ` 200
5 cm. The tangents at P and Q meet at a point T. Find the per sqm, if 30 sq m canvas was wasted during stitching.
length of TP. (4 Marks)  (1 Mark)

184 MATHS
EXPLANATIONS

1. Out of the Syllabus −11 −1 49


3. (a) Given A.P. is , −3, ,...,
2. (a) 2 2 2
A −11 11 5
Here, a = and d =−3 + =  ½ Mark)

2 2 2
P
60o 49
Given, an =  ½ Mark)

O 2
D −11 5 49
⇒ a + (n – 1) d = an ⇒ + ( n − 1) =
2 2 2
⇒ (n – 1) = 12 ⇒ n = 13 (1 Mark)
B C OR
Given, BC is tangent to the circle at B. (b) Since, a, 7, b, 23 are in A.P.
∠ABC = 90° [\ Tangent is ⊥ to the radius at the point \ b – 7 = 23 – b ⇒ b + b = 23 + 7
of contact] ⇒ 2b = 30 ⇒ b = 15 …(i) (1 Mark)
As we know that the line drawn through the centre of a Also, 7 – a = b – 7 ⇒ 7 – a = 15 – 7 [From (i)]
circle to bisect the chord is perpendicular to the chord. ⇒7–a=8⇒a=7–8=–1
\ ∠OPA = 90°[ OP bisects the chord AD] (½ ½ Mark) Hence, a = – 1 and b = 15 (1 Mark)
Now, in DAOP, 4. Given, an = 5 – 2n
For n = 1, a1 = 5 – 2 (1) = 3
∠OPA + ∠AOP + ∠OAP = 180° [Sum of int ∠'s of D]
For n = 2, a2 = 5 – 2 (2) = 1
⇒ 90° + 60° + ∠OAP = 180° ⇒ ∠OAP = 30° (½ ½ Mark)
Here, a = 1 and d = a2 – a1 = – 2 ½ Mark)

Now, in DABC,
n
∠ACB = 180° – ∠ABC – ∠BAC [⸪∠OAP = ∠BAC] Now, Sn = [2a + (n –1)d] ½ Mark)

2
= 180° – 90° – 30° = 60° (1 Mark) 20
S20 = [2(1) + (20 – 1)(–2)] = 10 [2 + 19 (–2)]
OR 2
(b) = 10 [2 – 38] = 10 × (– 36) = – 360 (1 Mark)
X 5. We have, x2 – 2ax + (a2 – b2) = 0
⇒ (x2 – 2ax + a2) – b2 = 0 ½ Mark)

O ⇒ (x – a) – b = 0 
2 2
[a – 2ab + b = (a – b)2]
2 2
A
40° ⇒ (x – a – b) (x – a + b) = 0  ½ Mark)

[a2 – b2 = (a – b) (a + b)]
Y B ⇒ x – a – b = 0 or x – a + b = 0
In DAOB, OA = OB [Radii of same circle] ⇒ x = – (–a –b) or x = – (–a + b) ½ Mark)

\ ∠BAO = ∠ABO [Equal sides have equal opp. ⇒ x = a + b or x = a – b ½ Mark)

angles] 6.
⇒∠BAO = 40° [∠AOB = 40° (Given)] (½ ½ Mark) Class 0–15 15–30 30–45 45–60 60–75 75–90
Now, ∠BAO + ∠AOB + ∠ABO = 180° Frequency 10 7 x 15 10 12
[Sum of interior angles of a D] Given, mode is 55. The maximum frequence is 15,
⇒ 40° + ∠AOB + 40° = 180° ⇒ ∠AOB = 100° (½ ½ Mark) therefore, Modal class is 45-60.
Given XAY is a tangent to the circle. ½ Mark)
So, l = 45, f1 = 15, f0 = x, f2 = 10 and h = 15(½
\ ∠OAY = 90° [\ Tangent is ⊥ to the radius at the f1 − f 0
Mode =
l+ ×h
point of contact] 2 f1 − f 0 − f 2
⇒ ∠BAO + ∠BAY = 90° ⇒ 40° + ∠BAY = 90° 15 − x
⇒ 55 = 45 + × 15 ½ Mark)

⇒ ∠BAY = 50° (1 Mark) 2 × 15 − x − 10 

CBSE 2022 (DELHI Term-II) 185


15 − x 15 − x ⇒ 25(44 + f) = 940 + 35f ⇒ 1100 +25f = 940 + 35f
⇒ 55
= − 45 × 15 ⇒ 10
= × 15 ⇒ 35f –25f = 1100 + 940 ⇒ 10f = 160 ⇒ f = 16
30 − x − 10 20 − x 
 ½ Mark)
(½ Hence, the value of f is 16. (1 Mark)
⇒ 200 – 10x = 225 – 15x ⇒ 15x – 10x = 225 – 200 OR
⇒ 5x = 25 ⇒ x = 5 ½ Mark)

Class Frequency Class di =xi–A fi di
7. (fi) Marks (xi)
Height Number of Cumulative 0-5 8 2.5 –10 –80
(in cm.) Students (fi) Frequency (c.f.)
5-10 7 7.5 –5 –35
130–135 4 4
10-15 10 12.5 = A 0 0
135–140 11 15
15-20 13 17.5 5 65
140–145 12 27
20-25 12 22.5 10 120
145–150 7 34
Total ∑fi = 50 ∑fi di = 70
150–155 10 44
(2 Marks)
155–160 6 50 Let, assumed mean, A = 12.5
Total ∑fi = N = 50
Mean= A +
∑ fi d=
i
12.5 +
70 ½ Mark)

(1 Mark)
N
∑ if 50 
Here, = 25 . The cumulative frequency just greater = 12.5 + 1.4 = 13.9
2
than 25 is 27 and corresponding class is 140–145. Hence, required mean is 13.9 ½ Mark)

\ The median class is 140–145 9.
B
So, l = 140, c.f. = 15, f = 12, and h = 5 ½ Mark)

N 
 2 − c. f .  ½ Mark)
(½ ½ Mark)

Median = l + × h 75 m
 f  30°
 60°
  C A
D 
 25 − 15  10 AB 1 75  1 
=140 +  × 5 = 140 + × 5 In DBAC, tan 30º = = ∵ tan 30° = 
 12  12 AC

3 AC  3
= 140 + 4.167 = 144.167 ⇒ AC = 75 3 m …(i) (1 Mark)
Hence, median height of the students is 144.167 cm. AB 75
 (1 Mark) Now, in DABD, tan 60º = ⇒ 3=
AD AD
8. 75 3 75 3
Class Frequency Class fi xi ⇒ AD = ×= = 25 3 m …(ii) (½ ½ Mark)
3 3 3
(fi) Marks (xi) Now, CD = AC + AD
0–10 5 5 25 = 75 3 + 25 3 = 100 3 m [From (i) & (ii)] (1 Mark)
10–20 18 15 270
Hence, the distance between two men is= 100 3 m m.
20–30 15 25 375
10. Out of the Syllabus
30–40 f 35 35f
11. Let first number be x, then other number be 34 – x
40–50 6 45 270 According to the question,
Total ∑fi = 44 + f ∑ fixi = 940 + 35f (x – 3) (34 – x + 2) = 260 (1 Mark)
 (1 Mark) ⇒ (x – 3) (36 – x) = 260 ⇒ –x2 + 36x + 3x – 108 = 260

Mean x =
∑x f
i i
⇒ x2 – 39x + 260 + 108 = 0 ⇒ x2 – 39x + 368 = 0
½ Mark)
(½ ⇒ x2 – 23x – 16x + 368 = 0 (1 Mark)
∑f i
 ⇒ x(x – 23) – 16 (x – 23) = 0 ⇒ (x – 16) (x – 23) = 0
940 + 35 f ⇒ x – 16 = 0 or x – 23 = 0 ⇒ x = 16 or x = 23 (1 Mark)
=⇒ 25 =
44 + f
[ Given, x 25] (½
½ Mark)
When, x = 16 then other number = 34 – 16 = 18

186 MATHS
When, x = 23 then other number = 34 – 23 = 11 AD 1 50 (1 Mark)
In DADC, sin 30° = ⇒ = 
Hence, the required number will be either 16 and 18 or AC 2 AC
23 and 11. (1 Mark) ⇒ AC = 100 m
OR BE 3 60
In DBEC, sin 60° = ⇒ =
(b) Let DABC be a right angled triangle, right angled at B, BC 2 BC
as shown in below figure. 60 × 2 20 × 3 × 2
A ⇒ BC= = = 40 3 (1 Mark)
3 3 
2x + 6 (2) Thus the length of strings used are 100 m and 40 3 m
3x – 6
Since DCE is a straight line
B x C ∠DCA + ∠ACB + ∠BCE = 180°
Let BC is the shortest side having length x. 30° + ∠ACB + 90° = 180°
Then, length of hypotenuse is 2x + 6 ...(i) ∠ACB = 180° – 90° = 90° (1 Mark)
Length of third side is 3x – 6 ...(ii) ACB is a right angled triangle
According to Pythagoras theorem, Applying pythagoras theorem
AC2 = AB2 + BC2 ⇒ (2x + 6)2 = (3x – 6)2 + x2 (1 Mark) In DACB, (AB)2 = (AC)2 + (BC)2
⇒ 4x2 + 36 + 24x = 9x2 + 36 – 36x + x2
d2 = (100)2 + ( 40 3 )2 = 10000 + 4800 = 14800
x = 10 cm [⸪ x can't be 0] (1 Mark)
Now, from equation (i), = 20 37 m  (1 Mark)
AC = 2x + 6 ⇒ 2(10) + 6 = 26 cm (1 Mark) 14.
and, from equation (ii),
AB = 3x – 6 = 3(10) – 6 = 24 cm
8m
Thus, the dimension of the triangle are 10 cm, 24 cm,
and 26 cm. (1 Mark)
12. Join OT.
Let it meets PQ at the point R. 9m
Then ∆TPQ is isosceles triangle, TO is the angle (1 Mark)
bisector of ∠PTO. 30 m 
[⸪ TP = TQ = Tangents from T upon the circle](1 Mark)
Here we have
⸫ OT ⊥ PQ
radius of conical part = radius of cylindrical part
⸫ OT bisects PQ.
30
PR = RQ = 4 cm = = 15m = r (say)
2
Now, OR = OP2 – PR2 = 52 – 42 = 3 cm  (1 Mark) Given height of conical = 8 m and height of cylindrical
Now, ∠TPR + ∠RPO = 90° (⸪ TPO = 90°) part = 9 m
∠TPR + ∠PTR (⸪ TRP = 90°) ∴
= l (15) 2 + =
82 225 +=
64 289 17 m  (1 Mark)
=
∠RPO = ∠PTR (1 Mark)
(1) The area of the canvas used in making the tent
⸫ Right triangle TRP is similar to the right triangle
= Curved suface area of cone + Curved surface area
PRO [By A–A rule of similar triangles] of cylinder
TP RP TP 4 20 22 22
⸫ = ⇒ = ⇒ TP = cm  (1 Mark) = prl + 2prh = × 15 × 17 + 2 × × 15 × 9
PO RO 5 3 3 7 7
13. (1) B 22 22
d = × 15(17 + 18) = × 15 × 35 = 1650 m 2 (1 Mark)
A 7 7 
60 m (2) Total canvas bought for the tent = 1650 + 30 = 1680 m2
50 m
30° ⸫ cost of canvas for the tent
60°
D C E = 1680 × 200 = ` 3,36,000 (1 Mark)

CBSE 2022 (DELHI Term-II) 187


Outside DELHI Term-II
2022 CBSE Solved Paper

Time allowed : 2 hours Maximum Marks : 40


GENERAL INSTRUCTIONS:
Read the following instructions very carefully and strictly follow them:
(i) This question paper contains 14 questions. All questions are compulsory.
(ii) This Question Paper is divided into 3 Sections - Section A, B and C.
(iii) Section-A comprises of 6 questions (Q. Nos. 1 to 6) of 2 marks each. Internal choice has been provided in
two questions.
(iv) Section-B comprises of 4 questions (Q. Nos. 7 to 10) of 3 marks each. Internal choice has been provided in
one question.
(v) Section-C comprises of 4 questions (Q. Nos. 11 to 14) of 4 marks each. An internal choice has been provided in one
question. It also contains two case study based questions.
(vi) There is no overall choice in the question paper. However, internal choice has been provided in some questions.
Attempt any one choice in such questions.
(vii) Use of calculator is not permitted.

3. (a) The curved surface area of a right circular cylinder is


Section-A 176 sq. cm and its volume is 1232 cu. cm. What is the
height of the cylinder? (2 Marks)
1. (a) If the sum of the roots of the quadratic equation
13 OR
ky2 – 11y + (k – 23) = 0 is more than the product (b) The largest sphere is carved out of a solid cube of side
21
of the roots, then find the value of k. (2 Marks) 21 cm. Find the volume of the sphere. [OS]
OR 4. If the first term of an A.P. is 5, the last term is 15 and the sum
of first n terms is 30, then find the value of n. (2 Marks)
(b) If x = –2 is the common solution of quadratic equations
ax2 + x – 3a = 0 and x2 + bx + b = 0, then find the value 5. For the following frequency distribution, find the mode:
of a2b. Class 25–30 30–35 35–40 40–45 45–50
2. In Fig., there are two concentric circles with centre O. If Frequency 12 5 14 8 9
ARC and AQB are tangents to the smaller circle from the
(2 Marks)
point A lying on the larger circle, find the length of AC, if
6. If the mean of the following frequency distribution is 18,
AQ = 5 cm. (2 Marks)
then find the missing frequency ‘f ’. (2 Marks)
B
Class 11–13 13–15 15–17 17–19 19–21 21–23 23–50
Frequency 3 6 9 13 f 5 4
Q
A O Section-B
R
7. (a) Find the value of ‘p’ for which the quadratic equation
p(x – 4)(x – 2) + (x – 1)2 = 0 has real and equal roots.
C (3 Marks)
OR A
(b) Had Aarush scored 8 more marks in a Mathematics
test, out of 35 marks, 7 times these marks would have
been 4 less than square of his actual marks. How
many marks did he get in the test?
8. Construct a pair of tangents to a circle of radius 4 cm
which are inclined to each other at an angle of 60o. O
 [OS] (3 Marks)
9. There is a small island in the middle of a 100 m wide river B C
and a tall tree stands on the island. P and Q are points 6 cm D 8 cm
directly opposite to each other on two banks and in line 13. Case Study-1
with the tree. If the angles of elevation of the top of the tree
Khurja is a city in the Indian state of Uttar Pradesh famous
from P and Q are respectively 30o and 45o, find the height
for the pottery. Khurja pottery is traditional Indian pottery
of the tree. (Use 3 = 1.732 ) (3 Marks)
work which has attracted Indians as well as foreigners
n with a variety of tea-sets, crockery and ceramic tile works.
10. In an A.P., the sum of first n terms is ( 3n + 5) . Find the A huge portion of the ceramics used in the country is
2
25th term of the A.P. (3 Marks) supplied by Khurja and is also referred as ‘The Ceramic
Town’.
Section-C One of the private schools of Bulandshahr organised an
Educational Tour for class 10 students to Khurja. Students
11. From the top of an 8 m high building, the angle of were very excited about the trip. Following are the few
elevation of the top of a cable tower is 60o and the angle pottery objects of Khurja.
of depression of its foot is 45o. Determine the height of the
tower. (Take 3 = 1.732 ) (4 Marks)
12. (a) In Fig., if a circle touches the side QR of ∆PQR
at S and extended sides PQ and PR at M and N,
respectively, then (4 Marks)

P
R N
1
prove that PM= ( PQ + QR + PR )
2
II I
OR
(b) In Fig. a triangle ABC is drawn to circumscribe a circle Students found the shapes of the objects very interesting
of radius 4 cm such that the segments BD and DC into and they could easily relate them with mathematical
which BC is divided by the point of contact D are of shapes viz sphere, hemisphere, cylinder etc. Maths teacher
lengths 6 cm and 8 cm respectively. If the area of ∆ABC who was accompanying the students asked following
is 84 cm2, find the lengths of sides AB and AC. questions:

CBSE 2022 (Outside DELHI Term-II) 189


(a) The internal radius of hemispherical bowl (filled
completely with water) in I is 9 cm and radius and
height of cylindrical jar in II is 1.5 cm and 4 cm
respectively. If the hemispherical bowl is to be
emptied in cylindrical jars, then how many cylindrical
jars are required? [OS] (2 Marks)
(b) If in the cylindrical jar full of water, a conical funnel of
same height and same diameter is immersed, then how
much water will flow out of the jar? [OS] (2 Marks)
14. Case Study-2
Yoga is an ancient practice which is a form of meditation Age Group 15–25 25–35 35–45 45–55 55–65 65–75 75–85
and exercise. By practising yoga, we not even make our Number of
8 10 15 25 40 24 18
body healthy but also achieve inner peace and calmness. People
The International Yoga Day is celebrated on 21st of June Based on the above, find the following:
every year since 2015. (a) Find the median age of people enrolled for the camp.
To promote Yoga, Green park society in Pune organised a  (2 Marks)
7-day Yoga camp in their society. The number of people of (b) If x more people of age group 65–75 had enrolled for
different age groups who enrolled for this camp is given the camp, the mean age would have been 58. Find the
as follows: value of x. (2 Marks)

EXPLANATIONS

1. (a) Given, quadratic equation ∴ From eqn (i), a(–2)2 + (–2) – 3a = 0


Ky2 – 11y + (k – 23) = 0 ⇒a=2
According to question, From eqn (ii), (–2)2 + b(–2) + b = 0
⇒4–b=0⇒b=4
13
α + β = αβ + Now, a2 b = 22 × 4 = 16 (1 Mark)
21
If the quadratic equation is ax2 + bx + c = 0 then the Sum 2. Here AC and AB are tangents from external point A to
−b c the smaller circle.
of roots is and product of the roots is (1 Mark) ∴ AC = AB
a a
−(−11) K − 23 13 11 21K − 483 + 13K Now, AB is chord of bigger circle and OQ is
∴ = + ⇒ = perpendicular bisector of chord AB. (1 Mark)
K K 21 K 21K
or AB = 2 AQ
⇒ 11 × 21 = 34 K – 483 Now, AB = 2 × 5 = 10 cm {Given AQ = 5 cm}
⇒ 231 + 483 = 34 K ⇒ 34 K = 714 ∴ AC = 10 cm  AB = AC (1 Mark)
714 3. (a) Given,
⇒ K= = 21
34  (1 Mark) Curved surface area of a right circular cylinder = 176 cm2
OR ⇒ 2prh = 176 ⇒ prh = 88 ...(i)
(b) Given equations are, also given, vol. of cylinder = 1232 cm 3

ax2 + x – 3a = 0 ...(i) ⇒ pr2h = 1232 ...(ii)


x + bx + b = 0
2
...(ii) On dividing equ.(ii) from (i), we get (1 Mark)
2
Since, x = –2, is the common solution of the given πr h 1232
⇒ =⇒ r = 14 cm
above quadratic equation therefore it must satisfied πrh 88
the given equation. (1 Mark) put the value of r in equ. (i), we get

190
190 MATHS
⇒ p × 14 × h = 88 Given, mean = 18
22 Now,
⇒ × 14 × h =88 ⇒ h = 2 cm (1 Mark)
7 Σfi = 40 + f and Σfixi = 704 + 20f (1 Mark)
OR Σfi xi
Out of the Syllabus ∴ Mean =
Σf i
4. Given,
a = 5, an = 15, Sn = 30 704 + 20 f
∴18 = ⇒ 720 + 18 f = 704 + 20 f
Now, 40 + f
 an = a + (n – 1)d ⇒ 16 = 2f ⇒ f = 8 (1 Mark)
∴ 15 = 5 + (n – 1)d 7. (a) Given,
⇒ (n – 1)d = 10 ...(i) ⇒ p(x – 4) (x – 2) + (x – 1)2 = 0
Now,  (1 Mark)
⇒ p(x2 – 4x – 2x + 8) + (x2 – 2x + 1) = 0
n
∵ S=
n ( 2a + (n − 1)d ) ⇒ (p + 1)x2 + (–6p – 2)x + 8p + 1 = 0 (1 Mark)
2
Now, equation has real and equal roots
n
⇒ 30= ( 2 × 5 + 10 ) . from eqn (i) ∴ D = 0  D = b2 – 4ac
2
⇒ b2 – 4ac = 0 (1 Mark)
⇒ 60 = 20n ⇒ n = 3 (1 Mark)
⇒ (–6p – 2)2 – 4(p + 1) (8p + 1) = 0
5.
⇒ 36p2 + 4 + 24p – 4(8p2 + p + 8p + 1) = 0
Class Frequency
⇒ 36p2 + 4 + 24p – 32p2 – 36p – 4 = 0
25 – 30 12
⇒ 4p2 – 12p = 0
30 – 35 5
35 – 40 14 ⇒ p2 – 3p = 0 ⇒ p(p – 3) = 0
40 – 45 8 ⇒ p = 0 or p = 3.
45 – 50 9 Hence, the values of p are 0 and 3 (1 Mark)

Highest frequency is 14 at 35 – 40 OR
So, modal class = 35 – 40 (b) Let Arush scare be x marks.
Therefore f1 = 14, f0 = 5, f2 = 8 and h = 5 (1 Mark) according to question
f1 − f 0 ⇒ 7(x + 8) = x2 – 4 (1 Mark)
∵ Mode =
l+ ×h
2 f1 − f 0 − f 2 ⇒ 7x + 56 = x – 4
2

⇒ x2 – 7x – 60 = 0
14 − 5
∴ Mode = 35 + × 5 = 38.  (1 Mark)
2 × 14 − 5 − 8 ⇒ x2 – 12x + 5x – 60 = 0 (1 Mark)
6. ⇒ (x – 12) (x + 5) = 0

Class xi fi xi fi ⇒ x – 12 = 0 or x + 5 = 0
∴ x = 12 or x = –5
11 – 13 12 3 36
But marks cannot be negative
13 – 15 14 6 84
15 – 17 16 9 144 So, x = 12
17 – 19 18 13 234 Hence, Arush score 12 marks in test (1 Mark)
19 – 21 20 f 20f 8. Out of the Syllabus
21 – 23 22 5 110 9. Let OA be the height of the tree
23 – 25 24 4 96 OA = h m
Total 40 + f 704 + 20 f In DPOA,

CBSE 2022 (Outside DELHI Term-II) 191


A Now, D
In DEAB,
8 E 60°
h tan 45° = C
x 45° hm
8
⇒1= 8m
30° 45° x
P O Q
100 m ⇒x=8m ...(i) 45°
 (1 Mark) Now In DECD, A xm B
OA h −8 (1 Mark)
∴ tan 30° = tan 60° =
OP x 
1 h h −8
⇒ = ⇒ OP = 3h ...(i) ⇒ 3=
3 OP x
from eqn (i), we have
and In DQOA,
h −8
OA 3= (1 Mark)
∴ tan 45° = 8 
OQ
⇒8 3 =
h −8
h
=1 = OQ
= h ...(ii) (1 Mark)
⇒ h =8 ( 3 + 1) h =8 × 2.732
OQ 

on adding eqn (i) and (ii), we get ∴ h = 21.856 m (1 Mark)


OP + OQ = 3h+h 12. (a) Given, A circle is touching a side QR of DPQR at
point S.
⇒ PQ = ( 3 + 1) h

100 M
⇒ h
⇒ 100 = 2.732 h = = 36.6 m
2.732
Q
Hence, height of tree is 36.6 m (1 Mark)
n S
10. Given,=
Sn ( 3n + 5 )
2
P
1 R N
for n = 1, S1 = a = ( 3 ×1 + 5) = 4  (1 Mark)
2 PQ and PR are produced at M and N respectively.
2 1
for n= 2, S2= ( 3 × 2 + 5=
) 11
2 To prove PM= ( PQ + QR + PR)  (1 Mark)
2
 S2 = a2 + a1 Proof,
⇒ 11 = a2 + 4 ⇒ a2 = 7 PM = PN  ...(i)
d = a2 – a1 = 7 – 4 ⇒ d = 3 (1 Mark) (tangents from external point P to a circle are equal)
Now, a = 4, and d = 3  (1 Mark)
 an = a + (n – 1)d Similarly, QM = QS...(ii)
∴ a25 = a + 24 d = 4 + 24 × 3  (tangents from external point Q to a circle are equal)
a25 = 76 (1 Mark) and, RS = RN...(iii)
11. Let BC be the height of the building and DC be the  (tangents from external point R to a circle are equal)
height of cable tower. Now, 2 PM = PM + PM
x = PM + PN [from eqn (i)]
Let BD = h m & AB =  (1 Mark)
m = PQ + QM + PR + RN
CD = (h – 8) m = PQ + QS + PR + RS [from eqn (ii), (iii)] (1 Mark)

192
192 MATHS
= PQ + PR + QS + SR 65–75 24 122
⇒ 2 PM = PQ + QR + PR 75–85 18 140
1 Σfi = N = 140
⇒ PM= ( PQ + QR + PR ) Proved (1 Mark) Total
2
(1 Mark)
OR
N 140
(b) Given, OD = OE = OF = 4 cm ∴ = = 70
2 2
The length of tangents drawn from an external point to So, median class = 55 – 65
A
a circle are equal
Now,
BD = BE = 6 cm x x
l = 55, CF = 58, f = 40 and h = 10
CD = CF = 8 cm E F N 
AE = AF = x (let)  − Cf 
O
l + 2
∵ median = × h
Now, BC = BD + DC  f 
= 8 + 6 = 14 cm B C
6 cm D 8 cm  70 − 58 
AB = AE + BE = (x + 6) cm (1 Mark) ∴ medium =
55 +   × 10
 40 
AC = AF + FC = (x + 8) cm 12
Now, = 55 + = 58  (1 Mark)
4
ar(DABC) = ar(DOBC) + ar(DOAB) + ar(DOCA) (b)
 (2 Marks) Age group fi xi xi fi
1 1 1 15–25 8 20 160
84 = ( BC )( OD ) + ( AB ) ( OE ) + ( AC )( OF )
2 2 2 25–35 10 30 300
⇒ 168 = 14 × 4 + (x + 6)4 + (x + 8) (4) 35–45 15 40 600
⇒ 42 = 14 + x + 6 + x + 8 45–55 25 50 1250
⇒ 42 = 2x + 28 55–65 40 60 2400
⇒ 2x = 14 ⇒ x = 7 65–75 24 + x 70 1680 + 70x
Hence, AB = x + 6 = 7 + 6 = 13 cm 75–85 18 80 1440
AC = x + 8 = 7 + 8 = 15 cm (1 Mark) Total 140 + x 7820 + 70x
13. Out of the Syllabus Now,
14. (a) Σxi fi
∵ mean =  (1 Mark)
No. of people Σf i
Age group Cf
(fi) 7820 + 70 x
15–25 8 8 ∴ 58 = 
140 + x
25–35 10 18 ⇒ 55 × 140 + 58x = 7820 + 70x
35–45 15 33 ⇒ 12x = 8120 – 7820
45–55 25 58 ⇒ 12x = 300
55–65 40 98 ⇒ x = 25 (1 Mark)

CBSE 2022 (Outside DELHI Term-II) 193


Term-I
2022 CBSE Solved Paper

Time allowed : 90 minutes Maximum Marks : 40


GENERAL INSTRUCTIONS:
Read the following instructions very carefully and strictly follow them:
(i) This question paper contains 50 questions out of which 40 questions are to be attempted. All questions carry equal
marks.
(ii) The question paper consists of three Sections. Section A, B and C.
(iii) Section - A contains of 20 questions. Attempt any 16 questions from Q. No. 1 to 20.
(iv) Section - B also contains of 20 questions. Attempt any 16 questions from Q. No. 21 to 40.
(v) Section - C contains of two Case Studies containing 5 questions in each case. Attempt any 4 questions from Q. No. 41
to 45 and another 4 from Q. No. 46 to 50.
(vi) There is only one correct option for every Multiple Choice Question (MCQ). Marks will not be awarded for answering
more than one option.
(vii) There is no negative marking.

Section-A 1
6. If cot θ = , the value of sec2 θ + cosec2 θ is (1 Mark)
3 40 38 1
1. The exponent of 5 in the prime factorisation of 3750 is: (a) 1 (b) (c) (d) 5
(1 Mark) 9 9 3
7. The area of a quadrant of a circle where the circumference
(a) 3 (b) 4 (c) 5 (d) 6 of circle is 176 m, is (1 Mark)
2. The graph of a polynomial P(x) cuts the x-axis at 3 points (a) 2464 m (b) 1232 m (c) 616 m (d) 308 m2
2 2 2
and touches it at 2 other points. The number of zeroes of
P(x) is: (1 Mark) 8. For an event E , P ( E ) + P ( E ) =
x , then the value of x3 – 3
(a) 1 (b) 2 (c) 3 (d) 5 is (1 Mark)
(a) –2 (b) 2 (c) 1 (d) –1
3. The values of x and y satisfying the two equations
32x + 33y = 34, 33x + 32y = 31 respectively are:(1 Mark) 9. What is the greatest possible speed at which a girl can
walk 95 m and 171 m in an exact number of minutes?
(a) –1, 2 (b) –1, 4 (c) 1, –2 (d) –1, –4
(1 Mark)
( )
4. If A 3, 3 , B ( 0, 0 ) and C(3, k) are the three vertices of (a) 17 m/min (b) 19 m/min
an equilateral triangle ABC, then the value of k is (1 Mark) (c) 23 m/min (d) 13 m/min
(a) 2 (b) –3 (c) − 3 (d) − 2 10. In figure, the graph of a polynomial P(x) is shown. The
5. In figure, DE || BC, AD = 2 cm A number of zeroes of P(x) is (1 Mark)
and BD = 3 cm, then ar(∆ABC): y
m
2c

ar(∆ADE) is equal to P(x)


[OS] (1 Mark) A B
 D E x' x
C
(a) 4 : 25 (b) 2 : 3
m

y'
3c

(c) 9 : 4 (d) 25 : 4
B C (a) 1 (b) 2 (c) 3 (d) 4
11. Two lines are given to be parallel. The equation of one of
the lines is 3x – 2y = 5. The equation of the second line can Section-B
be: (1 Mark)
21. The greatest number which when divides 1251, 9377 and
(a) 9x + 8y = 7 (b) –12x – 8y = 7 15628 leaves remainder 1, 2 and 3 respectively is
(c) –12x + 8y = 7 (d) 12x + 8y = 7  [OS] (1 Mark)
12. Three vertices of a parallelogram ABCD are A(1, 4), (a) 575 (b) 450 (c) 750 (d) 625
B(–2, 3) and C(5, 8). The ordinate of the fourth vertex D is 22. Which of the following cannot be the probability of an
(1 Mark) event? (1 Mark)
(a) 8 (b) 9 (c) 7 (d) 6 16 17
(a) 0.01 (b) 3% (c) (d)
1 17 16
13. In ∆ABC and ∆DEF, ∠F = ∠C, ∠B = ∠E and AB = DE. 23. The diameter of a car wheel is 42 cm. The number of
2 complete revolutions it will make in moving 132 km is
Then, the two triangles are (1 Mark) (1 Mark)
(a) Congruent, but not similar. (a) 104 (b) 105 (c) 106 (d) 103
(b) Similar, but not congruent. 24. If θ is an acute angle and tanθ + cotθ = 2, then the value of
(c) Neither congruent nor similar. sin3θ + cos3θ is (1 Mark)
1 2
(d) Congruent as well as similar. (a) 1 (b) (c) (d) 2
2 2
7 25. The ratio in which the line 3x + y – 9 = 0 divides the line
14. In ∆ABC right angled at B,sin A = , then the value of
cosC is: 25 (1 Mark) segment joining the points (1, 3) and (2, 7) is (1 Mark)
(a) 3 : 2 (b) 2 : 3 (c) 3 : 4 (d) 4 : 3
7 24 7 24
(a) (b) (c) (d) 26. If x – 1 is a factor of the polynomial p(x) = x3 + ax2 +2b
25 25 24 7
and a + b = 4, then [OS] (1 Mark)
15. The minute hand of a clock is 84 cm long. The distance
covered by the tip of minute hand from 10:10 am to (a) a = 5, b = –1 (b) a = 9, b = –5
10:25 am is: (1 Mark) (c) a = 7, b = –3 (d) a = 3, b = 1
(a) 44 cm (b) 88 cm (c) 132 cm (d) 176 cm 27. If a and b are two coprime numbers, then a3 and b3 are
(1 Mark)
16. The probability that the drawn card from a pack of
(a) Coprime (b) Not coprime
52 cards is neither an ace nor a spade is (1 Mark)
(c) Even (d) Odd
9 35 10 19
(a) (b) (c) (d) 28. The area of a square that can be inscribed in a circle of area
13 52 13 26
1408
17. Three alarm clocks ring their alarms at regular intervals of cm 2 is (1 Mark)
7
20 min, 25 min and 30 min respectively. If they first beep (a) 321 cm2 (b) 642 cm2 (c) 128 cm2 (d) 256 cm2
together at 12 noon, at what time will they beep again for
the first time? (1 Mark) 29. If A(4, –2), B(7, –2) and C(7, 9) are the vertices of a ∆ABC,
then ∆ABC is (1 Mark)
(a) 4:00 pm (b) 4:30 pm (c) 5:00 pm (d) 5:30 pm
(a) Equilateral triangle.
18. A quadratic polynomial, the product and sum of whose (b) Isosceles triangle.
zeroes are 5 and 8 respectively is (1 Mark)
(c) Right angled triangle.
(a) k[x2 – 8x + 5] (b) k[x2 + 8x + 5] (d) Isosceles right angled triangle.
(c) k[x2 – 5x + 8] (d) k[x2 + 5x + 8]
30. If α, β are the zeros of the quadratic polynomial
19. Points A(–1, y) and B(5, 7) lie on a circle with centre p(x) = x2 – (k + 6)x + 2(2k – 1), then the value of k, if
O(2, –3y). The values of y are (1 Mark) 1
α + β= αβ , is (1 Mark)
(a) 1, –7 (b) –1, 7 (c) 2, 7 (d) –2, –7 2
(a) –7 (b) 7 (c) –3 (d) 3
1 + tanθ
20. Given that sec θ = 2 , the value of is (1 Mark) 31. If n is a natural number, then 2(5n + 6n) always ends with
sinθ
(1 Mark)
(a) 2 2 (b) 2 (c) 3 2 (d) 2 (a) 1 (b) 4 (c) 3 (d) 2

CBSE 2022 (Term-I) 195


32. The line segment joining the points P(–3, 2) and Q(5, 7) is
divided by the y-axis in the ratio (1 Mark) Section-C
(a) 3 : 1 (b) 3 : 4 (c) 3 : 2 (d) 3 : 5 Case Study-I:
33. If a cot θ + b cosec θ = p and b cot θ + a cosec θ = q, then A book store shopkeeper gives books on rent for reading. He
p2 – q2 = (1 Mark) has variety of books in his store related to fiction, stories and
(a) a – b
2 2
(b) b – a
2 2
quizzes etc. He takes a fixed charge for the first two days and
(c) a + b
2 2
(d) b – a an additional charge for subsequent day. Amruta paid ` 22 for a
book and kept for 6 days; while Radhika paid ` 16 for keeping
34. If the perimeter of a circle is half to that of a square, then
the book for 4 days.
the ratio of the area of the circle to the area of the square is
 [OS] (1 Mark)
(a) 22 : 7 (b) 11 : 7 (c) 7 : 11 (d) 7 : 22
35. A dice is rolled twice. The probability that 5 will not come
up either time is (1 Mark)
11 1 13 25
(a) (b) (c) (d)
36 3 36 36
36. The LCM of two numbers is 2400. Which of the following
CANNOT be their HCF? (1 Mark)
(a) 300 (b) 400 (c) 500 (d) 600
37. In fig., PA, QB and RC are each perpendicular to AC. If
x = 8 cm and z = 6 cm, then y is equal to (1 Mark)
P Assume that the fixed charge be ` x and additional charge
(per day) be ` y.
R
x Q Based on the above information, answer any four of the
z following questions:
y
41. The situation of amount paid by Radhika, is algebraically
A C
B represented by (1 Mark)
(a) x – 4y = 16 (b) x + 4y = 16
56 7 25 24
(a) cm (b) cm (c) cm (d) cm (c) x – 2y = 16 (d) x + 2y = 16
7 56 7 7
42. The situation of amount paid by Amruta, is algebraically
38. In a ∆ABC , ∠A= x°, ∠B= (3 x − 2)°, ∠C = y° . Also represented by (1 Mark)
∠C – ∠B = 9°. The sum of the greatest and the smallest
(a) x – 2y = 11 (b) x – 2y = 22
angles of this triangle is (1 Mark)
(c) x + 4y = 22 (d) x – 4y = 11
(a) 107° (b) 135° (c) 155° (d) 145°
43. What are the fixed charges for a book ? (1 Mark)
39. If secθ + tanθ = p, then tanθ is (1 Mark)
(a) ` 9 (b) ` 10 (c) ` 13 (d) ` 15
p2 + 1 p2 −1
(a)
2p
(b)
2p 44. What are the additional charges for each subsequent day
for a book ? (1 Mark)
p2 −1 p2 + 1 (a) ` 6 (b) ` 5 (c) ` 4 (d) ` 3
(c) (d)
p2 + 1 p2 −1
45. What is the total amount paid by both, if both of them have
40. The base BC of an equilateral ∆ABC lies on the y-axis. The kept the book for 2 more days ? (1 Mark)
co-ordinates of C are (0, –3). If the origin is the mid-point (a) ` 35 (b) ` 52 (c) ` 50 (d) ` 58
of the base BC, what are the co-ordinates of A and B?
(1 Mark) Case Study-II:

(a) A ( 3, 0 ) , B ( 0,3) ( )
(b) A ±3 3, 0 , B ( 3, 0 )
A farmer has a field in the shape of trapezium, whose map with
scale 1 cm = 20 m, is given below :
(c) A ( ±3 3, 0 ) , B ( 0,3) (d) A(− )
3, 0 , B ( 3, 0 ) The field is divided into four parts by joining the opposite vertices.

196 MATHS
5 cm 48. If the ratio of the perimeter of ∆AOB to the perimeter of
A B
∆COD would have been 1:4, then [OS] (1 Mark)
O (a) AB = 2CD (b) AB = 4CD
(c) CD = 2AB (d) CD = 4AB
D C
10 cm AO AD OD
49. If in ∆AOD and BOC , = = , then (1 Mark)
Based on the above information, answer any four of the BC BO OC
following questions: (a) ∆AOD ~ ∆BOC (b) ∆AOD ~ ∆BCO
46. The two triangular regions AOB and COD are (1 Mark) (c) ∆ADO ~ ∆BCO (d) ∆ODA ~ ∆OBC
(a) Similar by AA criterion 50. If the ratio of areas of two similar triangles AOB and COD
(b) Similar by SAS criterion is 1 : 4, then which of the following statements is true?
(c) Similar by RHS criterion  [OS] (1 Mark)
(d) Not similar (a) The ratio of their perimeters is 3 : 4.
47. The ratio of the area of the ∆AOB to the area of ∆COD, is (b) The corresponding altitudes have a ratio 1 : 2.
 [OS] (1 Mark) (c) The medians have a ratio 1 : 4.
(a) 4 : 1 (b) 1 : 4 (c) 1 : 2 (d) 2 : 1 (d) The angle bisectors have a ratio 1 : 16.

EXPLANATIONS

1. (b) The prime factorisation of 3750 is; ⇒y=2


3750 = 54 × 2 × 3 ½ Mark)
(½ Putting y = 2 in equation (iii), we get
\ The exponent of 5 in the prime factorisation of x + 2 = 1 ⇒ x = 1 – 2 ⇒ x = –1 ½ Mark)

3750 is 4. ½ Mark)
(½ \ The values of x & y are –1 & 2 respectively.
2. (d) P(x) cuts x-axis at 3-points & touches x-axis at 2-points.
we know, number of zeroes of a polynomial 4. (c) The given points are A(3, 3); B ( 0, 0 ) is C(3, k) are
= no. of times graph touches the x-axis three vertices of an equilateral triangle ABC.
∴ No. of zeroes of P(x) = 5. (1 Mark) We know that the all sides of an equilateral triangle
are equal. (½½ Mark)
3. (a) Given, pair of linear equation
\ AB = CB
32x + 33y = 34 …(i)
By distance formula, we have
33x + 32y = 31 …(ii)
adding equation (i) from (ii), we get ⇒ (3 − 0) 2 + ( 3 − 0) 2 = (3 − 0) 2 + (k − 0) 2
32x + 33y = 34 ⇒ (3 – 0)2 + ( 3 – 0)2 = (3 – 0)2 + (k – 0)2 [Squaring
33x + 32y = 31  both sides]
65x + 65y = 65 ⇒ (3)2 + ( 3 )2 = (3)2 + k2
⇒ x + y = 1 ½ Mark)
…(iii) (½ ⇒ 9 + 3 = 9 + k2 ⇒ k2 = 3 ⇒ k = ± 3
Substracting equation (i) and (ii), we get But, k ≠ 3 , because A & C will be the same
32x + 33y = 34 ∴ k =− 3  ½ Mark)

33x
– +–32y = –31 5. (d) Out of the Syllabus
–x + y = 3 ...(iv) 6. (d) Given,
Now, adding equation (iii) and (iv), we get 1  1 
⸪ cot=
θ ⇒ cot=
θ cot60°  ∵ cot 60° = 
x+y=1 3  3
–x+y=3 ⇒ θ = 60º 
2y = 4 \ sec2 60º + cosec2 60º ½ Mark)

CBSE 2022 (Term-I) 197


2 Now, from the given options, we can see, option (c):
 2  4
(2) 2 +   =
= 4+ –12x + 8y = 7 ⇒ – 4 (3x – 2y) – 7 = 0
 3 3
where, k = – 4 and c = – 7 ½ Mark)

12 + 4 16 1 ½ Mark)
(½ It satisfies both conditions for being parallel.
= = = 5
3 3 3
12. (b) The three vertices of the parallelogram are A(1, 4);
7. (c) Given, circumference of circle = 176 m B(–2, 3); C(5, 8).
Let the radius of the circle be r Let the point D be D(x, y)
\ circumference of circle = 2pr Now, we know, the diagonals of a parallelogram
22 bisect each other,
⇒ 176 =2 × × r ⇒ r = 28 m ½ Mark)

7 \ Midpoint of BD = Midpoint of AC ½ Mark)

1 2
Now, the area of quadrant of the circle= ππ D C
4
1 22
= × × 28 × 28 m 2 = 616 m2 ½ Mark)

4 7
M
8. (a) Given, P(E) + P(E) = x
Let P(E) = u
then, P(E) = complement of P(E) A B
\ P(E) = 1 – P(E) ⇒ 1 – u ½ Mark)

 x − 2 y + 3  5 + 1 8 + 4 
\ P(E) + P(E) = x ⇒ , = , 
 2 2   2 2 
⇒u+1–u=x⇒x=1
 x − 2 y + 3   6 12  ⇒  x − 2 , y + 3  =
½ Mark)
\ The value of x3 – 3 = (1)3 – 3 = 1 – 3 = – 2 (½ ⇒ , = ,    ( 3, 6)
 2 2  2 2 2 2 
9. (b) Here, the given distance, comparing both sides, we get;
95 m = 5 × 19 m, 171 m = 3 × 19 m
2
x−2
=3⇒x–2=6⇒x=8
\ The greatest possible speed = HCF of (95, 171) = 19 2
\ The greatest possible speed = 19 m/min (1 Mark) y+3
= 6 ⇒ y + 3 = 12 ⇒ y = 9
2
10. (c)
y ½ Mark)
Now, the ordinate of the point D is y = 9. (½
13. (b)
A D
P(x)
A B
x′ x
C

y′
B C E F
Number of zeroes of a polynomial = Number of times
intersects the graph the x-axis. Given, in DABC and DDEF;
\ The no. of zeroes of the given polynomial = 3 ∠B = ∠E [given]
 (1 Mark) ∠C = ∠F [given]
11. (c) Given equation is: 3x – 2y = 5 ⸫ DABC ∼ DDEF [by AA criteria]
⇒ 3x – 2y – 5 = 0 But, since, AB ≠ DE
For another live to be parallel, the line must be: DABC ≠ DDEF
(i) scalar multiple coefficient of x & y of the given Hence, DABC & DDEF are similar but not congruent.
linear equation, i.e., for the given linear equation;  (1 Mark)
14. (a) Given, for DABC, right angled at B,
k(3x) – k(2y) + c = 0 ½ Mark)

(ii) a different constant term than the given linear 7 Perpendicular ( P ) 7
sinA = ⇒ =
equation i.e., 3x – 2y + c = 0 25 Hypotenuse ( H ) 25

198 MATHS
 BC  By distance formula
∵ sin A = AC   ½ Mark)

⇒ (−1 − 2) 2 + ( y − ( −3 y ) )=
2
(5 − 2) 2 + (7 − ( −3 y ) ) 2
  A
So, let, P = 7K and H = 25K ⇒ (−3) 2 + ( y + 3 y )=
2
(3) 2 + (7 + 3 y ) 2
BC = P = 7K ⇒ 9 + (4y) = 9 + (7 + 3y) [squaring on both sides]
2 2

AC = H = 25K ⇒ 16y2 = 49 + 9y2 + 42y


Base BC ⇒ 16y2 – 9y2 – 42y – 49 = 0
=cos C =
Hypotenuse AC B C ⇒ 7y2 – 42y – 49 = 0 ½ Mark)

7K 7 ⇒ y –6y – 7 = 0 ⇒ y – 7y + y – 7 = 0
2 2

= = ½ Mark)
(½ ⇒ y(y – 7) + 1 (y – 7) = 0 ⇒ (y + 1) (y – 7) = 0
25 K 25 
⇒ y + 1 = 0 or y – 7 = 0 ⇒ y = – 1 or y = 7
15. (c) The minute hand travels from 10:10 to 10:25
\ The values of y are –1, 7 ½ Mark)

Let the angle it covered be θ.
20. (a) Given, secθ = 2
15
∴θ
= × 360º = 90º ½ Mark)

60 ⇒ secθ = sec 45º [⸪ sec 45° = 2 ]
\ It covered a quarter of a circle Also, the minute ⇒ θ = 45º (½½ Mark)
hand is 84 cm long. 1 + tanθ 1 + tan45°
∴ =°
Now, the distance covered by the minute hand sinθ sin45
1 1 22 1+1 2
= × 2 × πr = × 2 × × 84 = 132 cm. ½ Mark)
(½ = = =2 2 
4 4 7 ½ Mark)

1/ 2 1/ 2
16. (a) Total no. of cards n(S) = 52.
21. (d) Out of the Syllabus
No. of cards that is neither an ace nor a spade n(E)= 36
22. (d) Probability of an event <1
n( E ) 36 9
\ P (neither a spade nor ace)
= = =  17
n( S ) 52 13 but, >1
 (1 Mark) 16
Hence, cannot be probability of an event. (1 Mark)
17. (c) The time after which they again ring together = LCM
23. (b) Diameter of car wheel = 42cm
of 20, 25, 30
42
=
Now, radius of the wheel, r = cm 21cm
5 20, 25, 30 2
2 4, 5, 6 ⸪ Circumference of the whee = 2pr
2, 5, 3 22
=2 × × 21 cm = 132 × 105 km ½ Mark)

7
\ LCM (20, 25, 30) = 52 × 22 ×3 = 300
Now, distance to be covered in one revolutions is
\ They ring again together after = 300 minutes 132 × 105 km
300 Distance to be covered
= = hr 5hr \ No. of revolutions =
60 circumference of wheel
If they first ring together at 12 noon, they again ring 132 × 105
= = 105 ½ Mark)

together after 5hrs at 5:00 pm. (1 Mark) 132
18. (a) Given, sum of zeroes = 8 and product of zeroes = 5 24. (c) Given, tanθ + cotθ = 2
We know, a quadratic polynomial is, 1  1 
⇒ tan θ += 2 ∵ cot θ +
K [x2 – (sum of zeroes) x + (product of zeroes] tan θ  tan θ 
\ The quadratic poly is: K [x2 – 8x + 5] (1 Mark) ⇒ tan2θ + 1 = 2tanθ
19. (b) The given points on circle are, ⇒ tan2θ – 2tanθ + 1= 0
A(–1, y), B(5, 7) and centre is O(2, –3y) ⇒ (tanθ – 1)2 = 0 ⇒ tanθ – 1 = 0
Now, A and B are equidistant from O [radii] ⇒ tanθ = 1 ⇒ tanθ = tan 45º ⇒ θ = 45º  ½ Mark)

\ AO = BO Now, sin3 θ + cos3 θ = sin3 45º + cos3 45º

CBSE 2022 (Term-I) 199


3 3 29. (c) Given vertices of triangle are A(4, –2), B(7, –2), C(7, 9)
 1   1 
=  +
 2   2  Now, AB
= (7 − 4) 2 + (−2 + 2)=
2
32 + 02
3
 1  1 1 2 = (3) 2 3
=
= 2 = 2⋅ = = ½ Mark)

 2  2 2 2 2 
25. (c) Let the ratio in which the live 3x + y – 9 =0 divides the BC = (7 − 7) 2 + (9 =
+ 2) 2 (11) 2 11
=
live segment joining the points (1,3) & (2,7) is k:1
CA= (7 − 4) 2 + (9 + 2) 2
\ The point of division is,
 2k + 1 7 k + 3  = (3) 2 + (11) 2 = 9 + 121 = 130  ½ Mark)

c ,
 k + 1 k + 1  We can see, CA2 = 130 and
Now, c also lies in 3x + y – 9 =0 AB2 + BC2 = (3)2 + (11)2 = 9 + 121 = 130
 2k + 1   7 k + 3  \ CA2 = AB2 + BC2
∴3 + −9 =0 ½ Mark)

 k +1   k +1   \ By converse of Pythagoras Theorem,
6k + 3 7 k + 3 DABC is a right angled triangle. ½ Mark)

⇒ + −9 = 0
k +1 k +1
30. (b) Given, p(x) = x2 – (k + 6) x + 2 (2k – 1).
6k + 3 + 7 k + 3 − 9k − 9
⇒ 0
= let a and β are zeroes of p(x).
k +1
3 \ a + b = k + 6 [sum of roots of quadratic polynomial]
⇒ 4k − 3 = 0 ⇒ k =
4 ab = 2(2k – 1) [product of roots of quadratic polynomial]
½ Mark)
\ The ratio at which the line divides is 3 : 4. (½ ½ Mark)

Also given,
26. (b) Out of the Syllabus 1 1
α + β= αβ ⇒ ( k + 6 ) = ⋅ 2 ( 2k − 1) 
27. (a) Given, a and b are coprimes. 2 2
Then, a3 and b3 are also coprimes ⇒ k + 6 = 2k – 1
Examples, 4 and 5 are coprimes ⇒ 2k – k = 6 + 1 ⇒ k = 7 ½ Mark)

43 = 64 and 53 = 125 31. (d) Given, n is a natural number and the resultant no of
64 + 125 are also coprimes. (1 Mark) 2(5n + 6n)
Let n = 1, then 2(5 + 6) = 2(11) = 22
1408
28. (c) Given, area of circle = cm 2 . Let the radius of Let n = 2, then 2(52 + 62) = 2 (25 + 36) = 2(61) = 122
circle be = r. 7
\ In both cases, the number ends with 2.
Then, area of circle = pr2 \ The correct answer is 2. (1 Mark)
1408 22 2
⇒ = × r ⇒ r2 = 64 32. (d) Let the ratio in which the y-axis divides, the live
7 7 joining the points P(–3, 2) and Q (5, 7) is k : 1.
⇒ r = 8 [r ≠ –8, as r represent length] \ The point of intersection is
Diameter of circle 2r = 16 cm ½ Mark)

 5k − 3 7 k + 2 
Now, the largest square that would fit inside the circle C , 
 k +1 k +1 
is such that, diagonal of square = diameter of circle.
But, since, the line is intersected by y-axis, its
Let the side of square be a.
x-coordinate will be 0.
\ Diagonal of square
= a 2 + a 2 = 2a i.e., x-coordinate of C = 0
∴ 2a = diameter of circle 5k − 3 3
⇒ = 0 ⇒ 5k – 3 = 0 ⇒ k =
k +1 5
16
⇒ 2a = 16 ⇒ a = cm  ⸫ The ratio of division is 3 : 5. (1 Mark)
2
\ Area of the square = a2 33. (b) Given, a cot q + bcosec q = p and b cot q + a cosec q = q
2 Now, p2 – q2
 16  256 2
= =  cm = 128 cm2 ½ Mark)
(½ = (a cot q + b cosec q)2 – (b cot q + a cosec q)2
 2 2

200 MATHS
= (a2cot2 q + b2cosec2 q + 2ab cot q cosec q) – (b2 cot2 q + y y CB AB y y CB + AB
a2 cosec2 q + 2ab cot qcosec q) ½ Mark)
(½ + = + ⇒ + =
x z AC AC x z AC
= a2cot2 q + b2cosec2 q + 2ab cot qcosec q – b2 cot2 q –
y y AC 1 1
a2cosec2   q – 2ab cot qcosec q ⇒ + = ⇒ y  +  =
1
= b2(cosec2  q – cot2 q) + a2 (cot2 q – cosec2 q) x z AC x z
= b2 (1) + a2 (–1)  [ q cot2 q + 1 = cosec2 q] 1 1 
⇒ y +  =1 (Given, x = 8 cm; z = 6 cm)
= b2 – a2. ½ Mark)
(½ 8 6 
34. (d) Out of the Syllabus 6 + 8  14 
⇒ y =1⇒ y   = 1
35. (d) A dice is rolled twice.  48   48 
∴ The sample space is; 48 24
⇒ y= = ½ Mark)

(1,1) ( 2,1) ( 3,1) ( 4,1) ( 5,1) ( 6,1) 14 7 
(1, 2 ) ( 2, 2 ) ( 3, 2 ) ( 4, 2 ) ( 5, 2 ) ( 6, 2 ) 38. (a)
(1,3) ( 2,3) ( 3,3) ( 4,3) ( 5,3) ( 6,3) A
(1, 4 ) ( 2, 4 ) ( 3, 4 ) ( 4, 4 ) ( 5, 4 ) ( 6, 4 )
(1,5) ( 2,5) ( 3,5) ( 4,5) ( 5,5) ( 6,5) x°
(1, 6 ) ( 2, 6 ) ( 3, 6 ) ( 4, 6 ) ( 5, 6 ) ( 6, 6 )
Total no. of out comes n(S) = 36.
No. of times, 5 will not come up n(E) = 25.
(3x–2)° y°
n( E ) 25
=
\ P(5 will not come up either time) P(E) =
n( S ) 36 B C
 (1 Mark) Given, ∠A = x°, ∠B = (3x – 2)° and ∠C = y°
36. (c) Since HCF is the factor of LCM Also, ∠C – ∠B = 9°
⸫ 500 is not the factor of 2400 ⇒ y° – (3x – 2)° = 9° ⇒ y° = 9° + (3x – 2)°
⸫ 500 cannot be HCF (1 Mark) = 9° + (3x)° – 2° = 7° + (3x)° ½ Mark)

37. (d) We know, sum of angles of a triangle = 180°
P ⸫ ∠A + ∠B + ∠C = 180°
⇒ x° + (3x – 2)° + (7 + 3x)° = 180°
R ⇒ x° + 3x° – 2° + 7° + 3x° = 180° ⇒ 7x° + 5° = 180°
x Q
180° − 5° 175°
z =⇒ x° =
= 25°
y 7 7
A C ∠B = (3(25) –2)° = (75 – 2)° = 73°
B
In DCAP and DCBQ, ∠CAP = ∠CBQ = 90o ∠C = (7 + 3x)° = (7 + 3(25°))° = (7 + 75°)° = 82°
∠PCA = ∠QCB = [common] \ Smallest angle + gratest angle
So, by AA criteria, ∠CAP ~ ∠CBQ ∠A + ∠C = 25° + 82° = 107°  ½ Mark)

BQ CB y CB 39. (b) Given, secq + tanq = p
Hence, = ⇒ =  ... (i)
AP CA x CA ⇒ 1 + tan 2 θ = p – tan q [Q 1 + tan2q = sec2q]
Again, in DACR and DABQ.
⇒ 1 + tan2 q = p2 + tan2q – 2 ptanq 

∠ACR =
∠ABC = 90
⇒ 1 = p2 – 2ptanq ⇒ 2 ptanq = p2 – 1
∠BAQ [common]
∠CAR =
p2 − 1
So, by AA criterion, DACR ~ DABQ ½ Mark)
(½ ⇒ tanθ =  (1 Mark)
2p
BQ AB 4 AB 40. (c) Given origin 0 is the midpoint of the have BC.
Hence, = ⇒ =  ... (ii)
CR AC z AC Therefore, coordinates of point B are (0, 3)
Now, (i) + (ii) [since, coordinate of C is (0, –3)B lies on y-axis]

CBSE 2022 (Term-I) 201


y Solving, (i) and (ii), we get
B
⇒ 2y = 6 ⇒ y = 3
The additional charges are ` 3. (1 Mark)
O x
A 45. (c) If Radhika Kept the book for two more days, she
would have paid = 10 + 4(3)
C = 10 + 12 = ` 22.
If Amrita kept the book for two more days, she would
Let the coordinates of A be (x, 0) using distance formula,
have paid = 22 + 2(3)
AB = BC [Q ABC is an equilater al triangle]
= 22 + 6 = ` 28
⇒ (0 − x) 2 + (3 − 0) 2 =
6 ⇒ x2 + 9 = ½ Mark)
6  (½ The total amount paid by both = ` (22 + 28) = ` 50
⇒ x +9 =2
36 [squaring on both sides]  (1 Mark)
⇒ x2 =
27 ⇒ x =±3 3 < x < 1 < x Care study II:
A 5 cm
\ The point A is A(±3 3, 0) B
∴ A(±3 3, 0); B(0,3)  ½ Mark)

O
Case study I:
D C
41. (d) Let the fixed charge for first two days be x and the 10 cm
additional charges thereafter be y. Given, 1 cm = 20 m
Radhika kept the book for 4 days and paid ` 16 46. (a) In DAOB and DCOD
The algebraic equation is: x + 2y = 16 (1 Mark) ∠BAO = ∠DCO [alternate interior angles]
42. (c) Let the fixed charge for first two days be x and the ∠AOB = ∠DOC [vertically opposite angle]
additional charges thereafter be y.
\ DAOB ~ DCOD [by AA criterion] (1 Mark)
Amrulta kept the book for 6 days and paid ` 22
47. (b) Out of the Syllabus
\ The algebraic equation is: x + 4y = 22 (1 Mark)
48. (d) Out of the Syllabus
43. (b) The two equations are
AO AD OD
x + 2y = 16 ...(i) 49. (b) Given, = =
BC BO OC
x + 4y = 22 ...(ii) We know, if two D’s are similar then, their
Now, 2 × (i) – (ii), gives corresponding sides are in same proportion
x = 10 Then, AO is corresponding to BC
\ The fixed charger is ` 10. (1 Mark) AD is corresponding to BO
44. (d) The two equations are OD is corresponding to OC
x + 2y = 16  ...(i) \ DAOD ~ DBCO (1 Mark)
x + 4y = 22 ...(ii) 50. (b) Out of the Syllabus

202 MATHS
DELHI
2020 CBSE Solved Paper

Time allowed : 3 hours Maximum Marks : 80


GENERAL INSTRUCTIONS:
Read the following instructions carefully and strictly follow them:
(i) This question paper comprises four sections – A, B, C and D. This question paper carries 40 questions. All questions
are compulsory.
(ii) Section A : Q. No. 1 to 20 comprises of 20 questions of one mark each.
(iii) Section B : Q. No. 21 to 26 comprises of 6 questions of two marks each.
(iv) Section C : Q. No. 27 to 34 comprises of 8 questions of three marks each.
(v) Section D : Q. No. 35 to 40 comprises of 6 questions of four marks each.
(vi) Three is no overall choice in the question paper. However, an internal choice has been provided in 2 questions of
one mark each, 2 questions of two marks each, 3 questions of three marks each and 3 questions of four marks each. You
have to attempt only one of the choices in such questions.
(vii) In addition to this, separate instructions are given with each section and question, wherever necessary.
(viii) Use of calculators is not permitted.

6. The distance between the points (a cos θ + b sin θ, 0) and


Section-A (0, a sin θ – b cos θ), is (1 Mark)
1. The HCF and the LCM of 12, 21,15 respectively are (1 Mark) (a) a2 + b2 (b) a2 – b2
(a) 3, 140 (b) 12, 420 a 2 − b2
(c) a 2 + b2 (d)
(c) 3, 420 (d) 420, 3
7. The total number of factors of a prime number is (1 Mark)
2. The value of x for which 2x, (x + 10) and (3x + 2) are the
(a) 1 (b) 0
three consecutive terms of an AP, is (1 Mark)
(c) 2 (d) 3
(a) 6 (b) –6
(c) 18 (d) –18 8. If the point P(k, 0) divides the line segment joining the
points A(2, –2) and B(–7, 4) in the ratio 1 : 2, then the
3. The value of k for which the system of equations x + y – 4 = 0
value of k is: (1 Mark)
and 2x + ky = 3, has no solution, is (1 Mark)
(a) 1 (b) 2
(a) –2 (b) ≠ 2
(c) 3 (d) 2 (c) –2 (d) –1
4. The first term of an AP is p and the common difference is 9. The value of p, for which the points A(3, 1), B(5, p) and
q, then its 10th term is  (1 Mark) C(7, –5) are collinear, is [OS] (1 Mark)
(a) q + 9p (b) p – 9q (a) –2 (b) 2
(c) p + 9q (d) 2p + 9q (c) –1 (d) 1
5. The quadratic polynomial, the sum of whose zeroes is –5 10. If one of the zeroe of the quadratic polynomial x2 + 3x + k
and their product is 6, is (1 Mark) is 2, then the value of k is (1 Mark)
(a) x2 + 5x + 6 (b) x2 – 5x + 6 (a) 10 (b) –10
(c) x2 – 5x – 6 (d) –x2 + 5x + 6 (c) –7 (d) –2
11. ABC is an equilateral triangle of side 2a, then length of one OR
of its altitude is ________. (1 Mark) In Fig., two tangents TP and TQ are drawn to a circle with
12. In Fig., ∆ABC is circumscribing a circle, the length of BC centre O from an external point T. Prove that ∠PTQ = 2
is ____________ cm. (1 Mark) ∠OPQ.
A P

4 cm T O

11 c
m
P R Q

3 cm 22. The rod AC of a TV disc antenna is fixed at right angles to


the wall AB and a rod CD is supporting the disc as shown
B C in Fig. If AC = 1.5 m long and CD = 3 m, find (i) tan θ
Q
(ii) sec θ + cosec θ. (2 Marks)
 2 1 
13. The value of  sin θ + 2 
= ________. (1 Mark)
 1 + tan θ  C
A
OR
The value of (1 + tan θ) (1 – sin θ)(1 + sin θ) = _______.
2

2 2 θ
sin 35°   cos 43° 
14.   +  − 2 cos 60° =_______. (1 Mark)
 cos 55°   sin 47°  D
15. ABC and BDE are two equilateral triangles such that D is B
the mid-point of BC. Ratio of the areas of triangles ABC
and BDE is _________. [OS] (1 Mark) Wall
16. A die is thrown once. What is the probability of getting a 23. If a number x is chosen at random from the numbers
number less than 3? (1 Mark) –3, –2, –1, 0, 1, 2, 3. What is probability that x2 ≤ 4?
OR (2 Marks)
If the probability of winning a game is 0.07, what is the 24. Find the mean of the following distribution: (2 Marks)
probability of losing it?
Class 3–5 5–7 7–9 9–11 11–13
17. If the mean of the first n natural number is 15, then find n.
(1 Mark) Frequency 5 10 10 7 8
18. Two cones have their heights in the ratio 1:3 and radii in OR
the ratio 3:1. What is the ratio of their volumes? (1 Mark) Find the mode of the following data:
19. The ratio of the length of a vertical rod and the length of Class 0-20 20-40 40-60 60-80 80-100 100-120 120-140
its shadow is 1: 3 . Find the angle of elevation of the sun
Frequency 6 8 10 12 6 5 3
at that moment? (1 Mark)
25. Find the sum of first 20 terms of the following AP: 1, 4, 7,
20. A die is thrown once. What is the probability of getting an
10, ____________. (2 Marks)
even prime number? (1 Mark)
26. The perimeter of a sector of a circle of radius 5.2 cm is
Section-B 16.4 cm. Find the area of the sector. (2 Marks)

BE BC
=
21. In Fig., DE || AC and DC || AP. Prove that
. Section-C
EC CP
(2 Marks) 27. A cone of base radius 4 cm is divided into two parts by
A drawing a plane through the mid-points of its height and
D
parallel to its base. Compare the volume of the two parts.
 [OS] (3 Marks)
28. In a triangle, if square of one side is equal to the sum of
the squares of the other two sides, then prove that the angle
B E C P opposite to the first side is a right angle. (3 Marks)

204 MATHS
29. Find the area of triangle PQR formed by the points Change the distribution to ‘a more than’ type distribution
P(–5, 7), Q( –4, –5) and R(4, 5). [OS] (3 Marks) and draw its ogive. [OS] (4 Marks)
OR OR
If the point C(–1, 2) divides internally the line segment The median of the following data is 525. Find the values of
joining A(2, 5) and B(x, y) in the ratio 3:4, find the x and y, if total frequency is 100:
coordinates of B. (3 Marks) Class

900-1000
100-200

200-300

300-400

400-500

500-600

600-700

700-800

800-900
0-100
30. Find a quadratic polynomial whose zeroes are reciprocals
of the zeroes of the polynomial f(x) = ax2 + bx + c, a ≠ 0,
c ≠ 0. (3 Marks) Frequency 2 5 x 12 17 20 y 9 7 4
OR (4 Marks)
Divide the polynomial f(x) = 3x2 – x3 – 3x + 5 by the
36. A vertical tower stands on a horizontal plane and is
polynomial g(x) = x – 1 – x2 and verify the division
surmounted by a vertical flag-staff of height 6 m. At a
algorithm. [OS] point on the plane, the angle of elevation of the bottom
31. Determine graphically the coordinates of the vertices of a and top of the flag-staff are 30° and 45° respectively. Find
triangle, the equations of whose sides are given by 2y – x = 8, the height of the tower. (Take 3 = 1.73 ) (4 Marks)
5y – x = 14 and y – 2x = l. (3 Marks)
37. Show that the square of any positive integer cannot be of
OR the form (5q + 2) or (5q + 3) for any integer q.
If 4 is a zero of the cubic polynomial x3 – 3x2 – 10x + 24,  [OS] (4 Marks)
find its other two zeroes. [OS] OR
32. A train covers a distance of 480 km at a uniform speed. If Prove that one of every three consecutive positive integers
the speed had been 8 km/h less, then it would have taken is divisible by 3. [OS]
3 hours more to cover the same distance. Find the original 38. The sum of four consecutive numbers in AP is 32 and
speed of the train. (3 Marks) the ratio of the product of the first and last terms to the
33. Prove that the parallelogram circumscribing a circle is a product, of two middle terms is 7 : 15. Find the numbers.
rhombus. (3 Marks) (4 Marks)
OR
34. Prove that: 2(sin6 θ + cos6 θ) – 3(sin4 θ + cos4 θ) + 1 = 0.
(3 Marks) Solve: 1 + 4 + 7 + 10 + ... + x = 287
39. A bucket is in the form of a frustum of a cone of height
Section-D 16 em with radii of its lower and upper circular ends as
8 cm and 20 cm respectively. Find the cost of milk which
35. The following table gives production yield per hectare (in can completely fill the bucket, at the rate of ` 40 per litre.
(Use π = 3.14) [OS] (4 Marks)
quintals) of wheat of 100 farms of a village:
40. Construct a triangle with sides 4 cm, 5 cm and 6 cm. Then
Production 2
40–45 45–50 50–55 55–60 60–65 65–70
yield/hect. construct another triangle whose sides are times the
3
Number 4 6 16 20 30 24 corresponding sides of the first triangle. [OS] (4 Marks)

CBSE 2020 (DELHI) 205


EXPLANATIONS

1. (c) Prime factorization of the given numbers are


a 2 cos 2 θ + b 2 cos 2 θ + 2a cos θ b sin θ
12 = 2 × 2 × 3 = 22 × 3 =
+ a 2 sin 2 θ + b 2 cos 2 θ − 2ab sin θ. cos θ
21 = 3 × 7
15 = 3 × 5 ½ Mark)
(½ = a 2 (sin 2 θ + cos 2 θ) + b 2 (sin 2 θ + cos 2 θ)
Hence, HCF (12, 21, 15) = 3 and
= a 2 + b2  (1 Mark)
LCM (12, 21, 15) = 22 × 3 × 7 × 5 = 420  (½½ Mark)
7. (c) The factors of a prime number is only 1 and itself,
2. (a) 2x, (x + 10) and (3x + 2) are in AP. therefore it has only two factors. (1 Mark)
Q If a, b and c are in AP.
8. (d) Given, The point (k, 0) divides the line segment AB in
\ 2b = a + c the ratio 1 : 2, then by section formula we have
Hence 2(x + 10) =2x + 3x + 2 ½ Mark)
(½ −7 × 1 + 2 × 2 −7 + 4
⇒ 2x + 20 = 2x + 3x + 2 k= =
1+ 2 3
⇒ 20 – 2 = 2x + 3x – 2x −3
⇒ 18 = 3x = ⇒ k = –1 (1 Mark)
3
18 9. (a) Out of the Syllabus
⇒ x= = 6 ½ Mark)

3 10. (b) Let f(x) = x2 + 3x + k and it is given that 2 is a zero of
3. (d) If a1 x b1 y + c1 = 0 and a2 x b2 y + c2 = 0 has no
solution. f(x).
a b c \ f(2) = 0.
Therefore 1 = 1 ≠ 1  ½ Mark)
(½ ⇒ 22 + 3 × 2 + k = 0
a2 b2 c2
Given a1 = 1, b1 = 1, c1 = 4 ⇒ k = – 4 – 6 = – 10. (1 Mark)
a2 = 2, b1 = k, c2 = –3 11. Let AM be the height on the base BC as shown in the below
fig. Since altitude of equilateral triangle its base.
1 1
⇒ = A
2 k
⇒ k =2 ½ Mark)

4. (c) Given a = p and d = q 2a 2a
a10 = ?
Q an = a + (n – 1)d B C
a M a
⇒ a10 = p + (10 – 1)q
= p + 9q (1 Mark) \ BM = MC = a
5. (a) Let a and b are the zeroes of the required quadratic Now, DAMB is a sight eagled triangle. Therefore by
polynomial. pythagors therm
b ⇒(AB)2 = (AM)2 + (MB)2 ½ Mark)

Given, a + b = – 5 = −
a ⇒(2a) = (AM) + (a) ⇒ (AM) = 4a – a
2 2 2 2 2 2
c
ab = 6 =
a ⇒AM = 3a 2 ⇒ AM = 3a  ½ Mark)

If a + b and a × b is given of any quadratic polynomial
12. Given AP = 4 cm and AP = AR = 4 cm
then, The required polynomial is x2 – (a + b)x + ab
\ x2 – (–5)x + 6 = x2 + 5x + 6 (1 Mark)  (Tangents from the same point are same)

6. (c) Given, x1 = a cos q + b sin q, y1 = 0 and \ RC = AC – AR


x2 = 0, y2 = a sin q – b cos q = 11 – 4 = 7 cm  (Given, AR = 11 cm)
Now CQ = CR 
By distance formula = ( x2 − x1 ) 2 + ( y2 − y1 ) 2
 (Tangents from the same point are same)
= [0 − (a cos θ + b sin θ)]2 + (a sin θ − b cos θ − 0) 2 ⇒ CQ = 7 cm ½ Mark)

206 MATHS
Again, PB = BQ  OR
 (Tangents from the same point are same) Given, P(E) = 0.07
⇒ BQ = 3 cm
Now, ∵ P( E ) + P( E ) =
1
BC = BQ + QC = 3 cm + 7 cm = 10 cm ½ Mark)
(½ ∴ 0.07 + P ( E ) =
1

2 1 ⇒ P( E ) =
1 − 0.07 =
0.93  (1 Mark)
13. Given sin θ +
1 + tan 2 θ 17. Given,
2 1 1 + 2 + 3 + ..... + n
= sin θ + [Q 1 + tan2 q = sec =
2
q] = 15
sec 2 θ n
 1  We know that the formula of sum of first n natural number
= sin q + cos q = 1 ∵ = cos θ [Q sin q + cos q = 1]
2 2 2 2
n(n + 1)
 sec  is
2 1 2
⇒ sin θ + 2 = 1 (1 Mark) n ( n + 1)
1 + tan θ ∴ = 15
2n
OR ⇒ n + 1 = 30 ⇒ n = 29 (1 Mark)
Given, 18. Let r1 and h1 is the radius and height of the first cone
(1 + tan2 q)(1 – sin q)(1 + sin q) respectively and r2 and h2 is the radius and height of the
= (1 + tan2 q)(1 – sin2 q)[Q (a + b)(a – b) = a2 – b2] second cone respectively.
½ Mark)
= (1 + tan2 q)cos2 q [Q 1 – sin2 q = cos2 q] (½ r1 3 h 1
Given, = and 1 =
= sec q cos q[Q 1 + tan q = sec2 q]
2 2 2 r2 1 h2 3
1 2
1  1  ∴ Volume of first cone (v1) = πr1 h1 and
= × cos 2 θ = 1 ∵ sec θ =  ½ Mark)
(½ 3
2
cos θ  cos θ  1 2
Volume of second cone (v2)= πr2 h2
14. Given, 3
2 2 1 2
 sin 35°   cos 43°  V
πr1 h1
  +  − 2 cos 60° ratio of their volume = 1 = 13  ½ Mark)

 cos 55°   sin 47°  V2
2 2 πr22 h2
 sin(90 − 55)   cos(90 − 47)  3
=   +  − 2 cos 60° 2 2
 cos 55°   sin 47°  r  h  3 1 9 1 3
=  1  1=    = × =
 [Q 90 – 55 = 35 and 90 – 47 = 43°]  r2   h2   1   3  1 3 1
2
 cos 55°   sin 47° 
2
⇒ V1 : V2 = 3 : 1 ½ Mark)

=   +  − 2 cos 60°
 cos 55°   sin 47°  19. Let AB be vertical rod and BC be shadow of its.
 [Q sin(90 – q) = cos q and cos(90 – q) = sin q] AB 1
Given, =
1  1 BC 3
=1+1–2×
2
 ∵ cos 60° = 2  Let q be the angle of elevation of the sum.
 
= 2 – 1 = 1 (1 Mark) A
15. Out of the Syllabus
16. If a die is thrown then outcome will be
1, 2, 3, 4, 5 and 6
∴ Total number of outcomes n(s) = 6 q
C B ½ Mark)

Number less then 3 is 1 and 2
AB
∴ Total number of favorable outcomes n(E) = 2  tan θ =
BC
½ Mark)
The probability of getting number less then 3 is(½ 1  AB 1 
⇒ tan θ =  Given, BC = 3 
Total number of favourable n( E ) 3 
∴ P( E )
= =  1 
Total number of outcomes n( s ) ⇒ tan q = tan 30° ∵ tan 30° = 3 
2 1 
P( E =
) =  ½ Mark)

6 3 ⇒ q = 30° ½ Mark)

CBSE 2020 (DELHI) 207


20. If a die is thrown then outcome will be 1, 2, 3, 4, 5 and 6. 22. In DACD
∴ Total number of outcomes n(S) = 6. A 1.5 m C
Number of even number n(E) = 3. ½ Mark)

∴ The probability of getting even number is q 3
m
Total number of favourableoutcome n( E ) D
P( E ) =
Total number of outcomes n( S ) B
Wall
3 1
⇒ P ( E ) == ½ Mark)
(½ CD2 = AC2 + AD2 ⇒ (3)2 = (1.5)2 + (AD)2
6 2
21. In DBPA A ⇒ 9 = 2.25 + (AD)2 ⇒ AD2 = 9 – 2.25
DC || AP 675 27
⇒ AD2 = 6.75 ⇒ AD 2 = =
By BPT D 100 4
3× 3× 3 3 3
BC BD AD
⇒= =
⇒ = ...(i) (1 Mark) B E C R 2× 2 2
CP DA AC
Now, In DBCA (i) We know that, tan θ =
AD
PF || AC 1.5 15 × 2 1
⇒ tan
= θ = =  (1 Mark)
by BPT 3 3 10 × 3 3 3
2

BE BD
=
CD 3 2
...(ii) (ii) We know, sec
=θ = =
EC DA AD 3 3 3
From equation (i) and eqn (ii), we get 2
CD 3
BC BE cosec=
θ =
= proved. (1 Mark) CA 1.5
CP EC
2 3 2 3 × 10 2
OR ∴ sec θ + cosec=
θ + = + = +2
3 1.5 3 15 3
TP = TQ...(i)
 1  2(1 + 3) 3 2
( 3 + 3) 
(Tangents from the same point are same) = 2  + 1=
 ×= (1 Mark)
 3  2 3 3
P
23. Given numbers –3, –2, –1, 0, 1, 2, 3
T O ½ Mark)
Square of the given numbers = 9, 4, 1, 0, 1, 4, 9 (½
Favourable numbers = 4, 1, 0, 1, 4
Q ∴ The number of favourable number n(E) = 5
∠TQP = ∠TPQ ...(ii) (1 Mark) Total number n(S) = 7 ½ Mark)

(angles of equal sides are equal) n( E ) 5
∴ probability of x2 ≤ 4 = =  (1 Mark)
n( S ) 7
Now TP is a tangent and OP is the radius. 24.
∴ OP is perpendicular to PT. C.I fi xi xi fi
∴ ∠OPT = 90° ⇒ ∠OPQ + ∠TPQ = 90° 3–5 5 4 20
⇒ ∠TPQ = 90 – ∠OPQ...(iii) 5–7 10 6 60
In DPTQ 7–9 10 8 80
∠TPQ + ∠PQT + ∠QTP = 180° 9 – 11 7 10 70
11 – 13 8 12 96
 [ sum of interior angle of a triangle is 180°]
Total Σfi = 40 Σxi fi = 326
From equation (ii) and (iii)
From the above table we have,
⇒ 2(90 – ∠OPQ) + ∠QTP = 180°
Σfi = 40 and Σxi fi = 326 (1 Mark)
⇒ 180° – 2∠OPQ + ∠QTP = 180°
Σxi fi 326
⇒ 2∠OPQ = ∠QTP Hence proved. (1 Mark)  Mean = = = 8.15  (1 Mark)
Σf i 40

208 MATHS
OR ⇒ AC2 = AB2 + BC2 ...(ii) ½ Mark)

C.I fi from equation (i) and (ii) we get
0 – 20 6 ⇒ PR2 = AC2
20 – 40 8 ∴ PR = AC ...(iii) ½ Mark)

40 – 60 10 Now, In DABC and DPQR.
60 – 80 12 AB = PQ, BC = QR and AC = PR
80 – 100 6
⇒ DABC ≅ DPQR (BY SSS Congruency)
100 – 120 5
∴ ∠B = ∠Q (By C.P.C.T)
120 – 140 3
∴ ∠Q = 90° and ∠B = 90° proved (1 Mark)
Highest frequency = 12
29. Out of the Syllabus
Modal class = 60 – 80
OR
l = 60, f0 = 10, f1 = 12, f2 = 6 and h = 20 (1 Mark)
Given A = (2, 5), B = (x, y), C = (–1, 2)
 f1 − f 0 
 Mode = l +  h  ½ Mark)
(½ 3 4
2
 1 − f0 − f 2 
f
A(2, 5) C(–1, 2) B(x, y)
 12 − 10  2
60 + 
=  × 20 = 60 + 8 × 20 = 60 + 5 = 65 By Section formula, we have
 2 × 12 − 10 − 6 
½ Mark)
(½ mx2 + nx1 my2 + ny1 
( x, y ) =  ,  (1 Mark)
25. Given, 1, 4, 7, 10, .......  m+n m+n 
a = 1, d = a2 – a1 = 4 – 1 = 3, n = 20  3x + 4 × 2 3 y + 4 × 5 
⇒ (−1, 2) = , 
n  3+ 4 3+ 4 
∵ S=
n ( 2a + (n − 1)d )  (1 Mark)
2 3x + 8 3 y + 20
⇒ −1
= = or 2  (1 Mark)
20 7 7
∴ S= 20 ( 2 × 1 + (20 − 1)3) = 10 (2 + 19 × 3)
2 ⇒ 3x + 8 = –7 or 3y + 20 = 14
= 10(2 + 57) = 10 × 59 = 590 (1 Mark)
⇒ 3x = – 7 – 8 or ⇒ 3y = 14 – 20
26. Given, r = 5.2 cm and perimeter of the sector = 16.4 cm.
⇒ 3x = – 15 or ⇒ 3y = –6
Let AOB be the sector with center O. ⇒ x = – 5 or ⇒ y = –2
∴ perimeter of the sector = AO + AB + OB ∴ B(–5, –2) (1 Mark)
⇒ 16.4 = 5.2 + 5.2 + AB 30. Let a and b are the zeroes of the given polynomial
⇒ AB = 16.4 – 10.4 = 6 cm (1 Mark) f(x) = ax2 + bx + c, a ≠ 0, c ≠ 0
1 1 b c
∴ Area of the sector = rl = × 5.2 × 6 = 15.6 cm2. ∴α + β = − and αβ =  ½ Mark)

2 2 a a
(1 Mark)
Let a' and b' are the zeroes of the required polynomial.
27. Out of the Syllabus
1 1
α′
Given that= and
= β′
28. Given, ABC is a triangle. α β
AC2 = AB2 + BC2 1 1 α + β −b / a b
Now, α′ + β′ = + = = =−  ½ Mark)

α β αβ c/a c
To prove ∠B = 90°
1 1 1 1 a
Construction: Construct a triangle PQR α′ ⋅=
β′ .= = =  ½ Mark)

α β αβ c / a c
right angled at Q such that ½ Mark)
∴ Required polynomial = x2 – (a′ + b′)x + a′b′(½
PQ = AB and QR = BC  b a b a
= x2 −  −  x + = x2 + x +
Proof  c c c c
In DPQR, by Pythagors theorem, (1 Mark) = cx2 + bx + a (1 Mark)
⇒ PR2 = PQ2 + QR2 OR
⇒ PR = AB + BC
2 2 2
...(i)( PQ = AB and QR = BC) Out of the Syllabus

CBSE 2020 (DELHI) 209


31. Given equations are A P B
2y – x = 8, 5y – x = 14 and y – 2x = 1
When x = 0 S Q

2y – 0 = 8, 5y – 0 = 14 and y – 2 × 0 = 1
D R C
8 14
⇒ y = ⇒ y = and y = 1 ⇒ y = 2 DR = DS( Tangents on the circle from same point D)
2 5
∴ The points are CR = CQ( Tangents on the circle from same point C)
 14  BR = BQ( Tangents on the circle from same point B)
(0, 2),  0,  and (0, 1) (1 Mark)
 5
When y = 0 AP = AS( Tangents on the circle from same point A)

2 × 0 – x = 8, 5 × 0 – x = 14 and 0 – 2x = 1 Adding all these equation, we get (1 Mark)


1 DR + CR + BP + AP = DS + CQ + BQ + AS.
⇒ x = – 8 ⇒ x = –14 ⇒ x = −
2 (DR + CR) + (BP + AP) = (CQ + BQ) + (DS + AS)
∴ The points are
CD + AB = AD + BC ½ Mark)

 −1 
(–8 , 0), (–14, 0) and  ,0   (1 Mark)
 2  Putting these values in eq(i) and (ii), we get
plot all the obtained points in the coordinate axes and join 2AB = 2BC ⇒ AB = BC...(iii)
these point.
from equation (i), (ii) and (iii)
AB = BC = CD = DA
6 ∴ ABCD is a Rhombus. (1 Mark)
(2, 5)
4 34. Given, 2(sin6q + cos6q) – 3(sin4q + cos4q) + 1 = 0
(–4, 2) L.H.S =2((sin2q)3 + (cos2q)3) – 3[(sin2q + cos2q)2 – 2 sin2q
2 (1, 3)
cos2q] + 1 (1 Mark)
–10 –8 –6 –4 –2 0 2 4 6 8 = 2{(sin2q + cos2q)3 – 3sin2q cos2q(sin2q + cos2q)}
(1 Mark) – 3(12 – 2sin2q cos2q) + 1 ( sin2q + cos2q = 1)
OR = 2(1 – 3 sin2q cos2q) –3(1 – 2sin2q cos2q) + 1 (1 Mark)
Out of the Syllabus = 2 – 6 sin2q cos2q – 3 + 6 sin2q cos2q + 1 = 3 – 3 = 0
32. Let the uniform speed of the train be x km/h. ∴ LHS = RHS (proved) (1 Mark)
480
∴ The time taken to cover the distance 480 km = h 35. Out of the Syllabus
x
Now the speed (x – 8) km/h. ½ Mark)
(½ OR
480 C.I fi C.f
The time taken to cover the same distance = h
x −8 0 – 100 2 2
 ½ Mark)

480 480 100 – 200 5 7
According to question, = +3 ½ Mark)

x −8 x 200 – 300 x 7+x
480 480  x − x +8 300 – 400 12 19 + x
⇒ − = 3 ⇒ 480  = 3 ½ Mark)

x −8 x  ( x − 8) x  400 – 500 17 36 + x
⇒ x2 – 8x = 160 × 8 ⇒ x2 – 8x – 1280 = 0
500 – 600 20 56 + x
⇒ x2 – 40x + 32x – 1280 = 0 ⇒ x(x – 40) + 32(x – 40) = 0
600 – 700 y 56 + x + y
⇒ (x – 40)(x + 32) = 0 ⇒ x = 40, –32 ⇒ x = –32 not possible 700 – 800 9 65 + x + y
∴ The speed is 40 km/h. (1 Mark) 800 – 900 7 72 + x + y
33. Since, ABCD is a parallelogram circumscribed in a circle 900 – 1000 4 76 + x + y
AB = CD...(i) Total 100
BC = AD  ½ Mark)
...(ii) (½  (2 Marks)

210 MATHS
We have, ⇒ 15(a2 – 9d2) = 7(a2 – d2) ⇒ 15 a2 – 135 d2 = 7a2 – 7d2
N = ∑fi = 100 ⇒ 76 + x + y = 100 ⇒ x + y = 24 ⇒ 15a2 – 7a2 = –7a2 + 135d2 ⇒ 8a2 = 128d2
It is given that the median is 525. Putting the value of a from eq(i) ½ Mark)

Clearly, it lies in the class 500 – 600 ⇒ 8 × (8) = 128d
2 2

l = 500, h = 100, f = 20, cf = 36 + x and N = 100


8×8×8
N d2
⇒= = 4 ⇒ d = ±2 ½ Mark)

− cf 128
Now, Median = l+ 2 ×h  (1 Mark) Case-I:
f When d = 2, then the A.P will be
50 − (36 + x)
⇒ 525 = 500 + × 100 8 – (3 × 2), 8 – 2, 8 + 2, 8 + (3 × 2)
20
i.e., 2, 6, 10, 14 ½ Mark)

⇒ 525 – 500 = (14 – x) × 5
⇒ 25 = 70 – 5x ⇒ 5x = 45 ⇒ x = 9 Case-II:
Putting x = 9 in x + y = 24, we get y = 15 When d = –2, then the A.P will be
Hence, x = 9 and y = 15 (1 Mark) 8 – (3 × (–2)), 8 + 2, 8 – 2, 8 × (3 × (–2))
36. Let AB be the height of the flag and BC be the height of the i.e 14, 10, 6, 2 ½ Mark)

vertical tower. OR
A
In DBCD,
Given, 1 + 4 + 7 + 10 + ... + x = 287
BC 1 BC
tan 30° = ⇒ = 6m Let a1 = 1, a2 = 4, a3 = 7, a4 = 10 and an = x
DC 3 DC
a2 – a1 = 4 – 1 = 3, a3 – a2 = 7 – 4 = 3
⇒ DC =3BC ...(i) B
a4 – a3 = 10 – 7 = 3
In DACD,
45°  a2 – a1 = a3 – a2 = a4 – a3 = d.
AC 30°
tan 45° =
DC D C ∴ The given series are in A.P. ½ Mark)

⇒ DC = 6 + BC ...(ii)  (1 Mark)  an = a + (n – 1)d ½ Mark)

from eq(i) and eq(ii), we get ⇒ x = 1 + (n – 1)3 ⇒ 3n – 3 + 1 = x ⇒ 3n = x + 2
3BC= 6 + BC  (1 Mark) x+2
⇒n= ...(i) ½ Mark)

6 3 +1 3
⇒ BC ( 3 − 1) =
6 ⇒ BC= ×
(1 Mark) n
( l + a ) (Where l is the last term)
3 −1 3 +1  ∵=Sn ½ Mark)

2
6 ( 3 + 1) 6 ( 3 + 1)
= = = 3 ( 3 + 1) n
∴ 287 = ( x + 1)
3 −1 2 2
= 3(1.73 + 1) = 3 × 2.73 = 8.19 m (1 Mark) from eq(i), we have
37. Out of the Syllabus ( x + 2)( x + 1)
⇒ 287 =  ½ Mark)

OR 6
Out of the Syllabus ⇒ x + 3x + 2 = 1722 ⇒ x + 3x – 1720 = 0
2 2

38. Let the four consecutive numbers in A.P be a – 3d, a – d, ⇒ x2 + 43x – 40x – 1720 = 0 ½ Mark)

a + d, a + 3d ⇒ x(x + 43) – 40(x + 43) = ⇒ (x + 43) (x – 40) = 0
According to questions,
⇒ x = –43 or 40.
⇒ a – 3d + a – d + a + d + a + 3d = 32 (1 Mark)
Since the given series is a increasing
⇒ 4a = 32 ⇒ a = 8 ...(i)
∴ x ≠ –43
(a − 3d )(a + 3 x) 7
Now, = (Given) (1 Mark) ∴ x = 40 (1 Mark)
(a − b)(a + d ) 15
a 2 − (3d ) 2 7 39. Out of the Syllabus
⇒ = [ (a – b) (a + b) = a2 = b2]
a2 − d 2 15 40. Out of the Syllabus

CBSE 2020 (DELHI) 211


Outside DELHI
2020 CBSE Solved Paper

Time allowed : 3 hours Maximum Marks : 80


GENERAL INSTRUCTIONS:
Read the following instructions very carefully and strictly follow them:
(i) This question paper comprises four sections – A, B, C and D. This question paper carries 40 questions.
All questions are compulsory.
(ii) Section A – Question no. 1 to 20 comprises of 20 questions of one mark each.
(iii) Section B – Question no. 21 to 26 comprises of 6 questions of two marks each.
(iv) Section C – Question no. 27 to 34 comprises of 8 questions of three marks each.
(v) Section D – Question no. 35 to 40 comprises of 6 questions of four marks each.
(vi) There is no overall choice in the question paper. However, an internal choice has been provided in 2 questions of one
mark, 2 questions of two marks, 3 questions of three marks and 3 questions of four marks. You have to attempt only
one of the choices in such questions.
(vii) In addition to this, separate instructions are given with each section and question, wherever necessary.
(viii) Use of calculators is not permitted.

6. The co-ordinates of the point which is reflection of point


Section-A (–3, 5) in x-axis are (1 Mark)
1. The value of k for which the system of linear equations (a) (3, 5) (b) (3, –5)
x + 2y = 3, 5x + ky + 7 = 0 is inconsistent is (1 Mark) (c) (–3, –5) (d) (–3, 5)
14 2 7. The point P on x-axis equidistant from the points A(–1, 0)
(a) − (b) (c) 5 (d) 10
3 5 and B(5, 0) is (1 Mark)
2. The zeroes of the polynomial x2 – 3x – m (m + 3) are (a) (2, 0) (b) (0, 2)
(1 Mark) (c) (3, 0) (d) (2, 2)
(a) m, m + 3 (b) –m, m + 3
8. The nth term of the A.P. a, 3a, 5a, ...... is (1 Mark)
(c) m, –(m + 3) (d) –m, –(m + 3)
(a) na (b) (2n – 1) a
3. Euclid’s division Lemma states that for two positive
(c) (2n + 1) a (d) 2na
integers a and b, there exists unique integer q and r
satisfying a = bq + r, and [OS] (1 Mark) 1 1− p 1− 2 p
9. The common difference of the A.P. , , , ……
(a) 0 < r < b (b) 0 < r ≤ b p p p
(c) 0 ≤ r < b (d) 0 ≤ r ≤ b is (1 Mark)
1 1
4. The sum of exponents of prime factors in the prime- (a) 1 (b) (c) –1 (d) −
factorisation of 196 is: (1 Mark) p p
(a) 3 (b) 4 (c) 5 (d) 2 10. The roots of the quadratic equation x2 – 0.04 = 0 are
5. If the point P(6, 2) divides the line segment joining A(6, 5) (1 Mark)
and B(4, y) in the ratio 3 : 1, then the value of y is (1 Mark) (a) ±0.2 (b) ±0.02
(a) 4 (b) 3 (c) 2 (d) 1 (c) 0.4 (d) 2
11. In fig. the angles of depressions from the observing 17. If a number x is chosen at random from the numbers –3,
positions O1 and O2 respectively of the object A are –2, –1, 0, 1, 2, 3, then find the probability of x2 < 4.
________, _________. (1 Mark) (1 Mark)
O2 O1 OR
60 o What is the probability that a randomly taken leap year has
52 Sundays?
18. A die is thrown once. What is the probability of getting a
45o prime number. (1 Mark)
A B C 19. If tan A = cot B, then find the value of (A + B).
ar ( ∆AMN )
12. In fig. MN || BC and AM : MB = 1 : 2, then  [OS] (1 Mark)
ar ( ∆ABC )
= _________. [OS] (1 Mark) 20. Find the class marks of the classes 15 – 35 and 45 – 60.
A (1 Mark)

Section-B

M N 21. A teacher asked 10 of his students to write a polynomial


in one variable on a paper and then to handover the paper.
The following were the answers given by the students:
B C 2x + 3, 3x2 + 7x + 2, 4x3 + 3x2 + 2, x 3 + 3 x + 7, 7 x + 7,
13. In given Fig. the length PB = _________ cm. (1 Mark) 2 5 1 3 2 1
5x3 – 7x + 2, 2 x + 3 − ,5 x − , ax + bx + cx + d , x +
x 2 x
Answer the following questions: (2 Marks)
(i) How many of the above ten, are not polynomials?
A 5 cm
O (ii) How many of the above ten, are quadratic polynomials?
3 cm 22. Compute the mode for the following frequency distribution:
P
(2 Marks)

B Size of items 0–4 4–8 8–12 12–16 16–20 20–24 24–28


14. In ∆ABC, AB = 6 3 cm, AC = 12 cm and BC = 6 cm, then
∠B = _________. [OS] (1 Mark) Frequency 5 7 9 17 12 10 6
OR
Two triangles are similar if their corresponding sides are 23. In fig. ABC and DBC are two triangles on the same base
_________. ar ( ∆ABC ) AO
BC. If AD intersects BC at O, show that =
15. The value of sin 23° cos 67° + cos 23° sin 67° is _________. ar ( ∆DBC ) DO
 [OS] (1 Mark)
 [OS] (2 Marks)
16. In fig. AOB is a sector of circle of radius 10.5 cm. Find the
A
 22 
perimeter of the sector.  Take π =   (1 Mark)
 7 

O C
A B
B

60o
D

O OR

CBSE 2020 (Outside DELHI) 213


In fig. if AD ⊥ BC, then prove that AB2 + CD2 = BD2 + AC2. 31. If the mid-point of the line segment joining the points
 [OS] A(3, 4) and B(k, 6) is P(x, y) and x + y – 10 = 0, find the
C value of k. (3 Marks)
D OR
Find the area of triangle ABC with A(1, –4) and the mid-
B A points of sides through A being (2, –1) and (0, –1). [OS]
32. If in an A.P., the sum of first m terms is n and the sum of
cot 2 α its first n terms is m, then prove that the sum of its first
24. Prove that 1 + cos ecα 
= (2 Marks)
1 + cos ecα (m + n) terms is –(m + n). (3 Marks)
OR OR
Show that tan θ + tan θ = sec4 θ – sec2 θ
4 2
Find the sum of all 11 terms of an A.P. whose middle term is 30.
25. A child has a die whose six faces show the letters as shown 33. A fast train takes 3 hours less than a slow train for a journey
below: (2 Marks) of 600 km. If the speed of the slow train is 10 km/h less
than that of the fast train, find the speed of each train.
A A B C C C
(3 Marks)
The die is thrown once.What is the probability of getting 34. If 1 + sin2 θ = 3 sin θ cos θ, prove that tanθ = 1 or 1/2.
(i) A, (ii) C? (3 Marks)
26. A solid is in the shape of a cone mounted on a hemisphere
of same base radius. If the curved surface areas of the Section-D
hemispherical part and the conical part are equal, then find
the ratio of the radius and the height of the conical part. 35. The mean of the following frequency distribution is 18.
(2 Marks) The frequency f in the class interval 19 – 21 is missing.
Determine f. (4 Marks)
Section-C Class interval 11–13 13–15 15–17 17–19 19–21 21–23 23–25
Frequency 3 6 9 13 f 5 4
27. In Fig. if ∆ABC ~ ∆DEF and their sides of lengths (in cm)
are marked along them, then find the lengths of sides of OR
each triangle. (3 Marks) The following table gives production yield per hectare of
wheat of 100 farms of a village:
A D
Production 40–45 45–50 50–55 55–60 60–65 65–70
yield
2x – 1 3x 18 6x
No. of farms 4 6 16 20 30 24

B C E F Change the distribution to a ‘more than’ type distribution


2x + 2 3x + 9 and draw its ogive. [OS]
36. Find the area of the shaded region in fig. 8, if PQ = 24 cm,
28. If a circle touches the side BC of a triangle ABC at P and PR = 7 cm and O is the centre of the circle. [OS] (4 Marks)
extended sides AB and AC at Q and R, respectively, prove Q
1
that AQ = ( BC + CA + AB )  (3 Marks)
2
O
29. The area of a circular play ground is 22176 cm2. Find the
cost of fencing this ground at the rate of ` 50 per metre.
 [OS] (3 Marks)
R P
30. If 2x + y = 23 and 4x – y = 19, find the value of (5y – 2x)
y  OR
and  − 2  . (3 Marks)
x  Find the curved surface area of the frustum of a cone, the
OR diameters of whose circular ends are 20 m and 6 m and its
height is 24 m. [OS]
1 1 11
Solve for x : − = , x #− 4, 7 .
x + 4 x + 7 30 37. Prove that 5 is an irrational number. (4 Marks)

214
214 MATHS
38. It can take 12 hours to fill a swimming pool using two OR
pipes. If the pipe of larger diameter is used for four hours Construct a triangle ABC with sides 3 cm, 4 cm and 5 cm.
and the pipe of smaller diameter for 9 hours, only half of Now, construct another triangle whose sides are 4/5 times
the pool can be filled. How long would it take for each the corresponding sides of ∆ABC. [OS]
pipe to fill the pool separately? [OS] (4 Marks)
40. The angle of elevation of the top of a building from the foot
39. Draw two tangents to a circle of radius 4 cm, which are of a tower is 30° and the angle of elevation of the top of a
inclined to each other at an angle of 60o. tower from the foot of the building is 60°. If the tower is
 [OS] (4 Marks) 50 m high, then find the height of the building.(4 Marks)

EXPLANATIONS

1. (d) A pair of linear equation is inconsistent if, for two 5. (d) Let P(a, b) divides AB internally in the ratio 3 : 1
linear equation.
3 1
a1x + b1 y + c1 = 0 & a2 x + b2 y + c2 = 0, we have A(6, 5) P(a, b) B(4, y)
a1 b1 c1
= ≠  3 × 4 + 1× 6 3 × y + 1× 5   9 3 y + 5 
a2 b2 c2 ∴ P(α, β) = ,  = , 
given, two linear equation are;  3 +1 3 +1   2 4 
x + 2y = 3 & 5x + ky + 7 = 0 ½ Mark)
(½  (½½ Mark)
Here, a1 = 1 ; b1 = 2 ; c1 = –3 and given P(6, 2)
a2 = 5 ; b2 = k ; c2 = 7 3y + 5
We have, = 2 ⇒ 3y + 5 = 8 ⇒ 3y = 3 ⇒ y = 1
a b 1 2 4
Now, for inconsistency, 1 = 1 ⇒ = ⇒ k = 10  ½ Mark)

a2 b2 5 k
a1 c2 1 −3 6. (c) The given point is (–3, 5)
And, ≠ ⇒ ≠ [satisfied]
a2 c2 5 7 For reflection through x-axis we take the (–)ve of the
10 
∴k = ½ Mark)
(½ y-coordinate only ½ Mark)

2. (b) Given, x – 3x – m(m + 3) is the quadratic equation.
2 Negative of y-coordinate = – (5) = –5
Now, x2 – 3x – m(m + 3) = 0 ∴ The reflection is (–3, –5) ½ Mark)

⇒ x2 – (3 + m – m)x – m(m + 3) = 0 7. (a) The given points are A(–1, 0) & B(5, 0)
 [by middle term factorization] Now, point P lies on x-axis, that means the
⇒ x – (m + 3) x + mx – m(m + 3) = 0
2
½ Mark)
(½ y-coordinate is zero. i.e. P(x, 0)
⇒ x(x – (m + 3)) + m(x – (m + 3)) = 0 given, let the point P(x, 0) is equidisplant from A and B
⇒ (x + m) (x – (m + 3)) = 0  ½ Mark)

⇒ x + m = 0 or x – (m + 3) = 0 ∴ According to question,
⇒ x = –m or x = m + 3 PA = PB.
∴ The zeroes are, –m, m + 3 ½ Mark)

⇒ ( x + 1) 2 + (0 − 0) 2 = ( x − 5) 2 + (0 − 0) 2
3. (c) Out of the Syllabus
4. (b) Prime factorization of 196 is, ⇒ x 2 + 1 + 2=
x x 2 + 25 − 10 x
196 = 22 × 72 ½ Mark)

⇒ x2 + 1 + 2x = x2 + 25 – 10x [squaring both sides]
Sum of exponents of prime factors
⇒ 12x = 24 ⇒ x = 2
= (exponent of 2) + (exponent of 7) = 2 + 2 = 4
 ½ Mark)
(½ ∴ The point P is P(2, 0) ½ Mark)

CBSE 2020 (Outside DELHI) 215


8. (b) The given AP is; a, 3a, 5a, ..... 14. Out of the Syllabus
First term = a. 15. Out of the Syllabus
common difference = 3a – a = 2a. ½ Mark)

16. Given, radius of circle = 10.5 cm
∴ The nth term = a + (n – 1)2a = a + 2an – 2a.
∠AOB = 60°
= 2an – a = a(2n – 1) ½ Mark)

Now, perimeter of sector
1 1− p 1− 2p A B
9. (c) The given AP is; , , ,....  ½ Mark)
(½ πrθ
p p p = 2r +
180° ½ Mark)

1− p 1 1− p −1 − p
=
Common difference − = = = −1 π(10.5)60
p p p p = 2(10.5) + 60°
180°
 ½ Mark)

22 1
10. (a) The given quadratic equation is: = 21 + × 10.5 × = 21 + 11 = 32 cm. O
7 3
∴ x2 – 0.04 = 0 ⇒ x2 = 0.04 ½ Mark)

∴ The perimeter of the sector is 32 cm. ½ Mark)

4 2
⇒ x 2 = ⇒ x =± ⇒ x = 0.2, –0.2
100 10 17. The given number are; –3, –2, –1, 0, 1, 2, 3
∴ The roots are ± 0.2 ½ Mark)
(½ Now, square of the numbers;
9, 4, 1, 0, 1, 4, 9 ½ Mark)

11. X O2 O1
Now, if we select x from the number, such that,
60° x2 < 4
Then no. of favourable outcomes = 3.
45°
3
A B C P(x2 < 4) =  ½ Mark)

7
Given, ∠O2AB = 45°, ∠AO1C = 60° ½ Mark)
(½ OR
Now, angle of depression at O1, ∠AO1O2 = 90° – ∠AO1C There are 52 weeks in a year, so there will be 52
= 90° – 60° = 30° Sundays.
And, angle of depression at O2, ∠AO2 X = 45° However, 52 × 7 = 364.
 [alternate angle to O2 AB] So, there still leaves two days in a leap year
∴ So angle of depression are 30°, 45° ½ Mark)
(½ So, the possible outcomes can be;
(Mon, Tue) ; (Tue, Wed) ; (Wed, Thur) ; (Thur, Fri) ;
12. Out of the Syllabus
(Fri, Sat) ; (Sat, Sun) ; (Sun, Mon) ½ Mark)

13. Here, OP ⊥ AB, since, the tangent is perpendicular to Now, to get 52 Sundays none of the last two days can
the radius at point contact be Sunday.
∴ No. of favourable outcomes = 5
No. of total outcomes = 7
A 5 O 5
3 ∴ P(52 Sundays) =  ½ Mark)

7
P
18. On throwing a die, total no. of outcomes = 6
B No. of prime numbers, on throwing a die = 3
Now, in DOAP;
3 1
∴ P(prime number) = =  (1 Mark)
AP
= 2
OA − OP 2 = (5) 2 − (3) 2 = 25 − 9 6 2
19. Out of the Syllabus
= 16 4cm
=
 ½ Mark)
(½ 20. Given, classes are, 15 – 35 and 45 – 60
Now, we know, the line segment perpendicular to a
chord & passing through the center bisects the chord Now, class mark of 15 – 35 = 15 + 35 50
½ Mark)
= = 25 (½
2 2
∴ AP = PB 45 + 60 105
Class mark of 45 – 60 = ½ Mark)
= = 52.5 (½
∴ PB = 4 cm ½ Mark)
(½ 2 2

216
216 MATHS
21. The answer given by the students are; Now, given
1 Curve surface area of conical part
2x + 3; 5 x − ; 3x2 + 7x + 2; ax3 + bx2 + cx + d;
2 = curve surface area of hemisphere
1 3
4x + 3x + 2; x + ; x + 3x + 7; 7 x + 7 ;
3 2
⇒ prl = 2pr2 ⇒ l = 2r (1 Mark)
x
5 r 2 + h2 ]
5x3 – 7x + 2; 2 x 2 + 3 − ;  (1 Mark) ⇒ r 2 + h2 = ∵l
2r [=
x
(i) No. of expressions that are not polynomials = 3 ⇒ r2 + h2 = 4r2 [squaring both sides]
r 1
5 1 ⇒ h2 = 3r2 ⇒ h =3 r ⇒ = ⇒ r : h =
1: 3
[ x3 + 3x + 7;2 x + 3 − ; x + ] ½ Mark)
(½ h 3
x x
The ratio of the radius & height of the conical part
(ii) No. of expressions that are quadratic polynomial = 1
is 1: 3  (1 Mark)
[3x2 + 7x + 2] ½ Mark)

27. A D
22. From the given distribution;
Modal class is 12 – 16 2x – 1 3x 18 6x
∴ l = 12 ; f0 = 9 ; f1 = 17 ; f2 = 12 ; h = 4
f1 − f 0
∴ Mode = l + ×h  (1 Mark) B 2x + 2 C E 3x + 9 F
2 f1 − f 0 − f 2
Now, given DABC ∼ DDEF
17 − 9 8 8
12 +
= ×4 =
12 + × 4 = 12 + × 4 AB AC 2 x − 1 3x
2(17) − 9 − 12 34 − 21 13 ∴ =⇒ =  (1 Mark)
DE DF 18 6x
32
=12 + =12 + 2.46 = 14.46 [approx] (1 Mark) ⇒
2x − 1 1
= ⇒ 2x − 1 =
18
13
18 2 2
23. Out of the Syllabus ⇒ 2x – 1 = 9 ⇒ 2x = 10 ⇒ x = 5 (1 Mark)
2 2
cot α cosec α − 1 In DABC, AB = 2x – 1 = 2(5) – 1 = 10 – 1 = 9 cm
24. LHS =
1+ 1+
=
1 + cosec α 1 + cosecα AC = 3x = 3(5) = 15 cm
(cosec α − 1)(cosec α + 1) BC = 2x + 2 = 2(5) + 2 = 10 + 2 = 12 cm
= 1+
(cosec α + 1)  (1 Mark)
Now, in DDEF, DE = 18 cm, DF = 6x = 6(5) = 30 cm
= 1 + (cosec a – 1) = 1 + cosec a – 1
EF = 3x + 9 = 3(5)+9 = 15 + 9 = 24 cm (1 Mark)
= cosec a = RHS.
28. Q
Hence, LHS = RHS (1 Mark) B
OR
P A
LHS = tan4 q + tan2 q = tan2 q (tan2 q + 1) (1 Mark)
C
= (sec2 q – 1) (sec2 q)[ sec2 q – tan2 q = 1] R
= sec4 q – sec2 q = RHS. (1 Mark) Given, AQ & AR are two tangent from A to the circle.
BC is also a tangent at P.
25. The six faces of the die are;
We have to prove that
A A B C C C
1
AQ=( BC + CA + AB)  (1 Mark)
∴ No. of sample space = 6 2
1 1 1
(i) No. of times we can get A = 2 we have, AQ = (2= AQ) ( AQ + AQ
= ) ( AQ + AR ) 
2 2 2
2 1  (1 Mark)
P(getting A) = =  (1 Mark)
6 3
[ length of tangent from an external point of a circle
(ii) No. of times we can get C = 3 point are equal]
3 1 1 1
P(getting C) = =  (1 Mark) = ( AB + BQ + AC + CR )= ( AB + BP + AC + CP)
6 2 2 2
26. Let the radius of the hemisphere & radius of cone be r. [ BP = BQ & CP = CR length of tangent from an
And, let the height of the conical part be h. external point of a circle are equal]

CBSE 2020 (Outside DELHI) 217


1 1 32. Given sum of first m terms is n & sum of first n terms
= ( AB + ( BP + CP ) + AC )= ( AB + BC + AC )
2 2 is m.
1 Let the first term of the AP be a & the common
⇒ AQ= ( BC + CA + AB)  (1 Mark)
2 difference be d.
29. Out of the Syllabus ∴ According to question,
30. Given, equation are m
2x + y = 23 ...(i) ∴ Sm = n ⇒ [2a + (m − 1)d ] =
n
2
4x – y = 19 ...(ii) 2n
⇒ 2a + (m − 1)d = and Sn = m...(i)
Now, (i) + (ii) m
6x = 42 ⇒ x = 7 (1 Mark) n
[2a + (n − 1)d ] =
m
Putting x = 7 in (i), we get 2
2x + y = 23 ⇒ 2(7) + y = 23 2m
⇒ 2a + (n − 1)d =  ...(ii) (1 Mark)
⇒ y = 23 – 14 = 9 n
∴ x = 7 and y = 9 (1 Mark) Now, equation (i) – equation (ii), we get
Now,
2n 2m
5y – 2x = 5(9) – 2(7) = 45 – 14 = 31 ⇒ (m – 1)d – (n – 1)d = −
m n
and y − 2 = 9= 9 − 14 −5
−2 =  (1 Mark)
x 7 7 7 2n 2 − 2m 2 2(n 2 − m 2 )
⇒ d [ m − 1 − n + 1] = ⇒ d (m − n) =
nm nm
OR
2(n − m)(n + m) −2(n + m)
1 1 11 ( x + 7) − ( x + 4) 11 ⇒d = = (1 Mark)
− = ⇒ = nm(m − n) nm
x + 4 x + 7 30 ( x + 4)( x + 7) 30 
x + 7 − x − 4 11 3 11 −2(n + m)
⇒ = ⇒ = (1 Mark) Putting, d = in (i), we get
( x + 4)( x + 7) 30 ( x + 4)( x + 7) 30  nm

⇒ 3(30) = 11(x + 4) (x + 7) (−2)(n + m) 2n


2a + (m − 1) =
⇒ 90 = 11(x2 + 7x + 4x + 28) nm m
⇒ 90 = 11x2 + 121x + 308 2n 2(m − 1)(n + m)
⇒ 2a = +
⇒ 11x2 + 121x + 308 – 90 = 0 (1 Mark) m nm
⇒ 11x2 + 121x + 218 = 0
2n 2 + 2(mn + m 2 − n − m)
⇒ 2a =
−121 ± 14641 − 9592 −121 ± 5049 nm
⇒x= ⇒x= (1 Mark)
2(11) 22
 n 2 + mn + m 2 − n − m 
31. Given, P(x, y) is the midpoint of A(3, 4) & B(k, 6) ⇒ 2a =
2 
 mn 
3+ k 6+ 4  3 + k 10  3+ k 
∴P , =P ,  = P ,5  (1 Mark) Now, sum of first (m + n) terms is
 2 2   2 2  2 
We can say after comparing both values of P ( m + n)
Sm + n
= [ 2a + (m + n − 1)d ]
3+ k 2
∴x
= = ;y 5
2
Also, given line x + y – 10 = 0 (m + n)   n 2 + mn + m 2 − n − m   −2(m + n)  
= 2   + (m + n − 1)  
2   mn   nm 
3+ k 3+ k
∴ + 5 − 10 =0 ⇒ −5=0
2 2  (1 Mark)
(m + n)  2(n 2 + mn + m 2 − n − m) − 2(m + n − 1)(m + n) 
3+ k =  
⇒ = 5 ⇒ 3 + k = 10 ⇒ k = 10 – 3 ⇒ k =
7 2  mn 
2
The value of k is 7. (1 Mark)  2n 2 + 2mn + 2m 2 − 2n − 2m − 2(m 2 + mn − m + 
OR  
( m + n)  nm + n 2 − n) 
=
Out of the Syllabus 2  nm 

218
218 MATHS
 2n 2 + 2mn + 2m 2 − 2n − 2m − 2m 2 − 4nm + 2m  34. Given, 1 + sin2q = 3 sin q cos q
 
( m + n)  + 2n − 2n 2  1 + sin 2 θ
3sin θ cos θ
= ⇒ = [Dividing both side by cos2q]
2  nm  cos θ 2
cos 2 θ
 (1 Mark)
(m + n)  −2mn  1 sin 2 θ
3sin θ
= −( m + n)
= ⇒ + = ⇒ sec2q + tan2q = 3 tan q
2  nm  2 2
cos θ cos θ cos θ
−(m + n) Hence, proved.
⇒ S( m + n ) = (1 Mark) ⇒ (1 + tan2q) + tan2q = 3 tan q[ sec2q = 1 + tan2q]
⇒ 2 tan2q – 3 tan q + 1 = 0 (1 Mark)
OR
⇒ 2 tan2q – 2 tan q – tan q + 1 = 0
Let the first term of the AP be a and the common
⇒ 2 tan q [tan q – 1] – 1(tan q – 1) = 0
difference be d.
Now, given the middle term is 30 ⇒ (2 tan q – 1) (tan q – 1) = 0
th ⇒ 2tan q – 1 = 0 or tan q – 1 = 0
 11 + 1  th
∴ The middle term =   = 6 term  (1 Mark) 1 1
 2  ⇒ tan θ = or tan q = 1 ⇒ tan θ = or 1 (1 Mark)
2 2
[∴ there are 11 terms in all]
Hence, proved.
∴ t6 = 30
35.
⇒ a + (6 – 1)d = 30 ⇒ a + 5d = 30  ...(i) (1 Mark)
Class fi xi fixi
Now, sum of all 11 terms
11–13 3 12 36
11 11 11
S11= [2a + (11 − 1)d ] = [2a + (10)d ] = [ 2(a + 5d )] 13–15 6 14 84
2 2 2
15–17 9 16 144
= 11(a + 5d) = 11(30) = 330 [from (i)]
17–19 13 18 234
∴ The sum of all 11 terms is 330 (1 Mark)
19–21 f 20 20f
33. Let the speed of fast train be x. km/hr
21–23 5 22 110
Then, the speed of slow train = (x – 10)km/hr.
23–25 4 24 96
Given, The fast train takes 3 hours less than the slow
Σfi = 40 + f Σfixi = 704 + 20f
train to cover a journey of 600 km
Now, according to question, Now, given, Mean = 18 (2 Marks)
600 600 distance Σfi xi
− 3
= [ ∵ time = ] (1 Mark) ⇒ 18 ⇒ 704 + 20 f =
= 18
x − 10 x speed Σf i 40 + f  (1 Mark)
 1 1  x − ( x − 10)  ⇒ 704 + 20f = 720 + 18f
⇒ 600  − 3 ⇒ 200 
= =1
 x − 10 x   x( x − 10)  ⇒ 20f – 18f = 720 – 704
 x − x + 10  ⇒ 2f = 16 ⇒ f = 8  (1 Mark)
⇒ 200  2 =1
 x − 10 x 
OR
⇒ 2000 = x2 – 10x [by cross multiplication]
Out of the Syllabus
⇒ x – 10x – 2000 = 0
2

⇒ x2 – 50x + 40x – 2000 = 0 36. Out of the Syllabus


⇒ x(x – 50) + 40(x – 50) = 0 37. Let 5 be a rational number
⇒ (x – 50) (x + 40) = 0 That means, we can write,
⇒ x + 40 = 0 or x – 50 = 0
p
⇒ x = – 40 or x = 50 (1 Mark) 5= , where, p & q are co-primes & q ≠ 0 (1 Mark)
q
Since, speed can't be (–)ve, x ≠ –40
∴ x = 50 km/hr ⇒ 5q =
p
∴ The speed of fast train = 50 km/hr ⇒ 5q2 = p2 [squaring both sides]
Speed of slow train = (50 – 10) km/hr ⇒ 5 divides p...(i) (1 Mark)
= 40 km/hr (1 Mark) Again, let, p = 5a, for some integer a.

CBSE 2020 (Outside DELHI) 219


⇒ p2 = 25a2 [squaring both sides] Let the height of the building be h.
⇒ 5q2 = 25a2 [from (i)] (1 Mark) Now, in DBDC;
⇒ q = 5a ⇒ 5 divides q also.
2 2
CD
tan 60° =  (1 Mark)
∴ 5 is a common factor of p & q, which is not possible BC
as p, q are co-primes.
50 50
Hence, our assumption is wrong ⇒ 3= ⇒ BC = (1 Mark)
BC 3
∴ 5 is irrational. (1 Mark)
Again, in DABC;
38. Out of the Syllabus
AB
39. Out of the Syllabus tan 30° =
BC
40. D 
1 h 50 1
⇒ = ⇒ h= ×
A 3 50 3 3
3
50 m
50
h = = 16.67 (1 Mark)
3 
60° 30°
B C ∴ The height of the building is 16.67 m. (1 Mark)

220
220 MATHS
DELHI
2019 CBSE Solved Paper

Time allowed : 3 hours Maximum Marks : 80


GENERAL INSTRUCTIONS:
Read the following instructions carefully and strictly follow them:
(i) All questions are compulsory.
(ii) This question paper consists of 30 questions divided into four sections A, B, C and D.
(iii) Section A contains 6 questions of 1 mark each, Section B contains 6 questions of 2 marks each, Section C contains
10 questions of 3 marks each and Section D contains 8 questions of 4 marks each.
(iv) There is no overall choice. However, an internal choice has been provided in two questions of 1 mark, two questions
of 2 marks, four questions of 3 marks each and three questions of 4 marks each. You have to attempt only one of the
alternative in all such questions.
(v) Use of calculators is not permitted.

Section-A Section-B
1. Find the coordinates of a point A, where AB is a diameter 7. Find the value of k for which the following pair of
of the circle with centre (–2, 2) and B is the point with linear equations have infinitely many solutions. 2x + 3y = 7,
coordinates (3, 4). (1 Mark) (k + l)x + (2k – 1)y = 4k + 1. (2 Marks)
2. Find a rational number between 2 and 3 . [OS] (1 Mark) 8. A die is thrown once. Find the probability of getting a
3. How many two digits numbers are divisible by 3? (1 Mark) number which (i) is a prime number (ii) lies between
4. Find A if tan 2A = cot (A – 24°) [OS] (1 Mark) 2 and 6. (2 Marks)
OR 9. Find the ratio in which the segment joining the points (l, – 3)
and (4, 5) is divided by x-axis? Also find the coordinates
Find the value of (sin 33° + sin2 57°)
2
[OS]
of this point on x-axis. (2 Marks)
5. For what values of k, the roots of the equation x2 + 4x + k = 0
10. A game consists of tossing a coin 3 times and noting the
are real? (1 Mark)
OR outcome each time. If getting the same result in all the
tosses is a success, find the probability of losing the game.
Find the value of k for which the roots of the equation
(2 Marks)
3x2 – 10x + k = 0 are reciprocal of each other.
11. Which term of the AP 3, 15, 27, 39, .... will be 120 more
6. In Figure, DE || BC, AD = 1 cm and BD = 2 cm. What is the than its 21st term? (2 Marks)
ratio of the ar(∆ABC) to the ar(∆ADE)? [OS] (1 Mark)
OR
A If Sn , the sum of first n terms of an AP is given by
Sn = 3n2 – 4n, find the nth term.
12. Find the HCF of 1260 and 7344 using Euclid's algorithm.
D E  [OS] (2 Marks)
OR
Show that every positive odd integer is of the form (4q + l)
B C or (4q + 3), where q is some integer. [OS]
20. Find the point on y-axis which is equidistant from the
Section-C points (5, –2) and (–3, 2). (3 Marks)
13. The arithmetic mean of the following frequency OR
distribution is 53. Find the value of k. (3 Marks) The line segment joining the points A(2, 1) and B(5, –8) is
Class 0–20 20–40 40–60 60–80 80–100
trisected at the points P and Q such that P is nearer to A.
If P also lies on the line given by 2x – y + k = 0, find the
Frequency 12 15 32 k 13 value of k.
14. Find the area of the segment shown in Fig., if radius of the 21. A father’s age is three times the sum of the ages of his two
 22  children. After 5 years his age will be two times the sum of
circle is 21 cm and ∠AOB = 120°  Use π =  their ages. Find the present age of the father. (3 Marks)
 7 
(3 Marks) OR
1
A fraction becomes when 2 is subtracted from the
A B 3
21 1
cm 120 1 cm
o
numerator and it becomes when 1 is subtracted from
2 2
O the denominator. Find the fraction.
22. Prove that (sin θ + cosec θ)2 + (cos θ + sec θ)2 = 7 + tan2 θ
+ cot2 θ. (3 Marks)
15. In Figure, ∠ACB = 90° and CD ⊥ AB, prove that OR
CD2 = BD × AD. [OS] (3 Marks)
Prove that (I + cot A – cosec A) (l + tan A + sec A) = 2
C
Section-D
tan 2 A cosec 2 A 1
23. Prove that + =
tan A − 1 sec A − cosec A 1 − 2 cos 2 A
2 2 2
A B
D (4 Marks)
OR 24. The first term of an AP is 3, the last term is 83 and the sum
If P and Q are the points on side CA and CB respectively of all its terms is 903. Find the number of terms and the
of ∆ABC, right angled at C, prove that (AQ2 + BP2) common difference of the AP. (4 Marks)
= (AB2 + PQ2) [OS] 25. Construct a triangle ABC with side BC = 6 cm, ∠B = 45°,
16. In Fig., a circle is inscribed in a ∆ABC having sides ∠A = 105°. Then construct another triangle whose sides
BC = 8 cm, AB = 10 cm and AC = 12 cm. Find the lengths 3
are times the corresponding sides of the ∆ABC.
BL, CM and AN. (3 Marks) 4
 [OS] (4 Marks)
A
26. If the median of the following frequency distribution is
32.5. Find the values of f1 and f2· (4 Marks)
12
10 cm

cm

10–20

20–30

30–40

40–50

50–60

60–70
0–10

Class Total
N M
Frequency f1 5 9 12 f2 3 2 40
B L C OR
8 cm
The marks obtained by 100 students of a class in an
17. Water in a canal, 6 m wide and 1.5 m deep, is flowing with examination are given below.
a speed of 10 km/hour. How much area will it irrigate in
10–15

15–20

20–25

25–30

30–35

35–40

40–45

45–50

30 m; if 8 cm standing water is needed?  [OS] (3 Marks)


5–10
0–5

Marks
18. Prove that 2 is an irrational number. (3 Marks)
No. of
19. Find the value of k such that the polynomial x – (k + 6)x2 2 5 6 8 10 25 20 18 4 2
Students
+ 2(2k – l) has sum of its zeros equal to half of their product.
Draw ‘a less than’ type cumulative frequency curves
(3 Marks) (ogive). Hence find median. [OS]

222
222 MATHS
27. Prove that in a right angle triangle, the square of the OR
hypotenuse is equal the sum of squares of the other two A boat goes 30 km upstream and 44 km downstream in
sides. [OS] (4 Marks) 10 hours. In 13 hours, it can go 40 km upstream and 55 km
downstream. Determine the speed of the stream and that of
28. A bucket open at the top is in the form of a frustum of a the boat in still water. [OS]
cone with a capacity of 12308.8 cm3. The radii of the top
and bottom of circular ends of the bucket are 20 cm and 30. A man in a boat rowing away from a light house 100 m
12 cm respectively. Find the height of the bucket and also high takes 2 minutes to change the angle of elevation of the
top of the light house from 60° to 30°. Find the speed of the
the area of the metal sheet used in making it. (Use π = 3.14)
boat in metres per minute. [Use 3 = 1.732 ] (4 Marks)
 [OS] (4 Marks)
OR
7 Two poles of equal heights are standing opposite each other
29. Two water taps together can fill a tank in 1 hours. The
8 on either side of the road, which is 80 m wide. From a point
tap with longer diameter takes 2 hours less than the tap between them on the road, the angles of elevation of the top
with smaller one to fill the tank separately. Find the time of the poles are 60° and 30° respectively. Find the height
in which each tap can fill the tank separately. (4 Marks) of the poles and the distances of the point from the poles.

EXPLANATIONS

1. Consider, the centre of the circle be O(–2, 2) and the co- Hence, there are 30 two-digit numbers which is divisible
ordinates of A be (x, y) and B be (3, 4) by 3. ½ Mark)

By using mid-point formula, we get
4. Out of the Syllabus
5. Given, quadratic equation is x2 + 4x + k = 0
A On comparing with ax2 + bx + c = 0, we get a = 1, b = 4, c = k
(x, y) B (3, 4)
O Given, roots are real ½ Mark)

∴ D ≥ 0 ⇒ b – 4ac ≥ 0
2

 x+3 y +4
⇒ (4)2 – 4 × 1 × k ≥ 0 ⇒ 16 – 4k ≥ 0
 , = ( −2, 2 )
 2 2  ⇒ –4k ≥ –16 ⇒ k ≤ 4
x+3 Hence, for k ≤ 4, the roots of equation x2 + 4x + k = 0 are real.
∴ −2
=
2  ½ Mark)

⇒ x + 3 = – 4 ⇒ x = –4 – 3 = –7 OR
y+4
And =2⇒ y + 4 = 4 ⇒ y = 0 We have, equation 3x2 – 10x + k = 0
2
Given, roots are reciprocal of each other.
Hence, coordinates of A are (–7, 0) (1 Mark) 1
Consider, roots are a and
2. Out of the Syllabus α
3. Since, two-digits number which is divisible by 3 are Here, on comparing with ax2 + bx + c = 0, we get
12, 15, 18 ........ 99 a = 3, b = –10, c = k ½ Mark)

Here, clearly these numbers form an A.P series in which a c
= 12, d = 15 – 12 = 18 – 15 = 3, l = 99 ½ Mark)
(½ Product of the roots =
a
1 k k
We know, l = a +(n – 1)d ∴α × = ⇒1= ⇒ k = 3
α 3 3
∴ 99 = 12 + (n – 1) × 3 ⇒ 3(n – 1) = 87
Hence, value of k is 3. ½ Mark)

87
⇒ (n − 1) = = 29 ⇒ n = 29 + 1 = 30 6. Out of the Syllabus
3

CBSE 2019 (DELHI) 223


7. Given, system of linear equations are 10. We know, on tossing a coin 3 times, total possible outcomes
are given by,
2x + 3y = 7
S = {HHH, HHT, HTH, HTT, TTT, TTH, THT, THH} = 8
(k + 1)x + (2k – 1)y = 4k + 1
 ½ Mark)

Since, the given system of equation is of the form Let favourable outcomes to the event E be getting the same
a1x + b1y + c1 = 0 and a2x + b2y + c2 = 0 result in all the tosses.
Where, a1 = 2, b1 = 3, c1 = –7 So, favourable outcomes are {TTT, HHH} ½ Mark)

a2 = k + 1, b2 = 2k – 1, c2 = –(4k + 1) ½ Mark)
(½ ∴ Number of favourable outcomes = 2
for infinitely many solutions 2 1
∵ P( E =
) =
8 4
a1 b1 c1 1 3
= = ∴ P(losing the game) = 1 – P(E) =1 − =  (1 Mark)
a2 b2 c2 4 4
2 3 −7 11. Given, A.P series is 3, 15, 27, 39
∴ = = ½ Mark)

k + 1 2k − 1 −(4k + 1)  Here, first term (a) = 3
⇒ 2(2k – 1) = 3(k + 1) or 3(4k + 1) = 7(2k – 1) Common difference (d) = 15 – 3 = 27 – 15 = 12 (½ ½ Mark)
⇒ 4k – 2 = 3k + 3 or 12k + 3 = 14k – 7 We know, an = a + (n – 1)d
⇒ k = 5 or k = 5 ∴ a21 = a + 20d = 3 + 20 × 12 = 3 + 240 = 243 (½ ½ Mark)
Hence, value of k is 5. (11 Mark) Consider, an be the term which is 120 more than 21 term
st

8. (i) We know, when a die is thrown, total possible outcomes = 6 ∴ an = 120 + a21 = 120 + 243 = 363 ½ Mark)

 Prime numbers are 2, 3 and 5 Also, an = a + (n – 1)d
∴ Total number of prime numbers = 3 ∴ 3 + (n – 1) × 12 = 363 ⇒ 12 (n – 1) = 360
360
∴ P(getting a prime number) ⇒ (n − 1)= = 30 ⇒ n = 30 + 1 = 31
12
Total no.of favourableoutcomes 3 1 Hence, the 31th term of the A.P will be 120 more than the
= = =  (1 Mark)
Total no.of possibleoutcomes 6 2 21st term ½ Mark)

(ii) Since, numbers between 2 and 6 are 3, 4, and 5 OR
∴ Total number between 2 and 6 = 3 Given, Sn = 3n2 – 4n
3 1 we know, an = Sn – Sn – 1 ½ Mark)

∴ P(Getting a number between 2 and 6)= = (1 Mark)
6 2 = 3n2 – 4n – [3(n – 1)2 – 4(n – 1)]
9. Consider, A and B be (1, –3) and (4, 5) respectively. = 3n2 – 4n – [3(n2 – 2n + 1) – 4n + 4] ½ Mark)

Since, the line segment joining by these points A and B is = 3n2 – 4n – [3n2 – 6n + 3 – 4n + 4]
divided by x-axis so the co-ordinate of point of intersection = 3n2 – 4n – [3n2 – 10n + 7]
will be C(x, 0) = 3n2 – 4n – 3n2 + 10n – 7 = 6n – 7 ½ Mark)

Let C(x, 0) divides line-segment AB in l : 1 ratio Hence, nth term will be 6n – 7 ½ Mark)

∴ By using section formula 12. Out of the Syllabus
 mx + nx1 my2 + ny1 
( x, y ) =  2 ,  ½ Mark)
(½ 13.
 m+n m+n 
 4 × λ + 1 × 1 5 × λ + 1 × (−3)  Class Frequency Mid-value fixi
⇒ ( x,0) =  ,  ½ Mark)
(½ (fi) (xi)
 λ +1 λ +1 
5λ − 3 3 0-20 12 10 120
⇒ = 0 ⇒ 5l – 3 = 0 ⇒ λ =
λ +1 5 20-40 15 30 450
3 40-60 32 50 1600
4× +1
5 17
∴x
= = ½ Mark)
(½ 60-80 k 70 70k
3
+1 8 
5 80-100 13 90 1170
Hence, required ratio is 3 : 5 and coordinate of point C on Total Σfi = 72+k Σfixi = 3340
x-axis is  17 ,0   ½ Mark)

+ 70k
 8  (2 Marks)

224
224 MATHS
Given, mean = 53 15. Out of the Syllabus
Σfi xi 3340 + 70k 16. Given, a circle is inscribed in a DABC
∵ Mean
= =
Σf i 72 + k where AB = 10 cm, BC = 8 cm and AC = 12 cm
3340 + 70k As, the lengths of tangents drawn from an external point to
⇒ 53 = ½ Mark)

72 + k  a circle are equal. ½ Mark)

⇒ 53(72 + k) = 3340 + 70k ⇒ 3816 + 53k = 3340 + 70k ∴ AN = AM = x
476
⇒ 70k – 53k = 3816 – 3340 ⇒ 17k = 476 ⇒ k= = 28 BN = BL = z
17 A
Hence, value of k is 28. ½ Mark)

x x
14. Given, radius = 21 cm 10 cm 12 cm
Let θ = ∠AOB = 120° N M
z y
A B
21 B z L y C
cm 120 1 cm
o

2 8 cm  ½ Mark)

O CL = CM = y
Since, AB + BC + CA = 10 + 8 + 12 = 30
⇒ x + z + z + y + y + x = 30 ½ Mark)

Area of segment AYB = Area of sector OAYB 30
– Area of DAOB ½ Mark)
(½ ⇒ 2(x + y + z) = 30 ⇒ x + y + z = = 15cm ...(i)
2
θ 120° 22 ( )2  AB = 10 ⇒ x + z = 10 ...(ii)
Area of sector OAYB = × πr 2 = × × 21 And AC = 12 ⇒ x + y = 12 ...(iii)
360° 360° 7
1 22
And BC = y + z = 8 cm ...(iv)
= × × 21 × 21 = 22 × 21 = 462 cm2 ½ Mark)
(½ From equation (i) and (iv), we get
3 7
x = 7 cm ½ Mark)

Now, for area of DAOB, draw OP ⊥ AB from equation (i) and (ii), we get
Note that, OA = OB (Radii of the circle) y = 5 cm ½ Mark)

Therefore, by R.H.S congruence from equation (i) and (iii), we get
z = 3 cm ½ Mark)

DAPO ≅ DBPO  (½ ½ Mark)
Hence, BL = 3 cm, CM = 5 cm, AN = 7 cm
So, P is the mid-point of AB and ∠AOP = ∠BOP
P 17. Out of the Syllabus
1 A B
= × 120°= 60° 21
2 cm 60° 60° cm 18. Let us assume, to the contrary, that 2 is rational.
Consider, OP = x cm 21 So, we can find integers r and s(≠ 0) such that
O r
OP x 1 21 2=  ½ Mark)

From DOPA, = cos 60° ⇒ = ⇒ x= cm s
OA 21 2 2 Suppose r and s have a common factor other than 1. Then,
a
AP 3 AP 21 3 we divide by common factor to get 2 = ,
Also, sin 60° = ⇒ = ⇒ AP = cm b
OA 2 21 2 where a and b are co-prime
∴ AB = 2 AP = 21 3 cm  ½ Mark)
(½ So, b 2 = a  ½ Mark)

1 1 21 Squaring on both sides and rearranging, we get 2b2 = a2.
∴ Area of DOAB = × AB × OP = × 21 3 × Therefore, 2 divides a2.
2 2 2
Now, it follows that 2 divides a. ½ Mark)

441 3 2 So, we can write a = 2c for some integer c.
= cm  ½ Mark)

4 Substituting for a, we get 2b2 = 4c2, that is b2 = 2c2
 441  2
=  462 −
∴ Area of segment AYB 3  cm This means that 2 divides b2, and so 2 divides b.
 4  Therefore, a and b have at least 2 as a common factor
 1848 − 441 3  2 21 ½ Mark)

 cm =( 88 − 21 3 ) cm ≅ 271.04 cm
2 2 
=
 4  4 But this contradicts the fact that a and b have no common
½ Mark)
(½ factors other than 1

CBSE 2019 (DELHI) 225


This contradiction has arisen because of our incorrect 21. Consider, age of first son be x and of second son be y
assumption that 2 is rational  ½ Mark)
(½  ½ Mark)

Then, the father's present age = 3(x + y) ½ Mark)

So, we conclude that 2 is irrational. ½ Mark)

After 5 years,
19. Given, polynomial is x2 – (k + 6)x + 2(2k – 1) Age of father = 3(x + y) + 5 ½ Mark)

On comparing with ax2 + bx + c, we get Sum of ages of children = (x + y + 5 + 5)
a = 1, b = –(k + 6) and c = 2(2k – 1) = (x + y + 10) years ½ Mark)

Let the zeros of the polynomial be a and b (11 Mark) According to question, we have
Given, sum of the zeros is equal to half to their product. 3(x + y) + 5 = 2(x + y + 10) ½ Mark)

⇒ 3x + 3y + 5 = 2x + 2y + 20
αβ −b c
∴α + β = ⇒ = ½ Mark)
(½ ⇒ 3x – 2x + 3y – 2y = 20 – 5 ⇒ x + y = 15
2 a 2a  ∴ Father's present age = 3(x + y) = 45 years ½ Mark)

−(−(k + 6)) 2(2k − 1) OR
⇒ =  ½ Mark)

1 2 ×1 x
⇒ k + 6 = 2k – 1 ⇒ 2k – k = 6 + 1 ⇒ k = 7 Consider, the fraction be  ½ Mark)

y
Hence, value of k is 7. (1 Mark) 1
20. Given, points be A(5, –2) and B(–3, 2) Given, fraction becomes when 2 is subtracted from
numerator. 3
Consider, coordinates of the point on y-axis be T(0, y)
 ½ Mark)
(½ x−2 1
=  ½ Mark)

Since, T is equidistant from A(5, –2) and B(–3, 2) y 3
Then, AT = BT ½ Mark)
(½ ⇒ 3x – 6 = y ⇒ 3x – y – 6 = 0 ...(i)
⇒ AT2 = BT2 1
Given, also fraction becomes when 1 is subtracted from
By distence formula we have, denominator. 2
⇒ (5 – 0)2 + (–2 – y)2 = (–3 – 0)2 + (2 – y)2 ½ Mark)

x 1
⇒ 25 + 4 + y2 + 4y = 9 + 4 + y2 – 4y ½ Mark)
(½ =  ½ Mark)

y −1 2
−16 ⇒ 2x = y – 1⇒ 2x – y + 1 = 0
⇒ 8y = –16 ⇒ y = = −2  ½ Mark)
(½ ...(ii)
8
½ Mark)
Hence, the coordinates of required point is (0, –2) (½ From equation (i) and (ii)
OR 3x – y – 6 = 0
Let the points A(2, 1) and B(5, –8) is trisected at the points 2x – y + 1 = 0
– + –
P(x, y) and Q(a, b) such that P is nearer to A. –––––––––––––––
x–7=0
So, P divides AB internally in the ratio 1 : 2
2x – y + k = 0 ⇒ x = 7 ½ Mark)

From equation (i) 3 × 7 – y – 6 = 0
Q
⇒ 21 – y – 6 = 0 ⇒ y = 15 ½ Mark)

(2, 1) A 1 2 B (5, –8)
P 7
Hence, fraction is . ½ Mark)

Using section formula, coordinates of P are (11 Mark) 15
 m x + m2 x1 m1 y2 + m2 y1  22. L.H.S = (sin q + cosec q)2 + (cos q + sec q)2
( x, y ) =  1 2 ,  ½ Mark)

 m1 + m2 m1 + m2  = sin2 q + cosec2 q + 2 sin q cosec q + cos2 q + sec2 q
+ 2 cos q sec q (1 Mark)
 1× 5 + 2 × 2 1× −8 + 2 × 1 
= ,  1 1
 1+ 2 1+ 2  [⸪ coses2 q = 1 + cot2 q, cosec q = and sec q = ]
sin θ cos θ
 9 −6 
( x, y=)  , =  ( 3, −2 ) 1
3 3  = (sin2 q + cos2 q) + (1 + cot2 q) + 2sin θ× +
Hence, P ≡ (3, –2) ½ Mark)
(½ sin θ
 P also lies on the line given by 2x – y + k = 0 1
 (1 + tan2 q) + 2 cos θ× (1 Mark)
 (3, –2) satisfies the equation 2x – y + k = 0 cos θ
\ 2 × 3 – (–2) + k = 0 ½ Mark)
(½  sin2 q + cos q = 1
⇒ 6 + 2 + k = 0 ⇒ k = –8 = 1 + (1 + cot2q) + 2 + (1 + tan2q) + 2
Hence, value of k is –8. ½ Mark)
(½ = 7 + tan2q + cot2q = R.H.S (1 Mark)

226
226 MATHS
Hence, L.H.S. = R.H.S. 24. Given, for an A.P a = 3, l = 83, Sn = 903 ½ Mark)

n
Hence, proved. ∵=Sn (a + l )  ½ Mark)

OR 2
n
L.H.S = (1 + cotA – cosecA)(1 + tanA + secA) ⇒ 903 = ( 3 + 83) ⇒ 903 × 2 = n(86)
2
 cos A 1   sin A 1 
=1 + −  1 + +  ½ Mark)
(½ 1806 ½ Mark)

 sin A sin A   cos A cos A  n
⇒= = 21 
86
 sin A + cos A − 1  cos A + sin A − 1   ½ Mark)
(½  an = a + (n – 1)d ½ Mark)

=  
 sin A  cos A  ⇒ 83 = 3 + (21 – 1)d ½ Mark)

 (a + b) (a – b) = a – b
2 2
80
⇒ 83 – 3 = 20d ⇒ 20d = 80 ⇒ d = = 4 ½ Mark)

( sin A + cos A )2 − (1) 2 20
=  ½ Mark)
(½ Hence, number of terms are 21 and common difference of
sin A cos A
the A.P. is 4. (1 Mark)
sin 2A + cos 2A + 2sin A cos A − 1
=  ½ Mark)
(½ 25. Out of the Syllabus
sin A cos A
26.
 sin2 A + cos2 A = 1
1 + 2sin A cos A − 1 2sin A cos A Class Frequency Cumulative frequency
= =  ½ Mark)

sin A cos A sin A cos A 0 – 10 f1 f1
= 2 = R.H.S ½ Mark)
(½ 10 – 20 5 f1 + 5
Hence, L.H.S. = R.H.S. 20 – 30 9 f1 + 14
Hence, proved. 30 – 40 12 f1 + 26
tan 2 A cosec 2 A 40 – 50 f2 f1 + f2 + 26
23. L.H.S = +
tan A − 1 sec A − cosec 2 A
2 2 50 – 60 3 f1 + f2 + 29
sin A 1 1 60 – 70 2 f1 + f2 + 31
∵ tan A
= = , cosecA = ,sec A ½ Mark)
 (½
cos A sin A cos A Total N = 40
2
sin A 1
(2 Marks)
2
= cos A + sin 2 A  ½ Mark)
(½ Total frequency = 40
2
sin A 1 1
−1 − ⇒ f1 + f2 + 31 = 40 ⇒ f1 + f2 = 9 ½ Mark)
...(i) (½
cos 2 A cos 2 A sin 2 A
N 40
sin 2 A 1 ∵ = = 20
2 2 2 2
= cos A + sin A  ½ Mark)
(½ Median = 32.5, which lies in the class interval (30-40)
sin A − cos A sin A − cos 2 A
2 2 2
∴ Median class = 30 – 40
cos 2 A sin 2 A cos 2 A Here, l = 30, f = 12, C.f = f1 + 14, h = 10
sin 2 A cos 2 A
= +  ½ Mark)
(½ N 
sin 2 A − cos 2 A sin 2 A − cos 2 A  − C. f 
We know, Median = l +  2 ×h  ½ Mark)

sin 2 A + cos 2 A  f 
=  ½ Mark)

sin 2 A − cos 2 A  20 − ( f1 + 14 ) 
1 ⇒ 32.5 =30 +  ×h  ½ Mark)

= sin 2 A + cos 2 A 1 
∵= ½ Mark)
(½  12 
sin 2 A − cos 2 A
Since, sin2A = 1 – cos2A  6 − f1  5
⇒ 32.5 =30 +   × 5 ⇒ 32.5 − 30 = ( 6 − f1 )
1  6  6
=  ½ Mark)

1 − cos A − cos 2 A
2 2.5 × 6
⇒ = 6 − f1 ⇒ 6 – f1 = 3 ⇒ f1 = 3
1 5
= = R.H.S  ½ Mark)
(½ From equation (i), we get
1 − 2 cos 2 A
Hence, L.H.S. = R.H.S. f2 = 9 – 3 = 6
Hence, proved. ∴ f1 = 3, f2 = 6 ½ Mark)

CBSE 2019 (DELHI) 227


OR
⇒ x+ y = 100 3 = ⇒ y 100 3 − x ...(ii) (½
½ Mark)
Out of the Syllabus From equation (i) and (ii)
27. Out of the Syllabus 100
= 100 3 − x  ½ Mark)

28. Out of the Syllabus 3
100 1 100 × 2 200
7 15 x= 100 3 − = 100 3= − =
29. Given two tads can fill a tank in 1 = hours 3 3 3 3
8 8
∴ x = 115.48 m ½ Mark)

1 8 ½ Mark)
 Time taken to cover 115.48 m = 2 min (Given) (½
Amount of water filled by two taps in 1 hour is = 
15 15
115.48
8 ∴ Speed of=
boat = 57.74 m/min  ½ Mark)

2
 (1 Mark)
OR
Let the time taken by longer diameter tap to fill the tank Let two poles be AB and CD
be x hrs the time taken by other tap to fill the tank be y hrs
So, length of pole be AB and CD
from the given condition
Given, length of road BC = 80 m ½ Mark)

x=y–2
A D
Amount of water filled by longer diameter tap in 1 hour is
1 1
= (1 Mark)
x y−2

Amount of water filled by longer diameter tap in 1 hour
1 60° 30°
is
y B P C
80 m  ½ Mark)

1 1 8 3
⇒ + = ⇒ 4 y 2 − 23 y + 15 = 0 ⇒ y = ,5 Let point P be on the road between poles.
y − 2 y 15 4 
Poles are perpendicular to ground.
 (1 Mark) ∠ABP = 90° and ∠DCP = 90° ½ Mark)

5 5 In DDPC,
⇒x= − ,3 but x =− is not possible
4 4 CD 1 CD CP
tan P = ⇒ = ⇒ CD = ½ Mark)
...(i) (½
So x = 3, y = 5 CP 3 CP 3
The time taken by longer diameter tap to fill the tank be In DAPB,
3 hrs the time taken by other tap to fill the tank be 5 hrs
AB AB
 (1 Mark) tan P = ⇒ tan 60° = ⇒ AB = 3PB
OR PB PB
Out of the Syllabus ∴ CD
= 3PB= [∵ AB CD] ...(ii) (½
½ Mark)
30. Let AB be the light house, C and D be the initial and final From equation (i) and (ii)
positions of the boat respectively, such that CD = x m and CP
BC = y m ½ Mark)
(½ = 3PB ⇒ CP = 3 BP
3
Now, in DABC, A Now, BC = BP + CP
AB 100 ⇒ 80 = BP + CP ⇒ 80 = BP + 3BP
tan 60° = ⇒ 3=
BC y 80
⇒ 80 = 4BP ⇒ BP = = 20  ½ Mark)

100 m

100 4
y= ½ Mark)
...(i) (½ Now,
3 CP = BC – BP = 80 – 20 = 60 m
In DABD, 30° 60°
B
From equation (ii),
D x C y
AB CD = 3BP = 3 × 20 = 20 3 m  ½ Mark)

tan 30° =  ½ Mark)

BD Hence, length of the pole = 20 3 m and distance of point
1 100 P from pole CD and AB are 60 m and 20 m respectively.
=
3 x+ y  ½ Mark)

228
228 MATHS
Outside DELHI
2019 CBSE Solved Paper

Time allowed : 3 hours Maximum Marks : 80


GENERAL INSTRUCTIONS:
Read the following instructions very carefully and strictly follow them:
(i) All questions are compulsory.
(ii) The question paper consists of 30 questions divided into four sections – A, B, C and D.
(iii) Section A contains 6 questions of 1 mark each. Section B contains 6 questions of 2 marks each, Section C contains
10 questions of 3 marks each and Section D contains 8 questions of 4 marks each.
(iv) There is no overall choice. However, an internal choice has been provided in two questions of 1 mark each, two
questions of 2 marks each, four questions of 3 marks each and three questions of 4 marks each. You have to attempt
only one of the alternatives in all such questions.
(v) Use of calculators is not permitted.

4. Find the nature of roots of the quadratic equation


Section-A 2x2 – 4x + 3 = 0. (1 Mark)
1. Write the coordinates of a point P on x-axis which is 5. If HCF (336, 54) = 6, find LCM (336, 54). (1 Mark)
equidistant from the points A(–2, 0) and B(6, 0).(1 Mark) 1 1
6. Find the 21st term of the A.P. −4 , −3, −1 ,...  (1 Mark)
2. Evaluate: sin2 60° + 2 tan 45° – cos2 30° (1 Mark) 2 2
OR
3 Section-B
If sin A = , calculate sec A.
4
3. In Figure, ABC is an isosceles triangle right angled at C 7. For what value of k, will the following pair of equations
with AC = 4 cm. Find the length of AB. [OS] (1 Mark) have infinitely many solutions:
A 2x + 3y = 7 and (k + 2) x – 3(1 – k)y = 5k + 1. (2 Marks)
8. Find the mode of the following frequency distribution:
(2 Marks)

Class Interval 25–30 30–35 35–40 40–45 45–50 50–55


C B
OR Frequency 25 34 50 42 38 14
In Figure, DE || BC. Find the length of side AD, given that
AE = 1.8 cm, BD = 7.2 cm and CE = 5.4 cm. 9. The larger of two supplementary angles exceeds the
A smaller by 18°. Find the angles. (2 Marks)
1.8 cm
D E OR
7.2 cm Sumit is 3 times as old as his son. Five years later, he shall
5.4 cm
B be two and a half times as old as his son. How old is Sumit
C at present?
10. The probability of selecting a blue marble at random from 19. In Figure, a square OABC is inscribed in a quadrant
1 OPBQ. If OA = 15 cm, find the area of the shaded region.
a jar that contains only blue, black and green marbles is .
5 (Use π = 3.14) [OS] (3 Marks)
The probability of selecting a black marble at random B
1
from the same jar is . If the jar contains 11 green marbles, Q P
4
find the total number of marbles in the jar. (2 Marks)
C A
11. Find a relation between x and y if the points A(x, y),
B(–4, 6) and C(–2, 3) are collinear. [OS] (2 Marks) cm
15
OR O
Find the area of a triangle whose vertices are given as OR
(1, –1) (–4, 6) and (–3, –5). [OS]
In Figure, ABCD is a square with side 2 2 cm and
12. Write the smallest number which is divisible by both 306 inscribed in a circle. Find the area of the shaded region.
and 657. (2 Marks) (Use π = 3·14) [OS]
D
Section-C
13. Point A lies on the line segment XY joining X(6, –6) and
XA 2 A C
Y(–4, –1) in such a way that = . If point A also lies
XY 5
on the line 3x + k (y + 1) = 0, find the value of k. (3 Marks)
14. Solve for x: x2 + 5x – (a2 + a – 6) = 0 (3 Marks) B

3 20. A solid is in the form of a cylinder with hemispherical ends.


15. Find A and B if sin (A + 2B) = and cos (A + 4B) = 0, The total height of the solid is 20 cm and the diameter of
where A and B are acute angles.2 (3 Marks) the cylinder is 7 cm. Find the total volume of the solid.
16. Prove that 2 + 5 3 is an irrational number, given that 3 22
(Use π = ) (3 Marks)
is an irrational number. (3 Marks) 7
OR 21. The marks obtained by 100 students in an examination are
given below: (3 Marks)
Using Euclid’s Algorithm, find the HCF of 2048 and 960.
 [OS]
30–35

35–40

40–45

45–50

50–55

55–60

60–65
17. Two right triangles ABC and DBC are drawn on the same Marks
hypotenuse BC and on the same side of BC. If AC and BD
intersect at P, prove that AP × PC = BP × DP. [OS] (3 Marks) No. of Students 14 16 28 23 18 8 3
OR
22. For what value of k, is the polynomial f(x) = 3x4 – 9x3 + x2
Diagonals of a trapezium PQRS intersect each other at the + 15x + k completely divisible by 3x2 – 5?
point O, PQ || RS and PQ = 3RS. Find the ratio of the areas  [OS] (3 Marks)
of triangles POQ and ROS. [OS]
OR
18. In Figure, PQ and RS are two parallel tangents to a circle 11 2
2
with centre O and another tangent AB with point of contact Find the zeroes of the quadratic polynomial 7 y − y−
3 3
C intersecting PQ at A and RS at B. Prove that ∠AOB = 90o.
 (3 Marks) and verify the relationship between the zeroes and the
P D A Q coefficients.

Section-D
O
C 23. Prove that the ratio of the areas of two similar triangles
is equal to the ratio of the squares on their corresponding
R E B S sides. [OS] (4 Marks)

230
230 MATHS
24. Two poles of equal heights are standing opposite to each sin θ sin θ
other on either side of the road which is 80 m wide. From Prove that: = 2+
cot θ + cosecθ cot θ − cosecθ
a point P between them on the road, the angle of elevation
of the top of a pole is 60o and the angle of depression from 28. Change the following data into ‘less than type’ distribution
the top of the other pole of point P is 30o. Find the heights and draw its ogive: [OS] (4 Marks)
of the poles and the distance of the point P from the poles.

90–100
(4 Marks)

30–40

40–50

50–60

60–70

70–80

80–90
Class Interval
25. The total cost of a certain length of a piece of cloth is
` 200. If the piece was 5 m longer and each metre of cloth Frequency 7 5 8 10 6 6 8
costs ` 2 less, the cost of the piece would have remained
unchanged. How long is the piece and what is its original 29. Construct an equilateral ∆ABC with each side 5 cm. Then
rate per metre? (4 Marks) 2
construct another triangle whose sides are times the
26. Which term of the Arithmetic Progression –7, –12, –17, 3
–22, ... will be –82 ? Is –100 any term of the A.P.? Give corresponding sides of ∆ABC. [OS] (4 Marks)
reason for your answer. (4 Marks) OR
OR Draw two concentric circles of radii 2 cm and 5 cm. Take a
How many terms of the Arithmetic Progression 45, 39, point P on the outer circle and construct a pair of tangents
33, ... must be taken so that their sum is 180? Explain the PA and PB to the smaller circle. Measure PA. [OS]
double answer. 30. A solid iron pole consists of a cylinder of height 220 cm
tan θ cot θ and base diameter 24 cm, which is surmounted by another
27. Prove that: + 1 sec θcosecθ
=+
1 − cot θ 1 − tan θ cylinder of height 60 cm and radius 8 cm. Find the mass of
(4 Marks) the pole, given that 1 cm3 of iron has approximately 8 gm
OR mass. (Use π = 3·14) [OS] (4 Marks)

EXPLANATIONS

1. Let the coordinates of P be (x, 0) 3 3k


Given, sin A= =
Given, A(– 2, 0) and B(6, 0) 4 4k
Now, according to question So, Perpendicular = 3k, Hypotenuse = 4k
PA = PB
(Hypotenuse)2 = (Perpendicular)2 + (Base)2
2 2 2 2
⇒ ( x + 2) + (0 − 0) = ( x − 6) + (0 – 0)  (½
½ Mark) ⇒ (4k)2 = (3k)2 + (Base)2
Take square both side
⇒ (Base)2 = 7k2 ⇒ Base = 7 k ½ Mark)

⇒ x2 + 4x + 4 = x2 – 12x + 36 ⇒ 16x = 32 ⇒ x = 2
Hypotenuse 4k 4
Hence, the coordinate of P is (2, 0) ½ Mark)
(½ Now, sec
= A = =  ½ Mark)

Base 7k 7
2. We know,
3. Out of the Syllabus
3 3
sin 60°
= , tan
= 45° 1, cos
= 30° ½ Mark)
(½ OR
2 2 
Given, DE || BC
Now, sin2 60° + 2 tan2 45° – cos2 30°
2 2 AD AE
 3  3 3 3 = (By thales theorem)
=   + 2 × 1 −   = + 2 − = 2 ½ Mark)
(½ DB EC ½ Mark)

 2   2  4 4
AD 1.8
⇒ = = 2.4 cm ½ Mark)

OR 7.2 5.4

CBSE 2019 (Outside DELHI) 231


4. Given, 2x2 – 4x + 3 = 0 9. Let the two angler be x° and y°
a = 2, b = –4, c = 3 Now, we know
Now, sum of supplementary angle is 180°
b2 – 4ac = (–4)2 – 4×2×3 = 16 – 24 = – 8 < 0 ½ Mark)
(½ so, x° + y° = 180° ⇒ x = 180 – y ...(i)
For D < 0, roots of given quadratic equation will be Also given, x – y = 18 ...(ii) (1 Mark)
imaginary. ½ Mark)
(½ put the value of x in equation (ii)
5. Given, HCF (336, 54) = 6 180 – y – y = 18 ⇒ 2y = 162 ⇒ y = 81° ½ Mark)

we know, put y in equation (i), we get
LCM × HCF = Ist number × IInd number ½ Mark)
(½ x = 180 – 81 = 99° ½ Mark)

336 × 54
LCM × 6 = 336 × 54 ⇒ LCM
= = 336 × 9 OR
6
LCM = 3024 ½ Mark)
(½ Let the age of sumit is x years and his son’s age is y years.
6. Given, Now, according to question
1 1 x = 3y ...(i)
−4 , −3, −1 ,....... After 5 years,
2 2
1 9  9 3 1 5
a= −4 = − , d =−3 −  −  = ½ Mark)
(½ x+5= 2 (y + 5) ⇒ x + 5 = (y + 5)
2 2  2 2  2 2
we know, an = a + (n – 1)d ⇒ 2x + 10 = 5y + 25 ...(ii) (1 Mark)
∴ a21 = a + 20d Put the value of x in equation (ii)
9 3 51 1 ⇒ 2(3y) + 10 = 5y + 25 ⇒ 6y – 5y = 25 – 10
⇒ a21 =− + 20 × = ⇒ a21 = 25 ½ Mark)

2 2 2 2 ⇒ y = 15  ½ Mark)

and x = 3y = 3 × 15 = 45
7. Given, 2x + 3y = 7 and (k + 2)x – 3(1 – k)y = 5k + 1
compare with a1x + b1y + c1 = 0 and a2x + b2y + c2 = 0 Hence, sumit’s age = 45 years and his son’s age = 15 years.
 ½ Mark)

a1 = 2, b1 = 3, c1 = –7, a2 = k + 2, b2 = –3 + 3k
10. Let the total number of marbles be x.
and c2 = –(5k + 1) (1 Mark)
1 1
Now for infinite many solution P(blue marble) = , P(black marbles) =
5 4
a1 b1 c1 = 2 3 −7 11
= = ⇒ k + 2 −= ½ Mark)
3(1 − k ) −(5k + 1)  (½
No. of green marbles = 11, P(green marbles) =
a2 b2 c2 x
Now, we know
Now,
Total probability = 1 (1 Mark)
2 1
= ⇒ 2k – 2 = k + 2 ⇒ k = 4 ½ Mark)
(½ P(blue) + P(black) + P(green) = 1
k + 2 k −1
1 1 11 4 x + 5 x + 220
8. + + = 1⇒ =1 ½ Mark)

5 4 x 20 x 
Class Interval 25–30 30–35 35–40 40–45 45–50 50–55
⇒ 9x + 220 = 20x ⇒ 11x = 220
Frequency 25 34 50 42 38 14
⇒ x = 20 ½ Mark)

f0 f1 f2 11. Out of the Syllabus
Highest frequency = 50, modal class = 35 – 40 (1 Mark) 12. Factors of 306 = 2 × 3 × 3 × 17
l = 35, f0 = 34, f1 = 50, f2 = 42, h = 5
Factors of 657 = 3 × 3 × 73 (1 Mark)
f1 − f 0 Now, LCM (306, 657) = 2 × 3 × 3 × 17 × 73
Mode = l + ×h
2 f1 − f 0 − f 2 = 306 × 73 = 22338
(50 − 34) × 5 16 × 5 Hence, 22,338 is the smallest number which is divisible by
35 +
= 35 +
= = 38.33 (1 Mark)
2 × 50 − 34 − 42 24 both 306 and 657. (1 Mark)

232
232 MATHS
13. Let point A(x, y) which lies on line joining X and Y such A + 2B = 60° ⇒ A = 60° – 2B  ...(i)
that: and cos (A + 4B) = 0
XA 2 5 cos (A + 4B) = cos 90° ⇒ A + 4B = 90° ...(ii) (1 Mark)
= ⇒ XA + AY = XY ⇒ XA + AY = XA
XY 5 2 Put A in equation (ii)
5 3 XA 2 ⇒ 60° – 2B + 4B = 90° ⇒ 2B = 30° ⇒ B = 15°(½ ½ Mark)
AY = XA − XA= XA or =  (1 Mark)
2 2 AY 3 Put B in equation (i), we get
Now, by section formula A = 60° – 2 × 15° ⇒ A = 30° ½ Mark)

 mx + nx1 my2 + ny1  p
A( x, y ) =  2 , 16. Let 2 + 5 3 is a rational number then, 2 + 5 3 =
 m+n m + n  q
(½ ½ Mark)
 2(−4) + 3 × 6 2(−1) + 3 × (−6) 
⇒ A( x, y ) =  , 
 2+3 2+3  p p − 2q p − 2q
⇒ 5 3= −2 = ⇒ 3=  (1 Mark)
q q 5q
 10 20  p − 2q
⇒ A( x=
, y)  , − 
 5 5  Now, p and q are integer and is a rational number
5q
but 3 is a irrational number.
⇒ A(x, y) = (2, –4) (1 Mark)
So, it creates a contradiction (1 Mark)
Now, (2, –4) lies on line 3x + k(y + 1) = 0
Hence, ( 2 + 5 3 ) is a irrational number. ½ Mark)

then 3 × 2 + k(–4 + 1) = 0
⇒ 6 – 3k = 0 ⇒ 3k = 6 ⇒ k = 2  (1 Mark) OR
14. x2 + 5x – (a2 + a – 6) = 0 Out of the Syllabus
we know, for px2 + qx + r = 0 17. Out of the Syllabus
18. Join O and C
−b ± b 2 − 4ac ½ Mark)

x= P D A Q
2a

a = 1, b = 5, c = –(a2 + a – 6) ½ Mark)

O
Now,
C
−5 ± 52 − 4 × 1× −(a 2 + a − 6)
x= R E B S
2 ×1
Now, In ∆ADO & ∆ACO,
−5 ± 25 + 4a 2 + 4a − 24 −5 ± 4a 2 + 4a + 1
⇒x= ⇒ x= AO = AC (tangent drawn from same external point)
2 2
AO = AO (common)
2
−5 ± (2a + 1) −5 ± (2a + 1) DO = CO (radius)
⇒ x= ⇒ x=
2 2  (1 Mark) ∆ADO ≅ ∆ACO (SSS cong.) (1 Mark)
−5 + 2a + 1 2a − 4 ∠AOD = ∠AOC (By CPCT)  ...(i)
⇒ x= = = a − 2 (Take ‘+’)
2 2 Similarly, ∠BOE = ∠BOC  ...(ii)
−5 − 2a − 1 −2a − 6 Also given, PQ || RS
⇒ x= = =−a − 3 = –(a +3) (Take ‘–’)
2 2 DE is diameter hence it is straingt line  (1 Mark)
x = (a – 2), x = –(a + 3) (1 Mark) ∠AOD + ∠AOC + ∠BOC + ∠BOE = 180°
3 2(∠AOC + ∠BOC) = 180°
15. Given, sin (A + 2B) = and cos (A + 4B) = 0
2 ∠AOB = 90° Proved (1 Mark)
3
we know, sin 60° = , cos 90° = 0 ½ Mark)
(½ 19. Out of the Syllabus
2
Now, 20. Total volume = Volume of cylinder + Volume of 2
3 hemispherical
sin (A + 2B) = ⇒ sin (A + 2B) = sin 60° ½ Mark)

2 Height of cylinder = 20 – 7 = 13 (1 Mark)

CBSE 2019 (Outside DELHI) 233


4 3 Let AB and CD be two poles and P be a point of road.
Total volume = πR2h +πR  (1 Mark) B D
3
22 7 7  4 22 7 7 7 
= × × × 13 +  × × × × 
7 2 2 3 7 2 2 2
= 500.5 + 179.6 = 680.66 cm3 (1 Mark)
60° 30°
21. A C
P
Marks No. of Students (f) xi fixi In ∆APB,

30–35 14 32.5 455 AB AB AB


tan 60
= ° 3
⇒ = ⇒ AP = ...(i) (1 Mark)
AP AP 3
35–40 16 37.5 600
In ∆PCD,
40–45 28 42.5 1190
CD AB 1 AB
tan 30
=° = = (1 Mark)
PC PC ⇒ 3 PC 
45–50 23 47.5 1092.5

50–55 18 52.5 945


PC = 3 AB  ...(ii)
55–60 8 57.5 460 Adding equation (i) and (ii), we get
60–65 3 62.5 187.5 AB AB + 3 AB
AP + PC = + 3 AB ⇒ AC =
∑ fi = 100 ∑ fi xi = 4930 3 3
(2 Marks) 4 AB 4 AB
⇒ AC = ⇒ 80 =  (1 Mark)
∑ fixi = 4930, ∑ fi = 110 3 3

mean
Now, =

=
fxi i 4930
= 44.82 (1 Mark)
⇒ AB = 20 3 = 34.64 m (1 Mark)
∑f i 110
25. Let the length of piece be x m then rate per metre =
200
x
22. Out of the Syllabus New length = (x + 5) m
OR 200
New rate per metre =  (1 Mark)
11 2 x+5
Given, 7 y 2 − y− 
3 3 Now, according to question
2 1 (21 y 2 − 14 y + 3 y − 2)
21 y 2 − 11 y −= 200 200 200 x + 1000 − 200 x
= 2  (1 Mark)
= − = 2 ⇒
3 3 x x+5 x( x + 5)
1 1 ⇒ 1000 = 2x(x + 5) ⇒ 2x2 + 10x – 1000 = 0
= {7 y (3 y − 2) + 1(3 y − 2)} = (7 y + 1)(3 y − 2) (1 Mark)
3 3 ⇒ x2 + 5x – 500 = 0 ⇒ x2 + 25x – 20x – 500 = 0
1 2 ⇒ x(x + 25) – 20(x + 25) = 0 ⇒ (x+ 25) (x – 20) = 0
The zeroes of the given polynomial are − ,  ½ Mark)

7 3
b ⇒ x = –25 or x = 20 ½ Mark)

Sum of roots = −  ½ Mark)

a length cannot be negative
11
− 200
1 2 −3 + 14 11 11 11
− + =− 3 ⇒ = ⇒ =  ½ Mark)
(½ So, x = 20 and rate per metre = ` = ` 10 (1 Mark)
7 3 7 21 21 21 21 20
c 26. Given sequence –7, –12, –17, .....
Product of zeroes =  ½ Mark)

a a = – 7, d = –12 – (–7) ⇒ d = – 12 + 7 = –5 (½ ½ Mark)
−2
−1 2 −2 −2 an = – 82
= × = 3 ⇒ =  ½ Mark)

7 3 7 21 21 Now, an = a + (n – 1)d ½ Mark)

23. Out of the Syllabus – 82 = – 7 + (n – 1) (– 5) ⇒ – 75 = (n – 1) (– 5)
24. Given, ∠APB = 60°, ∠CPD = 30° n – 1 = 15 ⇒ n = 16 ½ Mark)

∠APC = 180° So, – 82 is 16th term of the given sequence ½ Mark)

234
234 MATHS
Now, take an = – 100 1 + sin θ cos θ
½ Mark)
– 100 = – 7 + (n – 1)(–5) ⇒ – 93 = (n – 1)(–5) (½ = [⸪ sin2 θ + cos2 θ = 1]
sin θ cos θ
93 93 98
(n – 1) = ⇒ n= + 1=  ½ Mark)
(½ 1
5 5 5 = + 1 = 1 + sec θ.cosec θ (1 Mark)
sin θ cos θ
But n can't be a fractional number
So, –100 is not a part of the given sequence. (1 Mark) LHS = RHS
OR OR
Given sequence 45, 39, 33, ....... sin θ
LHS =
a = 45, d = 39 – 45 = –6, Sn = 180 cot θ + cosec θ
n
S=
n (2a + (n − 1)d ) ⇒ 180= n (2 × 45 + (n − 1)(−6))  sin θ sin 2 θ
2 2 = = (1 Mark)
 (1 Mark) cos θ 1 1 + cos θ
+
sin θ sin θ 
360 = 90n – 6n2 + 6n ⇒ 6n2 – 96n + 360 = 0 (½ ½ Mark)
n – 16n + 60 = 0 ⇒ n – 10n – 6n + 60 = 0
2 2
½ Mark)
(½ on rationalizing, we get
n(n – 10) – 6(n – 10) = 0 ⇒ (n – 10)(n – 6) = 0 sin 2 θ 1 − cos θ sin 2 θ(1 − cos θ)
= × =
n – 10 = 0 or n – 6 = 0 ⇒ n = 10, n = 6 (1 Mark) 1 + cos θ 1 − cos θ 1 − cos 2 θ
This is a decreasing sequence of AP so after summation of
(1 − cos θ) sin 2 θ
6 terms we get 180 then again after sum of 10 terms we get = = 1 – cos θ  ...(i) (1 Mark)
sum 180.  (1 Mark) sin 2 θ
sin θ
sin θ cos θ RHS= 2 +
cot θ − cosecθ
tan θ cot θ cos θ + sin θ
27. + =  (1 Mark) sin θ sin 2 θ
1 − cot θ 1 − tan θ cos θ sin θ
1− 1− = 2+ = 2+ (1 Mark)
sin θ cos θ cos θ 1 cos θ − 1

sin 2 θ cos 2 θ sin θ sin θ 
= +
cos θ(sin θ − cos θ) sin θ(cos θ − sin θ) sin 2 θ 1 − cos 2 θ (1 − cos θ)(1 + cos θ)
= 2− = 2− = 2−
sin 2 θ cos 2 θ 1 − cos θ 1 − cos θ 1 − cos θ 
= −
cos θ(sin θ − cos θ) sin θ(sin θ − cos θ) [⸪ a2 – b2 = (a – b)(a + b)]
= 2 – (1 + cos θ) = 2 – 1 – cos θ = 1 – cos θ ...(ii)
1  sin 2 θ cos 2 θ 
=  −  (1 Mark)
sin θ − cos θ  cos θ sin θ  From equation (i) and (ii), we get LHS = RHS

sin θ sin θ
1  sin 3 θ − cos3 θ  = 2+ (1 Mark)
=   cot θ + cosecθ cot θ − cosecθ (Proved)
sin θ − cos θ  sin θ cos θ 
28. Out of the Syllabus
1 (sin θ − cos θ)(sin 2 θ + cos 2 θ + sin θ cos θ)
= 29. Out of the Syllabus
sin θ − cos θ sin θ cos θ
 (1 Mark) 30. Out of the Syllabus

CBSE 2019 (Outside DELHI) 235


DELHI/Outside DELHI
2018 CBSE Solved Paper

Time allowed : 3 hours Maximum Marks : 80


GENERAL INSTRUCTIONS:
Read the following instructions very carefully and follow them:
(i) All questions are compulsory.
(ii) This question paper consists of 30 questions divided into four sections – A, B, C and D.
(iii) Section A contains 6 questions of 1 mark each. Section B contains 6 questions of 2 marks each, Section C contains
10 questions of 3 marks each. Section D contains 8 questions of 4 marks each.
(iv) There is no overall choice. However, an internal choice has been provided in four questions of 3 marks each and
3 questions of 4 marks each. You have to attempt only one of the alternatives in all such questions.
(v) Use of calculator is not permitted.

9. Find the ratio in which P(4, m) divides the line segment joining
Section-A the points A(2, 3) and B(6, –3). Hence find m.  (2 Marks)
AB 1 ar ∆ABC 10. Given that 2 is irrational, prove that (5 + 3 2 ) is an
1. Given ∆ABC ~ ∆PQR, if = , then find . irrational number. (2 Marks)
PQ 3 ar ∆PQR
 [OS] (1 Mark) 11. In Fig., ABCD is a rectangle. Find the values of x and y.
(2 Marks)
2. What is the value of (cos2 67° – sin2 23°)? [OS] (1 Mark) D x+y
C
3. Find the distance of a point P(x, y) from the origin. (1 Mark)
4. If x = 3 is one root of the quadratic equation x2 – 2kx – 6 = 0, 14 cm x–y
then find the value of k. (1 Mark)
5. What is the HCF of smallest prime number and the smallest A B
composite number? (1 Mark) 30 cm

6. In an AP, if the common difference (d) = –4 and the seventh 12. Find the sum of first 8 multiples of 3. (2 Marks)
term (a7) is 4, then find the first term. (1 Mark)
Section-C
Section-B 13. A plane left 30 minutes late than its scheduled time and in
order to reach the destination 1500 km away in time, it had
7. An integer is chosen at random between 1 and 100. Find to increase its speed by 100 km/h from the usual speed.
the probability that it is: (2 Marks) Find its usual speed. (3 Marks)
(i) divisible by 8. 14. Prove that the area of an equilateral triangle described on one
(ii) not divisible by 8. side of the square is equal to half the area of the equilateral
triangle described on one of its diagonal.  [OS] (3 Marks)
8. Two different dice are tossed together. Find the probability:
OR
(i) of getting a doublet (2 Marks)
If the area of two similar triangles are equal, prove that
(ii) of getting a sum 10 of the numbers on the two dice. they are congruent. [OS]
15. Prove that the lengths of tangents drawn from an external
point to a circle are equal. (3 Marks) Section-D
16. A wooden article was made by scooping 23. Draw a triangle ABC with BC = 6 cm, AB = 5 cm and
out a hemisphere from each end of a solid 3
∠ABC = 60°. Then construct a triangle whose sides are
cylinder, as shown in Fig. If the height 4
of the cylinder is 10 cm and its base is of of the corresponding sides of the ∆ABC. [OS] (4 Marks)
radius 3.5 cm. Find the total surface area of 24. The sum of four consecutive numbers in an AP is 32 and
the article. (3 Marks) the ratio of the product of the first and the last term to the
OR product of two middle terms is 7 : 15. Find the numbers.
A heap of rice is in the form of a cone of (4 Marks)
base diameter 24 m and height 3.5 m. Find 25. In an equilateral ∆ABC, D is a point on side BC such that
1
the volume of the rice. How much canvas BD = BC . Prove that 9(AD)2 = 7(AB)2 [OS] (4 Marks)
cloth is required to just cover the heap? 3
OR
17. The table below shows the salaries of 280 persons: Prove that, in a right triangle, the square on the hypotenuse is
(3 Marks) equal to the sum of the squares on the other two sides. [OS]
26. A motor boat whose speed is 18 km/hr in still water takes
10–15

15–20

20–25

25–30

30–35

35–40

40–45

45–50
Salary (in
5–10

1 hr more to go 24 km upstream than to return downstream


thousand `)
to the same spot. Find the speed of the stream. (4 Marks)
No. of Persons 49 133 63 15 6 7 4 2 1 OR
A train travels at a certain average speed for a distance of
Calculate the median salary of the data.
63 km and then travels at a distance of 72 km at an average
 4sin θ − cos θ + 1  speed of 6 km/hr more than its original speed. If it takes
18. If 4 tan θ = 3, evaluate   (3 Marks)
 4sin θ + cos θ − 1  3 hours to complete total journey, what is the original
OR average speed?
If tan 2A = cot (A – 18°), where 2A is an acute angle, find 27. As observed from the top of a 100 m high light house from
the value of A. [OS] the sea-level, the angles of depression of two ships are 30°
19. Find the area of the shaded region in Fig., where arcs drawn and 45°. If one ship is exactly behind the other on the same
with centres A, B, C and D intersect in pairs at mid-points side of the light house, find the distance between the two
P, Q, R and S of the sides AB, BC, CD and DA respectively of ships. [Use 3 = 1.732] (4 Marks)
a square ABCD of side 12 cm. [Use π = 3.14] [OS] (3 Marks) 28. The diameters of the lower and upper ends of a bucket
P in the form of a frustum of a cone are 10 cm and 30 cm
A B
respectively. If its height is 24 cm, find: 
(i) The area of the metal sheet used to make the bucket.
(ii) Why we should avoid the bucket made by ordinary
S Q
plastic? [Use π = 3.14] [OS] (4 Marks)
29. The mean of the following distribution is 18. Find the
frequency f of the class 19 – 21.
D C
R Class 11–13 13–15 15–17 17–19 19–21 21–23 23–25
20. If A(–2, 1), B(a, 0), C(4, b) and D(1, 2) are the vertices of Frequency 3 6 9 f 13 5 4
a parallelogram ABCD, find the values of a and b. Hence (4 Marks)
find the lengths of its sides. (3 Marks) OR
OR The following distribution gives the daily income of 50
If A(–5, 7), B(–4, –5), C(–1, –6) and D(4, 5) are the vertices of workers of a factory:
a quadrilateral, find the area of the quadrilateral ABCD. [OS] Daily Income (`) 100-120 120-140 140-160 160-180 180-200
21. Find HCF and LCM of 404 and 96 and verify that HCF × No. of Workers 12 14 8 6 10
LCM = Product of the two given numbers. (3 Marks)
Convert the distribution above to a less than type cumulative
22. Find all zeroes of the polynomial (2x – 9x + 5x2 + 3x – 1)
4 3
frequency distribution and draw its ogive. [OS]
if two of its zeroes are (2 + 3 ) and (2 – 3 ). sin A − 2sin 3 A
 [OS] (3 Marks) 30. Prove that: = tan A  (4 Marks)
2 cos3 A − cos A

CBSE 2018 (DELHI/Outside DELHI) 237


EXPLANATIONS

1. Out of the Syllabus 9. Let P divide line AB in ratio k : 1


2. Out of the Syllabus A(2, 3), B(6, –3)
3. Given, P(x, y) & O(0, 0) By, section formula
By, Distance formula = ( x2 − x1 ) 2 + ( y2 − y1 ) 2 (½
½ Mark)  mx + nx1 my2 + ny1 
( x, y ) =  2 ,  ½ Mark)

2 2
 m+n m + n 
\ PO = ( x − 0) + ( y − 0) ⇒ OP
2 2
= ½ Mark)
x + y (½
 k (6) + 1(2) k (−3) + 1(3) 
⇒ (4, m) =  , 
4. Given, x2 – 2kx – 6 = 0 and one root is x = 3  k +1 k +1 
\ x = 3 is the root of given equation then it will satisfy the
 6k + 2 –3k + 3 
quadratic equation. ⇒ (4, m) = , 
Now,  k +1 k +1   ½ Mark)

on comparing, we get
\ 32 – 2k(3) – 6 = 0 ½ Mark)

3 1 6k + 2
⇒ 9 – 6k – 6 = 0 ⇒ 6k = 3 ⇒ k= =  ½ Mark)
(½ ⇒ 4= ⇒ 4k + 4 = 6k + 2 ⇒ 4 – 2 = 6k – 4k
6 2 k +1
5. Smallest prime number is 2 and Smallest composite ⇒ 2 = 2k ⇒ k = 1 ½ Mark)

number is 4. ½ Mark)
(½ −3 × 1 + 3 −3 + 3
Now, Factors of 2 = 1 × 2, Factors of 4 = 1 × 2 × 2 =
and m = = 0 (put k = 1)
1+1 2
So, HCF (2, 4) = 1 × 2 = 2 ½ Mark)

Hence, ratio is 1 : 1 and m = 0. ½ Mark)

6. Given, d = –4, a7 = 4,
We know, an = a + (n – 1)d ½ Mark)
(½ 10. Given, 2 is an irrational number.
\ a7 = a + 6d ⇒ 4 = a + 6 × (–4) Let 5 + 3 2 is a rational number
⇒ 4 = a – 24 ⇒ a = 24 + 4 = 28 ½ Mark)

p p
7. (i)  Number of integers between 1 and 100 = 98 So, 5 + 3 2 = ⇒ 3 2= −5  ½ Mark)

q q
A = Numbers which are divisible by 8 = 12
p − 5q p − 5q
\ Probability of getting an integer which is divisible ⇒3 2= ⇒ 2=  (1 Mark)
q 3q
Favorable outcomes
by 8 P(A) = p − 5q
Total outcomes p & q are integer therefore is a rational number
12 6 3q
⇒ P( A= ) =  (1 Mark)
98 49 But 2 is irrational.

(ii) Let P(A ) is the probability of getting an integer which So, it becomes a contradiction
is not divisible by 8. Hence, ( 5 + 3 2 ) is an irrational number. ½ Mark)

 
P(A) + P(A ) = 1 ⇒ P(A ) = 1 – P(A) 11. Given, AD = 14 cm, AB = 30 cm, CD = x + y and BC = x – y
6 49 − 6 43 We know, In ABCD rectangle
=−1 = = (1 Mark)
49 49 49 AD = BC and AB = CD
8. When two dice are thrown together, total no. of possible \ x – y = 14  ...(i)
outcomes = 62 = 36 x + y = 30  ...(ii) (1 Mark)
(i) Number of doublets = 6 on adding equation (i) and (ii), we get
Number of favourable outcome ⇒ 2x = 44 ⇒ x = 22
Probability of doublets =
Total no. of possible outcome put the value of x in equation (i), we get
6 1
= =  (1 Mark) ⇒ 22 – y = 14 ⇒ y = 22 – 14 ⇒ y = 8 (1 Mark)
36 6
(ii) Number of outcomes whose sum is 10 = 3 12. Given, the multiple of 3 is 3, 6, 9, 12, ....
Number of favourable outcome \ a = 3, d = 6 – 3 = 3, n = 8
Probability of sum 10 = n
Total no. of possible outcome Now, S= ( 2a + (n − 1)d )  (1 Mark)
n
2
3 1 8
= = 
36 12
(1 Mark) \S=
8 ( 2 × 3 + (8 − 1) × 3) ⇒ S8 = 4(6 + 21) = 108 (1 Mark)
2
238
238 MATHS
13. Let, the usual speed be x km/hr 16. Given, height of the cylinder = 10 cm
Then, speed taken = (x + 100) km/hr, Distance = 1500 km radius of base = 3.5 cm
Total surface area of the article = curved surface area of
Distance 1500 1500
Usual time = = ⇒ Time taken = hrs the cylinder + 2 × surface area of hemisphere. ...(i)
Speed x x + 100 Now, the curved scarface area of the cylinder = 2πrh
 (1 Mark)
Now, according to question 22
= 2× × 3.5 × 10 = 220 cm2  (1 Mark)
7
1500 1500 1 1  1
⇒ − 0.5 ⇒ 1500  −
= = 22 2
x x + 100  x x + 100  2 Now surface area of the hemisphere = 2π r2 = 2 × × ( 3.5 )
7
 x + 100 − x  22 35 35
⇒ 3000   = 1 ⇒ 300000 = x + 100x
2
= 2× × × = 77cm 2  (1 Mark)
 x( x + 100)  7 10 10
⇒ x2 + 100x – 300000 = 0 (1 Mark) ∴ Total surface area of the article = 220 + 2 × 77 = 374 cm2
 (1 Mark)
−b ± b 2 − 4ac OR
x=
2a Given, Radius of conical heap = 12 m

−100 ± (100) 2 − 4 × 1× 300000 1 2 1 22


 volume of rice= πr h = × × (12) 2 × 3.5 = 528 m3
\x= 3 3 7
2 ×1 
(1 Mark)
Area of canvas cloth required = πrl
−100 ± 1100
⇒x= =l 122 + (3.5) 2 = 12.5 m (1 Mark)
2
⇒ x = –50 ± 550 ⇒ x = –600 or x = 550 22
Now, area of canvas required = × 12 × 12.5 = 471.4 m2
but x cannot be negative 7
 (1 Mark)
So, speed of plane usually = 500 km/hr (1 Mark) 17.
14. Out of the Syllabus Salary (in thousand `) No. of Persons fi C.f.
15. Let SK and SR drawn to circle with center O from an 5–10 49 49
external point K. 10–15 133 182

S 15–20 63 245
20–25 15 260

O K 25–30 6 266
30–35 7 273
35–40 4 277
R
40–45 2 279
Now, Normal and tangent at a point on the circle are
perpendicular to each other. (1 Mark) 45–50 1 280

∠OSK = ∠ORK = 90° Total N = ∑fi = 280


Using pythagoras theorem, in DOSK and DORK N 280
= = 140
OK = OS + SK 
2 2 2
...(i) 2 2  (1 Mark)
OK = OR + RK ...(ii) (1 Mark)
2 2 2 Median class = 10 – 15 and l = 10, h = 5, f = 133, C.f. = 49
N 
Subtracting equation (ii) from equation (i), we get  2 − C. f 
0 = OS2 + SK2 – OR2 – RK2 But OS = OR Median = l + × h  (1 Mark)
f
 
0 = OS2 – OS2 + SK2 – RK2 ⇒ 0 = SK2 – RK2 ⇒ SK2 = RK2  
⇒ SK = RK 140 − 49
=10 + × 5 = 10 + 3.42 = 13.42 (1 Mark)
Hence, proved. (1 Mark) 133

CBSE 2018 (DELHI/Outside DELHI) 239


18. Given, 4 tan θ = 3 ⇒ tan θ = 3/4, 21. Let a = 404 and b = 96
Perpendicular 3 Now,
= ∵ tan θ =
base 4 Prime factor of a = 2 × 2 × 101
\ By Pythagoras theorem, we have Prime factor of b = 2 × 2 × 2 × 2 × 2 × 3 (1 Mark)
So, HCF = 2 × 2 = 4
h = (3) 2 + 42 = 9 + 16 = 5  (1 Mark)
LCM = 2 × 2 × 2 × 2 × 3 × 2 × 101 = 9696 (1 Mark)
3 4
\ sin θ
= and
= cos θ  Now, LCM × HCF = 4 × 9696 = 38784
5 5
3 4 Product of 96 and 404 = 38784
4× − +1
4sin θ − cos θ + 1 5 5 Hence, LCM × HCF = Product of two numbers (1 Mark)
Now, =  (1 Mark)
4sin θ + cos θ − 1 4 × 3 + 4 − 1
22. Out of the Syllabus
5 5
12 − 4 + 5 13 23. Out of the Syllabus
= =  (1 Mark)
12 + 4 − 5 11 24. Let the four consecutive numbers of AP be a – 3d, a – d,
OR a + d, a + 3d
Out of the Syllabus Now, according to question
19. Out of the Syllabus a – 3d + a – d + a + d + a + 3d = 32
20. Given, Sides of parallelogram ⇒ 4a = 32 ⇒ a = 8  ...(i)
A(–2, 1), B(a, 0), C(4, b), D(1, 2) (a − 3d )(a + 3d ) 7
Now, =  (1 Mark)
We know that diagonals of parallelogram bisect each other. (a − d )(a + d ) 15
Midpoint let say O of diagonal AC is given by a 2 − 9d 2 7
⇒ = ⇒ 15a2 – 135d2 = 7a2 – 7d2 (1 Mark)
x1 + x2 y1 + y2 a 2 − d 2 15
=x = ,y
2 2 ⇒ 8a2 = 128d2 ⇒ 16d2 = a2
 −2 + 4 1 + b   ...(i) from equation (i) we have,
O , 
 2 2  16d2 = 82 = 64 ⇒ d2 = 4 ⇒ d = ±2 (1 Mark)
Midpoint let say M of diagonal BD is given by Now, d = 2 then 8 – 3 × 1, 8 – 2, 8 + 2, 8 + 3 × 2
 a + 1 0 + 2  ...(ii) (1 Mark) 2, 6, 10, 14
M , 
 2 2  and If d = –2 then 8 – 3 × (–2), 8 – (–2), 8 + (–2), 8 + 3 × (–2)
from (i) and (ii), we get
14, 10, 6, 2 (1 Mark)
 −2 + 4 1 + b   a + 1 0 + 2 
 , = ,  25. Out of the Syllabus
 2 2   2 2 
26. Given, speed of boat in still water = 18 km/hr
2 a +1 1+ b 2
=So, and − Let speed of stream = S
2 2 2 2
Speed of boat upstream = Speed of boat in still water
a + 1 = 2 and 1 + b = 2 ⇒ a = 1 and b = 1 (1 Mark)
– Speed of stream = 18 – S (1 Mark)
Now, A(–2, 1), B(1, 0), C(4, 1), D(1, 2) Speed of boat downstream = Speed in still water + Speed
of stream = 18 + S (1 Mark)
AB= (1 − (−2)) 2 + (0 − 1) 2= 10
Now, according to question
BC= 2
(4 − 1) + (1 − 0) = 2
10 Time taken for upstream = Time for downstream + 1
24 24
⇒ = +1  (1 Mark)
CD = (1 − 4) 2 + (2 − 1) 2 = 10 18 − S 15 + S

AD= (1 − (−2)) 2 + (2 − 1) 2= 10  ⇒ S2 + 48S – 324 = 0 ⇒ S2 + 54S – 6S – 324 = 0


(1 Mark)
⇒ (S + 54)(S – 6) = 0 ⇒ S = –54 or S = 6 (1 Mark)
OR but speed cannot be negative
Out of the Syllabus \ S = 6 km/hr

240
240 MATHS
OR 29.
Let the average speed be x km/hr Class xi fi fixi
Now, according to question
11–13 12 3 36
63 72
+ = 3 (1 Mark) 13–15 14 6 84
x x+6
⇒ 63(x + 6) + 72x = 3(x + 6)x ⇒ 63x + 378 + 72x = 3(x2 + 6x) 15–17 16 9 144
⇒ 135x + 378 = 3x2 + 18x ⇒ 3x2 – 117x – 378 = 0 (1 Mark) 17–19 18 13 234
⇒ x2 – 39x – 126 = 0 ⇒ x2 – 42x + 3x – 126 = 0(1 Mark) 19–21 20 f 20f
⇒ (x – 42)(x + 3) = 0 ⇒ x = 42 or x = –3
21–23 22 5 110
but speed cannot be negative,
23–25 24 4 96
\ x = 42 km/hr (1 Mark)
27. Let one height of light house be AB which is 100 m and let 40 + f 704 + 20f
the ships at positions be C and D.
A  (1 Mark)
30°
45° Given Mean = 18.

Now, Mean =
∑fx i i
 (1 Mark)
100 m

∑f i

704 + 20 f
⇒ 18 = ⇒ (40 + f)18 = 704 + 20f (1 Mark)
40 + f
30° 45°
B ⇒ 720 + 18f = 720 – 704 ⇒ 2f = 16 ⇒ f = 8 (1 Mark)
D C  (1 Mark)
OR
In ∆ABC,
AB Out of the Syllabus
100
tan 45° = ⇒ 1=  [ tan 45° = 1]
BC BC sin A(1 – 2sin 2 A)
sin A – 2sin 3 A
30. L.H .S = =  (1 Mark)
⇒ BC = 100 m (1 Mark) 2 cos3 A − cos A cos A(2 cos 2 A − 1)
AB
In ∆ABD, tan 30° =  1 – sin 2 A − sin 2 A 
BD = tan A    (1 Mark)
2 2
1
=
100  1   cos A + cos A − 1 

3 100 + CD
 ∵ tan 30° =  (1 Mark)
 3
 (1 – sin 2 A) − sin 2 A 
= tan A  2 2  (1 Mark)
⇒ 100 + CD = 100 3 ⇒ CD = 100 3 – 100  cos A − (1 − cos A) 
⇒ CD = 100 × 1.732 – 100  cos 2 A − sin 2 A 
= tan A   = tan A {⸪ 1 – cos A = sin2A,
2
⇒ CD = 173.2 – 100 ⇒ CD = 73.2 m (1 Mark) 2 2
 cos A − sin A  1 – sin2A = cos2A)}
28. Out of the Syllabus Hence, L.H.S = R.H.S proved (1 Mark)

CBSE 2018 (DELHI/Outside DELHI) 241


DELHI
2017 CBSE Solved Paper

Time allowed : 3 hours Maximum Marks : 90


GENERAL INSTRUCTIONS:
Read the following instructions very carefully and strictly follow them:
(i) All questions are compulsory.
(ii) The question paper consists of 31 questions divided into four sections – A, B, C and D.
(iii) Section A contains 4 questions of 1 mark each. Section B contains 6 questions of 2 marks each. Section C contains
10 questions of 3 marks each and Section D contains 11 questions of 4 marks each.
(iv) Use of calculators is not permitted.

Section-A 7. Find the roots of the quadratic equation 2 x 2 + 7 x + 5 2 =


0
(2 Marks)
1. A number is chosen at random from the numbers –3, –2,
–1, 0, 1, 2, 3. What will be the probability that square of 8. Find how many integers between 200 and 500 are divisible
this number is less then or equal to 1? (1 Mark) by 8. (2 Marks)

2. If the distance between the points (4, k) and (1, 0) is 5, then 9. Find the value of k for which the equation x2 + k(2x + k –1)
what can be the possible values of k? (1 Mark) + 2 = 0 has real and equal roots. (2 Marks)
3. The ratio of the height of a tower and the length of its 10. Draw a line segment of length 7 cm and divide it internally
shadow on the ground is 3 :1 . What is the angle of in the ratio 2 : 3. [OS] (2 Marks)
elevation of the sun? (1 Mark)
4. Volume and surface area of a solid hemisphere are Section-C
numerically equal. What is the diameter of hemisphere?
(1 Mark) 11. The area of a triangle is 5 sq units. Two of its vertices are
7 
(2, 1) and (3, –2). If the third vertex is  , y  find the
Section-B 2 
value of y. [OS] (3 Marks)
5. Prove that tangents drawn at the ends of a diameter of a
circle are parallel to each other. (2 Marks) 12. Show that ∆ABC, where A(–2, 0), B(2, 0), C(0, 2) and
∆PQR where P(–4, 0), Q(4, 0), R(0, 4) are similar triangles.
6. In the given figure, PA and PB are tangents to the circle
(3 Marks)
from an external point P. CD is another tangent touching
the circle at Q. If PA = 12 cm, QC = QD = 3 cm, then find 13. Two different dice are thrown together. Find the probability
PC + PD. (2 Marks) that the numbers obtained (3 Marks)
A (i) have a sum less than 7
C
(ii) have a product less than 16
(iii) is a doublet of odd numbers.
O Q P
14. Two tangents TP and TQ are drawn to a circle with centre
O from an external point T. Prove that ∠PTQ = 2∠OPQ.
D
B (3 Marks)
15. Find the sum of n terms of the series
 1  2  3
 4 –  +  4 –  +  4 –  + .....  (3 Marks) O
 n   n   n 

16. If the equation (1 + m2)x2 + 2mcx + c2 – a2 = 0 has equal


roots then show that c2 = a2 (1 + m2). (3 Marks)
O'
17. In the given figure, OACB is a quadrant of a circle with
centre O and radius 3.5 cm. If OD = 2 cm, find the area of
the shaded region. [OS] (3 Marks) 23. Peter throws two different dice together and finds the
A product of the two numbers obtained. Rina throws a die
and squares the number obtained. Who has the better
C chance to get the number 25. (4 Marks)
24. A chord PQ of a circle of radius 10 cm subtends an angle
D
of 60° at the centre of circle. Find the area of major and
minor segments of the circle. (4 Marks)

B 25. Prove that the lengths of tangents drawn from an external


O
point to a circle are equal. (4 Marks)
1 1
18. If the mth term of an A.P. is and nth term is then show 26. Speed of a boat in still water is 15 km/h. It goes 30 km
n m upstream and returns back at the same point in 4 hours 30
that its (mn)th term is 1. (3 Marks)
minutes. Find the speed of the stream. (4 Marks)
19. A metallic solid sphere of radius 10.5 cm is melted and
recasted into smaller solid cones, each of radius 3.5 cm 27. If a ≠ b ≠ 0, prove that the points (a, a2), (b, b2) (0, 0) will
and height 3 cm. How many cones will be made? not be collinear. [OS] (4 Marks)
 [OS] (3 Marks) 28. Draw a right triangle in which the sides (other than
20. From the top of a 7 m high building, the angle of elevation the hypotenuse) are of lengths 4 cm and 3 cm. Now
of the top of a tower is 60° and the angle of depression of construct another triangle whose sides are 3/5 times the
its foot is 45°. Find the height of the tower. (3 Marks) corresponding sides of the given triangle.[OS] (4 Marks)
29. If the sum of first m terms of an A.P. is the same as the sum
Section-D of its first n terms, show that the sum of its first (m + n)
terms is zero. (4 Marks)
21. In a hospital used water is collected in a cylindrical tank
of diameter 2 m and height 5 m. After recycling, this water 30. Two points A and B are on the same side of a tower and
is used to irrigate a park of hospital whose length is 25 m in the same straight line with its base. The angles of
and breadth is 20 m. If tank is filled completely then what depression of these points from the top of the tower are
will be the height of standing water used for irrigating the 60° and 45° respectively. If the height of the tower is 15 m,
park. Write your views on recycling of water. (4 Marks) then find the distance between these points. (4 Marks)
22. In the given figure, the side of square is 28 cm and radius 31. The height of a cone is 30 cm. From its topside a small
of each circle is half of the length of the sides of the square cone is cut by a plane parallel to its base. If volume of
where O and O' are centres of the circles. Find the area of smaller cone is 1/27 of the given cone, then at what height
shaded region. (4 Marks) it is cut from its base? (4 Marks)

CBSE 2017 (DELHI) 243


EXPLANATIONS

1. Given, Sample set = {–3, –2, 0, 1, 2, 3} To prove : PQ || RS


n(s) = 7 Proof : Since PQ is tangent at point A.
Square of these number = {9, 4, 1, 0} \ OA ⊥ PQ (Tangent at any point of cicle is perpendicular
So A is event of choosing one number at random whose to the radius through point of contact)
½ Mark)
square is less than or equal to 1 = {–1, 0, 1} (½ ∠OAP = 90°  ...(i)
n(A) = 3 Similarly, OB⊥RS
n( A) 3 ∠OBS = 90°  ...(ii) (1 Mark)
Probability P(A) = =  ½ Mark)

n( S ) 7 From equation (i) and (ii), we get
2. Given, A(4, k) and B (1, 0) ∠OAP = ∠OBS
AB = 5 units i.e., ∠BAP = ∠ABS
For line PQ & RS and transversal AB
⸫ AB = ( x2 − x1 ) 2 + ( y2 − y1 ) 2 ⇒ 5 = (4 − 1) 2 + (k − 0) 2
∠BAP = ∠ABS i.e., both alternate angles are equal
 ½ Mark)
(½ So, lines are parallel
On squaring, we get \ PQ || RS. (1 Mark)
⇒ 52 = (4 – 1)2 + k2 ⇒ 25 = 9 + k2
6. Given,
⇒ k2 = 16 ⇒ k = ± 4 A
C
Hence, the possible values of k is 4 and – 4 ½ Mark)

height of tower 3 A Q P
3. Given = O
length of shadow 1
In DABC, ∠B = 90° D
Tower

B
AB PA = 12 cm, QC = QD = 3 cm
tan θ = ½ Mark)

BC  PA and PB are two tangents to the circle from an external
q point P.  ½ Mark)

3
⇒ tan=
θ = tan 60° C Shadow B CD is tangent touches the circle at Q.
1
We known that, the lengths of the tangents drawn from
On comparing, we get q = 60° any external point on a circle are equal.
½ Mark)
Hence, the angle of elevation of the sun is 60°. (½ So, from the figure
4. According to question, PA = PB = 12 cm
Volume of hemisphere = surface area of hemisphere Also, CA = CQ = 3 cm and BD = DQ = 3 cm (½ ½ Mark)
2 3 Now, PC = PA – AC = 12 – 3 = 9 cm
⇒ πr =3πr 2 ⇒ 2pr3 = 9pr2
3 and PD = PB – BD = 12 – 3 = 9 cm
9 \ PC + PD = 9 + 9 = 18 cm (1 Mark)
⇒ r = ⇒ 2r = 9 ⇒ d = 9 (1 Mark)
2
7. Let 2 x2 + 7 x + 5 2 =
0
5. Given, A circle with centre O and diameter AB.
Let PQ & RS be the tangent at point A and B respectively. ⇒ 2 x2 + 2 x + 5x + 5 2 =
0
A ⇒ 2 x ( x + 2) + 5( x + 2) =
0 (1 Mark)
P Q
⇒ ( x + 2) ( 2 x + 5) =
0
O On comparing we get
+ 2 0 or =
⇒ x= 2x + 5 0

B − 2 or x =
⇒ x= −5 / 2  (1 Mark)
R S
244
244 MATHS
8. We have sequence from 200 to 500 divisible by 8 = {(1,1), (1,2), (1,3), (1,4), (1,5), (2,1), (2,2), (2,3),
200, 208, 216,............496 (2,4), (3,1), (3,2), (3,3), (4,1), (4,2), (5,1)}
Here, a = 200, d = 208 – 200 = 8, an = 496 (1 Mark) ⸫ n(A) = 15
We know, an = a + (n –1)d n( A) 15 5
So, P( A=
) = = . (1 Mark)
⇒ 496 = 200 + (n –1)8 ⇒ (n – 1)8 = 296 n( S ) 36 12
⇒ n – 1 = 37 ⇒ n = 38 (ii) B be the event of obtained product of less than 16
Hence, 38 integers are divisible by 8 between 200 and 500. 
{(1,1), (1,2), (1,3), (1,4), (1,5), (1,6), (2,1), (2,2),
 (1 Mark) (2,3), (2,4), (2,5), (2,6), (3,1), (3,2), (3,3), (3,4), (3,5),
(4,1), (4,2), (4,3), (5,1), (5,2), (5,3), (6,1), (6,2)}
9. Given, x2 + k(2x + k – 1) + 2 = 0
⸫ n(B) = 25
⇒ x2 + 2kx + k2 – k + 2 = 0 ½ Mark)

n( B) 25
For equal roots D = 0 ( B)
So, P= =  (1 Mark)
n( S ) 36
⇒ b2 – 4ac = 0 ...(i)
a = 1, b = 2k, c = k2 – k + 2 ½ Mark)
(½ (iii) C be the event of double of odd numbers
Put these values in equation (i), we get = {(1,1), (3,3) (5,5)}
(2k)2 – 4 × 1(k2 – k + 2) = 0 ⸫ n(C) = 3
⇒ 4k2 – 4k2 + 4k – 8 = 0 n(C ) 3 1
So, P(C=
) = =  (1 Mark)
⇒ 4k = 8 ⇒ k = 2 (1 Mark) n( S ) 36 12
10. Out of the Syllabus 14. We know that length of tangents drawn from on external
point to a circle are equal.
11. Out of the Syllabus
12. Given, A(–2, 0), B(2, 0), C(0, 2) and P
P(–4, 0), Q(4, 0), R(0, 4)
We know, q

Distance = ( x2 − x1 )2 + ( y2 − y1 )2  ½ Mark)
(½ T O

Now, AB = (2 + 2) 2 + (0 − 0) 2 = 4

BC = (0 − 2) 2 + (2 − 0) 2 = 4+4 = 2 2 Q

AC = (0 + 2) 2 + (2 − 0) 2 = 4+4 = 2 2  (1 Mark)
\ TP = TQ ...(i)
Similarly
\ ∠TPQ = ∠PQT (angles of equal sides are equal) ...(ii)
PQ = (4 + 4) 2 + (0 − 0) 2 = 8 Now, PT is tangent and OP is radius. (1 Mark)
\ OP⊥TP (tangent at any point circle is perpendicular to
QR = (0 − 4) 2 + (4 − 0) 2 = 16 + 16 = 4 2
the radius through point of contact)
PR = (0 + 4) 2 + (4 − 0) 2 =
16 + 16 = 4 2  ∠OPT = 90°
(1 Mark)
⇒ ∠OPQ + ∠TPQ = 90° ⇒∠TPQ = 90° – ∠OPQ ...(iii)
Now, on comparing both triangles we get
In DPTQ,
AB BC AC 1
= = = ∠TPQ + ∠PQT + ∠QTP = 180°  (1 Mark)
PQ QR PR 2 (\ sum of angles of triangle is 180°)
\DABC ~ DPQR (SSS similarity). ½ Mark)
(½ ⇒ (90° – ∠OPQ) + ∠TPQ + ∠PTQ = 180° [∠PQT = ∠TPQ]
13. Given; two dice are thrown together ⇒ (90° – ∠OPQ) + (90° – ∠OPQ) + ∠PTQ = 180°
So we have total 36 outcomes ⇒ 180° – 2∠OPQ + ∠PTQ = 180°
n(S) = 36 ⇒ ∠PTQ = 180° – 180° + 2 ∠OPQ
(i) A be the event of obtained sum of less than 7 ⇒ ∠PTQ = 2∠OPQ (1 Mark)

CBSE 2017 (DELHI) 245


15. Given series 20. Let AD be the height of building and EC be the height of
tower.
 1  2  3
⇒ Sn =  4 −  +  4 −  +  4 −  + ..... In DABE,
 n   n   n  E
1 2 3
⇒ Sn = (4 + 4 + 4 + .....) –  + + + .....   (1 Mark) h
n n n 
60°
A B
1 45°
⇒ Sn = 4n − (1 + 2 + 3 + ......)
n 7
1 n
⇒ Sn= 4n − . [2 × 1 + (n − 1).1] D C
n 2  (1 Mark) x
BE h h
1 (n + 1) ⇒ tan 60° = ⇒ 3= ⇒x= ...(i) (1 Mark)
⇒ Sn = 4n − (2 + n − 1) = 4n − AB x 3
2 2
In DABC,
7n − 1
⇒ Sn = BC 7
2  (1 Mark) ⇒ tan 45° = ⇒ 1 = ⇒ x = 7 [⸪ AB = CD = x]
AB x
16. Given, (1 + m2) x2 + 2mcx + c2 – a2 = 0 ...(ii) (1 Mark)
For equal roots, D = 0 (1 Mark) From (i) and (ii), we get
h
⇒ b – 4ac = 0
2
⇒ 7= ⇒h =7 3
3
⇒ (2mc)2 – 4 (1+ m2) (c2 – a2) = 0
and height of tower = 7 + h = 7 + 7 3
⇒ 4m2c2 – 4 (c2 – a2 + m2c2 – m2a2) = 0 (1 Mark)
= 7(1 + 1.73) = 7 × 2 .73 = 19.11 m (1 Mark)
⇒ –4c2 + 4a2 (1 + m2) = 0
21. Given, Dimeter of cylinder = 2 m
⇒ 4c2 = 4a2 (1 + m2)
Radius of cylinder = 1 m, Hight of cylinder = 5 m
⇒ c2 = a2 (1 + m2) (1 Mark)
Length of park = 25 m, breadth of park = 20 m
17. Out of the Syllabus Let the height of park be h m (1 Mark)
1 1 Now, according to question
18. Given, tm = ⇒ a + (m − 1)d = ...(i)
n n Volume of cylinder = Volume of water
1 1 22
and tn = ⇒ a + (n − 1)d = ...(ii) pr2h = l × b × h ⇒ × (1) 2 × 5 = 25 × 20 × h  (1 Mark)
m m 7
22 × 5
On subtracting eq. (ii) from (i), we get h= ⇒ h = 0.314 m.  (1 Mark)
25 × 20 × 7
1 1 Through recycling of water, better use of natural resource
a + (m – 1)d – a – (n – 1)d = − occurs without wastage. It helps in reducing and preventing
n m
pollution, also in conserving water.  (1 Mark)
m−n 1
⇒ d ( m − n) = ⇒d=  (1 Mark) 22. Given, Side of square = 28 cm
mn mn
Radius of circle = 14 cm (1 Mark)
Put the value of d in eq. (i) we get
Now, according to questions,
1 1 1 1 1 1
a + (m − 1) =⇒ a + − =⇒ a = (1 Mark)
mn n n mn n mn
Now, O
1 1
tmn = a + (mn – 1)d =+ (mn − 1)
mn mn
1 mn O'
= (1 + mn − 1)= = 1 ⇒ tmn = 1 (1 Mark)
mn mn Area of shaded region = area of square + 2 area of circle
19. Out of the Syllabus – 2 overlap area of quadrants  (1 Mark)

246
246 MATHS
1 2 25. Given, SK and SR drawn to circle with centre O from an
= (Side)2 + 2 × pr2 – 2 × pr (1 Mark) external point K.
4
3 3 22 S
= (28) 2 + πr 2 = (28) 2 + × × 14 × 14
2 2 7
= 784 + 924 = 1708 cm2  (1 Mark)
K O
23. Given, two dice are thrown together then total possible
outcomes n(S) = 36 (1 Mark)
The favorable outcome for getting the product of number R
on dice equal to 25 is {(5,5)} 
n(A) = 1 To prove: SK = RK
n( A) 1 Proof: Normal and tangent at a point on the circle are
Now, Probability P(A) = =  (1 Mark) perpendicular to each other.
n( S ) 36
∠OSK = ∠ORK = 90° (1 Mark)
The outcomes when Rina throws a dic are {1,2,3,4,5,6}
Using pythagoras theorem
Total number of out come n(S) = 6 (1 Mark)
OK2 = OS2 + SK2  ...(i)
Rina throws dice and square the number,
OK = OR + RK 
2 2 2
...(ii) (1 Mark)
So, to get product of two number whose square is 25, the
favorable outcomes is 5 From (i) and (ii), we get
Number of favorable outcome n(B) = 1 OS2 + SK2 = OR2 + RK2 ⇒ OR2 + SK2 = OR2 + RK2
n( B ) 1 SK2 = RK2 ⇒ SK = RK ( OS = OR both radius)
Probability P(B) = =  (1 Mark) Hence proved (1 Mark)
n( S ) 6
1 1 26. Given, Speed of boat in still water = 15 km/hr
As > , so Rina has better chance to get the number 25. Let the speed of stream = x km/hr
6 36
 (1 Mark) Now, speed of boat upstream = (15 – x) km/hr
speed of boat downstream = (15 + x) km/hr
24. Given, Radius of circle = 10 cm
30
Time taken for upstream T1 =  (1 Mark)
15 − x
O
Time taken for downstream T2 = 30
10 15 + x
10

60°
P Q 1
According to question, T1 + T2 = 4. hours
R 2
Angle of minor sector at centre = 60° 30 30 9  15 + x + 15 − x  9
⇒ + = ⇒ 30   = (1 Mark)
θ 15 − x 15 + x 2  225 − x 2  2
=
Now, Area of sector × πr 2
360° 30 × 30 9 2 × 900
⇒ = ⇒ 225 − x 2 =  (1 Mark)
area of circle = pr2 (1 Mark) 225 − x 2
2 9
Area of minor segment PRQ ⇒ x2 = 225 – 200
= Area of sector OPRQ – Area of ∆OPQ ⇒ x2 = 25 ⇒ x = 5
60 3 Hence, the speed of stream = 5 km/hr (1 Mark)
= × 3.14 × 102 − × (side) 2
360 4  (1 Mark) 27. Out of the Syllabus
3 28. Out of the Syllabus
= 52.33 – (10) 2 = 52.33 – 43.30 = 9.03 cm2 (1 Mark)
4
29. Given, Sm = Sn
Area of major sector = Area of circle – Area of minor
We know, S= n
segment PRQ n (2a + (n − 1)d )
2
= πr2 – 9.03 = 3.14 × 10 × 10 – 9.03 = 304.97 cm2 m n
Now, [2a + (m − 1)d=
] [2a + (n − 1)d ]  (1 Mark)
 (1 Mark) 2 2

CBSE 2017 (DELHI) 247


⇒ 2am + m2d – md = 2an + n2d – nd 31. Given, height of cone = 30 cm
⇒ 2am – 2an = – (m d – md) + n d – nd
2 2
D
⇒ 2a(m – n) = –(m – n ) d + d (m – n)
2 2

⇒ 2a(m – n) = (m – n)d (1 – m – n) h1

⇒ 2a = d(1 – m – n)  ...(i) (1 Mark) A


C r1
Now, h2=30

m+n
⇒ Sm + n 
=  [2a + (m + n − 1)d ]
 2 
m+n
⇒ Sm + n 
=  [d (1 − m − n) + (m + n − 1)d ]  (1 Mark) O B
 2  r2

m+n
=
⇒ Sm + n   d [1 − m − n + m + n − 1] Let the height of small cone be h1 cm and radius be r1 cm
 2 
and height of bigger cone be h2 cm and radius r2 cm
m+n
⇒= Sm + n d  =  × 0 0 Proved (1 Mark) We know,
 2 
1 2
30. Let the distance between C and D is x m. Volume of cone= πr h  (1 Mark)
3
D
According to questions, given that
1
Volume of small cone = volume of bigger cone
27
15m

1 2 1 1 2
⇒ πr1 h1= πr22 h2 ⇒ r12 h1 = r2 h2  (1 Mark)
(1 Mark) 3 27 × 3 27
2
60° 45° r  h  1
C ⇒ 1   1  =
y y
A B  r2   h2  27
Now, In DACD, 2
r  h  1 r 
2
10
⇒  1  ×  1  = ⇒  1  h1 = ...(i) (1 Mark)
CD 15  r2   30  27  r2  9
⇒ tan 60° = ⇒ 3=
AC x From the figure,
15
x = 5 3 m ...(i) (1 Mark)
⇒= DACD ~ DBOC (AA similarity)
3
r1 h1
again, In DBCD, ⸫ =
r2 30
CD 15
⇒ tan 45° = ⇒ 1=  (1 Mark) Put in equation (i), we get
BC x+ y
2
h  10
⇒ x + y = 15 m [From equation (i)] ⇒  1  h1 = ⇒ h13 = 1000 ⇒ h1= 10 cm
 30  9
y 5(3 − 3) m
y 15 − 5 3 ⇒ =
⇒ =
Height from the base = 30 – 10 = 20 cm
So, distance between points A & B are 5(3 − 3) m. Thus at height 20 cm above the base a small cone is cut.
 (1 Mark)  (1 Mark)

248
248 MATHS
Outside DELHI
2017 CBSE Solved Paper

Time allowed : 3 hours Maximum Marks : 90


GENERAL INSTRUCTIONS:
Read the following instructions very carefully and strictly follow them:
(i) All questions are compulsory.
(ii) The question paper consists of 31 questions divided into four sections – A, B, C and D.
(iii) Section A contains 4 questions of 1 mark each. Section B contains 6 questions of 2 marks each. Section C contains
10 questions of 3 marks each and Section D contains 11 questions of 4 marks each.
(iv) Use of calculators is not permitted.

10. Which term of the A.P. 8, 14, 20, 26, ... will be 72 more
Section-A than its 41st term? (2 Marks)

1. If a tower 30 m high, casts a shadow 10 3 m long on the


Section-C
ground, then what is the angle of elevation of the sun?
(1 Mark) 11. The dimensions of a solid iron cuboid are 4.4 m × 2.6 m ×
2. The probability of selecting a rotten apple randomly from 1.0 m. It is melted and recast into a hollow cylindrical pipe
a heap of 900 apples is 0.18. What is the number of rotten of 30 cm inner radius and thickness 5 cm. Find the length
apples in the heap? (1 Mark) of the pipe. [OS] (3 Marks)
3. What is the common difference of an A.P. in which a21 – a7 12. In the given figure, two concentric circles with centre O
= 84? (1 Mark) have radii 21 cm and 42 cm. If ∠AOB = 60o, find the area
 22 
4. If the angle between two tangents drawn from an external of the shaded region.  Use π =   [OS] (3 Marks)
point P to a circle of radius a and centre O, is 60o, then find  7
the length of OP. (1 Mark)

Section-B
O
5. A line intersects the y-axis and x-axis at the points P and 60o
D
C
Q respectively. If (2, –5) is the mid-point of PQ, then find
the coordinates of P and Q. (2 Marks) A B
6. If the distances of P(x, y) from A(5, 1) and B(–1, 5) are 13. Water in a canal, 5.4 m wide and 1.8 m deep, is flowing
equal, then prove that 3x = 2y. (2 Marks) with a speed of 25 km/hour. How much area can it irrigate
in 40 minutes, if 10 cm of standing water is required for
7. Find the value of p, for which one root of the quadratic
irrigation? (3 Marks)
equation px2 – 14x + 8 = 0 is 6 times the other. (2 Marks)
8. Prove that the tangents drawn at the end points of a chord  24 
14. In what ratio does the point  , y  divide the line
of a circle make equal angles with the chord. (2 Marks)  11 
segment joining the points P(2, –2) and Q(3, 7)? Also find
9. A circle touches all the four sides of a quadrilateral ABCD.
Prove that AB + CD = BC + DA. (2 Marks) the value of y. (3 Marks)
15. On a straight line passing through the foot of a tower, two 24. In a rain-water harvesting system, the rain-water from a
points C and D are at distances of 4 m and 16 m from the roof of 22 m × 20 m drains into a cylindrical tank having
foot respectively. If the angles of elevation from C and D diameter of base 2 m and height 3.5 m. If the tank is
of the top of the tower are complementary, then find the full, find the rainfall in cm. Write your views on water
height of the tower. (3 Marks) conservation. [OS] (4 Marks)
16. A bag contains 15 white and some black balls. If the 25. Prove that the lengths of two tangents drawn from an
probability of drawing a black ball from the bag is thrice external point to a circle are equal. (4 Marks)
that of drawing a white ball, find the number of black balls 26. In the given figure, XY and X’Y’ are two parallel tangents
in the bag. (3 Marks) to a circle with centre O and another tangent AB with point
17. Three semicircles each of diameter 3 cm, a circle of diameter of contact C, is intersecting XY at A and X’Y’ at B. Prove
4.5 cm and a semicircle of radius 4.5 cm are drawn in the that ∠AOB = 90o. (4 Marks)
given figure. Find the area of the shaded region. X P A Y
 [OS] (3 Marks)

O
C

C' Q B Y'
3 cm 3 cm 3 cm
27. If the ratio of the sum of the first n terms of two A.Ps is
18. From a solid right circular cylinder of height 2.4 cm and
(7n + 1) : (4n + 27), then find the ratio of their 9th terms.
radius 0.7 cm, a right circular cone of same height and
same radius is cut out. Find the total surface area of the (4 Marks)
remaining solid. (3 Marks) 1 1 1 3
28. Solve for x: + = 1 , x ≠ ,5  (4 Marks)
19. If the 10th term of an A.P. is 52 and the 17th term is 20 2x − 3 x − 5 9 2
more than the 13th term, find the A.P. (3 Marks)
29. A train covers a distance of 300 km at a uniform speed. If
20. If the roots of the equation (c2 – ab)x2 – 2(a2 – bc)x + b2 – the speed of the train is increased by 5 km/hour, it takes
ac = 0 in x are equal, then show that either a = 0 or a3 + b3 2 hours less in the journey. Find the original speed of the
+ c3 = 3abc.  (3 Marks) train. (4 Marks)
Section-D 30. A man observes a car from the top of a tower, which is
moving towards the tower with a uniform speed. If the
21. If the points A(k + 1, 2k), B(3k, 2k + 3) and C(5k – 1, 5k) angle of depression of the car changes from 30o to 45o in
are collinear, then find the value of k. [OS] (4 Marks) 12 minutes, find the time taken by the car now to reach the
tower. (4 Marks)
22. Two different dice are thrown together. Find the probability
that the numbers obtained have: (4 Marks) 31. In the given figure, ∆ABC is a right-angled triangle in
(i) even sum, and which ∠A is 90o. Semicircles are drawn on AB, AC and
BC as diameters. Find the area of the shaded region.
(ii) even product.
 [OS] (4 Marks)
23. Construct a triangle ABC with side BC = 7 cm, ∠B = 45o,
∠A = 105o. Then construct another triangle whose sides A
3 4c
are times the corresponding sides of the ∆ABC. m
m

4
3c

 [OS] (4 Marks)
B C

250 MATHS
EXPLANATIONS

1. Replacing both a21 and a7 in equation (i), we get,


A (a + 20d) – (a + 6d) = 84
⇒ a + 20d – a – 6d = 84 ⇒ 14d = 84
84
30 m ⇒d= ⇒ d = 6 [Dividing both sides by 14]
14
q \ The common difference of the A.P. is 6. (½ Mark)
B 10 3 m C 4.

Given, height of tower = AB = 30 m A

length of casted shadow = BC = 10 3 m a


30°
P
Let the angle of elevation of the sun be q 60° O
We know,
B
perpendicular AB 30 Given, PA & PB are two tangents drawn from an external
tanq = = ⇒ tanq =  (½ Mark)
base BC 10 3 point P to the circle with centre O.
3
= ⇒ tanq = 3 ⇒ tanq = tan60° ∵ tan 60° = 3 Radius of circle = a
3
Now, angle between the tangents = ∠APB = 60°. (given)
⇒ q = 60° We know, the line segment connecting the external point
\ The angle of elevation of sun is 60°. (½ Mark) & the centre of the circle bisects the angle between the
2. Given the total no. of apples in the heap = 900 tangents.
Probability that a random apple selected is rotten = 0.18 ∠APB 60°
\ ∠APO = = = 30°  (½ Mark)
Let the no. of rotten apples be x. 2 2
We know, Now, in DAPO,
No. of rotten apples ∠PAO = 90° [Q the radius at contact point of circle
P(rotten apples) =  (½ Mark)  & tangent makes 90° with the tangent]
Total no. of apples
x OA = a [radius of the cicle]
⇒ 0.18 =
900 \ In DAPO,
⇒ x = 0.18 × 900 [Multiplying both sides by 900]
OA
⇒ x = 162 sin 30° = [Q OA = perpendicular of DAPO
OP
\ The no. of rotten apples in the heap is 162. (½ Mark)  OP = hypotenure of DAPO]
3. Given, a 1 a 1
⇒ sin30° = ⇒ = [Q sin30° = ]
a21 – a7 = 84 ...(i) OP 2 OP 2
⇒ OP = 2a [By cross multiplying]
Let the first term of the A.P. be a & the common difference
of the A.P. be d. \ The length of OP is 2a. (½ Mark)
We know, general term of an A.P. is, 5. Given, the point P lies in y-axis the point Q lies in x-axis.
an = a + (n – 1)d (½ Mark) Let the point P be P(0, a) & the point Q be Q(b, 0)
Putting n = 21, we get, Given, (2, –5) is the mid-point of PQ
a21 = a + (21 – 1)d = a + 20d But we know, mid-point of PQ is,
And, putting n = 7 we get,
0+b a+0 b a
M , =M ,  (1 Mark)
a7 = a + (7 – 1)d = a + 6d  2 2  2 2

CBSE 2017 (Outside DELHI) 251


b a Multiplying both sides by p, since p ≠ 0.
Hence, comparing both M  ,  & (2, –5), we get,
2 2 [If, p = 0, the equation will not be quadratic]
b a we get,
= 2 and = –5 ⇒ b = 4 and a = –10
2 2 24
= 8 ⇒ p = 3 [cross multiplying both sides]
\ The coordinates of P & Q are, P(0, –10) and Q(4, 0) p
 (1 Mark) Hence, the value of p is 3. (1 Mark)
6. Given, distance of P(x, y) from A(5, 1) and B(–1, 5) are 8. Case I:
equal. To prove: 3x = 2y A
Proof:
Given, PA = PB P
By distance formula
B
⇒ ( x – 5)2 + ( y –1)=
2
( x – (–1))2 + ( y – 5)2  (1 Mark)
AB is a chord of the circle & the tangents are not parallel
⇒ ( x – 5)2 + ( y –1)2 = ( x + 1)2 + ( y – 5)2 to each other
Now, squaring on both sides, we get, Let us assume the two tangents meet each other at point P.
⇒ (x – 5)2 + (y – 1)2 = (x + 1)2 + (y – 5)2 \ DPAB is formed
⇒ x2 + 25 – 10x + y2 + 1 – 2y = x2 + 1 + 2x + y2 + 25 – 10y we know, tangents drawn to a circle from an external point
After cancelling the like terms on both sides, we get, are equal.

⇒ –10x – 2y = 2x – 10y ⇒ 2x + 10x = –2y + 10y Hence, PA = PB

12 x 8 y Now, we know, the angles opposite to equal sides are also


⇒ 12x = 8y ⇒ =  [Dividing both sides by 4] equal.
4 4
⇒ 3x = 2y (1 Mark) Hence, PA = PB
Hence, proved ⇒ ∠PBA = ∠PAB (1 Mark)
7. Given, one root of the quadratic equation px2 – 14x + 8 = 0 Case II:
is 6 times the other. A
Let one root be a
Then, the other root is 6a
Thus, the quadratic equation is, (x – a) (x – 6a) = 0
⇒ x2 – 6ax – ax + 6a2 = 0 ⇒ x2 – 7ax + 6a2 = 0 ...(i) B
The given quadratic equation is, px2
– 14x + 8 = 0 Tangents are parallel to each other & the chord formed is a
14 8 diameter of the circle.
⇒ x2 – x+ = 0  ...(ii) (1 Mark)
p p We know, the radius at a point of circle at the point of
 [dividing both sides by p] contact of tangent & the tangent makes 90° with each other.
Comparing the coefficients of x & the constant terms, in i.e., ∠A = ∠B = 90°
(i) & (ii), we get, Hence, from both the cases we can say, the tangents drawn
–14 2 8 at end points of a chord of a circle make equal angles with
–7a = ⇒a= ...(iii) and 6a2 = ...(iv) the chord. (1 Mark)
p p p
9. Given, a circle touches all the four sides of a quadrilateral
Substituting a in (iv), we get,
ABCD.
2
2 8 4 8 To prove,
6  = ⇒ 6. =
2
 p p p p AB + CD = BC + DA

252 MATHS
A P B 11. Out of the Syllabus
12. Out of the Syllabus
13. Given, dimensions of canal, width (b) = 5.4 m,
S Q
depth (b) = 1.8 m
flowing speed of water, S = 25 km/hr = 25000 m/hr.
D R C
Standing water required for irrigation = 10 cm = 0.1 m
Proof:
Now, volume of water that can be flown in 40 mins,
We know, the tangents drawn from an external point to a
40 3
circle are equal. V = 5.4 × 1.8 × 25000 × m (1 Mark)
60
Hence, here, AP = AS ...(i)  54 18 4
= × × 25000 × m3 = 162000 m3 (1 Mark)
BP = BQ ...(ii) 10 10 6
DR = DS ...(iii) Now, since standing water required = 0.1 m
CR = CQ ...(iv)  (1 Mark) \ Area that can be irrigated in 40 min,
Now, adding (i), (ii), (iii), (iv), we get, 162000 2
A= m = 1620000m2 (1 Mark)
AP + BP + DR + CR = AS + BQ + DS + CQ 0.1
⇒ (AP + BP) + (DR + CR) = (AS + DS) + (BQ + CQ) 14.
 24 
 [Rearranging both sides] A , y 
 11 
⇒ AB + CD = AD + BC ⇒ AB + CD = BC + DA (1 Mark)
Hence, proved. K 1
P(2, –2) Q(3, 7)
10. Given AP is,
8, 14, 20, 26, .... Let A divided PQ in the ratio K : 1.

\ first term, a = 8 \ By section fromula, we have

common difference, d = 14 – 8 = 6.  2 + 3K –2 + 7 K 
A ,  (1 Mark)
 an = a + (n – 1)d  K +1 K +1 

∴ The 41st term is,  24 


But, given, A  , y 
 11 
a41 = a + (41 – 1)d = 8 + 40(6)
Comparing both, we get,
= 8 + 240 ⇒ a41 = 248 (1 Mark)
2 + 3K 24
Let the nth term be 72 more than the 41st term. =
K + 1 11
\ an = a + (n – 1)d = 8 + (n – 1)6
⇒ 22 + 33K = 24K + 24 [by cross multiplication]
Now, according to question,
⇒ 33K – 24K = 24 – 22 ⇒ 9K = 2
an = = 72 + a41
2
⇒ 8 + (n – 1)6 = 72 + 248 [Q a41 = 248] ⇒K=  [Dividing both sides by 9]
9
⇒ 8 + (n – 1)6 = 320 2
Hence, the ratio is, : 1 = 2 : 9 (1 Mark)
⇒ (n – 1)6 = 320 – 8 [subtracting 8 from both sides] 9
⇒ (n – 1)6 = 312 i.e., The point A divides PQ in the ratio 2 : 9.
312 (–2)(9) + 7(2)
⇒ (n – 1) = = 52 [Dividing both sides by 6] y=
6 2+9
⇒ n – 1 = 52 ⇒ n = 52 + 1 [Adding 1 on both sides] –18 + 14 –4
= =
⇒ n = 53 11 11
Hence, the 53rd term of the A.P. is 72 more than the 41st –4
\ The value of y is  (1 Mark)
term. (1 Mark) 11

CBSE 2017 (Outside DELHI) 253


15. 17. Out of the Syllabus
B 18.

h
2.4 cm

q 90° – q
A D
4m C
16 m 0.7 cm
Let the height of the tower be h. Given, height of cylinder = 2.4 cm,
Given, C & D are at a distance of 4 m & 16 m from the foot radius of cylinder = 0.7 cm
of the tower respectively. Now, a right circular cone is cut out from the cylinder with
Let the angle of elevation from the top of the tower of same height & radius (1 Mark)
C & D are q & 90° – q respectively. \ Total surface area of the remaining solid = (curved
[Q The angles are complementary] surface area of cylinder) + (curved surface area of cone) +
\ In DABC: (area of top of the cylinder).
AB h = 2prh + pr + pr2
tan q = ⇒ tan q = ...(i) (1 Mark)
AC 4 Now, slant height of cone,  = h2 + r 2
& In DABD:
= (2.4)2 + (0.7)2 cm = 5.76 + 0.49 cm
AB h
tan (90° – q) = ⇒ cotq =  ...(ii) = 6.25 cm = 2.5 cm (1 Mark)
AD 16
[Q tan(90° – q) = cot q] \ Total surface area of remaining solid
Now, (i) and (ii): (1 Mark) = 2prh + pr + pr2 = pr + (2h +  + r)
h h 22
tan q ⋅ cot q = × = p · 0.7 (2(2.4) + 2.5 + 0.7) = × 0.7 × 8 = 17.6 cm2
4 16 7
h2 Hence, the total surface area of the remaining solid is 17.6 cm2.
⇒1⋅ ⇒ h2 = 64 [Q tan q ⋅ cot q = 1]  (1 Mark)
64
19. Let the first term of the A.P. be a & the common difference
⇒ h = 8 m[h can’t be negative since, it represents length]
be d.
\ The height of the tower is 8 m. (1 Mark)
Now, according to question,
16. Let the no. of black balls in the bag be n.
tenth term, a10 = 52
No. of white balls in the bag = 15
⇒ a + (10 – 1)d = 52 ⇒ a + 9d = 52  ...(i) (1 Mark)
\ Total no. of balls in the bag = 15 + n
Also, 17th term is 20 more than 13th term,
15
Now, P(drawing a white ball) =  i.e., a17 = 20 + a13
15 + n
⇒ a + (17 – 1)d = 20 + a + (13 – 1)d
n
  & P(drawing a black ball) =  (1 Mark) ⇒ a + 16d = 20 + a + 12d
15 + n
Now, according to question, ⇒ 16d = 20 + 12d[subtracting a on both sides]
P(drawing a black ball) = 3 × P(drawing a white ball) ⇒ 16d – 12d = 20 ⇒ 4d = 20 [subtracting 12d on both sides]

n 15 n 45 ⇒d=5 [dividing both sides by 4] (1 Mark)


⇒ = 3× ⇒ =  (1 Mark) Substituing d in equation (i), we get,
15 + n 15 + n 15 + n 15 + n
Q 15 + n ≠ 0, as n is non-negative integer. a + 9(5) = 52 ⇒ a + 45 = 52
So, multiplying both sides by 15 + n. we get, ⇒ a = 7 [subtracting 45 from both sides]
⇒ n = 45 \ The first term is 7 & the common difference is 5.
Hence, the no. of black balls in the bag are 45. (1 Mark) Hence, the AP is 7, 12, 17, 22, .... (1 Mark)

254 MATHS
20. Given, quadratic equation is, 25.
(c2 – ab)x2 – 2(a2 – bc)x + b2 – ac = 0 A
Given, The roots of the eqn are equal.
∴ its discriminant is 0. P O
i.e., D = 0 ⇒ B2 – 4AC = 0 (1 Mark)
B
\ (–2(a2 – bc))2 – 4(c2 – ab) (b2 – ac) = 0
Given, a circle with center O & tangent PA & PB drawn to
⇒ 4(a2 – bc)2 – 4(c2b2 – ac3 – ab3 + a2bc) = 0 the circle from the external point P.
⇒ 4(a4 + b2c2 – 2a2bc) – 4(c2b2 – ac3 – ab3 + a2bc) = 0 Construction, join OA, OP & OB. (1 Mark)
⇒ 4[(a4 + b2c2 – 2a2bc) – (c2b2 – ac3 – ab3 + a2bc)] =0 To prove, length of tangents drawn are equal i.e., PA = PB
 (1 Mark) Proof:
 [Taking 4 common from both terms] We know that, tangents drawn to a circle is perpendicular to
the radius of the circle at the point of contact.
⇒ a4 + b2c2 – 2a2bc – c2b2 + ac3 + ab3 – a2bc =0
\ ∠OAP = ∠OBP = 90° ...(i) (1 Mark)
⇒ a4 + ac3 + ab3 – 3a2bc = 0 ⇒ a[a3 + b3 + c3 – 3abc] = 0
\ In DOAP & DOBP;
 [Taking a common from all the terms]
∠OAP = ∠OBP [from (i)]
⇒ a = 0 or a3 + b3 + c3 – 3abc = 0
OP = OP [common]
⇒ a3 + b3 + c3 = 3abc Hence, prove. (1 Mark)
OA = OB [both are radius of circle]
21. Out of the Syllabus
\ by RHS congourency criteria,
22. The event is that two dies are thrown together
DOAP ≅ DOBP (1 Mark)
\ The sample space is,
Hence, PA = PB
(1, 1) (2, 1) (3, 1) (4, 1) (5, 1) (6, 1) i.e., the length of tangents drawn from an external point to
(1, 2) (2, 2) (3, 2) (4, 2) (5, 2) (6, 2) the circle are equal. (1 Mark)
(1, 3) (2, 3) (3, 3) (4, 3) (5, 3) (6, 3) 26.
P A
(1, 4) (2, 4) (3, 4) (4, 4) (5, 4) (6, 4) X Y
(1, 5) (2, 5) (3, 5) (4, 5) (5, 5) (6, 5) O
(1, 6) (2, 6) (3, 6) (4, 6) (5, 6) (6, 6) C Y′

\ Total no. of outcomes = 36. (2 Marks) X′ Q B

(i) No. of outcomes with even sum = 18 Given, XY & X ′Y ′ are tangents to the circle & are parallel
to each other
no. of outcomes with even sum
\ P(even sum) =
AB is also a tangent to the circle at C.
total no. of outcomes
PQ is the diameter of the circle. (1 Mark)
18 1

= =  (1 Mark) To prove: ∠AOB = 90°
36 2
(ii) No. of outcomes with even product = 27 Proof:
In DPOA & DOCA:
no. of outcomes with even product
\ P(even product) = ∠OPA = ∠OCA = 90° [angle between tangent & radius
total no. of outcomes
27 3  at point of contact is 90°]
= =  (1 Mark)
36 4 OA = OA [common]
23. Out of the Syllabus OP = OC [radii]
24. Out of the Syllabus \ DPOA ≅ DOCA [by RHS congurency]

CBSE 2017 (Outside DELHI) 255


\ ∠POA = ∠AOC ...(i) (1 Mark) 1 1 1 1 1 10
28. Given, + 1 ⇒
= + =
Again, in DOQB & DOCB: (2 x – 3) ( x – 5) 9 (2 x – 3) ( x – 5) 9
∠OQB = ∠OCB = 90° [angle between tangent & radius ( x – 5) + (2 x – 3) 10
 at point of contact is 90°] ⇒ = [Taking LCM on LHS](1 Mark)
(2 x – 3)( x – 5) 9
OB = OB[common] 3x – 8 10
⇒ =
OQ = OC[radii] (2 x – 3)( x – 5) 9
\ DOQB ≅ DOCB [by RHS conguency] ⇒ 9(3x – 8) = 10(2x – 3) (x – 5)
\ ∠QOB = ∠BOC ...(ii) (1 Mark)  [By using cross multiplication]
Therefore, ⇒ 27x – 72 = 10(2x2 – 10x – 3x + 15)
∠AOB = ∠AOC + ∠COB ⇒ 27x – 72 = 10(2x2 – 13x + 15)
1 1 1 ⇒ 27x – 72 = 20x2 – 130x + 150
= ∠POC + ∠COQ = (∠POC + ∠COQ)
2 2 2 ⇒ 20x2 – 130x – 27x + 150 + 72 = 0 (2 Marks)
1  [Arranging the terms on one side]
⇒ ∠AOB = × 180° = 90°
2
⇒ 20x2 – 157x + 222 = 0
Hence, proved: ∠AOB = 90°. (1 Mark)
⇒ 20x2 – 37x – 120x + 222 = 0[Middle term factorisation]
27. Let the first terms of the A.P’s be a & a′ and the common
⇒ x(20x – 37) – 6(20x – 37) = 0
difference be d & d ′.
⇒ (x – 6) (20x – 37) = 0
Given,
37
Ratio of sum of first n terms are (7n + 1) : (4n + 27). ⇒ x – 6 = 0 or 20x – 37 = 0 ⇒ x = 6 or x =
20
Sn 7n + 1 37
i.e., =  (1 Mark) Hence, the values of x are 6 & . (1 Mark)
Sn' 4n + 27 20
29. Let the original speed of the train be x km/hr.
n
(2a + (n – 1)d ) Given, train covers 300 km at uniform speed i.e., x.
7n + 1
⇒ 2 =
n 4 n + 27 Also, if the speed is increased by 5 km/hr, it takes 2 hrs
(2a' + (n – 1)d' )
2 less to complete the journey.
 n –1  Therefore,
a+ d
 2  = 7n + 1 300 300 distance
⇒ ...(i) (1 Mark)
 n –1  4n + 27 − 2 [Q Time =
= ] (1 Mark)
a' +   d' x x+5 speed
 2 
300( x + 5) – 300( x)
To get the ratio of 9th term, ⇒ = 2 [Taking LCM or LHS]
x( x + 5)
n –1
Let, = 8 [Q for 9th term (n – 1) = 9 – 1 = 8] ⇒ 300x + 1500 – 300x = 2x(x + 5) [Multiplying x(x + 5)
2
 on both sides, since, x(x + 5) ≠ 0] (1 Mark)
⇒ n = 17 (1 Mark)
⇒ 2x2 + 10x = 1500
\ Ratio of 9th terms,
⇒ 2x2 + 10x – 1500 = 0 [subtracting 1500 on both sides]
Ta a + 8d
= ⇒ x2 + 5x – 750 = 0 [dividing both sides by 2] (1 Mark)
Ta' d + 8d'
⇒ x2 + 30x – 25x – 750 = 0 [Middle term factorization]
from equation (i), we have
⇒ x(x + 30) – 25(x + 30) = 0 ⇒ (x – 25) (x + 30) = 0
7(17) + 1 120 24 ⇒ x – 25 = 0 or x + 30 = 0 ⇒ x = 25 or x = –30
= = =
4(17) + 27 95 19 Since, speed can’t be negatives, x = 25 km/hr.
\ The ratio of 9th terms of the APs is 24 : 19. (1 Mark) Hence, the original speed is 25 km/hr. (1 Mark)

256 MATHS
30. From, (i) and (ii), we get,
B
x + y = x 3 ⇒ y = x 3 – x [subtracting x from both sides]
30°
45° = x( 3 – 1) (1 Mark)
Therefore, the car took 12 minutes to reach from D to C
h i.e., the distance, x( 3 – 1).
distance x( 3 –1)
\ speed of the car, S = =  (1 Mark)
45° 30° time 12
A D
C Now, time taken by car to reach from C to D,
x y
i.e., to cover x units is,
Given, the angle of depression of the car at position D is
30° & at position C is 45°. distance x 12
time taken = = =
Clearly, the angle of elevation at D is 30° & at C is 45°. speed x( 3 –1) 3 –1
Let the distances AC & CD be x & y respectively & the 12
highest of tower be h. 12( 3 + 1) 12( 3 + 1) 12( 3 + 1)
= = = = 6( 3 + 1)
\ In DABC: ( 3 –1)( 3 + 1) 3 –1 2
h h = 16.4 min (approx)
tan45° = ⇒1= ⇒ x = h ...(i) (1 Mark)
x x
and in DABD: \ The time taken by the car to reach the tower is 16.4 min.
h 1 h  (1 Mark)
tan30° = ⇒ = ⇒ x + y = h 3 ...(ii)
x+ y 3 x + y 31. Out of the Syllabus

CBSE 2017 (Outside DELHI) 257


DELHI Term-II
2016 CBSE Solved Paper

Time allowed : 3 hours Maximum Marks : 90


GENERAL INSTRUCTIONS:
Read the following instructions very carefully and strictly follow them:
(i) All questions are compulsory.
(ii) The question paper consists of 31 questions divided into four sections – A, B, C and D.
(iii) Section A contains 4 questions of 1 mark each, Section B contains 6 questions of 2 marks each, Section C contains
10 questions of 3 marks each and Section D contains 11 questions of 4 marks each.
(iv) Use of calculators is not permitted.

6. In Fig., a circle is inscribed in a ∆ABC, such that it touches


Section-A the sides AB, BC and CA at points D, E and F respectively.
If the lengths of sides AB, BC and CA are 12 cm, 8 cm and
1. Cards marked with number 3, 4, 5, ....., 50 are placed in
10 cm respectively, find the lengths of AD, BE and CF.
a box and mixed thoroughly. A card is drawn at random
 (2 Marks)
from the box. Find the probability that the selected card C
bears a perfect square number. (1 Mark)
2. In Fig., AB is a 6 m high pole and CD is a ladder inclined at
an angle of 60° to the horizontal and reaches up to a point
D of pole. If AD = 2.54 m, find the length of the ladder.
(Use 3 = 1.73) (1 Mark) F E
A

2.54 A B
D D
7. In Fig., AP and BP are tangents to a circle with centre O,
6m such that AP = 5 cm and ∠APB = 60°. Find the length of
chord AB. (2 Marks)
P
B 60o C
60o
3. Find the 9 term from the end (towards the first term) of
th

the A.P. 5, 9, 13, ....., 185. (1 Mark)


4. From an external point P, tangents PA and PB are drawn
to a circle with centre O. If ∠PAB = 50o, then find ∠AOB. A B
(1 Mark)

Section-B O

5. The x-coordinate of a point P is twice its y-coordinate. 2


8. If x = and x = –3 are roots of the quadratic equation
If P is equidistant from Q(2, –5) and R(–3, 6), find the 3
coordinates of P. (2 Marks) ax2 + 7x + b = 0, find the values of a and b. (2 Marks)
9. Find the ratio in which y-axis divides the line segment 16. In Fig. are shown two arcs PAQ and PBQ. Arc PAQ is a
joining the points A(5, –6) and B(–1, –4). Also find the part of circle with centre O and radius OP while arc PBQ
coordinates of the point of division. (2 Marks) is a semi-circle drawn on PQ as diameter with centre M.
10. How many terms of the A.P. 27, 24, 21, .... should be taken If OP = PQ = 10 cm show that area of shaded region is
so that their sum is zero? (2 Marks)  π
25  3 −  cm 2 . [OS] (3 Marks)
 6
P
Section-C
cm
11. If the sum of first 7 terms of an A.P. is 49 and that of its 10 5 cm
first 17 terms is 289, find the sum of first n terms of the B
O M A
A.P. (3 Marks)
12. A well of diameter 4 m is dug 21 m deep. The earth taken 5 cm
out of it has been spread evenly all around it in the shape
of a circular ring of width 3 m to form an embankment. Q
Find the height of the embankment. [OS] (3 Marks) 17. The angles of depression of the top and bottom of a
13. In Fig., ABCD is a square of side 14 cm. Semi-circles are 50 m high building from the top of a tower are 45° and
drawn with each side of square as diameter. Find the area 60° respectively. Find the height of the tower and the
 22  horizontal distance between the tower and the building.
of the shaded region.  Use π =   [OS] (3 Marks) (use 3 = 1.73) (3 Marks)
 7  x +1 x − 2 2x + 3
A B 18. Solve for x: + = 4− ; x ≠ 1, −2, 2  (3 Marks)
x −1 x + 2 x−2
19. Two different dice are thrown together. Find the probability
of : (3 Marks)
(i) getting a number greater than 3 on each die
(ii) getting a total of 6 or 7 of the numbers on two dice
D C 20. A right circular cone of radius 3 cm, has a curved
14. In Fig., is a decorative block, made up of two solids–a surface area of 47.1 cm2. Find the volume of the cone.
cube and a hemisphere. The base of the block is a cube (use π = 3.14) (3 Marks)
of side 6 cm and the hemisphere fixed on the top has a
diameter of 3.5 cm. Find the total surface area of the block. Section-D
 22 
 Use π =   (3 Marks) 21. A passenger, while boarding the plane, slipped from the
 7  stairs and got hurt. The pilot took the passenger in the
emergency clinic at the airport for treatment. Due to
this, the plane got delayed by half an hour. To reach the
destination 1500 km away in time, so that the passengers
could catch the connecting flight, the speed of the plane
was increased by 250 km/hour than the usual speed. Find
the usual speed of the plane.
15. In Fig., ABC is a triangle coordinates of whose vertex A What value is depicted in this question? (4 Marks)
are (0, –1). D and E respectively are the mid-points of the 22. In Fig., O is the centre of a circle of radius 5 cm. T is a
sides AB and AC and their coordinates are (1, 0) and (0, 1) point such that OT = 13 cm and OT intersects circle at E.
respectively. If F is the mid-point of BC, find the areas of If AB is a tangent to the circle at E, find the length of AB,
∆ABC and ∆DEF. [OS] (3 Marks) where TP and TQ are two tangents to the circle. (4 Marks)
A(0, –1) P
A
5

D(1, 0) E(0, 1) O E
T
13
5
B
B C Q
F

CBSE 2016 (DELHI Term-II) 259


23. Prove that the lengths of tangents drawn from an external A
point to a circle are equal. (4 Marks)
5 cm
24. Prove that the area of a triangle with vertices (t, t – 2),
θ C
(t + 2, t + 2) and (t + 3, t) is independent of t. O P
 [OS] (4 Marks)
25. A game of chance consists of spinning an arrow on a
circular board, divided into 8 equal parts, which comes to B
rest pointing at one of the numbers 1, 2, 3, ..., 8 (Fig.), 27. A bucket open at the top is in the form of a frustum of a
which are equally likely outcomes. What is the probability cone with a capacity of 12308.8 cm3. The radii of the top
and bottom circular ends are 20 cm and 12 cm respectively.
that the arrow will point at (i) an odd number (ii) a number
Find the height of the bucket and the area of metal sheet
greater than 3 (iii) a number less than 9. (4 Marks)
used in making the bucket. (Use π = 3.14)[OS] (4 Marks)
28. The angles of elevation of the top of a tower from two
3 2 points at a distance of 4 m and 9 m from the base of the
tower and in the same straight line with it are 60° and 30°
4 1 respectively. Find the height of the tower. (4 Marks)
29. Construct a triangle ABC in which BC = 6 cm, AB = 5 cm
5 8 and ∠ABC = 60°. Then construct another triangle whose
6 7 sides are 3/4 times the corresponding sides of ∆ABC.
 [OS] (4 Marks)
30. The perimeter of a right triangle is 60 cm. Its hypotenuse
26. An elastic belt is placed around the rim of a pulley of is 25 cm. Find the area of the triangle. [OS] (4 Marks)
radius 5 cm. (Fig.) From one point C on the belt, the elastic
31. A thief, after committing a theft, runs at a uniform speed
belt is pulled directly away from the centre O of the pulley of 50 m/minute. After 2 minutes, a policeman runs to catch
until it is at P, 10 cm from the point O. Find the length of him. He goes 60 m in first minute and increases his speed
the belt that is still in contact with the pulley. Also find the by 5 m/minute every succeeding minute. After how many
shaded area. (Use π = 3.14 and 3 = 1.73) (4 Marks) minutes, the policeman will catch the thief? (4 Marks)

EXPLANATIONS

1. Given: Box contains marked, with number 3, 4,....50 Let the length of ladder be l meters ½ Mark)

Number of element n(s) = 48 Now, In DDBC,
Perfect square between 3 to 50 are 4, 9, 16, 25, 36, 49 BD 3 6 − 2.54
sin 60° = ⇒ =
favourable outcomes n(A) = 6 ½ Mark)
(½ DC 2 l
favourable outcomes 6 1 2 × 3.46 6.92
we know, Probability = = = 
total outcomes 48 8 ⇒ l
= = ⇒ l = 4 m ½ Mark)

3 1.73
 ½ Mark)

A 3. Given sequence 5, 9, 13, ....185
2. we need to find 9th term from the end So, we can reverse
2.54 the sequence 185, 181, 177, ... 9, 5 ½ Mark)

D Now a =185, d = 181 – 185 = –4 and n = 9
6m We know, tn = a + (n – 1)d
tn = 185 + (9 – 1) × (–4) = 185 – 32 = 153
Hence, 9th term from the end is 153 ½ Mark)

B 60o C
260
260 MATHS
4. then, AB = AD + BD ⇒ 12 = x + y ...(i)
A BC = BE + EC ⇒ 8 = y + z...(ii)
50° AC = AF + FC ⇒ 10 = x + z ...(iii) (1 Mark)
O adding eqn.(i), (ii) and (iii), we get
P
12 + 8 + 10 = x + y + y + z + x + z
⇒ 30 = 2(x + y + z) ⇒ x + y + z = 15 ...(iv)
B
put eqn. (i), in eqn. (iv), we get
Given; PA = PB and ∠PAB = 50° ⇒ 12 + z = 15 ⇒ z = 3
∠PBA = ∠PAB (angle opposite to equal sides) Put z = 3 in eqn. (iii)
Now, In DPAB ⇒ 10 = x + 3 ⇒ x = 7
∠PBA + ∠PAB + ∠APB = 180 (angle sum property) Put the value of x in eqn. (i), we get
⇒ ∠APB = 180° – ∠PBA – ∠PAB = 180° – 50° – 50° x + y =12 ⇒ 7 + y =12 ⇒ y = 5
⇒ ∠APB = 80° ½ Mark)
(½ Hence, AD = 7, BE = 5, CF = 3 (1 Mark)
Now ∠AOB + ∠APB = 180° (Supplementary angles) 7.
P
⇒ ∠AOB + 80° = 180° ⇒ ∠AOB = 100° ½ Mark)

5. Let the coordinate of P be (x, y) 60o
Given: x = 2y  ...(i)
Now, according to question
⸫ PQ = PR A B

( x − 2) 2 + ( y + 5) 2 = ( x + 3) 2 + ( y − 6) 2  (1 Mark) O
on squaring
⇒ (x – 2)2 + (y + 5)2 = (x + 3)2 + (y – 6)2 Given; AP = 5 cm and ∠APB = 60°
⇒ x2 + y2 – 4x + 10y + 29 = x2+ y2 + 6x – 12y + 45 also, PA = PB (tangents are equal from External Point)
⇒ 10x – 22y + 16 = 0 ⸫ ∠ABP = ∠BAP (angles opposite to equal sides)
put the value of x = 2y, we get Now, ∠ABP + ∠BAP + ∠APB = 180° (1 Mark)
⇒ 10(2y) – 22y + 16 = 0 ⇒ –2y = – 16 2∠ABP + 60° = 180° ⇒ 2∠ABP = 120 ⇒∠ABP = ∠BAP = 60°
⇒ y = 8 then x = 16 So, ∠ABP is Equilateral triangle
Hence the coordinate of P is (16, 8) (1 Mark) AP = BP = AB = 5 cm. (1 Mark)
6. C 2
8. Given x = and x = –3 are roots of ax2 + 7x + b = 0
3
then these points with satisfy the given quadratic equation
z z 2
at x =
3
F E 2
2 2 4 14
⇒ a  + 7  + b = 0 ⇒ a+ +b = 0
x y 3 3 9 3
A B ⇒ 4a + 9b + 42 = 0 at x = –3 ...(i)
x D y
⇒ a(–3) + 7 (–3) + b = 0
2

Given; AB = 12 cm, AC = 10 cm, BC = 8 cm


⇒ 9a + b = 21 ⇒ b = 21 – 9a...(ii) (1 Mark)
Now, from figure we can says that put the value of b in eqn. (i) we get
AD = AF = x (let) [⸪ tangent drawn from an external point ⇒ 4a + 9(21 – 9a) + 42 = 0 ⇒ 4a + 189 – 81a + 42 = 0
to a circle are equal]
231
Similarly BD = BE = y (let) and CF = CE = z (let) ⇒ 77a = 231 ⇒ a = =3
77

CBSE 2016 (DELHI Term-II) 261


put the value of a in eqn. (ii), we get put in eqn. (i), we get, a = 7 – 3 × 2 = 1
b = 21 – 3 × 9 = –6 (1 Mark) n
Now, Sn = [2 × 1 + (n – 1)2] = n(1 + n – 1) = n2 (1 Mark)
9. Let y-axis divides the line segment in the ratio k : 1 2
Given, A(5, –6), B(–1,–4) and y-axis (0, y) 12. Out of the Syllabus
We know 13. Out of the Syllabus
mx2 + nx1 my2 + ny1  14. Given; Side of a Cube = 6 cm
( x, y ) =  ,
 m+n m + n  Total surface area of cube = 6 × (Side)2 = 216 cm2
 k ⋅ ( −1) + 1× 5 k ( −4 ) + 1× ( −6 )  Area coverd on the face of cube by circular part of
( 0, y ) =  ,  22 3.5 3.5 77
 1+ k 1+ k  hemisphere = pr2 = × × = (1 Mark)
 −k + 5 −4k − 6   7 2 2 8
∴ ( 0, y ) =  ,  (1 Mark) curved surface area of hemisphere
 1+ k 1+ k 
On compairing, we get 22 3.5 3.5 77 
= 2pr2 = 2 × × × = (1 Mark)
7 2 2 4
−k + 5 Now total surface area = Surface area of cube – area of
= 0 ⇒ –k + 5 = 0 ⇒ k = 5
k +1 circular part + area of hemisphere
−4k − 6
and y = ...(i) 77 77 1728 − 77 + 154
k +1 = 216 − − = = 225.625 cm2
8 4 8
Put k = 5 in eqn. (i) 
(1 Mark)
−4 × 5 − 6 −20 − 6 −26 −13 15. Out of the Syllabus
⇒ y= = − = =
5 +1 6 6 3
16. Out of the Syllabus
 –13 
Hence point on y-axis is  0,   (1 Mark) 17. B
 3  45º
10. Given: Sequence 27, 24, 21, ...... 60º
Sn = 0, d = 24 – 27 = –3, a = 27 45º
h T P
we know,
n n 50
Sn = (2a + (n – 1)d) ⇒ 0 = (2 × 27 + (n – 1) (–3))
2 2 60º
 (1 Mark) A Q
x
0 = n(54 – 3n + 3) ⇒ 0 = n(57 – 3n)
on comparing we get BT h − 50
In DBTP tan 45° = ⇒ 1=
PT x
n = 0 and 57 – 3n = 0 ⇒ 3n = 57 ⇒ n = 19  (1 Mark)
⇒ x = h – 50 ...(i) (1 Mark)
11. Given S7 = 49 and S17 = 289 AB h h
In DBAQ, tan 60° = ⇒ 3= ⇒x= ...(ii)
n AQ x 3
We know, Sn = [2a + (n – 1)d]
2 From eqn. (i) and (ii), we get
7
Now S7 = [2a + (7 – 1)d] h h
7
2 ⇒ h − 50 =
3
⇒ h−
3
= 50 ⇒ h ( )
3 − 1 = 50 3 
49 = (2a + 6d) ⇒ 2a + 6d = 14 
2
a + 3d = 7 ⇒ a = 7 – 3d  ...(i) (1 Mark) 50 3 50 × 1.732
h
⇒= h
⇒= =118.25 m (1 Mark)
17 3 −1 1.732 − 1
also S17 = (2a + 16d)
2 118.25 118.25
17 (ii) x
From equation= = = 68.25 m
289 = (2a + 16d) ⇒ 2a + 16d = 34 3 1.732
2
⇒ a + 8d = 17 ⇒ 7 – 3d + 8d = 17 ⇒ 5d = 10 ⇒ d = 2 Hence, height as the Tower = 118.25 m. and distance
 (1 Mark) between tower and building = 68.25 m  (1 Mark)

262
262 MATHS
x +1 x − 2 2x + 3 21. Let the usual speed of the plane be S km/h and the usual
18. Given, + 4−
= time it takes to reach the destination be t hr
x −1 x + 2 x−2
Given, Distance d = 1500 km
( x + 1)( x + 2 ) + ( x − 2 )( x − 1)2x + 3
⇒ = 4−  (1 Mark) Distance 1500 1500
( x − 1)( x + 2 ) x−2 Speed =
time
⇒ S= ⇒t =  (1 Mark)
t S
x 2 + 3x + 2 + x 2 − 3x + 2 4 x − 8 − 2 x − 3
⇒ = The plane got delayed by half an hour and speed was
x2 + x − 2 x−2 increased by 250 km/hr to reach the destination on time.


( 2x 2
+ 4)
2 x − 11
=
Now, new speed (S + 250) km/hr
2
x + x−2 x−2 1  1500 1 
time = (t – ) hr =  −  (1 Mark)
⇒ (2x2 + 4)(x – 2) = (2x – 11)(x2 + x – 2) 2  S 2
⇒ 2x3 – 4x2 + 4x – 8 = 2x3 + 2x2 – 4x – 11x2 – 11x + 22 Now, distance = Speed × time
 (1 Mark) 1
⇒ (S + 250) (t – ) = 1500
⇒ 5x2 + 19x – 30 = 0 2
⇒ 5x2 + 25x – 6x – 30 = 0 ⇒ 5x(x + 5) – 6(x + 5)=0  1500 1 
⇒ (S + 250)  −  = 1500
⇒ (5x – 6)(x + 5) = 0  S 2
⇒ (S + 250)(3000 – S)= 3000 S
6
⇒ x = –5, x =  (1 Mark)
5 ⇒ 3000 S – S 2 + 250 × 3000 – 250 S = 3000 S (1 Mark)
19. Given, two dice are thrown simultaneously then maximum ⇒ –S 2 – 250 S + 250 × 3000 = 0
possible outcome. n(s) = 36
⇒ S2 + 250 S – 250 × 3000 = 0
(a) Let A be the event of getting a number greater than 3
on each die. ⇒ S2 – 750 S + 1000 S – 750 × 3000 = 0
favourable outcomes = {(4, 4), (4, 5), (4, 6), (5, 4), ⇒ S(S – 750) + 1000(S – 750) = 0
(5, 5), (5, 6), (6, 4), (6, 5), (6, 6)} ⇒ (S + 1000) (S – 750) = 0
n (A) = 9 ½ Mark)
(½ ⇒ S = –1000, S = 750 (Neglect)
Now, So, usual speed S = 750 km/hr
favourable outcomes 9 1 1500
Probability = = =  Usual time t = = 2 hrs (1 Mark)
Total possible outcomes 36 4 750
 (1 Mark) P
22.
A
(b) Let B be the event of getting a total of 6 or 7 of the 5
numbers on two dice.
O E
favourable outcomes = {(1, 5), (2, 5), (1, 6), (2, 4), T
13
(3, 3), (3, 4), (4, 3), (4, 2), (5, 1), (5, 2), (6, 1)} 5
 ½ Mark)
(½ B
Q
n ( B ) 11
=
P(B) =  (1 Mark)
n ( S ) 36
Given OP = OQ = 5 cm, OT =13 cm
20. Given, Radius 3 cm, curved surface area = 47.1 cm2 Also, PT = TQ (tangent from same external point)
we know, In DOPT, ∠P = 90° (1 Mark)
Curved surface area of cone = prl OT = OP + PT ⇒ 13 = 5 + PT ⇒ PT = 12cm = PQ
2 2 2 2 2 2

47.1 47.1 Since length of tangents drawn from a point to a circle are
⇒ 47.1 = p × 3 × l ⇒ l = = = 5 cm (1 Mark)
3π 3 × 3.14 equal.
Now,
Therefore
12 = r2 + h2 ⇒=
h =
l 2 − h2 ⇒ h 52 − 32 = 4 cm AP = AE = x (let)
 (1 Mark)
⇒ AP = AE = PT – AT = 12 – AT ⇒ AT = 12 – AP
1 1 ⇒ AT = (12 – x) cm
⸫ Volume of cone = pr2h = × 3.14 × 32 × 4
3 3
⇒ V = 37.68 cm3 (1 Mark) and OT = OE + ET ⇒ 13 = 5 + ET ⇒ ET = 8 cm (1 Mark)

CBSE 2016 (DELHI Term-II) 263


Now, In DAET, 26. A
AT = AE + ET ⇒ (12 – x) = x + 8
2 2 2 2 2 2
5 cm
⇒ 144 – 24x + x2 = x2 + 64 ⇒ 24x = 80
10 θ C
⇒x= cm (1 Mark) O P
3 10
10
Similarly BE = cm
3
B
10 10 20
\ AB = AE + BE = + = cm  (1 Mark) we know that, tangent of a circle is perpendicular to the
3 3 3
radius of the circle through the point of contact
23. S ∠OAP = 90°
OA 5 1
cosq = ⇒ cosq = = 
O K OP 10 2
⇒ cosq = cos 60° ⇒ q = 60°
R ⇒ ∠AOB = 60° + 60° = 120°
Let SK and RK are the two tangents on circle with Centre Reflex ∠AOB = 360° – 120° = 240° (1 Mark)
O from an external point K, (1 Mark) we know, length of belt in contact with pulley = ADB
OS = OR (Radii) 240°
⇒ Length of major arc = × 2 × 3.14 × 5
also, ∠OSK = ∠ORK = 90° (1 Mark) 360°
62.8
In DSOK, OK 2 = OS 2+ SK 2 ...(i) = = 20.93 cm
3
In DROK, OK 2 = OR2 + RK 2 ...(ii) (1 Mark) Now, In DOAP, we have
from eqn. (i) and (ii), we get AP 3 AP
sin 60° = ⇒ = ⇒ AP = 5 3 cm  (1 Mark)
OS + SK = OR + RK
2 2 2 2
OP 2 10
SK 2 = RK 2 ⇒ SK = RK  (\ OS = OR) 1 1
Area of triangle = × base × height = ×AP × OA
2 2
Hence, tangent from an external point to the circle are
equal (1 Mark) 1 25 3
= × 5 3 ×5= cm2 (1 Mark)
2 2
24. Out of the Syllabus
θ
25. Chances of arrow = {1, 2, 3, 4, 5, 6, 7, 8} Area of minor sector = × p r2
360°
n(s) = 8
120°
= × 3.14 × 5 × 5 = 26.170
we know, 360°
Favourable outcome Area of Shaded region: ar(DOAB) + ar(DOBP) – area of
Probability =  (1 Mark)
Total outcomes minor sector OABC
(a) Let A be a event of an odd number. 25 3 25 3
= + = 26.17 = 17.08 cm (1 Mark)
n ( A) 4 1 2 2
P ( A=
) = =  (1 Mark)
n(S ) 8 2 27. Out of the Syllabus
(b) Let B be a Event of a number greater than 2 28. A 
B = {4, 5, 6, 7, 8}
5 h
n(B) = 5 ⇒ P ( B ) =  (1 Mark)
8 60°
(c) Let C be a event of a number less than 9 30°
D C B
C = {1, 2, 3, 4, 5, 6, 7, 8}
4m
8 9m
n(C) = 8 ⇒ P ( C )=
= 1 (1 Mark)
8 Let h be the height of the tower.

264
264 MATHS
BC = 4 m, BD = 9 m (1 Mark) 31. According to question,
Distance between thief and police = 100 m
Now. In DABC,
Police starts chasing thief with initial speed of 60 and
AB h increasing his speed by 5 m/min. (1 Mark)
tan 60° = ⇒ 3= ⇒h=4 3 (1 Mark)
BC 4 Let police chase the thief fox t minutes
In DABD, ⇒ thief cover distance = (100 + 50t) m
Police runs with pattern 60, 65, 90, ..... (1 Mark)
AB h This is an Arithmetic progression
tan 30° = =
BD 9 a = 60, d = 5 and n = t
1 h 9 n
⇒ = ⇒h= =3 3m Sn = (2a + (n – 1)d) (1 Mark)
3 9 3 2
Here, question contains surplus data which does not lead t
⇒ (2 × 60+ (t – 1)5) = 50t + 100 ⇒ 5t2 + 15t – 200 = 0
to a unique solution. (2 Marks) 2
−3 ± 13
29. Out of the Syllabus ⇒ t2 + 3t – 40 = 0 ⇒ t = ⇒t=5
2
30. Out of the Syllabus Hence, police will catch thief in 5 minutes (1 Mark)

CBSE 2016 (DELHI Term-II) 265


Outside DELHI Term-II
2016 CBSE Solved Paper

Time allowed : 3 hours Maximum Marks : 90


GENERAL INSTRUCTIONS:
Read the following instructions very carefully and strictly follow them:
(i) All questions are compulsory.
(ii) This question paper consists of 31 questions divided into four sections – A, B, C and D.
(iii) Section A contains 4 questions of 1 mark each. Section B contains 6 questions of 2 marks each, Section C contains
10 questions of 3 marks each and Section D contains of 11 questions of 4 marks each.
(iv) Use of calculators is not permitted.

D R C
Section-A
1. A ladder, leaning against a wall, makes an angle of 60o S Q
O
with the horizontal. If the foot of the ladder is 2.5 m away
from the wall, find the length of the ladder. (1 Mark)
2. For what value of k will k + 9, 2k – 1 and 2k + 7 are the B
A P
consecutive terms of an A.P.? (1 Mark)
3. In fig., PQ is a tangent at a point C to a circle with centre 6. The 4th term of an A.P. is zero. Prove that the 25th term of
O. If AB is a diameter and ∠CAB = 30o, find ∠PCA. the A.P. is three times its 11th term. (2 Marks)
 (1 Mark) 7. In Fig., from an external point P, two tangents PT and PS
P are drawn to a circle with centre O and radius r. If OP =2r,
C show that ∠OTS = ∠OST = 30o. (2 Marks)

Q T
A B
O
P O
Q

4. A card is drawn at random from a well shuffled pack of 52


S
playing cards. Find the probability of getting neither a red
card nor a queen. (1 Mark)
8. Prove that the points (3, 0), (6, 4) and (–1, 3) are the
Section-B vertices of a right angled isosceles triangle. (2 Marks)
9. Let P and Q be the points of trisection of the line segment
5. In Fig., a quadrilateral ABCD is drawn to circumscribe a
joining the points A(2, –2) and B(–7, 4) such that P is
circle, with centre O, in such a way that the sides AB, BC,
nearer to A. Find the coordinates of P and Q. (2 Marks)
CD and DA touch the circle at the points P, Q, R and S
respectively. Prove that AB + CD = BC + DA. (2 Marks) 10. Solve for x: 2x + 9 + x =
13  (2 Marks)
18. The digits of a positive number of three digits are in A.P.
Section-C and their sum is 15. The number obtained by reversing
the digits is 594 less than the original number. Find the
11. If the point P(x, y) is equidistant from the points A(a + b,
number. (3 Marks)
b – a) and B(a – b, a + b). Prove that bx = ay. (3 Marks)
19. If the roots of the quadratic equation (a – b)x2 + (b – c)x +
12. A conical vessel, with base radius 5 cm and height 24 cm, is
(c – a) = 0 are equal, prove that 2a = b + c. (3 Marks)
full of water. This water is emptied into a cylindrical vessel
of base radius 10 cm. Find the height to which the water will 20. From a pack of 52 playing cards, Jacks, Queens and Kings
22 of red colour are removed. From the remaining, a card is
rise in the cylindrical vessel. (Use π = )  (3 Marks) drawn at random. Find the probability that drawn card is:
7
 (3 Marks)
13. In fig., O is the centre of a circle such that diameter
(i) a black King
AB = 13 cm and AC = 12 cm. BC is joined. Find the area
of the shaded region. (Take π = 3.14) [OS] (3 Marks) (ii) a card of red colour
A (iii) a card of black colour

Section-D
O
21. Due to heavy floods in a state, thousands were rendered
homeless. 50 schools collectively offered to the state
B C government to provide place and the canvas for 1500 tents
14. A sphere of diameter 12 cm, is dropped in a right circular to be fixed by the government and decided to share the
cylindrical vessel, partly filled with water. If the sphere is whole expenditure equally. The lower part of each tent is
completely submerged in water, the water level in the cylindrical of base radius 2.8 m and height 3.5 m, with
5 conical upper part of same base radius but of height 2.1 m.
cylindrical vessel rises by 3 cm. Find the diameter of If the canvas used to make the tents costs ` 120 per sq.m,
the cylindrical vessel. 9 (3 Marks) find the amount shared by each school to set up the
15. In fig., a tent is in the shape of a tents. What value is generated by the above problem?
22
2.

cylinder surmounted by a conical top


8

(Use π = )  (4 Marks)
m

of same diameter. If the height and 7


diameter of cylindrical part are 2.1 22. In Fig., two equal circles, with centres O and O', touch
m and 3 m respectively and the slant each other at X.OO' produced meets the circle with centre
height of conical part is 2.8 m, find 2.1 m
O' at A. AC is tangent to the circle with centre O, at the
the cost of canvas needed to make point C. O'D is perpendicular to AC. Find the value of
the tent if the canvas is available DO '
. (4 Marks)
at the rate of ` 500 per sq.metre. CO
22 C
(Use π = )  (3 Marks) 3m
7
16. In fig., find the area of the shaded region, enclosed D
between two concentric circles of radii 7 cm and 14 cm A O
where ∠AOC = 40o. (Use π = 22/7)  [OS] (3 Marks) O' X

23. A number x is selected at random from the numbers 1, 2,


O 3 and 4. Another number y is selected at random from the
40o numbers 1, 4, 9 and 16. Find the probability that product
B D of x and y is less than 16.  (4 Marks)
A C 24. In fig., the vertices of ∆ABC are A(4, 6), B(1, 5) and
C(7, 2). A line-segment DE is drawn to intersect the sides
17. A man standing on the deck of a ship, which is 10 m above
AD AE 1
water level, observes the angle of elevation of the top of AB and AC at D and E respectively such that = = .
a hill as 60o and the angle of depression of the base of hill AB AC 3
as 30o. Find the distance of the hill from the ship and the Calculate the area of ∆ADE and compare it with area of
height of the hill. (3 Marks) ∆ABC. [OS] (4 Marks)

CBSE 2016 (Outside DELHI Term-II) 267


A(4, 6) numbers of houses preceeding the house numbered X is
equal to sum of the numbers of houses following X.
  (4 Marks)
D E 27. A motor boat whose speed is 24 km/h in still water takes
1 hour more to go 32 km upstream than to return downstream
to the same spot. Find the speed of the stream. (4 Marks)
B(1, 5) C(7, 2) 28. Draw an isosceles ∆ABC in which BC = 5.5 cm and altitude
25. In Fig., is shown a sector OAP of a circle with centre AL = 3 cm. Then construct another triangle whose sides are
O, containing ∠θ. AB is perpendicular to the radius OA 3/4 of the corresponding sides of ∆ABC. [OS] (4 Marks)
and meets OP produced at B. Prove that the perimeter of 29. Prove that tangent drawn at any point of a circle is
 πθ  perpendicular to the radius through the point of contact.
shaded region is r  tan θ + sec θ + − 1 .  (4 Marks)
 180  (4 Marks)
B 30. As observed from the top of a light house, 100 m high
above sea level, the angles of depression of a ship,
P sailing directly towards it, changes from 30o to 60o. Find
the distance travelled by the ship during the period of
observation. (Use 3 = 1.73 ) (4 Marks)
θ
O A 31. A rectangular park is to be designed whose breadth is
r
3 m less than its length. Its area is to be 4 square metres
more than the area of a park that has already been made
in the shape of an isosceles triangle with its base as the
26. The houses in a row are numbered consecutively from 1 breadth of the rectangular park and of altitude 12 m. Find
to 49. Show that there exists a value of X such that sum of the length and breadth of the rectangular park. (4 Marks)

EXPLANATIONS

1. Given distance of foot of ladder from wall, x = 2.5 m 3. Given, ∠CAB = 30°
A P
angle of elevation, q = 60° 
Let the length of ladder be l C
 (½ Mark) l Q
BC
\ In DABC, cos 60° = A
30°
B
AC 60° O
B C
1 2.5 2.5m
⇒ = ⇒ l = 5 m  (½ Mark)
2 l
2. Given terms of A.P. are k + 9, 2k – 1, 2k + 7 PQ is a tangent to the circle at the point C, we know,
Now, by concept of Arithmetic mean, 2a2 = a1 + a3 ∠ACB = 90°  [diameter subtends 90° at any point on
 ½ Mark)
(½  the circle ] (½ Mark)
⇒ 2(2k – 1) = (k + 9) + (2k + 7) Also, since, ∠CAB = 30°, ∠BCQ = 30°
⇒ 4k – 2 = 3k + 16 ⇒ 4k – 3k = 16 + 2 ⇒ k = 18 \ ∠PCA = 180° – ∠ACB – ∠BCQ
\ k = 18 ½ Mark)
(½ = 180° – 90° – 30° = 60° (½ Mark)

268
268 MATHS
4. Total number of card = 52 \ In DOTS, OT = OS [both are radii]
No. of cards that are neither a red card nor a queen = 24 ⇒ ∠TOS + ∠OTS + ∠OST = 180°
No. of cards that are neither red nor queen ⇒ 120° + ∠OTS + ∠OST = 180°
Probability =
total no. of cards
⇒ 2∠OTS = 60° ⇒ ∠OTS = 30°
24 6 \ ∠OTS = ∠OST = 30° Hence proved. (11 Mark)
= =  (1 Mark)
52 13 8. The given paints are A(3,0), B(6,4), C(–1, 3).
5. Given, a circle touches all the four sides of a quadrilateral
ABCD at point P, Q, R and S Here, AB = ( 6 − 3)2 + ( 4 − 0 )2
D R C
= 32 + 42 = 9 + 16 = 25 = 5
S Q 2 2
O BC = (−1 − 6) + (3 − 4)
= (−7) 2 + (−1)=
2
49 +=
1 50
= 5 2 and
A B
P
We know, tangents drawn from an external point to a circle CA = (−1 − 3) 2 + (3 − 0) 2
are equal. So,
= (−4) 2 + (3) 2= 16 + 9= 25= 5  (11 Mark)
AP = AS ...(i), BP = BQ ...(ii)
CR = CQ ...(iii), DR = DS ...(iv)(1 Mark) Now, AB2 + AC2 = (5)2 + (5)2 = 25 + 25 = 50 = BC2
Now equation (i) + (ii) + (iii) + (iv) i.e., DABC satisfies Pythagoras theorem
AD + BP + CR + DR = AS + BQ + CQ + DS ⇒ DABC is right angled triangle
⇒ (AP + BP) + (CR + DR) = (AS + DS) + (BQ + CQ) Hence, DABC is isosceles right triangle.  (11 Mark)
⇒ AB + CD = AD + BC 9.
⇒ AB + CD = BC + DA (1 Mark) A(2,–2) P(x1,y1) Q(x2,y2) B(–7,4)
6. Let the first term of the A.P. be a and the common difference Let P(x1,y1) and Q(x2,y2)
be ‘d ’. Given, A(2, –2) & B(–7, 4) are this trisection by P & Q, P
Given, 4th term of the AP is 0, i.e t4 = 0 being nearer to A.
⇒ a + (4 – 1) d = 0 ⇒ a + 3d = 0 ⇒ a = –3d  ...(i) \ Ratio at which P divides AB is 1 : 2
Now, a25 = a + (25 – 1)d (1 Mark)  −7(1) + 2(2) 4(1) + (−2)2 
\ P( x1 , y1 ) =  , 
= a + 24d = – 3d + 24d = 21d  [from (i)]  1+ 2 1+ 2 
Also, 3a11 = 3[a + (11 – 1)d] = 3[a + 10d]  −7 + 4 4 − 4   −3 0  1 Mark)
=3[–3d + 10d] = 3[7d] = 21d [from (i)] P( x1 , y1 ) =  ,  =  ,  = (−1, 0)  (1
 3 3   3 3
∴ a25 = 3a11 (1 Mark) Again, ratio at which Q divides AB is 2 : 1
7. Here, two tangents PT & PS are drawn to the circle from
external point P. Given, radius = r, OP = 2r \ Q( x , y ) =  −7(2) + 2(1) , 4(2) + (−2)1 
2 2  
 2 +1 2 +1 
Let ∠TOP = θ
\ In DTOP, [\ right angled at ∠OTP]  −14 + 2 8 − 2   −12 6 
Q( x2 , y2 ) =  , = , = (−4, 2)
r 1  3 3   3 3 
OT
cosq = ⇒ cosq = =
OP 2r 2 \ The coordinates of P(–1,0) & Q(–4,2). (11 Mark)
10. 13 ⇒
2x + 9 + x = 2 x + 9 = 13 − x
T
On squaring both sides, we get

( )
2
P O 2x + 9 =(13 − x) 2 ⇒ 2x + 9 = 169 + x2 – 26x(1
1 Mark)
Q
⇒ x2 –26x –2x + 169 – 9 = 0 ⇒ x2 – 28x + 160 = 0
S ⇒ x2 – 20x – 8x + 160 = 0 [Middle term factorisation]
⇒ cosq = cos60° ⇒ q = 60° (11 Mark) ⇒ x(x – 20) –8 (x – 20) = 0 ⇒ (x –8) (x – 20) = 0
\ ∠TOS – 2∠TOP = 2 × 60° = 120° \ x = 8, 20 (1 Mark)

CBSE 2016 (Outside DELHI Term-II) 269


11. Given, P(x,y) is equidistant from A(a + b, b – a) & B(a – b,

2.
a + b). So, AP = BP (1 Mark)

8
m
⇒ ( x – (a + b)) 2 + ( y – (b − a )) 2= (a − b − x) 2 + (a + b − y ) 2

(1 Mark)
Squaring and solving
2.1 m
⇒ x2 + (a + b)² – 2x(a + b) + y2 + (b – a)2 – 2y(b – a)
= (a – b)2 + x2 – 2x(a – b) + (a + b)2 + y2 – zy(a + b)
⇒ – 2ax – 2bx – 2by + 2ay = – 2ax + 2bx – 2ay – 2by
⇒ – 4bx = – 4ay ⇒ bx = ay (Proved) (1 Mark) 3m
12. Given radius of conical vessel (r) = 5 cm, height (h) = 24 cm Slant height, l = 2.8 m;
and radius of cylindrical vessel (R) = 10 cm (1 Mark) Diameter of cone d2
We know, volume of cylindrical vessel = volume of water = diameter of cylinder d1 = 3 cm (1 Mark)
in conical vessel Now, Area of canvas needed = curve surface area of tent
1 2 22 1 22 \ Area of canvas needed = curved surface area of canvas
⇒ πR 2 H = πr h ⇒ × 10 × 10 × H = × × 5 × 5 × 24 + curved surface area of cylinder.
3 7 3 7
d d
 (1 Mark) = prl + 2prh ⇒ π 2 l + 2π 1 h
2 2
1 22 22 3
× × 5 × 5 × 24 = × × [2.8 + 2 × 2.1] = 33 m2 (1 Mark)
8
⇒ H =3 7 = =2 cm 7 2
22 4
× 10 × 10 \The area of canvas needed = 33 m2
7
Given, cost of convas is `500 /m2
\ Height to which the water will rise in the cylindrical
vessel is 2 cm. (1 Mark) \ Total cost of canvas = `(500 × 33) = `16500. (1 Mark)
13. Out of the Syllabus 16. Out of the Syllabus
14. Given diameter of sphere, d = 12 cm, so radius = 6 m 17. Given angle of elevation = 60°
Q
Now, weight by which water level rises in cylindrical
5 32
vessel is h = 3 cm = cm  (1 Mark) y
9 9
A 60° C
Let the diameter of the cylindrical vessel be d' 30°
4 3 4 60°
10m
Now, Volume of sphere, V = πr = π(6)3 10m
3 3
30°
Now, Volume of water displaced in cylindrical vessel B x P
= volume of sphere (1 Mark)
angle of depression = 30°
2
 d ′  32 4 height of the man from water level = 10m
V = pr2h ⇒ π   × = × π× 63
2 9 3 From the figure we can clearly say, CP = 10m (1 Mark)
(d ′) 2 32 4 Let the distance of the hill from the ship be x & the height
⇒π × = × π× 6 × 6 × 6 QC be y.
4 9 3
AB 1 10
⇒ d′2 = 3 × 3 × 6 × 6 ⇒ d′ = 3 × 6 \ In DABP; tan 30° = ⇒ = ⇒ x= 10 3
BP 3 x
[d′ can't be (–)ve as it represents lengths]
⇒ d′ = 18  [by cross multiplication] (1 Mark)
⇒The diameter the cylindrical vessel is 18 cm.  (1 Mark) CQ y
In DAQC; tan 60° = 3
⇒ = [∵ AC = x]
= BP
15. Given dimensions of cylindrical part, AC x
Height, h1 = 2.1 m; Diameter, d1 = 3m ⇒ 3=
10 3
y
⇒ y = 10 3 ( 3 ) [by cross multiplication]
Dimensions of conical part,

270
270 MATHS
⇒ y = 10 × 3 ⇒ y = 30 21. Given dimensions of costs tent
\ Distance of the hill from the ship = x = 10 3 Radius of cylindrical & conical part = 2.8 m = r
Height of the hill = (y + 10) m = (30 + 10) m = 40 m. height of cylindrical part, h1 = 3.5 m
 (1 Mark) height of conical part, h2 = 2.1 m
18. Given, the three digits of a positive number of three digits Now, Slant height of conical part, l = r 2 + h22
are in AP.
Let the three digits be a – d, a, a + d = (2.8) 2 + (2.1) 2 m = 3.5 m (1 Mark)
Given their sum is 15 \ Area of canvas required for are tent
\ a – d + a + a + d = 15 ⇒ 3a = 15 ⇒ a = 5 (1 Mark) = curve surface area of cylindrical part + curve surface
Now, the number obtained by reversing the digit is area of cenical part
100 (a – d) + 10 a + (a + d) 22 22
= 2prh1 + prl = 2 × × 2.8 × 3.5 + × 2.8 × 3.5
Reversing the digits we get, 7 7
100 (a + d) + 10a + (a – d) ...(i) (1 Mark) 22
Given, an reversing the digits, the number formed is 594 = 3× × 2.8 × 3.5 = 92.4 m2  (1 Mark)
7
less than the original number. There are 1500 tents that are needed to be fixed
⇒ 100(a – d) + 10a + (a + d) = 100(a + d) + 10a + (a – d) – 594 \ Total area of canvas required = 1500 × 92.4 m2
⇒  100a – 100d + 10a + a + d = 100a + 100d + 10a + a – d – 594 = 138600 m2 (1 Mark)
⇒ –100d – 100d + d + d = –594 Given, cost of canvas = ` 120/m 2

⇒ –198d = –594 ⇒ d = 3 \ Total cost of canvas = ` (120 ×138600) = ` 16632000


Hence orginal number is 100(a + d) + 10a + a – d 16632000
Now, share of each school = ` = ` 332640
= 100(5 + 3) + 10 × 5 + 5 – 3 = 800 + 50 + 2 = 852 50
 (1 Mark)  The cost shared by each school is = ` 332640. (1 Mark)
19. Given quadratic eq. is. 22. Given, are two circles with centres O and O'.
(a – b)x2 + (b – c) x + (c – a) = 0 they touch each other at X.
Given the roots of the quadratic equation are equal
C
\ Discriminant, D = O (1 Mark)
⇒ (b – c)2 – 4 (a – b) (c – a) = 0 D
⇒ b² + c2 – 2bc – 4 (ac – a² – bc + ab) = 0 A O
O' X
⇒ b² +c2 – 2bc – 4ac + 4a2 + 4bc – 4ab = 0
⇒ b² + c2 + 2bc – 4a(b + c) + 4a2 = 0 (1 Mark)  (1 Mark)
⇒ (b + c)2 – 2.2a (b + c) + 4a2 = 0 AC is a tangent to the circle with centre O.
⇒ (b + c – 2a)2 = 0 [ (a – b)2 = a2 – 2ab + b2] \ ∠ACO = 90° [ the tangent makes 90° with the radius
⇒ b + c – 2a = 0 ⇒ b + c = 2a (1 Mark)
at point of contact]
∠ADO′ = 90° [given]
Hence, proved.
\ O′D || OC (1 Mark)
20. From a pack of 52 playing cards, Jacks, Queens & Kings of
red colour are removed. Now, in DAO′D & DAOC
\ No. of cards left = 52 – 6 = 46. ∠ADO′ = ∠ACO = 90°
∠O′AD = ∠OAC [common]
(i) No. of cards that are black king = 2
∠AO′D = ∠AOC [corresponding angles]
2 1 \ DAO′D ~ DAOC [by AAA criteria] (1 Mark)
\ P (black king)
= =  (1 Mark)
46 23 AO′ DO′
\ = 
(ii) No. of cards that are of red colour = 20 AO CO
20 10 Now, AO′ = r [radius of circle]
=
\ P (a card of red colour) =  (1 Mark)
46 23 AO = AO′ + O′X + XO
(iii) No. of cards that are of black colour = 26 = r + r + r = 3r [all of them are radii of the circles]
26 13 ′ AO′ r 1 
\ DO= (1 Mark)
\ P (card of black colour) = =  (1 Mark) = =
46 23 CO AO 3r 3

CBSE 2016 (Outside DELHI Term-II) 271


23. A number x is selected at random from 1, 2, 3, 4. 26. It is given that, the houses of a row are numbered from 1, 2,
A number y is selected at random from 1, 4, 9, 16 3, 4, 5…….49. Hence, the houses numbered in a row form
\ The sample space is, (1 Mark) an AP
Therefore, First term, i.e., a = 1;
(1, 1) (2, 1) (3, 1) (4, 1)
Common difference, i.e., d = 1
(1, 4) (2, 4) (3, 4) (4, 4)
(1, 9) (2, 9) (3, 9) (4, 9) n
Sum of n terms of AP = [2a + (n – 1)d] (1 Mark)
(1, 16) (2, 16) (3, 16) (4, 16) 2
Sum of number of houses preceding x houses = Sx–1
\ Total no. of cases = 16. (1 Mark) x −1 x −1
Sx–1 =  2a + ( ( x − 1) − 1)d )  =  2 (1) + ( x − 1 − 1)1
The favourable out comes (1 × 1, 1 × 4, 1 × 9, 2 × 1, 2 × 2 2 
4, 3 × 1, 3 × 4, 4 × 1). x 1 x 1
No. of out causes when the product of x & y is less than 16 =
2
2  x  2 = 2  x  ...(i) (1 Mark)
= 8. (1 Mark) According to the question, sum of number of houses
No. of favourable outcomes
\ P(product is less than 16) = Total no. of possible outcomes following xth house is S49 – Sx. 
49 x
8 1 =  2 ×1 + ( 49 − 1) × 1 −  2 × 1 + ( x − 1) × 1
= =  (1 Mark) 2  2
16 2
49 x
24. Out of the Syllabus = [ 2 + 48] − [ 2 + x − 1] = 49 × 50 − x [ x + 1]
2 2 2 2
25. Given, sector OAP contains ∠θ x  x  1
B = 1225  ...(ii) (1 Mark)
2
According to the given condition, eq.(i) and eq.(ii) must be
P
equal to each other. Hence,
x ( x − 1) x ( x + 1)
= 1225 − 
θ 2 2
O A
r x2 x x 2 x ⇒ x2 = 1225 ⇒ x = ±35
− = 1225 − −
2 2 2 2
The number of houses can never be negative. Therefore,
Radius of circle, OA = r the value of x is 35. (1 Mark)
θ πr θ 27. Let the speed of the stream be x.
Now, length of AP= 2 πr =  (1 Mark)
360 180 and speed of the boat in still water = 24 km/h
Now, given AB⊥OA, given motor boat takes 1 hour more to go upstream than
\ In DOAB; downstream.
The distance between the places = 32 km
AB AB
tan=
θ ⇒ tan=
θ ⇒ AB = r tanq (1 Mark) Speed in upstream = (24 – x) km\hrs
OA r
Speed in downstream = (24 + x) km/hrs. (1 Mark)
Again in DOAB,
According to questions;
OA r r
cos=
θ ⇒ cos=
θ ⇒ OB = ⇒ OB = r secq 32 32 distance  (1 Mark)
OB OB cos θ ⇒ − = 1 . Time =
24 − x 24 + x speed
Now, PB= OB – OP = r secq – r = r (secq – 1) (1 Mark)
32(24 + x) − 32(24 − x)
\ Perimeter of shaded region ⇒ = 1 [taking LCM]
(24 − x) (24 + x)
πr θ
= AP + PB + AB = + r (sec θ − 1) + r tan θ ⇒768 + 32x –768 + 32x = (24 – x) (24 + x)
180
[by cross multiplication]
 πθ  πθ 
= r + sec θ − 1 + tan θ  = r  tan θ + sec θ + −1 ⇒64x = 576 – x2[ a2 – b2 = (a – b)(a + b)]
180   180  ⇒x2 + 64x – 576 = 0 ⇒x2 + 72x – 8x – 576 = 0 (1 Mark)
Hence, proved. (1 Mark)  [Middle term factorization]

272
272 MATHS
⇒x(x + 72) – 8(x + 72) = 0 ⇒ (x – 8) (x + 72) = 0 Angle of depression of ship changes from 60° to 30°.
\ x = 8 km/hrs (1 Mark) So, Let ∠QBP = 60° and ∠QAP = 30°
\ speed of stream is 8 km/hrs. QP = 100 m.  (1 Mark)
28. Out of the Syllabus QP
Now, in DQPB, tan 60° =
29. Given, xx′ is a tangent to the circle with centre O at point of BP
contact A. 100 100
⇒ 3= ⇒ BP =
BP 3  (1 Mark)
QP
O And, in DQAP, tan 30° =
AP
C 1 100
⇒ = ⇒ AP = 100 3
3 AP  (1 Mark)
x A B x′ \ The distance travelled by the ship AB = AP – BP
B is a point on xx′ such that OC is product to xx′ 100 300 − 100 200
= 100 3 − = =
To prove: ∠OAB = 90° (1 Mark) 3 3 3
Proof: Here, OA = OC [radii]
200 3 (1.73)
Now, OB = OC + BC AB = = 200 = 115.3 m
3 3
clearly, OB > OC [ OC is the radius, B is any point on
the tangent] The distance travelled by the ship during time of
observation is 115.3 m.  (1 Mark)
⇒ OB > OA (1 Mark)
31. Let the length of the rectangular park be x m
Now, B is any arbitrary point on tangent.
\ The breadth of the park = (x – 3) m
Thus, OA is shorter than any other line segment joining O
to any point on tangent  (1 Mark) Now, Area of rectangular park = x(x – 3) m2 (1 Mark)
and, we known shortest distance of point from a given line 1
\ Area of isosceles triangular park= ( x − 3) × 12 = 6 (x – 3)
is the perpendicular distance from that line. 2
Now, Area of rectangular parks
Hence, the tangent at any point of circle is perpendicular
= Area of isosceles triangular park + 4 (1 Mark)
to the radius.  (1 Mark)
⇒ x(x – 3) = 6(x – 3) + 4 ⇒ x2 – 3x = 6x – 18 + 4
30. Given, weight of light house = 100 m
⇒ x2 – 3x = 6x – 14 ⇒ x2 – 9x + 14 = 0
Q
60° 30° ⇒ x2 – 7x – 2x + 14 = 0 ⇒ x (x – 7) – 2 (x –7) = 0
⇒ (x –2) (x –7) = 0 ⇒ (x – 2) = 0 or (x – 7) = 0
100 m ⇒ x = 2 or x –7 (1 Mark)
It x = 2m, then breadth = 2 – 3 = –1 m, which is not possible
\x=7
30° 60° i.e., length of rectangular park is = 7 m
A B P breath of rectangular park = (7 – 3) m = 4 m. (1 Mark)

CBSE 2016 (Outside DELHI Term-II) 273


Term-I
2016 CBSE Solved Paper

Time allowed : 3 hours Maximum Marks : 90


GENERAL INSTRUCTIONS:
Read the following instructions very carefully and strictly follow them:
(i) All questions are compulsory.
(ii) The question paper consists of 31 questions divided into four sections – A, B, C and D.
(iii) Section A contains 4 questions of 1 mark each, Section B contains 6 questions of 2 marks each, Section C contains
10 questions of 3 marks each and Section D contains 11 questions of 4 marks each.
(iv) Use of calculators is not permitted.

Section-A sin 3 θ + cos3 θ


9. Prove the following identity: 1 sin θ.cos θ
=−
sin θ + cos θ
1. In ∆DEW, AB || EW. If AD = 4 cm, Now, DE = 12 cm and (2 Marks)
DW = 24 cm, then find the value of DB. (1 Mark)
10. Show that the mode of the series obtained by combining
2. If ∆ABC is right angled at B, what is the value of sin (A + C). the two series S1 and S2 given below is different from that
(1 Mark) of S1 and S2 taken separately:  [OS] (2 Marks)
3cos 2 θ + 2 cos θ S1: 3, 5, 8, 8, 9, 12, 13, 9, 9; S2: 7, 4, 7, 8, 7, 8, 13
3. If 3 sin=
θ cos θ , find the value of
.
3cos θ + 2 Section-C
(1 Mark)
4. From the following frequency distribution, find the median class: 11. The length, breadth and height of a room are 8 m 50 cm,
6 m 25 cm and 4 m 75 cm respectively. Find the length
Cost of Living of the longest rod that can measure the dimensions of the
1400–1550 1550–1700 1700–1850 1850–2000
Index room exactly. (3 Marks)
No. of weeks 8 15 21 8 12. Solve by elimination: 3x – y = 7, 2x + 5y + 1 = 0. (3 Marks)
(1 Mark) 13. Find a quadratic polynomial, the sum and product of whose
3
Section-B zeroes are 0 and − respectively. Hence find the zeroes.
5
 (3 Marks)
5. Show that 3 7 is an irrational number.  (2 Marks) 14. The sum of the digits of a two digit number is 8 and
6. Explain why (17 × 5 × 11 × 3 × 2 + 2 × 11) is a composite the difference between the number and that formed by
number? (2 Marks) reversing the digits is 18. Find the number. (3 Marks)
15. In given figure, EB ⊥ AC, BG ⊥ AE and CF ⊥ AE. Prove that:
7. Find whether the following pair of linear equations is (3 Marks)
consistent or inconsistent: 3x + 2y = 8, 6x – 4y = 9. (2 Marks) E

8. X and Y are points on the sides AB and AC, respectively F


AX 1 G D
of a triangle ABC such that = , AY = 2 cm and
AB 4
YC = 6 cm. Find whether XY || BC or not. (2 Marks) A C
B
(i) ∆ABG ~ ∆DCB 26. In the given figure, AD = 3 cm, AE = 5 cm, BD = 4 cm,
BC BE CE = 4 cm, CF = 2 cm, BF = 2.5 cm, then the pair of
(ii) = parallel lines and hence their lengths. (4 Marks)
BD BA
16. In triangle ABC, if AP ⊥ BC and AC2 = BC2 – AB2, then A
prove that PA2 = PB × CP. [OS] (3 Marks)
3 cm
12 5 cm
17. If sin θ = , 0o < θ < 90o, find the value of:
13
D E
sin θ – cos 2 θ
2
1
× (3 Marks)
tan 2 θ 
4 cm 4 cm
2sin .cos
θ θ
p2 −1 C
18. If sec θ + tan θ = p, prove that: sin θ = 2 . (3 Marks) B 2.5 cm F 2 cm
p +1
19. Find the mean of the following distribution by Assumed 1
Mean Method: (3 Marks) 27. If tan (A + B) = 3 and tan (A – B) = , where
3

90–100
0 < A + B < 90o, A > B, find A and B. Also calculate
10–20

20–30

30–40

40–50

50–60

60–70

70–80

80–90
Class Interval
tan A.sin (A + B) + cos A.tan (A – B). (4 Marks)
Frequency 8 7 12 23 11 13 8 6 12 28. Prove that:
20. The average score of boys in the examination of a school sec3 A − cosec3 A
(1 + cot A + tan A).(sin A – cos A) =
is 71 and that of the girls is 73. The average score of sec 2 A.cosec 2 A
the school in the examination is 71.8. Find the ratio of  (4 Marks)
number of boys in the number of girls who appeared in the
examination. (3 Marks) sin A + cos A sin A − cos A 2
29. Prove that:
+ =
sin A − cos A sin A + cos A 1 − 2 cos 2 A
Section-D (4 Marks)
21. Find HCF of numbers 134791, 6341 and 6339 by Euclid 30. The following table gives the daily income of 50 workers
division algorithm. [OS] (4 Marks) of a factory. Draw both types ("less than type" and "greater
22. Draw the graph of the following pair of linear equations: than type") ogives. [OS] (4 Marks)
x + 3y = 6 and 2x – 3y = 12. Find the ratio of the areas
of the two triangles formed by first line, x = 0, y = 0 and Daily
100–120 120–140 140–160 160–180 180–200
second line, x = 0, y = 0. [OS] (4 Marks) Income
No. of
23. If the polynomial (x4 + 2x3 + 8x2 + 12x + 18) is divided by 12 14 8 6 10
Workers
another polynomial (x2 + 5), the remainder comes out to be
(px + q), find the values of p and q. [OS] (4 Marks) 31. In a class test, marks obtained by 120 students are given
24. What must be subtracted from p(x) = 8x4 + 14x3 – 2x2 + in the following frequency distribution. If it is given that
8x – 12 so that 4x2 + 3x – 2 is factor of p(x)? This question mean is 59, find the missing frequencies x and y.
was given to group of students for working together. Do (4 Marks)
you think teacher should promote group work?
90–100
10–20
20–30
30–40
40–50
50–60
60–70
70–80
80–90
0–10

 [OS] (4 Marks) Marks


25. Prove ‘If a line is drawn parallel to one side of a triangle
to intersect the other two sides in distinct points, the other No. of
1 3 7 10 15 x 9 27 18 y
Students
two sides are divided in the same ratio’. (4 Marks)

CBSE 2016 (Term-I) 275


EXPLANATIONS

1.
D 5. Let us assume that 3 7 is rational.
So, we can find co-primes a & b, such that,
4cm
a a
12cm 24cm 3 7= ⇒ 7=  ½ Mark)

B b 3b
A a
Since, 3, a & b are integers, can be written in the form
p a 3b
E W of . So, is rational. ½ Mark)

a 3b
Given, AD = 4 cm, DE = 12 cm, DW = 24 cm (½ ½ Mark)
Implies, 7 is also rational.
Let DB be x cm
Now, in DDEW, AB || EW ½ Mark)
But, this is a contradiction, as 7 is irrational. (½
AD DB That means, out assumption was wrong.
\ =  [by Thale's Theorem] Thus, 3 7 is irrational. (½½ Mark)
DE DW
4 x 4 6. 17 × 5 × 11 × 3 × 2 + 2 × 11
⇒ = ⇒x= × 24 = 8 cm ½ Mark)
(½ = 17 × 5 × 3 × 22 + 22 = 22(17 × 5 × 3 + 1)
12 24 12
2. Given, DABC is right angled at B. = 22(255 + 1) = 2 × 11 × 256 (1 Mark)
\ ∠B = 90° \ The given number is divisible by 2, 11 and 256.
We know, i.e., it is divisible by more than 2 primes factors.
∠A + ∠B + ∠C = 180°[angle sum property of triangle] \ (17 × 5 × 11 × 3 × 2 + 2 × 11) is a composite number.
⇒ ∠A + 90° + ∠C = 180° ½ Mark)
(½ (1 Mark)
⇒ ∠A + ∠C = 90° 7. Given pair of linear equation is:
\ sin (∠A + ∠C) = sin (90°) = 1 ½ Mark)
(½ 3x + 2y = 8 ⇒ 3x + 2y – 8 = 0
sin  1 6x – 4y = 9 ⇒ 6x – 4y – 9 = 0 (½½ Mark)
3. Given, 3 sinq = cosq  
So, here,
cos  3
1 a1 = 3; b1 = 2; c1 = –8
⇒ tanq = ⇒ tanq = tan30°⇒ q = 30° ½ Mark)

3 a2 = 6; b2 = –4; c2 = –9 (½½ Mark)
3cos 2 θ + 2 cos θ cos θ(3cos θ + 2) a1 3 1
∴ = cos θ
= Now, = =
3cos θ + 2 (3cos θ + 2) a2 6 2
b1 2 1 a b
3   ⇒ 1 ≠ 1  ½ Mark)

= cos30°= [ q = 30°]  ½ Mark)
(½ b2 4 2 a2 b2
2
\The pair of linear equation will have unique solution &
4. hence, is consistent. ½ Mark)

Cost of Living Index No. of Weeks (f) cf 8.
1400 – 1150 8 8 A
1550 – 1700 15 23
1700 – 1850 21 44 Y
X
1850 – 2000 8 52
N = ∑f = 52
B C
Here, N = 52 ½ Mark)
(½ AX 1
Given, =
N 52 AB 4
⇒ = = 26 
2 2 AY = 2 cm, YC = 6 cm ½ Mark)

\ 26 will lie in the class interval 1700 – 1850. AX 1
Now, =
\ Median class is 1700 – 1850. ½ Mark)
(½ AB 4

276
276 MATHS
Let, AX = K and AB = 4K Given, sum of zeroes = 0
Now, XB = AB – AX = 4K – K = 3K ½ Mark)
(½ −3
and Product of zeroes =  (1 Mark)
AX K 1 5
\ = =
XB 3K 3 \ The quadratic polynomial is;
AY 2 1  3  3
Also, = =  ½ Mark)
(½ x2 – (0)x +   = x2 –
YC 6 3  5  5
Now, to find the zeroes of the polynomial;
AX AY
\ = 2
XB YC 3 3  3
x –
2
= 0 ⇒ x2 = ⇒ x2 =   (1 Mark)
½ Mark)
⇒ XY || BC [by converse of Thale's Theorem] (½ 5 5  5 
 
3 3
sin 3   cos3  ⇒x=+ ⇒x= or x = – 3
9. L.H.S. =  ½ Mark)
(½ 5 5 5
sin   cos 
3 5 3 5
(sin   cos )(sin 2   cos 2   sin  cos ) ⇒x= × or x = – ×
 5 5 5 5
(sin   cos )
 (1 Mark) 15 − 15
[Q a3 + b3 = (a + b)(a2 + b2 – ab)] ⇒x= or x =  (1 Mark)
5 5
= sin2 q + cos2 q – sin q cos q
= 1 – sin q cos q   [Q sin2 q + cos2 q = 1] ½ Mark)
(½ 15 − 15
\ The zeroes of the polynomial are or
= R.H.S. 5 5
Hence, L.H.S. = R.H.S. 14. Let the unit digit be x & the tense digit be y ½ Mark)

10. Out of the Syllabus \ original number = 10y + x
11. Given, dimensions of room, Now, according to question, sum of digits is 8.
Length, L = 8 m 50 cm = 850 cm = 2 × 52 × 17 cm i.e., x + y = 8 ½ Mark)
...(i) (½
 ½ Mark)
(½ And, also the differences between the number & that
Breadth, B = 6 m 25 cm = 625 cm = 5 × 5 cm (½
2 2
½ Mark) formed by reversing the digits is 18.
Height, H = 4 m 75 cm = 475 cm = 5 × 19 cm (½
2
½ Mark) If the digits are reversed, then, unit digit = y, tens digit = x
To find the length of the longest rod that can measure the \ The reversed number = 10x + y. ½ Mark)

dimension of the room exactly, we have to find the HCF. \ According to question (x + 10y) – (10x + y) = 18
 ½ Mark)
(½ ⇒ x + 10y – 10x – y = 18 ⇒ –9x + 9y = 18
\ HCF of L, B, H = 52 = 25 ½ Mark)
(½ ⇒ 9(–x + y) = 18 ⇒ –x + y = 2 ½ Mark)
...(ii) (½
\ Length of the longest rod = 25 cm ½ Mark)
(½ Adding equation (i) and (ii), we get
12. Given, 3x – y = 7 ...(i) 2y = 10 ⇒ y = 5
2x + 5y + 1 = 0 ⇒ 2x + 5y = –1 ...(ii) (1 Mark) Putting y = 5 in (i), we get,
Now, 5 × (i) + (ii), we get, x + 5 = 8 ⇒ x = 8 – 5 = 3 ½ Mark)

15x – 5y = 35 \ Original no. is: 10y + x = 10(5) + 3 = 50 + 3 = 53
2x + 5y = –1  ½ Mark)

+ +
____________ + 15.
17x = 34 E
34 F
⇒x= =2
17 G 7 D
\ x = 2 (1 Mark) 3
2
Putting the value of x in (i), we get 1 6 5 4
3(2) – y = 7 ⇒ 6 – y = 7 ⇒ y = 6 – 7 = –1 A B C
\ x = 2, y = –1 (1 Mark) Given, EB ⊥ AC
13. We know, quadrartic polynomial: BG ⊥ AE, CF ⊥ AE
x2 – (sum of zeroes) x + (product of zeroes) We have to prove that

CBSE 2016 (Term-I) 277


(i) DABG ~ DDCB ½ Mark)
(½ 18. Given, sec q + tan q = p.

(ii)
BC BE
=  p2 − 1 (sec θ + tan θ)2 –1
BD BA R.H.S. = = 2
 ½ Mark)

p 2 + 1 (sec θ + tan θ) + 1
Proof: (i) In DABG and DDCB, ½ Mark)
(½ [Q secq + tanq = p]
BG || CF, as corresponding angles are equal sec2 θ + tan 2 θ + 2sec θ tan θ –1
\ ∠2 = ∠5 [both are 90°] =  ½ Mark)

sec2 θ + tan 2 θ + 2sec θ tan θ + 1
∠6 = ∠4 [corresponding angles]
\ DABG ~ DDCB [by AA similarity] (sec2 θ –1) + tan 2 θ + 2sec θ tan θ
=
\ ∠1 = ∠3 [by CPCT] ...(i) (1 Mark) sec2 θ + (tan 2 θ + 1) + 2sec θ tan θ
(ii) In DABE and DDBC, tan 2 θ + tan 2 θ + 2sec θ tan θ
= [ sec2q – tan2q = 1]
∠1 = ∠3 [from (i)] sec2 θ + sec2 θ + 2sec θ tan θ
∠ABE = ∠5 [both are 90°]
2 tan 2 θ + 2sec θ tan θ
\ DABE ~ DDBC [both AA similarity] =  (1 Mark)
2sec2 θ + 2sec θ tan θ
BC BD
\ =  2 tan θ(tan θ + sec θ) tan θ
BE BA = =
[Q in similar triangles, the corresponding sides are 2sec θ(sec θ + tan θ) sec θ
proportional] sin θ cos θ sin θ 1
= × [Q tanq = & secq = ]
cos θ 1 cos θ cos θ
BC BE
⇒ = Hence, proved. (1 Mark) = sin q = L.H.S.
BD BA
16. Out of the Syllabus p 2 –1
i.e., sinq = 2 Hence, L.H.S. = R.H.S.proved.
12 P  p +1 (1 Mark)
17. Given, sinq = = , 0° < q < 90° ½ Mark)

13 H
perpendicular 12 19.
P 12
⇒ = ⇒ =
hypotenuse 13 H 13 Class Frequency xi di = xi fidi
Interval fi – 55
Let, P = 12k, H = 13k
10 – 20 8 15 –40 –320
2 2
\ Base, B = H –P 20 – 30 7 25 –30 –210
30 – 40 12 35 –20 –240
= (13k )2 – (12k )2  [by Pythagoras theorem]
40 – 50 23 45 –10 –230
2 2 2 55 = A
= 169k –144k = 25k = 5 k ½ Mark)
(½ 50 – 60 11 0 0

Base B 60 – 70 13 65 10 130
\ cosq = = 70 – 80 8 75 20 160
Hypotenure H
80 – 90 6 85 30 180
5k 5
= =  ½ Mark)
(½ 90 – 100 12 95 40 480
13k 13
∑fi = 100 ∑fidi = –50
Perpendicular P 12k 12
tanq = = = =  ½ Mark)
(½ (2 Mark)
Base B 5k 5
Let A = 55[Assumed Mean]
2 2
 12   5  Σf i d i (–50)
2 2   –  \ Mean = A + = 55 + = 55 – 0.5 = 54.5
\ sin θ – cos
=
θ 1  13   13  × 1 (½ ½ Mark) Σf i 100
×
2
2sin θ ⋅ cos θ tan θ  12   5   12 2 
2⋅ ⋅ 
 13   13   5  \ The mean is 54.5. (1 Mark)
144 25 20. Let the number of boys be x & the number of girls be y.

25 119 25 119 × 5 595
= 169 169 × = × = =  ½ Mark)

60 144 120 144 24 × 144 3456
2⋅ Given,
 169 Average boy’s score = 71 = x1 (suppose)
½ Mark)

278
278 MATHS
Average girl’s score = 73 = x2 (suppose) 26.
½ Mark)
Given, combined mean marks is given to be 71.8 (½ A

xx1 + yx2 3 5
Also, combined mean =
x+ y D E
x(71) + y (73)
⇒ 71.8 =  ½ Mark)
(½ 4 4
x+ y
⇒ 71.8x + 71.8y = 71x + 73y [cross multiplication] B 2.5 F 2 C
⇒ 71.8x – 71x = 73y – 71.8y ⇒ 0.8x = 1.2y Given, AD = 3 cm
x 1.2 12 3 AE = 5 cm, BD = 4 cm , CE = 4 cm, CF = 2 cm
⇒ = = = ⇒ x : y = 3 : 2 (1 Mark) BF = 2.5 cm (1 Mark)
y 0.8 8 2
\ The ratio of no. of boys to no. of girls is 3 : 2. EC 4 CF 2 4
Now, = & = = 
i.e., No. of boys : No. of girls = 3 : 2. ½ Mark)
(½ EA 5 FB 2.5 5
EC CF
21. Out of the Syllabus \ =
CA FB
22. Out of the Syllabus
\ EF || AB [by converse of Thale's Theorem] (1 Mark)
23. Out of the Syllabus
Now, In DCFE & DCBA;
24. Out of the Syllabus
∠C = ∠C [common]
25. Given, in DABC, DE || BC
∠CFE = ∠CBA [corresponding angles]
A
∠CEF = ∠CAB [corresponding angles]
M N \ DCFE ~ DCBA [by AAA similarity]
D E EF CE
Q = [by CPCT] (1 Mark)
AB CA

B C EF CE EF 4 4(7) 28
⇒ = ⇒ = ⇒ EF = = 
AD + DB CE + AE 3+ 4 4+5 9 9
AD AE
To prove, = 
DB EC 28
\ EF = cm ⇒ AB = 7 cm
Here, EM ⊥ AB and DN ⊥ AC (1 Mark) 9
\ The pair of parallel lines is EF and AB
Now, In DADE and DBDE,
28
1 and EF = cm and AB = 7 cm (1 Mark)
× AD × EM AD 9
ar.(∆ADE ) 2
= = ...(i) (1 Mark) 27. Given, tan (A + B) = 3  ½ Mark)

ar.(∆BDE ) 1 × DB × EM DB
2 ⇒ tan (A + B) = tan (60°)⇒ A + B = 60° ...(i) (½ ½ Mark)
In DADE & DCDE, 1
and tan (A – B) =
1 3
× AE × DN ⇒ tan (A – B) = tan (30°) ⇒ A – B = 30°  ...(ii) (1 Mark)
ar (∆ADE ) 2 AE ..(ii)
= =
ar (∆CDE ) 1 × EC × DN EC Now, adding equation (i) & (ii), we get
2 A = 45 and B = 15 ½ Mark)

Since DE||BC (Given) \ tan A sin (A + B) + cos A tan (A – B)
\ ar.(DBDE) = ar.(DCDE)...(iii) (1 Mark) = tan (45°) sin (60°) + cos (45°) tan (30°) ½ Mark)

[Q triangles on the same Base & between the same parallel [From (i) & (ii)]
sides are equal in area]
3 1 1 3 1 3 6 3 3+ 6
\ from equation (i), (ii) and (iii) = 1⋅ + ⋅ = + = + = 
2 2 3 2 6 2 6 6
AD AE
=  (1 Mark) 3 3+ 6
DB EC \ tan A sin (A + B) + cos A tan (A – B) = 
6
Hence, proved.  (1 Mark)

CBSE 2016 (Term-I) 279


28. L.H.S. = (1 + cot A + tan A) (sin A – cos A) 30. Out of the Syllabus
 cos A sin A  31.
= 1 + +  (sin A – cos A) (1 Mark)
 sin A cos A 
Marks No. of Student xi di = xi fidi
2 2
=  sin A cos A + cos A + sin A  (sin A – cos A) (fi) – 55
 cos A sin A  (1 Mark) 0–10 1 5 –50 –50
10–20 3 15 –40 –120
(sin A cos A + cos 2 A + sin 2 A)(sin A – cos A) 20–30 7 25 –30 –210
=
cos A sin A 30–40 10 35 –20 –200
3 3
sin A – cos A 40–50 15 45 –10 –150
= [Q a3 – b3 = (a – b)(a2 + b2 + ab)]
sin A cos A 50–60 x 55 = A 0 0
(1 Mark)
60–70 9 65 10 90
1 1
− 3 3 70–80 27 75 20 540
cosec3 A = sec3 A sec A − cosec A × cosecA ⋅ sec A
80–90 18 85 30 540
1 cosec3 A ⋅ sec3 A
cosecA ⋅ sec A 90–100 y 95 40 40y
∑fi = 90 + x + y ∑fidi = 440 + 40y
sec3 A – cosec3 A
= = R.H.S. (1 Mark) (2 Marks)
sec2 Acosec2 A 
Hence, L.H.S. = R.H.S. proved. Here, h = 10, A = 55

sin A + cos A sin A – cos A Σf i d i 440 + 40 y


29. L.H.S. = +  (1 Mark) We know, Mean = A + ⇒ 59 = 55 +
sin A – cos A sin A + cos A Σf i 90 + x + y

(sin A + cos A)2 + (sin A – cos A)2 440 + 40 y


⇒ 4 ⇒ 440 + 40y = 360 + 4x + 4y
=
=  (1 Mark) 90 + x + y
(sin A – cos A)(sin A + cos A)
⇒ 4x – 36y = 80 ⇒ x – 9y = 20 ...(i)
sin 2 + cos 2 A + 2sin A cos A + sin 2 A + cos 2 A
−2sin A cos A and given ∑fi = 120
= 2 2
sin A – cos A ⇒ 90 + x + y = 120 ⇒ x + y = 30 ...(ii) (1 Mark)
(1 Mark)
1+1 Adding equation (i) + 9 × (ii), we get
= [Q sin2A + cos2A = 1]
1 – cos A – cos 2 A
2
10x = 290 ⇒ x = 29
2
= = R.H.S. (1 Mark)
1 – 2 cos 2 A From equation (ii)
Hence, L.H.S. = R.H.S. proved. 29 + y = 30 ⇒ y = 1 (1 Mark)

280
280 MATHS
DELHI Term-II
2015 CBSE Solved Paper

Time allowed : 3 hours Maximum Marks : 90


GENERAL INSTRUCTIONS:
Read the following instructions carefully and strictly follow them:
(i) All questions are compulsory.
(ii) The question paper consists of 31 questions divided into four sections – A, B, C and D.
(iii) Section A contains 4 questions of 1 mark each, Section B contains 6 questions of 2 marks each, Section C contains
10 questions of 3 marks each and Section D contains 11 questions of 4 marks each.
(iv) Use of calculators is not permitted.

7. In Figure, AB is the diameter of a circle with centre O and


Section-A AT is a tangent. If ∠AOQ = 58°, find ∠ATQ. (2 Marks)
1. A letter of English alphabet is chosen at random. Determine B
the probability that the chosen letter is a consonant. (1 Mark)
2. In Figure, PA and PB are tangents to the circle with centre
O such that ∠APB = 50°. Write the measure of ∠OAB. O
 (1 Mark)
A 58o
Q
A T
P 50° O 8. Solve the following quadratic equation for x : 4x2 – 4a2x +
(a4 – b4) = 0. (2 Marks)

B 3 5 
9. Find the ratio in which the point P  ,  divides the
 4 12 
3. The tops of two towers of height x and y, standing on 1 3
level ground, subtend angles of 30° and 60° respectively line segment joining the points A  ,  and B(2, –5).
2 2
at the centre of the line joining their feet, then find x : y. (2 Marks)
 (1 Mark)
10. Find the middle term of the A.P. 213 , 205, 197, ......, 37.
1
4. If x = − , is a solution of the quadratic equation (2 Marks)
2
3x + 2kx – 3 = 0, find the value of k.
2 (1 Mark) Section-C
Section-B 11. In Figure, APB and AQO are Q
semicircles, and AO = OB. If the
5. If A(5, 2), B(2, –2) and C(–2, t) are the vertices of a right angled perimeter of the figure is 40 cm,
triangle with ∠B = 90°, then find the value of t. (2 Marks) O B
find the area of the shaded A
6. From a point T outside a circle of centre O, tangents TP  22 
region.  Use π = 
and TQ are drawn to the circle. Prove that OT is the right  7
bisector of line segment PQ. (2 Marks) (3 Marks) P
12. A solid wooden toy is in the form of a hemisphere 17. The angle of elevation of the top of a building from the
surmounted by a cone of same radius. The radius of foot of the tower is 30° and the angle of elevation of the
hemisphere is 3.5 cm and the total wood used in the top of the tower from the foot of the building is 45°. If the
5 tower is 30 m high, find the height of the building.
making of toy is 166 cm3 . Find the height of the toy.
6  (3 Marks)
Also, find the cost of painting the hemispherical part of the 1
18. If the sum of the first n terms of an A.P. is (3n2 + 7n),
 22  2
toy at the rate of ` 10 per cm2.  Use π =   (3 Marks) then find its nth term. Hence write its 20th term.(3 Marks)
 7
19. Three distinct coins are tossed together. Find the probability
13. Find the area of the triangle ABC with A(1, –4) and mid-points of of getting (3 Marks)
sides through A being (2, –1) and (0, –1). [OS] (3 Marks) (i) at least 2 heads
14. In Figure, from the top of a solid cone of height 12 cm and (ii) at most 2 heads
base radius 6 cm, a cone of height 4 cm is removed by a plane 20. Find that value of p for which the quadratic equation
parallel to the base. Find the total surface area of the remaining (p + l)x2 – 6(p + l)x + 3(p + 9) = 0, p ≠ –1 has equal roots.
 22  Hence find the roots of the equation. (3 Marks)
=
solid.  Use π = and 5 2.236  [OS] (3 Marks)
 7 
Section-D
4 cm 21. In Figure, tangents PQ and PR are drawn from an external
point P to a circle with centre O, such that ∠RPQ = 30°.
12 cm A chord RS is drawn parallel to the tangent PQ. Find ∠RQS.
(4 Marks)

S R
6 cm
O
15. In Figure, from a cuboidal solid metallic block, of
dimensions 15 cm × 10 cm × 5 cm, a cylindrical hole of
diameter 7 cm is drilled out. Find the surface area of the 30°
 22  P
remaining block.  Use π =   (3 Marks) Q
 7
22. From a point P on the ground the angle of elevation of the
7 cm top of a tower is 30° and that of the top of a flag staff fixed
on the top of the tower, is 60°. If the length of the flag staff
is 5 m, find the height of the tower. (4 Marks)
23. Ramkali required ` 2500 after 12 weeks to send her
5 cm
daughter to school. She saved ` 100 in the first week and
increased her weekly saving by ` 20 every week. Find
whether she will be able to send her daughter to school
after 12 weeks. What value is generated in the above
10 cm
15 cm situation? (4 Marks)

16. In Figure, find the area of the shaded region [Use π = 3.14] 24. A box contains 20 cards numbered from 1 to 20. A card is
drawn at random from the box. Find the probability that
(3 Marks)
the number on the drawn card is:  (4 Marks)
14 cm
(i) divisible by 2 or 3
3 cm
(ii) a prime number
25. Water is flowing at the rate of 2.52 km/h through a
14 cm

cylindrical pipe into a cylindrical tank, the radius of whose


base is 40 cm, If the increase in the level of water in the
3 cm
tank, in half an hour is 3.15 m, find the internal diameter
of the pipe. (4 Marks)

282
282 MATHS
26. A well of diameter 4 m is dug 14 m deep. The earth taken pool separately, if the pipe of smaller diameter takes 10
out is spread evenly all around the well to form a 40 cm hours more than the pipe of larger diameter to fill the pool.
high embankment. Find the width of the embankment. (4 Marks)
(4 Marks) 29. Prove that the lengths of tangent drawn from an external
point to a circle are equal. (4 Marks)
2 2 23
27. Solve for x: + = , x ≠ 0, −1, 2  (4 Marks) 30. Construct an isosceles triangle whose base is 6 cm and
x + 1 2( x − 2) 5 x
altitude 4 cm. Then construct another triangle whose sides are
28. To fill a swimming pool two pipes are to be used. If the 3/4 times the corresponding sides of isosceles triangle.
pipe of larger diameter is used for 4 hours and the pipe  [OS] (4 Marks)
of smaller diameter for 9 hours, only half the pool can be 31. If P(–5, –3), Q(–4, –6), R(2, –3) and S(1, 2) are the vertices
filled. Find, how long it would take for each pipe to fill the of a quadrilateral PQRS, find its area. [OS] (4 Marks)

EXPLANATIONS

1. Total number of possible outcomes in an English alphabet = 26 In DAOB


Total number of favourable outcomes of chosen letter is a AB 1 x
∴ tan 30º = ⇒ = ⇒ OB = 3 x ...(ii)
consonant = 21 OB 3 OB
21 From figure, O is mid point of QB
\ P(Chosen letter is a consonant) =  (1 Mark)
26 y x 1 1
\ OQ = OB ⇒ 3 x = = ⇒ = ½ Mark)

2. PA = PB [\ Tangent drawn from an external 3 y 3× 3 3
point to a circle is equal] −1
4. Given x = is one roots and quadratic equations is
2
A
3x2 + 2kx – 3 = 0 ½ Mark)
...(i) (½
Put value of x in equations (i)
1 1 3 −9
P 50o O ⇒ 3 − 2k − 3 = 0 ⇒ k =−3 + =  ½ Mark)

4 2 4 4
5. AC2 = AB2 + BC2 A (5, 2)
B
⇒ (–2 – 5)2+(t – 2)2 (1 Mark)
\ ∠PAB = ∠PBA
= (2 – 5)2+(– 2– 2)2
We know +(–2 – 2)2 + (t + 2)2
∠PAB + ∠PBA + ∠APB = 180°
⇒ 49 + t2 + 4 – 4t B (2, –2) C (–2, t)
⇒ ∠PAB + ∠PAB + 50º = 180°  (∠APB = 50°)
⇒ 2∠PAB = 130° ⇒ ∠PAB = 65° ½ Mark)
(½ = 9 + 16 + 16 +t2 + 4 + 4t
From figure ∠PAO = ∠PAB + ∠OAB ⇒ 8t = 49 – 41 ⇒ 8t = 8 ⇒ t = 1 (1 Mark)
⇒ 90° = 65° + ∠OAB  [\PA ⊥ OA] P
⇒ ∠OAB = 25° ½ Mark)

3. P A 6.  ½ Mark)

T M O

y x
Q

60° 30°
PT and QT are tangent ½ Mark)

Q B
O We have to prove that OT is right bisector of the line
In ∆PQO segment PQ
PQ y y we have PT = QT and MT = MT [common sides] (½½ Mark)
∴ tan 60° = ⇒ 3= ⇒ OQ = ½ Mark)
...(i) (½
QO OQ 3 and ∠PTM = ∠QTM

CBSE 2015 (DELHI Term-II) 283


\ DMTP ≅ DMTQ n 3n
2m + −5m +
MP = MQ and ∠PMT = ∠QMT 3
2 = 2 5

= and
\ ∠PMT + ∠QMT = 180° and ∠PMT = ∠QMT = 90° m+n 4 m+n 12
So, OT is right bisector of the line segment PQ. ½ Mark)
(½ 4m + n 3
∴ = ⇒ 16m + 4n = 6m + 6n  ½ Mark)

7. Given that ∠AOQ = 58° 2(m + n) 4

B m 2 1
⇒ 10m = 2n ⇒ = =
n 10 5
⇒ P divides AB in the ratio 1: 5 ½ Mark)

O 10. Given AP is 213, 205, 197, ......, 37
58o a = 213, d = 205 – 213 = – 8 and tn = 37
Q \ tn = a + (n –1) d ⇒ 37 = 213 + (n –1) × (–8)
A T ⇒ 37 = 213 – 8n + 8 ⇒ 8n = 184 ⇒ n = 23 (1 Mark)
AB is the diameter and AT is a tangent of the given circle (n + 1) th
\ middle term of the given A.P = term = 12th term
From the figue, we have 2
1 \ t12= 213 + 11 × (–8) = 213 – 88 = 125 (1 Mark)
∠ABT = ∠AOT (Angle subtended on the arc is half of
2 11. From the figure
the angle subtended at centre) Q
1 Let OB = radius = r
⇒ ∠ABT = × 58° = 29° (1 Mark) O
2 then radius of semicircle A B
and ∠A = 90  [ BA ⊥ AT] r
Now In DABT AQO =
2
∠ABT + ∠BAT + ∠ATB = 180° P
Given perimeter of the figure = 40 cm (1 Mark)
⇒ 29 + 90 + ∠ATB = 180° i.e., length of arc AQO + length of arc APB + length of
⇒ ∠ATB = 180° – 90° – 29° = 61° (1 Mark) OB = 40
8. Given quadratic equation 1 r 1 πr
⇒ × 2π   + × 2πr + r =40 ⇒ + πr + r = 40
4x2 – 4a2x + (a4 – b4) = 0  ...(i) 2 2 2 2
compare equation (i) with Ax2 + Bx + C = 0 ⇒ 3pr + 2r = 80 ⇒ 3 × 22 × r + 2r =80
A = 4, B = –4a2, C = a4 – b4  (1 Mark) 7
⇒ 80r = 80 × 7 ⇒ r = 7 (1 Mark)
− B ± B − 4 AC 4a ± 16a − 16a + 16b 4
2 2 4 4
=x = Area of shaded region = Area of semi-circle AQO + Area
2A 8
of semicircle APB
a 2 ± b2 a 2 + b2 a 2 − b2
= x ⇒
= x or x =  (1 Mark) 2
2 2 2 1  r  1 2 1 22  7 2 1 22 2
= π   + πr = ×   + × × 7
9. m n 2 2 2 2 7 2 2 7
A 1 3 P 3 5  B (2, – 5)
  77 385 1
 ,   ,  = + 77 = = 96 cm 2  (1 Mark)
 2 2  4 12  4 4 4
Let P divides AB internally in the ratio m:n 12. r = radius of the hemisphere = 3.5 cm
We have, and let, ‘h’ is the hight of the cone
 n 3n  Total volume of solid wooden toy
 2m + 2 −5m + 2  h
\ P= ,   (1 Mark) = volume of cone
 m + n m+n 

  + volume of hemisphere
3 5  5 1 2
and given P =  ,  ⇒ 166 cm3 = πr 2 h + πr 3
 4 12  6 3 3
Equating the corresponding co-ordinate from the point P, 1001 1 2
⇒ =πr (h + 2r )
we get 6 3
284
284 MATHS
1001 1 22 17. Let AB = 30 m is the height of the tower
⇒ = × × (3.5) 2 (h + 2 × 3.5)  (1 Mark)
In DAQB
A
6 3 7
AB P
1001 462 \ tan 45° =
⇒ = h + 7 ⇒ h = ⇒ h = 6 cm (1 Mark) BQ 30 m
77 77
30
Height of to y = h + r = 6 + 3.5 = 9.5 cm ⇒1 = ⇒ BQ = 30m
BQ
Surface area of hemisphere = 2pr2 = 2p(3.5)2 = 77 cm2 (1 Mark) Q 45° 30° B
Cost of painting the hemispherical part of the toy
and In DPQB (1 Mark)
= 77 × 10 = ` 770  (1 Mark)
PQ 1 PQ 30
13. Out of the Syllabus \ tan 30° = ⇒ = ⇒ PQ = = 10 3m
BQ 3 30 3
14. Out of the Syllabus \ Height of building = 10 3m . (1 Mark)
15. Given cuboidal solid metalic dimensions 15 cm × 10 cm × 5 cm 18. Let Sn be the sum of n term and tn be the last term of an A.P
7 cm 1
=
Given, Sn (3n 2 + 7 n)  (1 Mark)
2
1
S1 = (3 + 7) =5 =a (first term)
5 cm
2
1
S2 = (12 + 14) = 13  (1 Mark)
2
\ a + d = second term = S2 – S1 = 13 – 5 = 8
10 cm ⇒5+d=8⇒d=3
15 cm
\ tn = a + (n –1)d ⇒ t20 = 5 + (n – 1) × 3 = 3n + 2
Diameter of cylindrical hole = 7cm \ t20 = 3 × 20 + 2 = 62 (1 Mark)
\ Total surface area of cuboidal solid metalic
19. The possible outcomes = {H,T} × {H,T} × {H,T}
= 2 (lb + bh + hl) = 2(15 × 10 + 10 × 5 + 5 × 15) = {H,T} × {HH, HT, TH, TT}
= 2 × 275 = 550 cm2 (1 Mark) = {HHH, HHT, HTH, HTT, THH, THT, TTH, TTT}
2
22  7  (1 Mark)
Area of circular hole = 2× pr2 =2 × ×   =77cm 2 \ Total no. of possible outcomes = 8
7 2
 (1 Mark) (i) Number of favourable outcomes at least two heads
Surface area of the cylindrical hole = 2prh = {HHT, HHH, HTH, THH} = 4
22 7 4 1
= 2 × × × 5 = 110 cm 2 \ Probability of at least 2 heads = =  (1 Mark)
7 2 8 2
\ Surface area of remaining block (ii) Number of favourable outcomes at most two heads
= 550 – 77 + 110 = 660 – 77 = 583 cm2 (1 Mark) = {HHT, HTH, HTT, THH, THT, TTH, TTT} = 7
7
16. Area of the square = 14 × 14 = 196 cm2  (1 Mark) \ Probability of at most 2 head =  (1 Mark)
8
Area of square PQRS = (2 + 2)2 A B 20. Given quadratic equation.
3 cm (p + 1)x2 – 6 (p + 1)x + 3 (p + 9) = 0, p ≠ –1...(i)
= 16 cm2
We know for the equal roots of the quadratic equation
Area of one semicircle P 2 Q
discriminant us equal to zero. (1 Mark)
14 cm

2 2
πr 2× 2 2 22 2 i.e., D = 0
= = 3.14 × = 6.28 cm2
2 2 S 2 R
 (1 Mark) ⇒ b2 – 4ac = 0 ⇒ {–6 (p +1)}2 – 4 × (p + 1) ×3 (p + 9) = 0
3 cm ⇒ 36(p2 + 2p + 1) – 12(p2 + 10p + 9) = 0 ⇒ 2p2 – 4p – 6 = 0
\Area of shaded region = Area
of square ABCD – Area of D C
⇒ p2 – 2p – 3 = 0 ⇒ p2 –3p + p – 3 = 0
PQRS – 4 × Area of semicircle ⇒ p (p – 3) + 1 (p – 3) = 0 ⇒ (p + 1) (p – 3) = 0
= 196 cm2 – 16 cm2 – 4 × 6.28 cm2 = (180 – 25.12) cm2 ⇒ p = – 1, 3,
= 154.88 cm2  (1 Mark) \ p = 3 [ p ≠ –1] (1 Mark)

CBSE 2015 (DELHI Term-II) 285


put p = 3 in equation (i), we get 23. Ramkali required money for daughter admission = 2500
4x2 – 24x + 36 = 0 ⇒ x2 – 6x + 9 = 0 ⇒ x2 – 3x – 3x + 9 = 0 After weekly saving, it form an A.P.
⇒ x (x–3) – 3 (x–3) = 0 ⇒ (x–3) (x–3) = 0 ⇒ x = 3, 3 So A.P is 100, 120, 140, ........... to 12 terms.
 (1 Mark) Here a = 100, d = 20 and n = 12
21. Sum of money after 12 weeks. (2 Marks)
S n 12
R S=
n [ 2a + (n − 1)d ]= [ 2 ×100 + 11× 20]
2 2
O
= 6 [200 + 220] = ` 2520. (2 Marks)
24 Given box contains 20 cards number from 1 to 20
30o Total number of out comes = 20
P
(1 Q Mark) (i) Total number of favourable outcomes which is divisible
From the figure, we have by 2 or 3
PQ = PR (equal tangent) = (2, 3, 4, 6, 8, 9, 10, 12, 14, 15, 16, 18, 20) = 13
13
and ∠PQR = ∠PRQ (angle opposite to equal side) (1 Mark) \ Probability of getting no. divisible by 2 or 3 =
20
In DPQR,
 (2 Marks)
∠PRQ + ∠PQR + ∠QPR = 180° (sum of interior angle of (ii) Total no. of favourable outcomes (a prime number)
a triangle = 180°) (1 Mark) = (2, 3, 5, 7, 11, 13, 17, 19) = 8
⇒ ∠PQR + ∠PQR + 30° = 180° ⇒ ∠PQR = 75° 8 2
\ Probability of getting a prime number = =
\ we have ∠SRQ = ∠PQR = 75° (by alternative angle) 20 5
 (2 Marks)
In DSQR
25. Given, increase in the level of water in the tank in half an
\ ∠SQR + ∠QRS + ∠RSQ = 180° hour is 3.15 m = 315 cm = h
⇒ ∠SQR + 75° + 75° = 180°
and radius of the cylindrical tank = 40 cm = r
⇒ ∠SQR = 180 – 150 = 30°  (1 Mark)
Volume of water in the tank = pr2h = p(40)2 × 315
22. P
= 504000p cm3 (2 Marks)
Water following rate = 2.52 km/h
5m

We know volume of the water flow through the pipe in


A
half an hour = volume of water falls in the tank in half an
h (1 Mark) hour  (1 Mark)
60 o
2
d  504000π× 4
30o R = 504000π ⇒ d 2 =
⇒ π×   × 126000
 Q 2
  126000 × π
Let AQ = h (height of the tower) and PA = 5m (height of ⇒ d2 = 16 ⇒ d = 4 cm. (1 Mark)
flag staff)
\ In DAQR 4
26. \ radius (r) = =2m
2
AQ 1 h
tan 30° = ⇒ = ⇒ QR = 3h ...(i) (1 Mark) depth of the well (h) = 14 m
QR 3 QR
and In DPQR Volume of the earth taken out after digging the well = pr2h
22
PQ 5+ h 5+ h = × (2) 2 × 14 = 176 m3  (1 Mark)
tan 60° = ⇒ 3= ⇒ QR = ...(ii) (1 Mark) 7
QR QR 3
Let x be the width of the embankment formed by the earth
From equation (i) & (ii) Total width of well including embankment(R) = 2 + x
5+ h 5 40 2
= 3h ⇒ 5 + h = 3h ⇒ 2h = 5 ⇒ h = =
and height of embankment = 40 cm = m m
3 2 100 5
\ Height of tower = 2.5 m (1 Mark)  (1 Mark)

286
286 MATHS
Now, volume of well = volume of embankment ⇒ 8x + 80 + 18x = x2 + 10x ⇒ x2 – 16x – 80 = 0
22  2 ⇒ x2 – 20x +4x – 80 = 0 ⇒ x(x – 20) + 4 (x – 20) = 0
⇒ 176 = p (R2 – r2) × H ⇒ 176 = (2 + x) 2 − (2) 2  ×
7  5 ⇒ (x – 20) (x + 4) = 0 ⇒ x = 20, – 4
176 × 7 × 5
⇒ x2 + 4x + 4 – 4 = ⇒ x2 + 4x – 140 = 0 \ x = 20 (x ≠ – 4)
2 × 22
Hence larger diameter pipe fill the pool in 20 hours and
⇒ x2 + 14x – 10x – 140 = 0 ⇒ x(x + 14) – 10 (x + 14 ) = 0 smaller diameter pipe fill the pool in 30 hours. (2 Marks)
⇒ (x + 14) (x – 10 ) = 0 29. Let PQ and PR be the two tangents and O be the centre of
\ x = –14 or 10 the circle. 
Q
So, x = 10 (x ≠ –14) (2 Marks)
2 3 23 4( x − 2) + 3( x + 1) 23
27. Given + = ⇒ =
x + 1 2( x − 2) 5 x 2( x + 1) ( x − 2) 5x O P (1 Mark)
⇒ 5x (7x – 5) = 46 (x2 – x – 2) (1 Mark)
R
⇒ 35x2 – 25x = 46x2 – 46x – 92 

⇒ 11x2 – 21x – 92 = 0 (1 Mark) Now we have to prove that PQ = PR


In DPRO and DPQO
21 ± (−21) 2 + 4 × 11× 92
\ x=  (1 Mark) OQ = OR (1 Mark)
22
and ∠PQO = ∠PRO = 90° (Tangent at any point of a circle
21 ± 441 + 4048 21 ± 67 −23 is perpendicular to the radius through in point of contect
= = = 4,  (1 Mark)
22 22 11 \ We have OP = OP (common)
28. Given pipe of larger diameter used to fill the swimming
So, DOQP ≅ DORP (By R.H.S congruence)  (1 Mark)
pool = x hours
then smaller diamer fill the pool = (x + 10) hours. (1 Mark) \ PQ = PR
then by the questions So length of tangents drown from an external point of a
4 9 1 4( x + 10) + 9 x 1 circle are equal.  (1 Mark)
⇒ + = ⇒ =
x x + 10 2 x( x + 10) 2 30. Out of the Syllabus
⇒ 8 (x + 10) + 18x = x (x + 10)  (1 Mark) 31. Out of the Syllabus

CBSE 2015 (DELHI Term-II) 287


Outside DELHI Term-II
2015 CBSE Solved Paper

Time allowed : 3 hours Maximum Marks : 90


GENERAL INSTRUCTIONS:
Read the following instructions carefully and strictly follow them:
(i) All questions are compulsory.
(ii) The question paper consists of 31 questions divided into four sections - A, B, C and D.
(iii) Section A contains 4 questions of 1 mark each. Section B contains 6 questions of 2 marks each, Section C contains 10
questions of 3 marks each and Section D contains 11 questions of 4 marks each.
(iv) Use of calculators is not permitted.

Section-A Section-B
1. In Figure, a tower AB is 20 m high and BC, its shadow on 5. In an AP, if S5 + S7 = 167 and S10 = 235, then find the AP,
the ground, is 20 3 m long. Find the Sun’s altitude.  where Sn denotes the sum of its first n terms. (2 Marks)
 (1 Mark) 6. The points A(4, 7), B(p, 3) and C(7, 3) are the vertices of a
A right triangle, right-angled at B. Find the value of p. (2 Marks)
7. In Figure, two tangents RQ and
RP are drawn from an external
point R to the circle with centre P
O. If ∠PRQ = 120o, then prove
that OR = PR + RQ. (2 Marks) O R
8. In Figure, a triangle ABC is
C B drawn to circumscribe a circle Q
2. Two different dice are tossed together. Find the probability of radius 3 cm, such that the
that the product of the two numbers on the top of the segments BD and DC are
dice is 6. (1 Mark) respectively of lengths 6 cm and 9 cm. If the area of ∆ABC
is 54 cm2, then find the lengths of sides AB and AC.  (2 Marks)
3. If the quadratic equation px2 – 2 5 px + 15 = 0 has two A
equal roots, then find the value of p. (1 Mark)
4. In Figure, PQ is a chord of a circle with centre O and PT is
a tangent. If ∠QPT = 60o, find ∠PRQ. (1 Mark)
O
3 cm
Q
B C
D
6 cm 9 cm
O
9. Find the relation between x and y if the points A(x, y),
R B(– 5, 7) and C(– 4, 5) are collinear.  [OS] (2 Marks)
10. Solve the following quadratic equation for x: x2 – 2ax –
P T (4b2 – a2) = 0. (2 Marks)
Section-C Section-D
11. Due to sudden floods, some welfare associations
21. At a point A, 20 metres above the level of water in a lake,
jointly requested the government to get 100 tents fixed
the angle of elevation of a cloud is 30o. The angle of
immediately and offered to contribute 50% of the cost. If
depression of the reflection of the cloud in the lake, at A is
the lower part of each tent is of the form of a cylinder
60o. Find the distance of the cloud from A. (4 Marks)
of diameter 4.2 m and height 4 m with the conical upper
22. A card is drawn at random from a well-shuffled deck of playing
part of same diameter but of height 2.8 m, and the canvas
cards. Find the probability that the card drawn is (4 Marks)
to be used costs ` 100 per sq.m, find the amount, the
(i) a card of spade or an ace.
associations will have to pay. What values are shown by
(ii) a black king.
 22 
these associations ?  Use π =   (3 Marks) (iii) neither a jack nor a king.
 7 (iv) either a king or a queen.
12. A hemispherical bowl of internal diameter 36 cm contains 23. In Figure, PQRS is a square lawn with side PQ = 42
liquid. This liquid is filled into 72 cylindrical bottles of metres. Two circular flower beds are there on the sides PS
diameter 6 cm. Find the height of the each bottle, if 10% and QR with centre at O, the intersection of its diagonals.
liquid is wasted in this transfer. (3 Marks) Find the total area of the two flower beds (shaded parts).
13. A cubical block of side 10 cm is surmounted by a  [OS] (4 Marks)
hemisphere. What is the largest diameter that the S R
hemisphere can have ? Find the cost of painting the total
surface area of the solid so formed, at the rate of ` 5 per O
100 sq.cm. [ Use π = 3.14 ] (3 Marks)
14. 504 cones, each of diameter 3.5 cm and height 3 cm, are
melted and recast into a metallic sphere. Find the diameter P Q
 22  24. From each end of a solid metal cylinder, metal was
of the sphere and hence find its surface area.  Use π =  scooped out in hemispherical form of same diameter. The
 7
(3 Marks) height of the cylinder is 10 cm and its base is of radius
4.2 cm. The rest of the cylinder is melted and converted
15. Solve for x: 3x 2 − 2 2 x − 2 3 =
0 (3 Marks)
into a cylindrical wire of 1.4 cm thickness. Find the length
16. The angle of elevation of an aeroplane from a point A on  22 
the ground is 60o. After a flight of 15 seconds, the angle of the wire.  Use π =   [OS] (4 Marks)
 7
of elevation changes to 30o. If the aeroplane is flying at a 25. The diagonal of a rectangular field is 16 metres more than
constant height of 1500 3 m, find the speed of the plane the shorter side. If the longer side is 14 metres more than
in km/hr. (3 Marks) the shorter side, then find the lengths of the sides of the
field.  (4 Marks)
17. Find the area of the minor segment of a circle of radius
26. Prove that the lengths of the tangents drawn from an
14 cm, when its central angle is 60o. Also find the area of the
external point to a circle are equal. (4 Marks)
 22 
corresponding major segment.  Use π =   (3 Marks) 27. Prove that the tangent drawn at the mid-point of an arc of a circle
 7 is parallel to the chord joining the end points of the arc. (4 Marks)
18. The 13 term of an AP is four times its 3 term. If its fifth
th rd
28. A truck covers a distance of 150 km at a certain average
term is 16, then find the sum of its first ten terms. (3 Marks) speed and then covers another 200 km at an average speed
19. Find the coordinates of a point P on the line segment which is 20 km per hour more than the first speed. If the
truck covers the total distance in 5 hours, find the first
2
joining A(1, 2) and B(6, 7) such that AP = AB. (3 Marks) speed of the truck. (4 Marks)
5
29. An arithmetic progression 5, 12, 19, ... has 50 terms. Find its
20. A bag contains, white, black and red balls only. A ball is last term. Hence find the sum of its last 15 terms. (4 Marks)
drawn at random from the bag. If the probability of getting 30. Construct a triangle ABC in which AB = 5 cm, BC = 6 cm
3 2 and ∠ABC = 60o. Now construct another triangle whose
a white ball is and that of a black ball is , then find
10 5 sides are 5/7 times the corresponding sides of ∆ABC.
the probability of getting a red ball. If the bag contains  [OS] (4 Marks)
20 black balls, then find the total number of balls in 31. Find the values of k for which the points A(k + 1, 2k),
the bag. (3 Marks) B(3k, 2k + 3) and C(5k – 1, 5k) are collinear.[OS] (4 Marks)

CBSE 2015 (Outside DELHI Term-II) 289


EXPLANATIONS

1. Given tower (AB) = 20 m and shadow (BC) = 20 3 ⇒ 5a + 10d + 7a + 21d = 167



A ⇒ 12a + 31d = 167
...(i) (1 Mark)
AB
∴ tan θ = ½ Mark)

BC and given S10 = 235
20 1 10

20 m
⇒=tan θ = [2a + 9d ] =
235
20 3 3 2
⇒ tan θ = tan 30 ⇒ 2a + 9d = 47
...(ii)
θ
⇒ θ = 30 ½ Mark) C
(½ B Multiplying eqn (ii) by 6 and substracting from (i), we get
20 3m
2. Total number of outcomes –23d = –115 ⇒ d = 5 ⇒ From (2) a = 1
½ Mark)
= {1, 2, 3, 4, 5, 6} × {1, 2, 3, 4, 5, 6} = 6 × 6 = 36 (½ AP is 1, 6, 11, 16, 21, ... (1 Mark)
Total favourable outcomes (getting product are 6) 6. We know
= {(2, 3) (3, 2) (1, 6) (6, 1)} = 4 (4, 7)
A
4 1
=
Probability of favourable outcomes = ½ Mark)
 (½
36 9
3. Given quadratic equation
px 2 − 2 5 px + 15 =
0
For the real and equal roots B C (7, 3)
(p, 3)
\D=0
AB2 + BC2 = AC2 ⇒ (p – 4)2 + (3 – 7)2 + (7 – p)2 + (3 – 3)2

⇒ b2 – 4ac = 0 ½ Mark)

= (7 – 4)2 + (3 – 7)2 (1 Mark)
⇒ (−2 5 p ) 2 − 4 × p × 15 =0
⇒ p + 16 – 8p + 16 + 49 + p – 14p = 9 + 16
2 2
⇒ 20p2 – 60p = 0
⇒ 20p (p – 3) = 0 ⇒ p = 0, 3 ½ Mark)
(½ ⇒ 2p2 – 22p = 9 + 16 – 16 – 16 – 49

4. Given that PQ is chord and ∠QPT = 60° ⇒ 2p2 – 22p = –56 ⇒ p2 – 11p = –28 ⇒ p2 – 11p + 28 = 0

Q ⇒ p2 – 4p – 7p + 28 = 0 ⇒ p(p – 4) –7 (p – 4) = 0

From the figure, we have
∠OPT = ∠QPO + ∠QPT
⇒ (p – 4) (p – 7) = 0

O ⇒ p = 4, 7
(1 Mark)
⇒ 90° = ∠QPO + 60°

R
⇒ ∠QPO = 90° – 60° = 30°
7.
∴ ∠OQP = 30°

P T P
[OPQ is isosceles triangle]  ½ Mark)

In ∆ OPQ
⇒ ∠QPO + ∠POQ + ∠OQP = 180° O R
[Sum of interior ∠'s of a triangle is 180°.]
⇒ 30° + ∠POQ + 30° = 180° Q
⇒ ∠POQ = 180° – 30° – 30° = 120°
∴ reflex ∠POQ = 360° – 120° = 240°
1 Given that ∠PRQ = 120°
1
∴ ∠PRQ = × reflex ∠POQ = × 240 = 120° (½ ½ Mark) We have to prove that OR = PR + RQ
2 2
5. Let a be the first and d be the common differennce 120
∴∠PRO = ∠QRO =
= 60  ½ Mark)

Given S5 + S7 = 167 2
5 7 [∴ Centre lies on the bisector of the angle between the two
⇒ [2a + 4d ] + [2a + 6d ] = 167
2 2 tangents]

290 MATHS
From figure 9. Out of the Syllabus
∠OPR = ∠OQR = 90
10. Given quadratic equation
[Radius of the circle is perpendiculas to the tangent to the x2 – 2ax – (4b2 – a2) = 0
circle through the point of contact] Here a′ = 1, b′ = –2a, c′ = –(4b2 – a2) (1 Mark)
Now In ∆OPR
−b′ ± (b′) 2 − 4a′c′
PR 1 PR ∴x =
cos 60
=° ⇒= = 2 PR  ...(i) (½
⇒ OR ½ Mark) 2a′
OR 2 OR
In ∆OQR 2a ± 4a 2 + 4 × 1× (4b 2 − a 2 )
=
RQ 1 RQ 2 ×1
cos 60=
⇒ = ⇒ OR= 2 RQ ...(ii) (½
½ Mark)
OR 2 OR
2a ± 16b 2 2a ± 4b
Adding eqn (i) and (ii) = =
2 2
OR + OR = 2 PR + 2 RQ
⇒ x = a ± 2b (1 Mark)
⇒ 2 OR = 2 (PR + RQ)
11. Given diameter of cylinderical tent = 4.2 cm
⇒ OR = PR + RQ
½ Mark)

8. A l
x x 2.8 m
P Q
O r = 2.1
3 cm

B D C
6 cm 9 cm 4m
.
Let AP = AQ = x [By the tangents drawn from an external
point to a circle are equal.]
Similarly
4.2 m
DC = QC = 9 cm and BD = BP = 6 cm and OD = OP = OQ

= 3 cm  ½ Mark)
(½ diameter 4.2
∴ radius = = = 2.1 m
Now 2 2
Area of ∆ABC = area of ∆BOC + area of ∆AOC + area of l = slant height = (2.8) 2 + (2.1) 2 = 3.5 m  (1 Mark)
∆AOB
Total area of cloth to make tent
1 1 1
⇒ 54 cm2 =
× BC × OD + × AC × OQ + × AB × OP = curved surface area of cylinder + curved surface area of
2 2 2 conical tent
1 1
⇒ 5 4 cm2 =
(BD + DC) × OD + × (AQ + QC) × OQ = 2πrh + πrl
2 2
1 22 22
+ (AP + BP) × OP. ½ Mark)
(½ =2 × × 2.1× 4 + × 2.1× 3.5
2 7 7
1 1 1 22
⇒ 54cm 2 = × 15 × 3 + ( x + 9) × 3 + ( x + 6) × 3
= (16.8 + 7.35)
2 2 2 7
3 22 531.3
⇒=
54 [2 x + 30] = × 24.15 = =75.9  (1 Mark)
2 7 7
⇒ 54 = 3 (x + 15)
Cost of one tent = 75.9 × 100 = 7590
⇒ 3x = 54 – 45
Total cost of 100 tent = 7590 × 100 = 759000
⇒ 3x = 9 ⇒ x = 3
½ Mark)

50
∴ AB = AP + BP = x + 6 = 3 + 6 = 9 cm
Cost by welfare associations= 759000 ×
= 379,500
100
and AC = AQ + QC = x + 9 = 3 + 9 = 12 cm ½ Mark)
(½ (1 Mark)

CBSE 2015 (Outside DELHI Term-II) 291


12. Given diameter = 36 cm 15. Given (1 Mark)
3x2 − 2 2 x − 2 3 =
0
36
∴ radius = = 18 cm ⇒ 3x2 − 3 2 x + 2 x − 2 3 =
0
2
36 cm ⇒ 3 x( x − 6) + 2( x − 6) =
0 (1 Mark)
⇒ ( x − 6)( 3 x + 2) =
0
− 2
⇒ x =6 or  (1 Mark)
3
2 3 3 16. Q
Volume of hemispherical bowl= πr cm R
3

= × (18)3 ⋅ cm3  (1 Mark)
3 1500 3m
Volume of hemispherical bowl of 10% wastage °
60
2π 90 30° ym S
= × 183 ×
3 100 P x T
Volume of cylinder = πr2h Let QT = height of Aeroplane = 1500 3m
Volume of 72 bottles = π × 32 × h × 72  (1 Mark) After 15 seconds R be the position of the aeroplane which
[ radius of cylinder = 3 cm] makes the angle of elevation 30°.
2π 90 Let PT = x m and TS = y m
× (18)3 ×
∴h =
3 100 In DPQT
π× 32 × 72
QT
2π  18 
3
9 tan 60 =

=
× = 5.4 cm  (1 Mark) PT
 
π  3  72 × 10
1500 3m
13. Given diameter = 10 cm ⇒ 3
=
= ⇒ x 1500m  (1 Mark)
x
10 In DPRS
∴ Radius(r) of hemisphere
= = 5 cm  (1 Mark)
2 RS
Total Surface are = Total surface area of cube – Area of tan 30° =

PS
base of hemisphere + Curved surface area of hemisphere.
= 6 × 102 + 3.14 × 52 (1 Mark) 1 1500 3 1 1500 3

= ⇒ =
= 600 + 3.14 × 25 3 PT + TS 3 x+ y

= 600 + 78.5 = 678.5 cm2 ⇒ x + y = 1500 × 3 = 4500 m



5 678.5
Cost of printing = 678.5 × = = 33.925 (1 Mark) ∴ y = 4500 – x = 4500 – 1500 = 3000 m
(1 Mark)
100 20
distance y 3000 m
14. Given diameter = 3.5 cm and height of cone is 3 cm. ∴ Speed of aeroplane = = = = 200 m/s
3.5 time 15 15 sec
radius = 18
2 = 200 ×
km/h = 40 × 18 = 720 km/h (1 Mark)
volume of 504 cones = volume of sphere 5
2 17. Let PRQ be the arc which subtending an angle of 60°
1 22  3.5  4 22
504 × × ×  × 3 = × × r3  (1 Mark)
3 7  2  3 7
35 35 4 3 504 × 35× 35× 3
⇒ 504 × × = × r ⇒ r3 = O
20 20 3 20× 20× 4
14
cm

21 60°
⇒ r= = 10.5 cm
c


m
14

2 P Q
∴ diameter = 21 cm (1 Mark) R
Surface area of sphere = 4 πr2
Area of the minor segment PRQP (1 Mark)
22 9702 = Area of the sector OPRQ – Area of ∆OPQ
=4 ×
= = 1386 cm 2 
× 10.5 × 10.5 (1 Mark)
7 7

292 MATHS
θ 1 20. Let A, B and C be the event of getting white, black and red
=
πr 2 × − r 2 sin θ ball.
360 2
22 60 1 3 2
∴ Given P ( A) = , P( B) =  (1 Mark)
=
× (14) 2 × − × (14) 2 sin 60 10 5
7 360 2
3 2 3
1 3 So P (C ) =1 − − =  (1 Mark)
= 22 × 2 × 14 × − 7 × 14 × 10 5 10
6 2
2
308 ∴ × Total numbers of balls = 20
= − 49 3 =17.89 cm 2  (1 Mark) 5
3
20 × 5
Area of major segment = Area of circle – Area of minor ∴ Total numbers of balls = 50 balls.  (1 Mark)
=
segment PRQP. 2
21. Let P be the position of the cloud which makes an angle
= πr2 – 17.89 = 22 × 14 × 14 − 17.89 of elevation from A is 30° and Q be the position of
7 reflection of the cloud which makes angle of depression
= 616 – 17.89 = 598.11 cm2 (1 Mark) from A is 60°.
P
18. Let a, d be the first and common difference and Sn be the
surn of n-terms of an A.P h
Given t13 = 4t3 30°
A x
⇒ 9 + 12d = 4 (9 + 2d)
60° B

⇒ 3a = 4d
...(i) (1 Mark) 20m 20m

and given t5 = 16 C D

⇒ a + 4d = 16 ⇒ a + 3a = 16 (from eqn. (i))



⇒ 4a = 16 ⇒ a = 4

Q
3a 3 × 4 (1 Mark) Here AC = BD = 20 m
∴d = = = 3
4 4
PD = 20 + h = DQ (1 Mark)
We know
n PB h
S=
n [2a + (n − 1)d ] In DAPB, tan 30
=° =
2 AB x
10 1 h
∴ S10
= [2 × 4 + (10 − 1) × 3] ⇒ = ⇒x= 3h  ...(i) (1 Mark)
5 3 x
= 5[8 + 27] = 5 × 35 =175 (1 Mark)
In DAQB
19. 5 BQ BD + DQ 20 + 20 + h
2 3 tan=
60 = =
AB x x
A(1,2) P(x,y) B(6,7) 40 + h 40 + h
⇒= 3 ⇒ = x  ...(ii) (1 Mark)
x 3
Let P(x, y) be the any point on AB From eqn. (i) of (ii)
Given AP = 2 AB ⇒ AP =
2
∴ PB = 5 – 2 = 3 (1 Mark) 40 + h
5 AB 5 = 3h ⇒ 40 + h = 3h ⇒ h = 20
3
So P divides AB internally in the ratio 2:3
Put h = 20 in eqn (i), we get x = 20 3
 mx + nx1 my2 + ny1 
P ( x, y ) =  2 ,  (1 Mark)
 m+n m + n  From the figure

 2 × 6 + 3 × 1 2 × 7 + 3 × 2   15 20  AP = AB 2 + PB 2 = =
x2 + h2 (20 3) 2 + (20) 2
= , = ,
 2+3 2 + 5   5 5 
= 20 3 + 1 = 20 × 2 = 40 m
P(x, y) = (3, 4) (1 Mark) ∴ Distance of the cloud from A = 40 m. (1 Mark)

CBSE 2015 (Outside DELHI Term-II) 293


22. Let S = Sample Space = 52 26. Given: Two tangents PQ and PR are drawn from a point P
(i) Let A be the event of drawing a card of spade or an to a circle having centre O.
ace. Q
= 13 + 4 – 1 = 16
16 4
∴ P( A) = =  (1 Mark) P O
52 13
(ii) Let B be the event of drawing a black king = 2
2 1 R
∴ P( B) = =  (1 Mark)
52 26 To prove: PQ = PR
(iii) Let C be the event of drawing neither a jack nor a king Construction: Join OP, OQ and OR. (1 Mark)
= 52 – (4 + 4) = 44 Proof: PQ is a tangent at Q and OQ is the radius.
44 11 ∴ OQ ⊥ PQ [tangent is ⊥ to the radius]
P(C= ) = (1 Mark)
52 13
Similarly, OR ⊥ PR (1 Mark)
(iv) Let D be the event of drawing either a king or a queen.
=4+4=8 In the right triangle OPQ and OPR, we have
OQ = OR  [radii of the same circle]
8 2
P( D=
)
= (1 Mark)
52 13 OP = OP  [common]
23. Out of the Syllabus ∠OQP = ∠ORP [90° each]
24. Out of the Syllabus ∴ DOPQ ≅ DOPR [by RHS-congruence] (1 Mark)
25. D C Hence, PQ = PR. [by C.P.C.T] (1 Mark)
27. Let D be the mid point of the minor arc AB.
O be the centre and
PQ is tangent. We
A B
have to prove that O
Let the length of shorter side i.e., BC = x
AB||PQ
then, length of diagonal i.e., AC = x + 16 ...(i) A B
arc AD = arc BD
and, length of other side i.e., AB = x + 14 ...(ii) (1 Mark)
AD = BD
As, we know, each angle of a rectangle is 90°.
Now, in ∆ABC using pythagoras theorem, we get From the figure DABD P D Q
is an isosceles triangle. Thus the perpendicular bisector of
AC2 = AB2 + BC2
the side
⇒ (x + 16)2 = (x + 14)2 + x2 (1 Mark) (1 Mark)
⇒ x2 + 256 + 32x = x2 + 196 + 28x + x2
AB of DABD passes through vertex D. The perpendicular
⇒ x2 – 4x – 60 = 0 ⇒ x2 – 10x + 6x – 60 = 0
bisector of a chord passes through the centre of the circle
⇒ x (x – 10) + 6 (x – 10) = 0 ⇒ (x + 6) (x – 10) = 0
so perpendicular bisector of AB passes through the centre
O of the circle. Thus perpendicular bisector of AB passes
⇒ x + 6 = 0 or x – 10 = 0 ⇒ x = –6 or x = 10
through the points O and D. (1 Mark)
Since, length of side cannnot be negative. ∴ AB ⊥ OD
∴ x = 10 m
(1 Mark) PQ is the tangent to the circle through the point D on the
From eq. (i) circle.
AC = x + 16 = 10 + 16 = 26 m and from eq. (ii) PQ ⊥ OD (1 Mark)
AB = x + 14 = 10 + 14 = 24 m The chord AB and the tengent PQ of the circle are
perpendicular to the same line OD
Hence, length of the sides of the field are 10 m and 24 m.
 (1 Mark) ∴ AB || PQ. (1 Mark)

294 MATHS
28. Let the average speed of the truck be x km/h. ⇒ x + 10 = 0 or x – 60 = 0
Then, new average speed of the truck = (x + 20) km/h. ⇒ x = –10 or x = 60
150  Speed cannot be negative
Time taken by truck to cover 150 km = hrs. (1 Mark)
x ∴ First speed = 60 km/h (1 Mark)
200
And, Time taken by truck to cover 200 km = hrs 29. Given A.P. is 5, 12, 19, ...
x + 20
Total time taken by truck = 5 hrs. Here, a = 5 and d = 12 – 5 = 7
According to the question, As we know, tn = a + (n – 1) d

150 200 ∴ t50 = a + (50 – 1) d = 5 + 49 × 7 = 5 + 343 = 348


+ 5
= (1 Mark) (1 Mark)
x x + 20 Hence, its last term is 348.
150( x + 20) + 200( x)
⇒ 5
= Not 15th from last = (n – 15 + 1)th term from beginning
x( x + 20)
= (50 – 15 + 1)th
⇒ 150x + 3000 + 200x = 5x (x + 20)
= 36th term from beginning (1 Mark)
⇒ 350x + 3000 = 5x + 100x
2
Hence, t36 = a + (36 – 1) d = 5 + 35 × 7 = 5 + 245 = 250
⇒ 5x2 – 250x – 3000 = 0 (1 Mark)
⇒ 5(x2 – 50 x – 600) = 0 (1 Mark) n 15
=
∴ Sum of last 15 term [=
a + ] [250 + 348]
⇒ x2 – 50x – 600 = 0 2 2
15
⇒ x2 – 60x + 10x – 600 = 0 = × 598 = 15 × 299 = 4485 (1 Mark)
2
⇒ x(x – 60) + 10 (x – 60) = 0 30. Out of the Syllabus
⇒ (x + 10) (x – 60) = 0 31. Out of the Syllabus

CBSE 2015 (Outside DELHI Term-II) 295


Term-I
2015 CBSE Solved Paper

Time allowed : 3 hours Maximum Marks : 90


GENERAL INSTRUCTIONS:
Read the following instructions very carefully and follow them:
(i) All questions are compulsory.
(ii) The question paper consists of 31 questions divided into four sections - A, B, C and D.
(iii) Section A contains 4 questions of 1 mark each, Section B contains 6 questions of 2 marks each, Section C contains
10 questions of 3 marks each and Section D contains 11 questions of 4 marks each.
(iv) Use of calculators is not permitted.

7. Complete the following factor tree and find the composite


Section-A number x. (2 Marks)
1. In the given figure if DE || BC, AE = 8 cm, EC = 2 cm and x
BC = 6 cm, then find DE. (1 Mark)
A 3 195

3 y
8c
m

5 13
D E
8. In a rectangle ABCD, E is middle point of AD. If AD = 40 m
2c

and AB = 48 m, then find EB. (2 Marks)


m

B C 9. If x = p sec θ + q tan θ and y = p tan θ + q sec θ, then prove


6 cm
that x2 – y2 = p2 – q2. (2 Marks)
1 − cot 2 45o
2. Evaluate: 10.  (1 Mark) 10. Given below is the distribution of weekly pocket money
1 + sin 2 90o received by students of a class. Calculate the pocket
5 money that is received by most of the students.(2 Marks)
3. If cosec θ = , find the value of cot θ. (1 Mark)
4
Pocket Money
100–120

120–140
4. Following table shows sale of shoes in a store during one month:
80–100
20–40

40–60

60–80
0–20

(in `)
Size of Shoe 3 4 5 6 7 8
Number of Pairs Sold 4 18 25 12 5 1 No. of Students 2 2 3 12 18 5 2
Find the modal size of the shoes sold. (1 Mark)
Section-C
Section-B
11. Prove that 3 + 2 3 is an irrational number. (3 Marks)
5. Find the prime factorisation of the denominator of rational 12. Solve by elimination: 3x = y + 5, 5x – y = 11. (3 Marks)
number expressed as 6.12 in simplest form.[OS] (2 Marks)
13. A man earns ` 600 per month more than his wife. One-
6. Find a quadratic polynomial, the sum and product of tenth of the man’s salary and one-sixth of the wife’s salary
1 amount to `1,500, which is saved every month. Find their
whose zeroes are 3 and respectively. (2 Marks)
3 incomes. (3 Marks)
14. Check whether polynomial x –1 is a factor of the polynomial 25. In the figure, there are two points D and E on side AB of
x3 – 8x2 + 19x – 12. Verify by division algorithm. ∆ABC such that AD = BE. If DP || BC and EQ || AC, then
 [OS] (3 Marks) prove that PQ || AB. (4 Marks)
15. If the perimeters of two similar triangles ABC and DEF are A
50 cm and 70 cm respectively and one side of ∆ABC = 20 cm,
then find the corresponding side of ∆DEF. (3 Marks)
D P
16. In the figure if DE || OB and EF || BC, then prove that
DF || OC. (3 Marks)
A E
D
E F B Q C
26. In ∆ABC, altitudes AD and CE intersect each other at the
O point P. Prove that (4 Marks)
B C (i) ∆APE ~ ∆CPD (ii) AP × PD = CP × PE
17. Prove the identify: (iii) ∆ADB ~ ∆CEB (iv) AB × CE = BC × AD
(sec A – cos A)(cot A + tan A) = tan A.sec A. (3 Marks) 27. Prove that: (cot A + sec B)2 – (tan B – cosec A)2
18. Given 2 cos 3θ = 3 , find the value of θ. (3 Marks) = 2(cot A.sec B + tan B.cosec A). (4 Marks)
19. For helping poor girls of their class, students saved pocket 28. Prove that:
money as shown in the following table:  (sin θ + cos θ + 1). (sin θ – 1 + cos θ).sec θ.cosec θ = 2.
Money Saved (in `) 5–7 7–9 9–11 11–13 13–15  (4 Marks)
29. If tan (20° – 3α) = cot (5α – 20°), then find the value of α
Number of Students 6 3 9 5 7
and hence evaluate.  [OS] (4 Marks)
Find mean and median for this data. (3 Marks) 30. The frequency distribution of weekly pocket money received
20. Monthly pocket money of students of a class is given in by a group of students is given below:
the following frequency distribution:
Pocket Money (in `) No. of Students
Pocket
100–125 125–150 150–175 175–200 200–225 More than or equal to 20 90
Money (in `)
Number of More than or equal to 40 76
14 8 12 5 11
Students
More than or equal to 60 60
Find mean pocket money using step deviation method.
(3 Marks) More than or equal to 80 55
More than or equal to 100 51
Section-D
More than or equal to 120 49
21. If two positive integers x and y are expressible in terms of primes
as x = p2q3 and y = p3q, what can you say about their LCM and More than or equal to 140 33
HCF. Is LCM a multiple of HCF ? Explain. (4 Marks) More than or equal to 160 12
22. Sita Devi wants to make a rectangular pond on the road side More than or equal to 180 8
for the purpose of providing drinking water for street animals.
The area of the pond will be decreased by 3 square feet if More than or equal to 200 4
its length is decreased by 2 ft. and breadth is increased by Draw a ‘more than type’ ogive and from it, find median.
1 ft. Its area will be increased by 4 square feet if the length Verify median by actual calculations. [OS] (4 Marks)
is increased by 1 ft. and breadth remains same. Find the
31. Cost of living Index for some period is given in the
dimensions of the pond. What motivated Sita Devi to provide
following frequecy distribution:
water point for street animals? (4 Marks)
23. If a polynomial x + 5x + 4x – 10x – 12 has two zeroes as
1500–1600

1600–1700

1700–1800

1800–1900

1900–2000

2000–2100

2100–2200

4 3 2

–2 and –3, then find the other zeroes. [OS] (4 Marks) Index
24. Find all the zeroes of the polynomial 8x4 + 8x3 –18x2 – 20x – 5,
5 5
if it is given that two of its zeroes are and − . No. of Weeks 3 11 12 7 9 8 2
2 2
 [OS] (4 Marks) Find the mode and median for above data. (4 Marks)

CBSE 2015 (Term-I) 297


EXPLANATIONS

1. Given, DE || BC 7. Given, composite tree.


AE = 8 cm, EC = 2 cm, BC = 6 cm x

Now, In ∆ADE and ∆ABC,


3 195
∠DAE = ∠BAC (common) ½ Mark)

∠ADE = ∠ABC (corresponding angles) 3 y

∠ADE ~ ∠BAC (AA similarity)  (1 Mark)


5 13
∴ AE =DE ⇒ 8 =DE From above tree, we can say that
AC BC 8+2 6 3 × 195 = x ⇒ x = 585
⇒ 48 = 10 DE ⇒ DE = 4.8 cm ½ Mark)
(½ Similarly, 5 × 13 = y ⇒ y = 65 (1 Mark)
8. 40m
 1 − cot 45° 
2
A E D
2. Given, 10  2 
 1 + sin 90° 
we know, cot 45° = 1, sin 90° = 1 ½ Mark)
(½ 48m
 1 − (1) 
2
 1−1  0
= 10  2 
= 10  =  10  ½ Mark)

 1 + (1)   1 + 1 
=  0 B C
2
Given, AD = 40 m and E is mid point of AD
5
3. Given; cosec q = 1
4 then, AE = AD = 20 m.
we know, 2
1 + cot q = cosec q 
2 2
½ Mark)
(½  AB = 48 cm ½ Mark)

5
2
now in, DABE, ∠A = 90°
⇒ cot q = cosec q –1 ⇒ cot q =   –1
2 2 2
⇒ (BE)2 = (AE)2 + (AB)2 (By pythagoras theorem)
4
25 25 –16 9 ⇒ (EB)2 = (20)2 + (48)2 ½ Mark)

⇒ cot2q = –1 = ⇒ cot2q =
16 16 16 ⇒ (EB) = 400 + 2304 = 2704
2

3 ⇒ EB = 52 m (1 Mark)
⇒ cot q =  ½ Mark)

4 9. Given, x = p sec q + q tan q , y = p tan q + q sec q
4. From the given table, L.H.S: x2 – y2 = (p sec q + q tan q)2 – (p tan q + q sec q)2 (1 Mark)
Maximum frequency is 25 for shoes size 5 ½ Mark)
(½ = p2sec2q + q2tan2q + 2p q tan q sec q – p2tan2q – q2sec2q
Hence, modal size of shoes is 5 ½ Mark)
(½  – 2p qtan q sec q
5. Out of the Syllabus = p (sec q – tan q) – q (sec q – tan q)
2 2 2 2 2 2

6. Let a and b be the zeroes of required quadratic polynomial. = p2 ∙ 1 – q2 ∙ 1[ sec2q – tan2q = 1]
= p2 – q2 = R.H.S
Given sum of zeroes (a + b) = 3
Hence, L.H.S = R.H.S (Proved.) (1 Mark)
1
Product of zeroes (ab) = 10.
3
Now we know, The required polynomial is x2 – (sum of Pocket money in ` No. of students
zeroes) x + (product of zeroes) = 0(1 Mark) 0 – 20 2
20 – 40 2
1
⇒ x 2 – 3x + 0
= 40 – 60 3
3
60 – 80 12
3x 2 − 3x + 1 80 – 100 18
⇒ =0
3 100 – 120 5
120 – 140 2
⇒ 3x 2 − 3x + 1 =0.  (1 Mark)  ½ Mark)

298
298 MATHS
modal class is 80 – 100 14. Out of the Syllabus
f1 − f 0 15. Given: DABC ~ DDEF
l+
Now, mode = ×h ½ Mark)

2 f1 − f 0 − f 2 perimeter of DABC = 50 cm
l = 80, f1 = 18, f0 = 12, f2 = 5, h = 20 perimeter of DDEF = 70 cm
18 − 12 one side of DABC = BC=20 cm ½ Mark)

mode = 80 + × 20  ½ Mark)

2 × 18 − 12 − 5
now,If DABC ∼ DDEF
6 × 20
= 80 + = 86.32(approx) BC Perimeter of ∆ABC
19 then =  (1 Mark)
½ Mark)
Hence, required pocket money = ` 86.32 (approx)(½ EF Perimeter of ∆DEF
11. Let 3 + 2 3 is a rational number 20 50
=  ½ Mark)

p EF 70
∴3+ 2 3 =  ½ Mark)

q 20 × 70
EF = = 28 cm (1 Mark)
p p – 3q 50
2 3 =
= –3
q q 16. Given, DE || OB and EF || BC
p − 3q now in DAOB, DE || OB
3=  (1 Mark)
2q AE AD
= (Thale’s theorem) ...(i) (1 Mark)
p – 3q EB DO
Since, p and q are integer, so is rational number
2p similarly in DABC, EF || BC
but 3 is irrational. ½ Mark)

AE AF
= (Thale’s theorem) ...(ii) (1 Mark)
So, this is contradictory situation Hence, 3 + 2 3 is an EB FC
irrational number. (1 Mark) from eqn (i) and (ii) we get
12. Given,
AD AF
3x – y = 5  ...(i) =
DO FC
and 5x – y = 11 ...(ii)
Solving eqn.(i) and eqn.(ii), we get DE || OC (by converse of Thale’s theorem) (1 Mark)
3x – y = 5 17. L.H.S = (secA – cosA) (cotA + tanA)
5x – y = 11 (1 Mark)
– +
_________–  1   cos A sin A 
=
 − cos A   +  (1 Mark)
–2x = –6  cos A   sin A cos A 
⇒ x = 3 (1 Mark)
 1 − cos 2 A   cos 2 A + sin 2 A 
put the value of x in eqn (i), we get =  
 cosA   sinA ⋅ cosA 
3×3–y=5 ⇒y=9–5
⇒ y = 4 (1 Mark) sin 2 A 1
= ⋅  (1 Mark)
13. Let the wife’s income be ` x cosA sin A × cos A

then man’s income = ` (x + 600) ½ Mark)


(½ sinA
= = tanA × secA = R.H.S.
cosA ⋅ cosA
Now according to question,
Hence, L.H.S = R.H.S proved. (1 Mark)
1 1
⇒ x + ( x + 600 ) =
1500 (1 Mark)
6 10 18. Given, 2cos3q = 3
1  x x + 600  3
⇒  + = 1500 ⇒ cos3q =  (1 Mark)
23 5  2
⇒ 8x + 1800 = 45000 ½ Mark)
(½  3
⇒ 8x = 43200 ⇒ x = 5400
⇒ cos3q = cos30° ∵ cos 30° =  (1 Mark)
 2 
Hence, wife’s income = ` 5400 30°
⇒ 3q = 30° ⇒ q = =10° (1 Mark)
Man’s income = ` (5400 + 600) = ` 6000 (1 Mark) 3

CBSE 2015 (Term-I) 299


19. 21. Given, x = p2q3 = p × p× q × q × q and
Money No. of xi di fidi C.f y = p3q = p × p × p × q (1 Mark)
saved students HCF (x, y) = p × p× q = p q2

(fi)
LCM (x, y) = p × p × p × q × q ×q = p³q³ (1 Mark)
5-7 6 6 –2 –12 6
7-9 3 8 –1 –3 9 Now, LCM (x, y) = p3q3 = p2q . pq2 = pq2 = HCF (x, y)
9-11 9 10 0 0 18 ∴ LCM is a multiple of HCF. (1 Mark)
11-13 5 12 1 5 23 LCM (x, y) = pq² HCF (x, y)
13-15 7 14 2 14 30 Let take x = 60, y = 90
Total Σfi = 30 Σfidi = 4 Now, x = 2 × 2 × 3 × 5
xi − 10 y=2×3×3×5
a = 10 (assumed mean), di = , (1 Mark)
2 HCF = 2 × 3 × 5 = 30
∑ fi di 4 LCM = 2 × 2 × 3 × 3 × 5 = 180 = 6 × 30
Now, mean = a+ × h = 10 + × 2 = 10.27
∑ fi 30 ∴ LCM = 6 HCF (1 Mark)
 (1 Mark)
Assumed mean a = 10, for median 22. Let the length of rectangular Pond be x and the breadth be y.
x − 10 Area of rectangle = l × b = xy (1 Mark)
∵ di = i
2 Now, according to question, (x – 2)(y + 1) = xy – 3
N 30 ⇒ x(y + 1) –2(y + 1) = xy – 3 (1 Mark)
 = = 15
2 2
⇒ x – 2y = –1  ...(i)
Cumulative frequency greater than 15 is 18. Thus
corresponding class is 9 – 11 and (x + 1)y = xy + 4 (1 Mark)
⇒ xy + y = xy + 4 ⇒ y = 4
∴ Median class = 9 – 11, l = 9, C.f = 9, h = 2
Put the value of y in eqn.(i), we get
N
− C. f ⇒ x – 2 × 4 = –1
15 − 9 4
Median =l+ 2 ×h = 9+ × 2 = 9 + =10.33
f 9 3 ⇒ x = 8–1 = 7
 (1 Mark) Hence, the length of pond is 7 ft. and breadth of pond
20. is 4 ft. (1 Mark)
Pocket No. of xi (let) a = 162.5 fidi 23. Out of the Syllabus
Money students x – 162.5
di = i
25
24. Out of the Syllabus
25. Given, DP || BC
100-125 14 112.5 –2 –28
We have to prove that PQ || AB
125-150 8 137.5 –1 –8
150-175 12 162.5 0 0 AD AP
 = (By Thale’s theorem)
175-200 5 187.5 1 5 BD PC
AD AP
200-225 11 212.5 2 22 = ...(i) (1 Mark)
AB AC
Total ∑ fi = 50 ∑ fidi = –9
Similarly, EQ || AC
(1 Mark)
BQ BE
Given, h = 25, Σfi = 50 and ∑ fidi = –9 = (By Thale’s theorem) (1 Mark)
QC EA
∑ fi di
Mean = a+ ×h  (1 Mark)
∑ fi BQ BE
=
 –9  BC AB
= 162.5 +   × 25 = 162.5 – 4.5 = 158 (1 Mark)
 50  BE = AD (given)

300
300 MATHS
BQ AD 28. We have from the given equation,
=  ...(ii) (1 Mark)
BC AB L.H.S = (sin q+cos q +1) (sin q –1 + cos q) sec q cosec q (1 Mark)
from eqn. (i) and (ii) we get =[(sinq + cosq) + 1) [(sin q+cos q) – 1]sec q cosec q
BQ AP BQ AP We know, (a + b) (a – b) = a2 – b2
= ⇒ = 
BC AC QC PC = [(sin q+cos q)² –12] secq cosecq (1 Mark)
By converse of Thale’s theorem = (sin²q + cos²q + 2sinq cosq – 1) secq cosecq
Hence, PQ || AB proved (1 Mark) We know, sin²q + cos²q = 1
26. = (1 + 2sin q cos q – 1)sec q cosec q
A
1 1
sec q = and cosec q =  (1 Mark)
cos θ sin θ
E 12 1 1
= 2 sin q cos q· ⋅ = 2 = R.H.S
7 P cos θ sin θ
3 Hence, L.H.S = R.H.S
6 5 4
B D C Hence proved. (1 Mark)
Given, In D ABC 29. Out of the Syllabus
AD ⊥ BC, CE ⊥ AB 30. Out of the Syllabus
(i) In DAPE and DCPD 31.

∠1 = ∠4 (each 90°) Index No. of weeks C.F


1500–1600 3 3
∠2 = ∠3 (vertically opposite angles)
1600–1700 11 14
∴ DAPE ~ DCPD (AA similarity) (1 Mark)
1700–1800 12 26
(ii) If DAPE ~ DCPD which we had proved them
1800–1900 7 33
AP PE 1900–2000 9 42
=
PC PD
2000–2100 8 50
AP × PD = CP × PE (1 Mark)
2100–2200 2 52
(iii) In DADB and DCEB which we had proved them Total Σfi = 52
∠5 = ∠7 (each 90°) N
∠6 = ∠6 (common) N = Σfi = 52 ⇒ = 26  (1 Mark)
2
DADB ~ DCEB (AA similarity) (1 Mark) The cumulative frequency just greater than 26 is 33 and
the corresponding class is 1800–1900
(iv) From above data (DADB ~ DCEB) we can say that
⸫ Median class = 1800–1900
AB AD
= N
CB CE − cf
 26 − 26 
AB × CE = AD × CB (1 Mark) Median = l + 2 × h= 1800 +   × 100
27. We have from the given equation,
f  12 
 (1 Mark)
L.H.S = (cot A+ sec B)2 – (tan B – cosec A)2
Median = 1800 ½ Mark)

Since,
­
(a + b)2 = a2 + b2 + 2ab and (a – b)2 = a2 + b2 – 2ab Now, maximum frequency = 12
= cot2A + sec2B + 2cot A sec B – tan2B – cosec2A ∴ modal class is 1700–1800
+ 2 tan B cosec A (1 Mark)
f1 − f 0
We know, cot2 q = cosec2 q – 1, sec2q = 1 + tan2q (1 Mark) Mode = l + ×h  (1 Mark)
2 f1 − f 0 − f 2
= cosec2 A – 1 + tan2B + 1 + 2 cotA secB – tan2B
– cosec2 A + 2 tan B cosecA (1 Mark) 12 − 11
1700 +
= × 100
= 2(cot A sec B + tan B cosec A) = R.H.S 2 × 12 − 11 − 7
Hence, L.H.S = R.H.S 1
Hence proved. (1 Mark) = 1700 + × 100 = 1716.67 ½ Mark)

6

CBSE 2015 (Term-I) 301


Term-II
2014 CBSE Solved Paper

Time allowed : 3 hours Maximum Marks : 90

GENERAL INSTRUCTIONS:
Read the following instructions carefully and strictly follow them:
(i) All questions are compulsory.
(ii) The question paper consists of 34 questions divided into four sections – A, B, C and D.
(iii) Section A contains 8 questions of 1 mark each, which are multiple choice type questions, Section B contains 6 questions
of 2 marks each, Section C contains 10 questions of 3 marks each and Section D contains 10 questions of 4 marks each.
(iv) Use of calculators is not permitted.

7. ABCD is a rectangle whose three vertices are B(4, 0),


Section-A C(4, 3) and D(0, 3). The length of one of its diagonals is:
1. If k, 2k – 1 and 2k + 1 are three consecutive terms of an  (1 Mark)
A.P., the value of k is: (1 Mark) (a) 5 (b) 4 (c) 3 (d) 25
(a) 2 (b) 3 (c) – 3 (d) 5 8. A chord of a circle of radius 10 cm subtends a right angle
2. Two circles touch each other externally at P. AB is a at its centre. The length of the chord (in cm) is: (1 Mark)
common tangent to the circles touching them at A and B. 5
(a) 5 2 (b) 10 2 (c) (d) 10 3
The value of ∠APB is: (1 Mark) 2
(a) 30o (b) 45o (c) 60o (d) 90o
3. In a right triangle ABC, right-angled at B, BC = 12 cm and Section-B
AB = 5 cm. The radius of the circle inscribed in the triangle
9. Find the values of p for which the quadratic equation
(in cm) is: (1 Mark)
4x2 + px + 3 = 0 has equal roots. (2 Marks)
(a) 4 (b) 3 (c) 2 (d) 1
4. In a family of 3 children, the probability of having at least 10. Find the number of natural numbers between 101 and 999
one boy is: (1 Mark) which are divisible by both 2 and 5. (2 Marks)
7 1 5 3 11. In Figure, common tangents AB and CD to the two circles
(a) (b) (c) (d)
8 8 8 4 with centres O1 and O2 intersect at E. Prove that AB = CD.
5. The angle of depression of a car parked on the road from  (2 Marks)
the top of a 150 m high tower is 30o. The distance of the
car from the tower (in metres) is: (1 Mark) A
(a) 50 3 (b) 150 3 (c) 150 2 (d) 75 D
6. The probability that a number selected at random from the
numbers 1, 2, 3, ..., 15 is a multiple of 4, is: (1 Mark)
O1 E O2
4 2
(a) (b)
15 15 B
(c)
1
(d)
1 C
5 3
12. The incircle of an isosceles triangle ABC, in which 24. In Figure, ABCD is a trapezium of area 24.5 sq. cm. In it,
AB = AC, touches the sides BC, CA and AB at D, E and F AD || BC, ∠DAB = 90o, AD = 10 cm and BC = 4 cm. If
respectively. Prove that BD = DC. (2 Marks) ABE is a quadrant of a circle, find the area of the shaded
22
13. Two different dice are tossed together. Find the probability region. [Take π = ] (3 Marks)
7
(i) that the number on each die is even. (2 Marks)
(ii) that the sum of numbers appearing on the two dice is 5. D E A
14. If the total surface area of a solid hemisphere is 462 cm , 2

22
find its volume. [Take π = ] (2 Marks)
7

Section-C
C B
16 15
15. Solve for x: =
−1 ; x ≠ 0, −1  (3 Marks)
x x +1 Section-D
16. The sum of the 5th and the 9th terms of an A.P. is 30. If its
25th term is three times its 8th term, find the A.P. (3 Marks) x − 2 x − 4 10
25. Solve for x: + = ; x ≠ 3,5 . (4 Marks)
17. Construct a triangle with sides 5 cm, 5.5 cm and 6.5 cm. x−3 x−5 3
3 26. In a school, students decided to plant trees in and around the
Now construct another triangle, whose sides are times the school to reduce air pollution. It was decided that the number
5
corresponding sides of the given triangle. [OS] (3 Marks) of trees, that each section of each class will plant, will be
18. The angle of elevation of an aeroplane from a point on double of the class in which they are studying. If there are 1 to
the ground is 60o. After a flight of 30 seconds the angle 12 classes in the school and each class has two sections, find
of elevation becomes 30o. If the aeroplane is flying at how many trees were planted by the students. Which value is
shown in this question?  (4 Marks)
a constant height of 3000 3 m, find the speed of the
aeroplane. (3 Marks) 27. The angle of elevation of the top of a tower at a distance
of 120 m from a point A on the ground is 45o. If the angle
19. If the point P(k – 1, 2) is equidistant from the points A(3, k)
of elevation of the top of a flagstaff fixed at the top of
and B(k, 5), find the values of k. (3 Marks)
the tower, at A is 60o, then find the height of the flagstaff.
20. Find the ratio in which the line segment joining the points [Use 3 = 1.73] (4 Marks)
A(3, –3) and B(–2, 7) is divided by x-axis. Also find the
28. Red queens and black jacks are removed from a pack
coordinates of the point of division.  (3 Marks)
of 52 playing cards. A card is drawn at random from
21. In Figure, two concentric circles with centre O, have radii the remaining cards, after reshuffling them. Find the
21 cm and 42 cm. If ∠AOB = 60o, find the area of the probability that the drawn card is: (4 Marks)
22
shaded region. [Take π = ] (3 Marks) (i) a king (ii) of red colour
7
(iii) a face card (iv) a queen
29. If A(–3, 5), B(–2, –7), C(1, –8) and D(6, 3) are the vertices
of a quadrilateral ABCD, find its area. [OS] (4 Marks)
O 30. A motorboat whose speed in still water is 18 km/h, takes
1 hour more to go 24 km upstream than to return downstream
60o to the same spot. Find the speed of the stream. (4 Marks)
C D
31. In Figure, PQ is a chord of length 16 cm, of a circle of
A B radius 10 cm. The tangents at P and Q intersect at a point
22. The largest possible sphere is carved out of a wooden T. Find the length of TP. (4 Marks)
solid cube of side 7 cm. Find the volume of the wood left. P
22
[Take π = ] (3 Marks)
7
23. Water in a canal, 6 m wide and 1.5 m deep, is flowing at T O
R
a speed of 4 km/h. How much area will it irrigate in
10 minutes, if 8 cm of standing water is needed for irrigation?
 [OS] (3 Marks) Q

CBSE 2014 (Term-II) 303


32. Prove that the tangent at any point of a circle is 34. A container open at the top, is in the form of a frustum of
perpendicular to the radius through the point of contact. a cone of height 24 cm with radii of its lower and upper
 (4 Marks)
circular ends as 8 cm and 20 cm respectively. Find the cost
33. 150 spherical marbles, each of diameter 1.4 cm, are dropped
in a cylindrical vessel of diameter 7 cm containing some of milk which can completely fill the container at the rate
water, which are completely immersed in water. Find the 22
rise in the level of water in the vessel. [OS] (4 Marks) of Rs 21 per litre. [Take π = ] [OS] (4 Marks)
7

EXPLANATIONS

1. (b) If k, 2k – 1, 2k + 1 are consecutive terms of an AP then 1


(2k – 1) – k = (2k + 1) – (2k – 1) Divide by both sides and put the values, we get
2
⇒ k – 1 = 2k + 1 – 2k + 1 ⇒ 5 × 12 = x × 5 + x × 12 + x × 13
⇒ k – 1 = 2 ⇒ k = 2 + 1 ⇒ k = 3 (1 Mark) ⇒ 60 = 30x ⇒ x = 2 cm ½ Mark)

2. (d) Given, two circles touch each other externally at P. 4. (a) A family has 3 children then total no. of possible
x y outcomes = 23 = 8
Now, favourable case, atlease one of the them is boy
means all those cases in which we have 1, 2 or 3 boys.
P
= {GGB, GBG, BGG, GBB, BGB, BBG, BBB}
α β
α β
n(5) = 7 (½½ Mark)
No. of favourable outcomes 7
A C B Now, Probability = =
AB is common tangent between the circles at point A Total no. of possible outcomes 8
and B respectively.  ½ Mark)

Let ∠CAP = α and ∠CBP = b 5. (b) Let AC be the height of tower and AB be the distance
CA = CP (length of tangents from external points C) between tower and car.
In ∆APB,  (½½ Mark) Given, Height = 150 m, Angle of depression = 30°
∠PAB + ∠PBA + ∠APB = 180o (Angle sum property) C
⇒ α + β + (α + β) = 180o 30o
⇒ 2α + 2β = 180o
⇒ α + β = 90o ½ Mark)

150m
3. (c) In right triangle ABC,
A
30o
A B
R
In ∆ABC, ½ Mark)

x
x O AC
P tan 30o =
AB
x
B C 1 150
Q ⇒ = ⇒ AB = 150 3 m  ½ Mark)

3 AB
By using Pythagoras theorem
6. (c) Given numbers {1, 2, 3, 4, ...... 15}
AC2 = AB2 + BC2
Total no. of sample space n(s) = 15
= 52 + 122 = 25 + 144 = 169 ⇒ AC = 13 cm (½ Mark) ½ Mark)
Let A be the event, multiple of 4 = {4, 8, 12} (½
Now, ar(∆ABC) = ar(∆AOB) + ar(∆BOC) + ar(∆AOC) \ n(A) = 3
1 1 1 1 n( A) 3 1
AB × BC= AB × OP + OQ × BC + OR × AC =
Probability = =  ½ Mark)

2 2 2 2 n( S ) 15 5

304 MATHS
7. (a) Given, ABCD is a rectangle 12. A
Y
a a
D (0, 3) C (4, 3)
F E

b c

X B c C
B (4, 0) b D
A (0, 0)
Given, ∆ABC is isosceles triangle in which
CD = AB = 4 units, AD = CB = 3 units ½ Mark)
(½ AB = AC with a circle inscribed in triangle
By Pythagoras theorem,
Now, AF and AE are tangents drawn to the circle from point A.
AC2 = AB2 + BC2 = 42 + 32 = 16 + 9 = 25  (1 Mark)
AC = 5 units. ½ Mark)
(½ Since two tangents drawn to a circle from the same exterior
8. (b) Let O be the centre, radius 10 cm. AB is the Chord point are equal.
which subtends 90o at the centre. ½ Mark)
(½ So, AF = AE, BF = BD, CD = CE
Now, In ∆OAB, ∆ABC is an isosceles triangle in which AB = AC.
a + b = a + c ⇒ b = c
Hence, BD = CD Proved. (1 Mark)
O 13. Two different dice are thrown then total possible outcomes
n(S) = 36
10 10
(a) Let A be the event that the number of each die is even.
A B Favourable outcomes = {(2, 2), (2, 4), (2, 6), (4, 2), (4, 4),
(4, 6), (6, 2), (6, 4), (6, 6)}
⇒ OA2 + OB2 = AB2 (using pythagoras theorem) \ n(A) = 9
⇒ 102 + 102 = AB2 ⇒ AB = 10 2 cm ½ Mark)
(½ n( A) 9 1
\ P(A)= = =  (1 Mark)
9. Given quadratic equation is 4x2 + px + 3 = 0 n( S ) 36 4
we know for equal roots, D = 0 (1 Mark) (b) Let B be the event that the sum of numbers appearing
⇒ b2 – 4ac = 0 ⇒ p2 – 4×4×3 = 0 on two dice is 5.
⇒ p2 = 48 ⇒ p = 4 3  (1 Mark) B = {(1, 4), (2, 3), (3, 2), (4, 1)}
10. Numbers between 101 and 999 those are divisible by 2 and n(B) = 4
5 are: 110, 120, 130, ......, 990 n( B ) 4 1
=
\ P(B) = =  (1 Mark)
The numbers are in AP, a = 110, d = 10, tn = 990(1 Mark) n( S ) 36 9
tn = a + (n – 1)d 14. Total surface area of hemisphere = 462 cm2
⇒ 990 = 110 + (n – 1)10 We know, total surface area of hemisphere = 3πr2
⇒ 990 – 110 = 10n – 10 22 2
⇒ 462 = 3 × × r ⇒ r = 7 cm (1 Mark)
⇒ 880 + 10 = 10n ⇒ 10n = 890 ⇒ n = 89 (1 Mark) 7
2 3
11. From given figures, Volume of hemisphere = πr
3
AB and CD are common tangents on both circles 2 22 3
V = × ×7 ⇒ V = 718.67 cm3 (1 Mark)
which intersect at E 3 7
Now, EA = EC  ...(i) 16 15
15. = −1 , x ≠ 0, −1  (1 Mark)
(tangents drawn on circle from an external point) x x +1
EB = ED  ...(ii) 16 − x 15
⇒ =  (1 Mark)
(tangents drawn on circle from an external point) (1 Mark) x x +1
⇒ (16 – x)(x + 1) = 15x ⇒ 16x + 16 – x2 – x = 15x
Adding equation (i) and (ii), we get
EA + EB = EC + ED ⇒ AB = CD (1 Mark) ⇒ –x2 + 15x + 16 = 15x ⇒ x2 = 16 ⇒ х = ± 4 (1 Mark)

CBSE 2014 (Term-II) 305


16. Given, t5 + t9 = 30 ⇒ 16 + k2 – 8k + k2 + 4 – 4k = 10

⇒ (a + 4d) + (a + 8d) = 30 [ tn = a + (n – 1)d] ⇒ 2k2 – 12k + 10 = 0 ⇒ k2 – 6k + 5 = 0
⇒ 2a + 12d = 30 ⇒ a + 6d = 15 ⇒ k2 – 5k – k + 5 = 0 ⇒ k(k – 5) – 1 (k – 5) = 0 (1 Mark)

⇒ a = 15 – 6d ...(i) (1 Mark) ⇒ (k – 1)(k – 5) = 0 ⇒ k = 1, 5 (1 Mark)
Now, t25 = 3t8 20. Let P(x, 0) divides AB in the ratio m : n

⇒ a + 24d = 3(a + 7d) ⇒ a + 24d = 3a + 21d Now, by section formula A (3,–3)
⇒ 2a = 3d ⇒ 2(15 – 6d) = 3d ⇒ 30 – 12d = 3d my2 + ny1
y= (1 Mark) m
⇒ 15d = 30 ⇒ d = 2 (1 Mark) m+n
putting value of d in eqn (i) m(7) + n(−3) x-axis
0= 0 P (x, 0)
a = 15 – 6 × 2 = 3 m+n n
Hence, the AP is a, a + d, a + 2d, ... ⇒ 7m = 3n
m 3 B (–2,7)
3, 5, 7, 9, .......... (1 Mark)
⇒ =  (1 Mark)
17. Out of the Syllabus n 7
mx2 + nx1 3(−2) + 7(3) −6 + 21 15 3
18. Given, Height of plane = 3000 3 m. and x
= = = = =
m+n 3+ 7 10 10 2
D C
Hence, the coordinate at x-axis is (3/2, 0). (1 Mark)
21. Given, Radius of bigger circle = 42 cm
Radius of smaller circle = 21 cm
3000 3

60o
30o
A x E B O
y
In ∆ABC, 60o
C D
BC
\ tan 30o = A B
AB
1 3000 3 Now, Area of shaded region = Area of bigger circle – Area
⇒ = ⇒ y = 9000  ...(i) (1 Mark)
3 y of smaller circle – [Area of larger sector – Area of smaller
sector] (1 Mark)
In ∆AED,
DE  θ θ 
= πR 2 − πr 2 −  o
× πR 2 − × πr 2 
tan 60 = o
 360 360o 
AE
22 22  60o 22 60o 22 
3000 3 = × (42) 2 − × (21) 2 −  × × (42) 2 − × × (21) 2 
⇒ 3 = ⇒ x = 3000  ...(ii) (1 Mark) 7 7  360
o
7 360o 7 
x
Distance covered = y – x (1 Mark)

= 9000 – 3000 = 6000 m 22 22 1 22 1 22


= × (42) 2 − × (21) 2 − × × (42) 2 + × × (21) 2
Distance 6000 7 7 6 7 6 7
∴ Speed = = = 200 m/sec. (1 Mark)
Time 30 5 22 5 22
= × × (42) 2 − × × (21) 2 = 3465 cm2  (1 Mark)
19. Given, P(k – 1, 2), A(3, k), B(k, 5) and P is equidistant 6 7 6 7
from the point A and B 22. Diameter of sphere = Side of cube (S) = 7 cm
⇒ PA = PB⇒ PA2 = PB2 (1 Mark) Radius of sphere = 7/2 cm (1 Mark)
⇒ (3 – (k – 1))2 + (k – 2)2 = (k – (k – 1))² + (5 – 2)2 Volume of wood left = Volume of cube – Volume of sphere
⇒ (4 – k)2 + (k – 2)2 = 12 + 32  (1 Mark)

306 MATHS
4 22 7 7 7 Now, 4, 8, 12, ........ 48 (1 Mark)
4
= S 3 − πr 3 = 7 × 7 × 7 − × × × × This series forms an AP
3 3 7 2 2 2
a = 4, d = 8 – 4 = 4, an = 48, n = 12 (1 Mark)
539 490
= 343 − = = 163.33 cm3 (1 Mark) Now,
3 3 n 12
23. Out of the Syllabus Sn
= (a + an ) ⇒= Sn (4 + 48) = 6 × 52 = 312
2 2
24. Given, Trapezium ABCD, ∠DAB = 90° Hence, total number of trees planted by students is 312.
AD = 10 cm, BC = 4 cm, Area = 24.5 cm2 (1 Mark)
1 27. Let the height of flagstaff be h m.
Area of trapezium = (sum of parallel side) ×
perpendicular 2 side Height of tower be x m.

1 ∠DAB = 45o, ∠CAB = 60o, AB = 120 m


⇒ 24.5 = × (10 + 4) × AB
2 C
49
⇒ AB = = 3.5 cm (1 Mark)
14 h
θ
Area of quadrant ABE = π(AB)2 D
360o
o
90 22 1 22 35 35 77 x
= o × × (3.5) 2 = × × × =
360 7 4 7 10 10 8 45o 60
o

= 9.625 cm² (1 Mark) A 120 B  (1 Mark)


Area of shaded portion = area of trapezium – area of quadiant BD x
In ∆ABD, tan 45o = ⇒ 1= ⇒ x= 120 m 
= 24.5 – 9.625 = 14.875 cm = 15 cm (Approx.) (1 Mark)
2 2 
AB 120
(1 Mark)
x − 2 x − 4 10
25. + = BC
x−3 x−5 3 In ∆ABC, tan 60o =
AB
( x − 2)( x − 5) + ( x − 4)( x − 3) 10 x+h
⇒ =  (1 Mark) ⇒ 3= ⇒ x + h = 120 3 ⇒ 120 + h = 120 3
( x − 3)( x − 5) 3 (1 120 Mark)
x 2 − 7 x + 10 + x 2 − 7 x + 12 10 ⇒ h = 120( 3 – 1) = 120(1.73 – 1) = 120 × 0.73

⇒ =
x 2 − 8 x + 15 3
⇒ h = 87.6 m
2 x 2 − 14 x + 22 10 x 2 − 7 x + 11 5 (1 Mark)
⇒ = ⇒ 2 =  (1 Mark) Hence, height of flagstaff = 87.6 m.
2
x − 8 x + 15 3 x − 8 x + 15 3
28. Given: Total no. of cards in pack = 52
⇒ 3x2 – 21x + 33 = 5x2 – 40x + 75
After removing red queen (2) and black jacks (2)
⇒ 2x2 – 19x + 42 = 0 (1 Mark)
⇒ 2x2 – 12x – 7x + 42 = 0 ⇒ 2x(x – 6) – 7(x – 6) = 0 Number of cards = 52 – 4 = 48
⇒ (x – 6) (2x – 7) = 0 ⇒ x – 6 = 0 or 2x – 7 = 0 (a) No. of kings = 4
⇒ x = 6 or x = 7/2 (1 Mark) 4 1
Probability of king = =  (1 Mark)
26. According to question, 48 12
There are two sections of each class so the number of trees (b) No. of red cards = 26 – 2 = 24
that are planted by students of class I, class II class III, 24 1
........ Class XI are. Probability of red card = =  (1 Mark)
48 2
No of trees planted by students of two sections of class I (c) No. face cards = 12 – 4 = 8
= 2 × 2 = 4 (1 Mark) 8 1
Probability of face card = =  (1 Mark)
No. of trees planted by students of two sections of class II 48 6
=2×4=8 (d) No. of queen cards = 4 – 2 = 2
Similarly, for class III = 2 × 6 = = 12 2 1
Probability of queen card = =  (1 Mark)
for class XII = 2 × 24 = 48 48 24

CBSE 2014 (Term-II) 307


29. Out of the Syllabus In ∆TPR,
30. Given, Speed of boat = 18 km/hr and Distance = 24 km. ∠TPR + ∠PTR + ∠PRT = 180° [Sum of ∠' of D]
Let the speed of stream be a x km/hr and t1, t2 be the time
⇒ ∠TPR + ∠PTR = 180° – 90° = 90° ...(ii)
of upstream and down stream.  (1 Mark)
∠RPO = ∠PTR [From (i) and (ii)]
distance
We know, Time = ∠TRP = ∠ORP = 90°
speed
For upstream, Speed = (18 – x) km/hr Distance = 24 km \ ∆TRP ~ ∆PRO (AA Similarity) (1 Mark)

24 TP RP
⇒ =
Time = t1 So, t1 =  ...(i) (1 Mark) PO RO
(18 − x)
For downstream, Speed = (18 + x) km/hr, Distance = 24 km TP 8 80 40
⇒ = ⇒ TP = =  (1 Mark)
24 10 6 6 3
time = t2 So, t2 =  ...(ii) (1 Mark)
(18 + x) 32. Given: A circle with centre O and a tangent XY at a point P
Now, according to question, t1 – t2 = 1 of the circle. (½½ Mark)
24 24
⇒ − =1
(18 − x) (18 + x)
⇒ 24(18 + x – 18 + x) = (18 – x) (18 + x)
⇒ 24 × 2x =324 – x2 ⇒ x2 + 48x – 324 = 0

⇒ x2 + 54x – 6x – 324 = 0

⇒ x(x + 54) – 6(x + 54) = 0
⇒ (x + 54) (x – 6) = 0 ⇒ x = –54, x = 6
To prove: OP ⊥ XY.  ½ Mark)

Then the speed of stream is 6 km/hr. (1 Mark)
Construction: Other than P, take a point Q, on XY. Join OQ.
31. Q Proof: Q is a point on the tangent XY, other than the point
of contact P. Hence Q lies outside the circle.
O R
T Let OQ intersect the circle at M.  (1 Mark)
10 8 Then, OM < OQ [a part (OM) of line segment (OQ) is less
P than the whole line segment (OQ)]  ...(i)
As we know that the lengths of tangent drawn from or But, OP = OM [radii of the same circle]. ...(ii)
external point are equal. From equation (i) and (ii), we get
\ PT = QT OP < OQ (1 Mark)
Hence, DPTQ is an isoscales triangle. Also, the tangents
Therefore, OP is shorter than any other line segment
drawn to a circle from an external point are equally inclined
drawn from O to any point on XY, except P.
to the line joining the external point and the centre.
\ ∠PTR = ∠QTR (1 Mark) Or, we can say that, OP is the shortest distance between
In isosceles triangle, the angle bisector of the vertex angle the point O and the line XY.
is ⊥ bisector of base. And also we know that the shortest distance of a point
Hence, OR ⊥ PQ, bisect PQ from the given line is the perpendicular distance.

\ PR = 8 cm Therefore, OP ⊥ XY. Hence, proved. (1 Mark)
In right ∆ORP, OR = 102 − 82 = 6 33. Out of the Syllabus
∠TPR + ∠RPO = 90 [Tangent is ⊥ to radius] ...(i) (1 Mark)
o
34. Out of the Syllabus

308 MATHS

You might also like